You are on page 1of 754

Microsoft Word - ACJC_JC2_H2_Math_2016_Prelim_P1_Questions 4

Microsoft Word - ACJC_JC2_H2_Math_2016_Prelim_P1_Solutions 9


Microsoft Word - ACJC_JC2_H2_Math_2016_Prelim_P2_Questions 22
Microsoft Word - ACJC_JC2_H2_Math_2016_Prelim_P2_Solutions 29
Microsoft Word - AJC 2016 H2 Math prelim P1 41
Microsoft Word - AJC 2016 H2 Math prelim P1 solution 46
Microsoft Word - AJC 2016 H2 Math prelim P2 56
Microsoft Word - AJC 2016 H2 Math prelim P2 Solutions 62
CJC P1 Questions 71
CJC P1 Solutions 77
CJC P2 Questions 103
CJC P2 Solutions 110
Microsoft Word - DHS_JC2_H2_Maths_2016_Prelim_Paper_1_Questions 135
DHS P1 Solutions 141
Microsoft Word - DHS_JC2_H2_Maths_2016_Prelim_Paper_2_Questions 158
DHS P2 Solutions 164
sgexamguru.com
Microsoft Word - 2016 HCI JC2 Prelim Paper 1 (Question) 176
Microsoft Word - 2016 HCI JC2 Prelim Paper 1 (Solution) 181
Microsoft Word - 2016 HCI JC2 Prelim Paper 2 (Question) 196
Microsoft Word - 2016 HCI JC2 Prelim Paper 2 (Solution) 202
IJC P1 Questions 212
ANNEX B 218
IJC P1 Solutions 220
IJC P2 Questions 236
ANNEX B 242
IJC P2 Solutions 244
Microsoft Word - 2016 JJC H2 Math P1 Prelim_Qn(updated) 259
Microsoft Word - 2016 JJC H2 Math P1 Prelim_Soln 265
Microsoft Word - 2016 JJC H2 Math P2 Prelim_Question 278
Microsoft Word - 2016 JJC H2 Math P2 Prelim_Solution 285
MJC P1 Questions 299
Microsoft Word - MJC_JC2_H2_Maths_Prelim_P1_Solutions 307
MJC P2 Questions 319
Microsoft Word - MJC_JC2_H2_Maths_Prelim_P2_Solutions 326
Microsoft Word - 2016 NJC SH2 H2 Mathematics Paper 1 Question Paper 336
Microsoft Word - 2016 NJC SH2 H2 Mathematics Paper 1 Solutions 342
Microsoft Word - 2016 NJC SH2 H2 Mathematics Paper 2 Question Paper 358
Microsoft Word - 2016 NJC H2 Mathematics Paper 2 Solutions 364
Microsoft Word - NYJC 2016 H2 Mathematics P1 378
Microsoft Word - NYJC 2016 H2 Mathematics P1 Solutions 384
Microsoft Word - NYJC 2016 H2 Mathematics P2 399
Microsoft Word - NYJC 2016 H2 Mathematics P2 Solutions 405
Microsoft Word - Prelim Paper 1 questions and solutions for PJC 414
Microsoft Word - Prelim Paper 2 questions and solutions for PJC 433
Microsoft Word - RI_JC2_H2Maths_2016_Prelim_P1 449
Microsoft Word - RI_JC2_H2Maths_2016_Prelim_P1_Solutions (updated) 455
Microsoft Word - RI_JC2_H2Maths_2016_Prelim_P2 468
Microsoft Word - RI_JC2_H2Maths_2016_Prelim_P2_Solutions(updated) 475
Microsoft Word - RVHS_JC2_H2_Maths_2016_Prelim_P1_Question 485
sgexamguru.com
Microsoft Word - RVHS_JC2_H2_Maths_2016_Prelim_P1_Solutions 490
Microsoft Word - RVHS_JC2_H2_Maths_2016_Prelim_P2_Question 502
Microsoft Word - RVHS_JC2_H2_Maths_2016_Prelim_P2_Solutions 509
Microsoft Word - 2016 SAJC Prelim Paper 1(updated) 524
Microsoft Word - 2016 SAJC Prelim Paper 1 Solutions 531
Microsoft Word - 2016 SAJC Prelim Paper 2 549
Microsoft Word - 2016 SAJC Prelim Paper 2 Solutions 556
SRJC P1 Questions 575
SRJC P1 Solutions 581
SRJC P2 Questions 600
SRJC P2 Solutions 607
Microsoft Word - 2016 TJC Prelim H2 Maths Paper 1 (Questions) 623
Microsoft Word - 2016 TJC Prelim H2 Maths Paper 1 (Solutions) 629
Microsoft Word - 2016 TJC Prelim H2 Maths Paper 2 (Questions) 639
Microsoft Word - 2016 TJC Prelim H2 Maths P2 (Solution) 645
Microsoft Word - 2016 TPJC H2 JC2 Prelim Paper 1 (Questions) 658
Microsoft Word - 2016 TPJC H2 JC2 Prelim Paper 1 (Solutions) 664
Microsoft Word - 2016 TPJC H2 JC2 Prelim Paper 2 (Questions) 675
Microsoft Word - 2016 TPJC H2 JC2 Prelim Paper 2 (Solutions) 681
Microsoft Word - 2016 VJC H2 JC2 Prelim Paper 1 (Solutions) 691
Microsoft Word - 2016 VJC H2 JC2 Prelim Paper 1 (Questions) 701
Microsoft Word - 2016 VJC H2 JC2 Prelim Paper 2 (Questions) 707
Microsoft Word - 2016 VJC H2 JC2 Prelim Paper 2 (Solutions) 712
9740/1 721
YJC P1 Solutions 726
9740/1 739
YJC P2 Solutions 744

sgexamguru.com
ANGLO-CHINESE JUNIOR COLLEGE
MATHEMATICS DEPARTMENT

MATHEMATICS
Higher 2 9740 / 01
Paper 1 18 August 2016
JC 2 PRELIMINARY EXAMINATION
Time allowed: 3 hours

Additional Materials: List of Formulae (MF15)

READ THESE INSTRUCTIONS FIRST

Write your Index number, Form Class, graphic and/or scientific calculator model/s on the cover page.
Write your Index number and full name on all the work you hand in.
Write in dark blue or black pen on your answer scripts.
You may use a soft pencil for any diagrams or graphs.
Do not use paper clips, highlighters, glue or correction fluid.

sgexamguru.com
Answer all the questions.
Give non-exact numerical answers correct to 3 significant figures, or 1 decimal place in the case of
angles in degrees, unless a different level of accuracy is specified in the question.
You are expected to use a graphic calculator.
Unsupported answers from a graphic calculator are allowed unless a question specifically states
otherwise.
Where unsupported answers from a graphic calculator are not allowed in the question, you are
required to present the mathematical steps using mathematical notations and not calculator commands.
You are reminded of the need for clear presentation in your answers.

The number of marks is given in brackets [ ] at the end of each question or part question.
At the end of the examination, fasten all your work securely together.

This document consists of 5 printed pages.

[Turn Over

Microsoft Word - ACJC_JC2_H2_Math_2016_Prelim_P1_Questions 1


2w  5
1 Without the use of a calculator, solve the inequality 0. [2]
w2  3

Hence solve
 2 y  5 sin x  0 , given that   x  3 . [3]
y2  3 2

x
2 The equation of the curve C is given by y  ln x . The line  with the equation y  is
e
tangential to the curve C at the point  e,1 . The region R is bounded by the curve C, the

line  and the x-axis. The solid S is formed by rotating the region R through 2 radians
about the x-axis. Find the exact volume of the solid S in terms of  and e. [5]

3 (i) Every year Warren Gate’s net worth increases by 100% of the previous year. His net
worth was estimated to be $1 000 000 on 31st December 1993. In what year will his
fortune first surpass 2.5 billion dollars (1 billion = 109). [3]
(ii) On 1st January 2005, Warren Gates deposits $100 000 in an investment account and
receives an interest of $1000 on 31st December 2005. After that the amount of interest

sgexamguru.com
earned at the end of the year is 1.5 times the amount of interest earned in the previous
year. Taking year 2005 as the first year, find the amount of savings that Warren Gates
has in his account at the end of the 15th year giving your answer to the nearest integer.
[2]
1
4 (a) It is given that g( x)  where x is sufficiently small for x 3 and
cos( 4  x) cos( 4  x)
 

higher powers of x to be neglected.


Show that g( x)  2  ax  bx 2 , where a and b are constants. [3]
m m

 g( x) dx   (2  ax  bx ) dx
2
Comment on the value of m for this expression to be
m m

valid. [1]
1
Find the first four non-zero terms of the expansion of 1  x 2 

(b) 2 in ascending powers of

x where x  1 . [2]

Hence find the first four non-zero terms of the Maclaurin’s series for cos 1 x in
ascending powers of x. [3]
[Turn Over

Anglo-Chinese Junior College


H2 Mathematics 9740: 2016 JC 2 Preliminary Examination Paper 1
ACJC P1 Questions Page 2 of 5 2
5 A sequence u1 , u2 , u3 , ... is given by

1  ur 1 
u1  1 and r 
 ur   2r for all r  1 .
3  3 

Use the method of differences to prove that un1  3n (3n2  3n  1) for all n  1. [4]

6 An investor deposits $K in a bank account. The bank offers an annual interest rate of 5%
compounded continuously. No further deposits are made. The amount of money in the
account at time t years is denoted by M. Both M and t are taken to be continuous variables.
Money is withdrawn at a continuous rate of $4000 per year. Set up a differential equation and
dM
show that for t > 0,  aM  b , where a and b are constants to be determined. [1]
dt
For t > 0, find M in terms of t and K. [4]
On a single clearly labelled diagram, show the graph of M against t for
(i) K > 80 000, [1]
(ii) K < 80 000. [1]
Hence state the condition for which the money deposited initially will be completely

sgexamguru.com
withdrawn in a finite period of time. [1]

7 (i) Use the substitution x  5sin  to find  25  x 2 dx . [5]

(ii) The circle with equation x 2  ( y  b) 2  25 where 0  b  5 , cuts the positive x-axis at
A(a, 0). The region R is bounded by the x and y axes, and the part of the circle lying in
the fourth quadrant as shown in the diagram below.
y

0 A(a, 0) x
R

Use your result in (i) to find the area of the region R in terms of a. [3]

Anglo-Chinese Junior College


H2 Mathematics 9740: 2016 JC 2 Preliminary Examination Paper 1
ACJC P1 Questions Page 3 of 5 3
8 The complex number z is given by z  k  i where k is a non-zero real number.
(i) Find the possible values of k if z  k  i satisfies the equation z 3  iz 2  2 z  4i  0 . [3]
(ii) For the complex number z found in part (i) for which k  0 , find the smallest integer
value of n such that z n  100 and z n is real. [3]

9 Use the method of mathematical induction to prove that


n
3r  1 n  7n  9 
 r  r  1 r  2   4  n  1 n  2  , for all n  1. [5]
r 1

n  7n  9  7
(i) Show that  . [2]
4  n  1 n  2  4
n
3r 7
(ii) Hence show that   . [2]
r 1  r  13 4

x 2  4k 2
10 The curve C has equation y  f ( x) , where f ( x)  . It is given that k is a constant and
xk
x  k.
Find the set of possible values that y can take. [3]
For the case k  1 ,
(i) sgexamguru.com
Sketch the graph of C, stating in terms of k, the coordinates of any points of intersection
with the axes and equations of any asymptotes. [3]

 f  x  dx in terms of k.
1
(ii) Hence find [3]
1

(iii) The graph of curve C is transformed by a scaling of factor 2 parallel to the x-axis,
followed by a translation of 2k units in the negative x-direction. Find the equation of the
new curve. You need not simplify your answer. [2]

11 Referred to the origin O, the position vectors of the points A, B and C are a, b and c
respectively. Given that a  b  4a  c , where b  4c and a is a non-zero vector,
(i) show that b  4c   a where  is a scalar. [1]
(ii) Hence evaluate b  c , given that the area of triangle OAB is 126 and  = 3 . [2]

(iii) Give the geometrical meaning of b  c . [1]

It is also given that b is a unit vector, a  5 , c  2 and b  4c  3a .

(iv) By considering  b  4c    b  4c  , find the angle between b and c . [3]

[Turn Over

Anglo-Chinese Junior College


H2 Mathematics 9740: 2016 JC 2 Preliminary Examination Paper 1
ACJC P1 Questions Page 4 of 5 4
12 (i) Solve the equation z 5  i  0, giving the roots in the form rei , where r  0 and
     . Show the roots z1 , z2 , z3 , z4 and z5 on an Argand diagram where

  arg( z1 )  arg( z2 )  arg( z3 )  arg( z4 )  arg( z5 )   . [5]

(ii) Find the exact cartesian equation of the locus of all points z such that z  z2  z  z3

and sketch this locus on an Argand diagram. Find the least possible value of z  z1 . [4]

(iii) Sketch on the same Argand diagram in (ii), the locus arg( z  z1 )  arg( z4 ) . [1]

(iv) Find the complex number z that satisfy the 2 equations z  z2  z  z3 and

arg( z  z1 )  arg( z4 ) , giving your answer in the form a  ib . [2]

13
4k
A D

sgexamguru.com B 2x C

The diagram shows the cross-section of a container. It is in the shape of a semicircle of fixed
radius 4k metres with a hole in the shape of a trapezium ABCD.
(i) If BC  2 x metres, show that the area S of the trapezium ABCD is given by

S   x+4k  16k 2  x 2 . [2]

(ii) Use differentiation to show that the area of the trapezium is maximum when
x  2 k metres. [4]
It is given that x  2 k metres and the length of the container is given as 3 metres. This
container is filled with water at a constant rate of 0.2 m3/s. At time t seconds the depth of
water in the container is h metres as shown.
 h 
(iii) Show that the volume V of water in the container is given by V  3h  4 k   . [2]
 3
(iv) Find, in terms of k, the rate at which the depth is increasing at the instant when the
depth is k 3 metres. [3]

- End of Paper -

Anglo-Chinese Junior College


H2 Mathematics 9740: 2016 JC 2 Preliminary Examination Paper 1
ACJC P1 Questions Page 5 of 5 5
 
2016 ACJC JC2 H2 Mathematics 9740
Preliminary Examination Paper 1 Markers Report

Qn Solution
1 2w  5
0
w2  3
–  –  OR
+  + 
           
 3 3 5  3 3 5
2 2
5
 3  w 3 or w
2
 2 y  5 sin x  0 ,
y2  3
2 y 5 3
Since sin x  0 for   x  0 ,
y2  3 2
5
From above,  3  y  3 or y
2
5
0  y  3 or y
2
5 5
 3  y  3 or y or y
2
2
sgexamguru.com
2

y
y = x/e
1 y = ln x

R
0 1 e x

Method 1 - Integration Method 2 – Volume of Cone


Volume of solid S
Volume of solid S
2
e x 
 Vol of cone     ln x  dx  
e 2
     dx     ln x  dx  
e 2
1
 
0 e 1

 1
x 2 dx     ln x  dx   (1) 2 (e)     ln x  dx
e e

2 e 2

e2 0 1 3 1
e
  x3 
    ln x  dx  
e

2
1
  e     ln x  dx  
e 2
e 2  3  0 1
3 1

  e3 
 2       ln x  dx  
e 2

e 3 1


 
Microsoft Word - ACJC_JC2_H2_Math_2016_Prelim_P1_Solutions 6
 
1
  e     ln x  dx  
e 2

3 1

     

  ln x 
e 2
     dx  
u1   ln x   
1 2
d v1
1
e
  x  ln x    2  ln x dx  
2 e

 1 1 dx
 
du1 1
  x  ln x    2  x ln x 1   1 dx
2 e e e
 2  ln x  ( ) v 1  x
 1 1 dx x

  x  ln x    2  x ln x 1  2  1 dx
e
u2  ln x  
2 e e
dv2
 1 1 1
dx  
  x  ln x    2  e ln e  ln11  2  x 1  
e
2 e e du2 1
 1    v2  x
dx x
  x  ln x    2  e ln e  ln1  2  e  1
2 e

 1
 e  2e  2e  2  
 e2 
 
Hence, volume of solid S
1
  e   (e  2)  
3
1
  e   e  2
3
2
 2   e
3
2
  (3  e)
3
sgexamguru.com
3 (i) 2 500 000 000 ≤ 1000 000(2)n-1
2n-1 ≥ 2500
ln 2500
n-1 ≥ = 11.2877
ln 2
n ≥ 12. 2877
His net worth will first exceed 2.5 billion when n = 13
The year 1993 + (13 -1)(1) = 2005 or 1993 + 13 – 1 = 2005

3 100 000 + 1000 + 1000(1.5) + 1000(1.52) + .......(15 terms) =


(ii) 1000(1.515  1)
100 000 + = $ 973 787.7808 = $ 973 788
1.5  1
4  
(a) 1 1
g ( x)  = =
 1 1  1 1   1 1 
 cos x  sin x  cos x  sin x   cos 2 x  sin 2 x 
 2 2  2 2   2 2 
1 2 2
= 2  = = 2(1  2 x 2 ) 1
1 (cos 2 x  sin 2 x) cos 2 x ( 2 x) 2 1  2x2
2 (1  )
2
2
 2(1  2 x 2 ) = 2 + 4x

m must be sufficiently small for g(x)  2 + ax + bx2 to be used


 
ACJC P1 Solutions 7
 
4   12    12    32 
(b) (1 – x2) -1/2 = 1 +
1
  x2  + 2!
x 
2 2
+

  12    32   52 
3!
x  =
2 3
1 1 x2  3 x4 
2 8
5 6
16
x  .....

1
cos 1 x   dx    (1  12 x 2  83 x 4  165 x 6  ....) dx
1 x 2

1 3 5 5 7
 ( x  x 3  x  x )C
6 40 112

When x = 0 , cos 1 0  =C
2

1 3 3 5 
cos 1 ( x )  ( x  x  x )
6 40 2

5 1  ur 1 
r 
 ur   2 r
3  3 
n n
 ur 1 ur 
  r 1  r
r 1  3 3
2 r
 r 1

u2 u1

32 31
u u
 33  22
3 3 sgexamguru.com
...
un un1
 
3 3n1
n

u u n 
 nn11  nn  2  1  n  
3 3 2 
un1 1
  n  n  1
3n1 3


1
un1  3n1  n  n  1    3n 3n2  3n  1
3
 
6 Rate of change = rate of growth – rate of decrease
Rate of change = rate of earning interest – rate of withdrawal
dM
 0.05M  4000
dt
1
 0.05M  4000 dM   1dt
1
ln 0.05M  4000  t  C
0.05
t C
ln 0.05M  4000  
20 20


 
ACJC P1 Solutions 8
 
t  20
C
0.05M  4000  e 20
C
0.05M – 4000= Ae 20 where A  e 20
t

M  80000  20 Ae 20
t

When t = 0 , M = K
K = 80000 + 20A
20A = K – 80000
Hence M  80000  ( K  80000)e 20
t

M K > 80000

80000 

t
K <  80000 
Money is completely withdrawn if K < 80000
7(i) Let x  5sin   
dx
 5 cos 
d
 25  x 2 dx  

  25  (5sin  ) 2 (5cos  )d  

  25  25sin 2  (5cos  )d  

  25(1  sin 2  )(5cos  )d  


sgexamguru.com
  25(cos 2  )(5cos  )d  
  (5 cos  ) 2 d  
 25 cos 2  d  
Let x  5 sin   
25
2 
 1  cos 2 d  
x
         sin    
25  1  5
    sin 2   c   x
2  2 
x
          sin
1
25  
   sin  cos    c   θ
5
2
25  x  x   25  x 2 
cos  
1
25  x 2  
  sin 1       c   5
2  5  5   5 


25 x 1
 sin 1  x 25  x 2  c
2 5 2
7(ii) x  ( y  b) 2  25  
2

          ( y  b) 2  25  x 2  
Since y  0 , y  b  0 , 
y  b   25  x 2  
y  b  25  x 2  


 
ACJC P1 Solutions 9
 
Area of region R
a
 
0
b  25  x 2 dx  
Substituting x = a
a a and y = 0 into the
 
0
b dx  
0
25  x 2 dx  
equation
a
 25 x 1  x 2  ( y  b) 2  25 ,
 bx 0   sin 1  x 25  x 2   
a

2 5 2 0 we have
25 1 a 1
 ab  sin  a 25  a 2   a 2  (0  b)2  25
2 5 2
1 25 1 a b  25  a 2
 a 25  a  a 25  a  sin
2 2
 
2 2 5
1 25 a 25 a 1
 a 25  a 2  sin 1  sin 1  a 25  a 2 .  
2 2 5 2 5 2
8 (i) z  k i  
z 2  (k  i ) 2  k 2  2(k )(i )  (i ) 2  (k 2  1)  (2k )i
z 3  (k  i )3  k 3  3(k ) 2 (i )  3(k )(i ) 2  (i )3  
       (k 3  3k )  (3k 2  1)i  

z 3  iz 2  2 z  4i  0
[(k 3  3k )  (3k 2  1)i ]  i[(k 2  1)  (2k )i ]  2[k  i ]  4i  0  
[(k 3  3k )  2k  2k ]  i[(3k 2  1)  (k 2  1)  2  4]  0

sgexamguru.com
(k 3  3k )  i (2k 2  6)  0   y
k (k 2  3)  0  and 2k 2  6  0   1
( k  0 or k   3)  and k   3  
 
Hence, k   3 0 x
 

8 z  3 i  k  0 
(ii)
z  1 3  2

arg( z )   
6
 
Method 1: By Polar Form & Trigonometry

  
z  2ei 6  2  cos  i sin   
 6 6
 n n 
z n  2n ein 6  2n  cos  i sin  
 6 6 
n
z n  is real  sin  0 
6
n
                       k , where k    
6
                      n  6k , where k    
Hence, n  0,  6,  12,  18, ...  


 
ACJC P1 Solutions 10
 
 
Method 2: By Properties of arg(z)

n
arg( z n )  n arg( z )   
6
z n  is real, the point representing z n  on the Argand diagram is on
the x-axis.
n
Thus,   arg( z n )   k , where k  
6
 n  6k , where k    
i.e. n  0,  6,  12,  18, ...  
 
Given  z n  100 . 
z n  z  2n  
n

Hence, 2 n  100  
But n is a multiple of 6. We then have
26  64  100
 
212  4096  100
The least value of n is 12.
9 n
3r  1 n  7n  9 
Let Pn be the statement  r  r  1 r  2   4  n  1 n  2  for
r 1
all integers n ≥ 1.

sgexamguru.com 3r  1 3 1 2
1
When n = 1, LHS =
r 1

r (r  1)(r  2)
 
1(2)(3) 3
79 2
RHS =  = LHS  P1 is true.
4(2)(3) 3
Assume that Pk is true for some positive integer k, k ≥ 1,
k
3r  1 k  7k  9 
i.e.  r  r  1 r  2   4  k  1 k  2 
r 1
Need to prove Pk 1 is true,
k 1
3r  1 (k  1)  7 k  16 
i.e.  r  r  1 r  2   4(k  2)  k  3
.
r 1
k 1
3r  1
LHS of Pk 1 =  r  r  1 r  2 
r 1
k
3r  1 3  k  1  1
=  r  r  1 r  2 
r 1

(k  1)(k  2)(k  3)
k (7 k  9) 3k  4
= 
4(k  1)(k  2) (k  1)(k  2)(k  3)
7k 3  30k 2  39k  16
=
4(k  1)(k  2)(k  3)


 
ACJC P1 Solutions 11
 
(7k 2  23k  16)(k  1)
=
4(k  1)(k  2)(k  3)

(k  1)  7 k  16 
= = RHS
4(k  2)  k  3
 Pk is true  Pk 1 is true.
Since P1 is true, and Pk is true Pk 1 is true, by the Principle of
n
3r  1 n  7n  9 
Mathematical Induction,  r  r  1 r  2   4  n  1 n  2  is
r 1
true for all integers n ≥ 1
9 (i) n  7n  9  7 6n  7
 
4  n  1 n  2  4 2  n  1 n  2 
6n  7
Since n is a positive integer, 0
2  n  1 n  2 
n
3r  1 n  7n  9  7 6n  7 7
    
r 1 r  r  1 r  2  4  n  1 n  2  4 2  n  1 n  2  4
 
9 (r  1)3  r 3  3r 2  3r  1
(ii)
r (r  1)(r  2)  r 3  3r 2  2r  (r  1)3  r (r  1)(r  2)

sgexamguru.com
1 1
So 
(r  1) 3
r (r  1)(r  2)
n
3r n
3r n
3r  1 7
 (r  1)3  r (r  1)(r  2)  r (r  1)(r  2)  4 .
 <
r 1 r 1 r 1

10
x 2  4k 2
y where k is a constant such that k  0
xk
xy  ky  x 2  4k 2
x 2  xy  (ky  4k 2 )  0
x is real  discriminant  0
y 2  4  ky  4k 2   0  

y 2  4ky  16k 2  0
 y  2k 
2
 12k 2  0
This inequality is true for all values of y.
Therefore y can take the set of all real numbers.
Alternative Method: 
dy x 2  2 xk  4k 2 ( x  k ) 2  3k 2
  0
x k ( x  k )2
2
  dx  
y is increasing, so y  .


 
ACJC P1 Solutions 12
 
10
(i) x 2  4k 2 3k 2
y  xk   
xk xk
Asymptotes: y  x  k and x  k

Points of intercept with axes: (0, 4k ), ( 2k , 0), (2k , 0)  

 
y
 

  4k  
 

  2k   O  2k   x
 

10
 
sgexamguru.com 3k 2
f  x  dx  2  f  x dx  2  x  k 
1 1 1
(ii)
 1 0 0 xk
dx
1
 x2 
 2   kx  3k 2 ln x  k 
2 0  
1 
 2   k  3k 2 ln 1  k  3k 2 ln  k 
2 
k
 1  2k  6k 2 ln .
k 1
10 2
(iii) x
   4k 2
x  4k
2 2
y 
2
y (scaling)
xk x
 k
2
2
 x  2k 
  4k
2
  
 x  2k   16k 2
2
 2 
translation  y  
 x  2k  2x
 k
 2 


 
ACJC P1 Solutions 13
 
11 a  b  4a  c
(i)  a  b    4a  c   0
 a  b    a  4c   0
a   b  4c   0
 a = 0 or  b  4c  = 0 or a is parallel to b – 4c
But we are given b ≠ 4c and a≠0, hence b – 4c =  a
(ii) 1
a  b  126
2
1
4a  c  126
2
126
ac 
2
 b  4c  126
 c  2
 3 
3 126 cc= 0
 b  c    4c  c  
2
378
b  c 
2
(iii) Area of parallelogram with adjacent sides OB and OC.
(iv) (b – 4c) . (b – 4c) = 3|a|2
|b|2 – 8 b . c + 16|c|2 = 3|a|2
b.c= 
10
8 sgexamguru.com
10

b.c
cos    8
| b || c | 1(2)
  128.7
12 z5  i  0
(i)
z5  i    
  
i (2 k  )
z 5  ei 2  e 2
, where k    
2 k   
i(  )
ze 5 10
   
7 3   9
 i  i i i i  
Putting  n  2, 1, 0,1, 2 ,  z  e 10
,e 10
,e ,e ,e
10 2 10
 
 
   
Given   arg( z1 )  arg( z2 )  arg( z3 )  arg( z4 )  arg( z5 )   .   
7 3   9  
 i  i i i i
i.e. z1  e 10
, z2  e 10
, z3  e10 , z4  e 2 , z5  e 10    
.
Let the points P1, P2, P3, P4 and P5 on the Argand diagram
represent the complex numbers z1 , z2 , z3 , z4 , and z5 .


 
ACJC P1 Solutions 14
 
y

P4(0, 1)

P5     P3

   
-1   1 x
0

P1 P2
-1
(ii) The locus  z  z2  z  z3  is a perpendicular bisector of the line  
 
segment P2P3 where P2  z2  and  P3  z3 .  
Since OP2P3 is an isosceles triangle, the perpendicular cuts  
through the origin and bisect the angle P2OP3.  
  
Thus the perpendicular bisector is inclined at angle  radian
10  
below the positive x-axis.  
  
Cartesian equation of locus z  z2  z  z3  is y   x tan( ) . 

sgexamguru.com
 
10

10 
 
ACJC P1 Solutions 15
 
 y

P4(0, 1)

P5     P3
     
-1   1 x
0

P1 P2
-1

(ii) z  OP  

z1  OP1  
  
z  z1  OP  OP1  i.e. z  z1  PP
1  

The least value of z  z1 is the shortest distance from P1 to the


perpendicular bisector (line OP5).
  N
  P5 0
4
sgexamguru.com
P1 N
 sin
OP1 10
4 2
P1 N  OP1 sin  sin  0.951
10 5
 
P1

(iii) The locus arg( z  z1 )  arg( z4 ) is a half-line with its initial point
at P1 and above and excluding P1, parallel to the y-axis. (Refer to
diagram.)
12 The intersection point between the 2 loci has the same x-
(iv) 7 3
coordinates as the point P1, i.e.   x  cos   cos . 
10 10
3
Substituting x   cos  into equation of the perpendicular
10

bisector y   x tan( ) , we have  
10
3  3 
y  ( cos ) tan  cos tan  0.191  
10 10 10 10
3
x   cos  0.588  
10
Coordinates of the intersection point are  ( 0.588, 0.191) . 

11 
 
ACJC P1 Solutions 16
 
13
A E D
(i)

H 4k

B F x C

Let E be the centre of the circle and F be the foot of


perpendicular from the point E to BC.
Since EC and ED are both radii of the circle, ED  EC  4k .
Let H be the length of EF, which is the height of the trapezium.
By Pythagora’s theorem,
H 2  x 2  (4k ) 2
H 2  16k 2  x 2
H  16k 2  x 2
1 1
S H ( BC  AD )  H  2 x  8k 
2 2
 ( x  4k ) 16k 2  x 2 ( shown)

(ii) 1  
 16k 2  x 2   x  4k  x 16k 2  x 2  2
dS 
 
dx


sgexamguru.com
2 x 2  4kx  16k 2
0
 

16k 2  x 2
x 2  2kx  8k 2  0
 x  2k  x  4k   0
 x  2k or x  4k (N.A, x  0)
dS 2 x 2  4kx  16k 2 2( x 2  2kx  8k 2 )
 
dx 16k 2  x 2 16k 2  x 2
 2 1 
 16k  x  2 x  2k   ( x  2kx  8k ) 2 16k  x  (2 x) 
1
2 2 2 2 2  2
d 2S
 2  
dx 2  16k 2  x 2 
 
 2(16k 2  x 2 ) 3 2  2( x  k )(16k 2  x 2 )  x( x 2  2kx  8k 2 ) 
 2(16k 2  x 2 ) 3 2  2( x 3  kx 2  16k 2 x  16k 3 )  ( x3  2kx 2  8k 2 x) 
 2(16k 2  x 2 ) 3 2   x 3  24k 2 x  32k 3 
when x  2k
d 2S 2(72k 3 )
2
 2(16k 2  4k 2 ) 3 2  8k 3  48k 3  32k 3      12  0
dx 12 12k 3
Area of trapezium is maximum when x=2k.

12 
 
ACJC P1 Solutions 17
 
(iii) 2k 2k 2k 2k
2k

2k 2k H H y

h h

y 2k 2k y

Given x  2k  and from (i) we have  H  16k 2  x 2 . 


 H  16k 2  x 2  16k 2  (2k ) 2
 16k 2  4k 2  12k 2  2 3k
Using similar triangles:
y h

2k H
 h  h
y  2k 
 2 3k  3
V  area of trapezium  length of container
1
=  h  4k  (4k  2 y )  (3)
2
3
=  h  8k  2 y 
2
sgexamguru.com
=3  h  4k  y 
 h 
=3h  4k   ( shown)
 3

(iv)
dV  h   1 
  4k   3  3h  
dh  3  3
6h
 12k 
3
dV
When h  3k ,  18k
dh
dV dV dh
 
dt dh dt
dh
0.2  18k 
dt
dh 1
 m/s
dt 90k

13 
 
ACJC P1 Solutions 18
ANGLO-CHINESE JUNIOR COLLEGE
MATHEMATICS DEPARTMENT

MATHEMATICS
Higher 2 9740 / 02
Paper 2 22 August 2016
JC 2 PRELIMINARY EXAMINATION
Time allowed: 3 hours

Additional Materials: List of Formulae (MF15)

READ THESE INSTRUCTIONS FIRST

Write your Index number, Form Class, graphic and/or scientific calculator model/s on the cover page.
Write your Index number and full name on all the work you hand in.
Write in dark blue or black pen on your answer scripts.
You may use a soft pencil for any diagrams or graphs.
Do not use paper clips, highlighters, glue or correction fluid.

sgexamguru.com
Answer all the questions.
Give non-exact numerical answers correct to 3 significant figures, or 1 decimal place in the case of
angles in degrees, unless a different level of accuracy is specified in the question.
You are expected to use a graphic calculator.
Unsupported answers from a graphic calculator are allowed unless a question specifically states
otherwise.
Where unsupported answers from a graphic calculator are not allowed in the question, you are
required to present the mathematical steps using mathematical notations and not calculator commands.
You are reminded of the need for clear presentation in your answers.

The number of marks is given in brackets [ ] at the end of each question or part question.
At the end of the examination, fasten all your work securely together.

This document consists of 8 printed pages.

[Turn Over

Microsoft Word - ACJC_JC2_H2_Math_2016_Prelim_P2_Questions 19


Section A: Pure Mathematics [40 marks]

1 A triangular region R is drawn on a large sheet of graphing paper marked in 1mm squares.

The region R is bounded by the x-axis, the line y = 1 x and the line y =  1 x  40 . The
20 20

scales on both the axes are such that 1 mm represents 1 unit.


By using the table below or otherwise, find the number of complete 1mm squares which lie
inside region R. [3]
Range of x Number of complete 1 mm squares which lies inside region R
20  x  40
40  x  60
60  x  80
.
.
.

2 The parametric equations of curve C are


x  a cos3 t , y  a sin 3 t for 0  t   ,

(i) sgexamguru.com
where a is a positive constant.
Find the coordinates of the points on the curve where the tangent is parallel to the
x-axis and the coordinates of the points at which the tangent is parallel to the y-axis. [5]
(ii) Hence sketch the curve C. [1]
(iii) If the cartesian equation of curve C is y  f ( x) , use the curve in part (ii) to sketch the
graphs of
1
(a) y  , (b) y  f '( x) ,
f ( x)
stating in each case, the equations of any asymptotes and the coordinates of any points
of intersection with the axes. [4]
(iv) The point P on the curve has parameter p. Show that the equation of tangent at P is
x sin p  y cos p  a sin p cos p . [2]
(v) The tangent at P is perpendicular to the tangent at another point Q, on the curve.

If p  , find the value of the parameter t at point Q. [2]
3

[Turn Over

Anglo-Chinese Junior College


H2 Mathematics 9740: 2016 JC 2 Preliminary Examination Paper 2
ACJC P2 Questions Page 2 of 7 20
3 The functions h and g are defined by
2 x 1
h:x  e  1 , x  , x  k ,

 2x for 0  x  12 ,
g:x  
 2  2 x for 2  x  1.
1

(i) Given that the function h has an inverse, state the greatest value of k. Find h 1  x  and

write down the domain of h 1 . [4]


(ii) Explain why the composite function gg exist. [1]
(iii) Sketch the graph of y  gg  x  . [2]
2 x 1
(iv) Given that f : x  e  1 , x  , find the range of fg exactly. [1]

4 The equations of three planes p1 , p2 , p3 are


2 x  3 y  6 z  10,
2x  3 y  6 z  a ,

sgexamguru.com
respectively, where a, b are constants.
x  y  bz  5,

5 x y
The planes p1 and p3 intersect in the line l with cartesian equation   z.
3 4
(i) Show that b  1 . [2]
(ii) The point S lies on p1 and the point R has coordinates  2, 4, 1 . Given that RS is

perpendicular to p3 , find the coordinates of S. [4]

8
The planes p1 and p2 are units apart.
7
(iii) Given that a < 0, find the possible values of a. [4]
(iv) The point P with coordinates  5, 2, c  lies on p1. Find the value of c. [1]

(v) The point F is the foot of the perpendicular from P to the line l. The point Q is the
reflection of F in the plane p2 . Find the distance PF and hence find the area of
triangle FPQ. [4]

Anglo-Chinese Junior College


H2 Mathematics 9740: 2016 JC 2 Preliminary Examination Paper 2
ACJC P2 Questions Page 3 of 7 21
Section B: Statistics [60 marks]

5 Florida fitness club wants to carry out a survey to find out from their members the facilities
that the club can improve on. The club has a list of all the 15000 members’ names.
(i) Describe how to obtain a systematic sample of 500 members from the list to take part in
the survey. [2]
(ii) State one disadvantage of using a systematic sample in this context. [1]

6 A factory manufactures rectangular glass panels. The length and breadth of each panel, in cm,
are modelled as having independent normal distributions with means and standard deviations
as shown in the table.
Glass Panel Mean (cm) Standard Deviation (cm)
Length 300 0.5
Breadth 150 0.2

The probability that the total perimeter of 2 randomly selected glass panels exceeds the mean
length of n randomly selected glass panels by more than 1501cm is less than 0.2576. Find the
least value of n. [5]

7 sgexamguru.com
The mean number of guests checking into a hotel in an hour is 3.6 and can be modelled by a
Poisson distribution.
(i) Find the probability that not more than 4 guests checked into the hotel in a given hour.[1]
(ii) Given that three non-overlapping one-hour blocks are chosen at random, find the
probability that one of the blocks has not more than 4 guests checking into the hotel and
the remaining two blocks have no guests checking into the hotel. [2]
(iii) Given that each day consists of 24 non-overlapping one-hour blocks. Use a suitable
approximation, to find the probability that between 85 and 90 guests checked into the
hotel in a particular day. State the parameters of the distribution that you use. [3]
(iv) The probability of at least n one-hour blocks in a day of 24 non-overlapping one-hour
blocks has not more than 4 guests checking into the hotel is less than 0.124. Find the
least value of n. [3]
(v) Explain why the Poisson distribution may not be a good model for the number of guests
checking into the hotel in a year. [1]

[Turn Over

Anglo-Chinese Junior College


H2 Mathematics 9740: 2016 JC 2 Preliminary Examination Paper 2
ACJC P2 Questions Page 4 of 7 22
8 Tandao Café has an outlet at North Vista and another outlet at South Parc. On weekdays, the
waiting time during lunch periods in each outlet follows a normal distribution with mean 
minutes.
(i) Hono has lunch regularly at the North Vista outlet. On 10 randomly selected weekdays,
his waiting times per visit were recorded, in minutes, as follows:
49, 38, 43, 70, 45, 51, 57, 85, 39, 44
Test, at the 10% significance level, whether the mean waiting time is less than one
hour. [4]
(ii) Lulu has lunch regularly at the South Parc outlet. On 56 randomly selected weekdays,
her waiting times per visit were recorded, in minutes. It was found that the sample mean
waiting time is t minutes and the sample variance is 69.8 minutes2. A test is to be
carried out at the 5% level of significance to determine whether the average waiting
time at the South Parc outlet is not one hour.
Find the range of values of t for which the result of the test would be that the null
hypothesis is rejected, leaving your answers in 2 decimal places. (Answers obtained by
trial and improvement from a calculator will obtain no marks.) [4]

9
sgexamguru.com
A B

(a) The diagram above shows a goalkeeper and a front view of a goal post labelled A on
one side and B on the other side. Aaron is a goalkeeper in a football club. Based on his
past experiences as a goalkeeper in penalty shoot-outs, the probability that he dives to
side B is 0.72. In a particular match, Aaron’s team went into a penalty shoot-out. The
probability that a penalty kicker kicks the ball to side B is p, where 0  p  1 . Assume
that Aaron's choice of direction to dive is independent of the penalty kicker’s choice of
direction to kick the ball.
(i) Show that the probability Aaron dives in the same direction as the ball is kicked is
0.44 p  0.28 . [1]
(ii) If Aaron dives in the same direction as the ball is kicked, the probability that he
saves the ball is 0.4. Find, in terms of p, the probability that Aaron fails to save
the ball. [3]
Hence find the values between which this probability must lie. [1]

Anglo-Chinese Junior College


H2 Mathematics 9740: 2016 JC 2 Preliminary Examination Paper 2
ACJC P2 Questions Page 5 of 7 23
(b) In any football match, the expected number of saves made by Aaron is 4.8, with a
standard deviation of 1.2. Find the probability that in 50 matches, the average number of
saves per match made by Aaron is less than 5. [2]
(c) Aaron can either be a goalkeeper or a forward in a match, depending on the strategy that
the team uses. In the team, there is another goalkeeper, 6 defenders, 5 midfielders and 5
forwards. Altogether, there are 18 members in the team.
(i) Find the number of ways to select 4 defenders, 4 midfielders, 2 forwards and a
goalkeeper from the team, given that Aaron is selected. [3]

After a match, the team stands in a straight line to take a photo.


(ii) Find the number of ways such that all the defenders are standing alternately with
all the midfielders. [2]
The team is then asked to sit in a circle for a debrief.
(iii) Find the probability that the two goalkeepers sit opposite each other given that
a group consisting of 2 particular midfielders and 6 defenders are seated together.
[3]

10 (i)
sgexamguru.com
Sketch a scatter diagram that might be expected when h and s are related approximately
as given in each of the models (A) and (B) below. In each model, your diagram should
include 6 points, approximately equally spaced with respect to h, and all h- and s-
values positive. The letters a, b, c and d represent constants.
(A) s  a  b ln h , where a is negative and b is positive.
d
(B) s  c  , where c is positive and d is negative. [2]
h
A company recently launched a new product in Singapore and wanted to know more about
the relationship between the number of promoters, h, and the product’s monthly sales, s, in
Singapore dollars. They collected data for the past 9 months and the results are given in the
table.
h 50 60 70 80 90 100 110 120 130
s 40 000 47 000 52 000 55 000 57 800 60 000 61 500 62 500 63 000

(ii) Draw a scatter diagram for these values, labelling the axes. [1]
(iii) Comment on whether a linear model would be appropriate, referring to both the scatter
diagram and the context of the question. [2]

[Turn over
Anglo-Chinese Junior College
H2 Mathematics 9740: 2016 JC 2 Preliminary Examination Paper 2
ACJC P2 Questions Page 6 of 7 24
(iv) It is required to estimate the number of promoters needed to achieve a monthly sales of
$75,000. Using an appropriate model in part (i) to find the equation of the suitable
regression line, correct to 3 decimal places. Use your equation to find the required
estimate, correct to the nearest integer. [2]
(v) Comment on the reliability of your estimate. [1]
(vi) Given that 1 US dollar = 1.34 Singapore dollars, re-write your equation from part (iv),
correct to 3 decimal places, so that it can be used to estimate the number of promotors
when the product’s monthly sales is given in US dollars. [1]

11 The manager of a car show room wants to study the number of cars sold by the 2 car salesman
under his charge. The number of potential car-buyers that they meet in a particular week and
the average probabilities that each salesman is successful in closing a deal with each customer
is given in the table below.
Salesman Number of potential car-buyers Probability of closing a deal
X 60 0.2
Y 50 0.3

(i)
sgexamguru.com
It is assumed that the deals closed are independent of one another. State, in context,
another assumption needed for the number of deals closed by a car salesman to be well
modelled by a binomial distribution. [1]
(ii) Explain why the assumption that the deals closed are independent of one another may
not hold in this context. [1]
Assume now that the assumptions stated in part (i) do in fact hold and the deals closed by
salesman X is independent of the deals closed by salesman Y.
(iii) Use suitable approximations to find the probability that both salesmen collectively
closed a total of more than 20 deals in a particular week. State the parameters of the
distributions that you use. [4]
A new salesman joined the company. During his probation week, he met 60 potential car-
buyers. The number of car deals he closed during his probation week is denoted by C with the
distribution B(60, p ) .
(iv) Given that P(C  30)  0.03014 . Find an equation for p. Hence find the value of p,
correct to 1 decimal place, given that p  0.5 . [2]
(v) Given that p  0.05 , use a suitable approximation, which should be stated, to find the
probability that he sold more than 4 cars. [2]
- End of Paper -

Anglo-Chinese Junior College


H2 Mathematics 9740: 2016 JC 2 Preliminary Examination Paper 2
ACJC P2 Questions Page 7 of 7 25
Anglo-Chinese Junior College
H2 Mathematics 9740
2016 JC 2 Pelim Paper 2 Solutions
Qn Solutions Remarks
1    
 
 
  y=  
 

 

 
  20  40 
 
Number of squares for 20  x  40 is 20
 

Column Range of x Number of squares


1 20  x  40 20
2 40  x  60 40
3 60  x  80 60
.
.

n 380  x  400
380
Number of columns = n = = 19 for 0  x  400
20
OR 380 = 20 + (n-1)(20)  n =19
OR
sgexamguru.com
400 = 40 + (n-1)(20)  n =19
AP sequence : 20, 40, 60, …….. 19 terms
Number of complete squares in region R for 0  x  400 is
19
 2(20)  18(20) = 3800
2
Region R is symmetrical about the line x = 400.

Total number of squares in region R = 3800 X 2 = 7600


 
 
 
2(i)
x  a cos3 t , y  a sin 3 t for 0  t  
dy dy dt 3a sin 2 t cos t
     tan t
dx dt dx 3a cos2 t sin t
dy
Let 0
dx
 tan t  0  t  0,   
Points on curve where tangent is parallel to x-axis are (a,0) and (  a,0)
dx 1 
Let 0 0t 
dy  tan t 2
Point on curve where tangent is parallel to y -axis are (0,a)


 
Microsoft Word - ACJC_JC2_H2_Math_2016_Prelim_P2_Solutions 26
Anglo-Chinese Junior College
H2 Mathematics 9740
2016 JC 2 Pelim Paper 2 Solutions
2
 
(ii)
  y
  (0,a)
 
 
 
  x
(-a,0) O (a,0)
 
 
 
2
  y
(iii)
(a)  
 
 
  (0,1/a)
O x
 
x= -a x=a
2 y
 
(iii)
(b)  
 
  sgexamguru.com y=f ’(x)

  (a,0)
x
  (-a,0) O
 
 

 
2
Equation of tangent at point P with parameter p:
(iv)
y  a sin 3 p   tan p( x  a cos3 p)
y cos p  a sin 3 p cos p   sin p( x  a cos3 p)
 
x sin p  y cos p   a sin p cos p  (cos 2 p  sin 2 p)
x sin p  y cos p  a sin p cos p ( shown)
2
 
(v) Given p  , gradient of tangent at P is  tan  3
3 3
1
 gradient of tangent at Q   
3
1 5
Value of parmeter at Q :  tan t  t 
3 6


 
ACJC P2 Solutions 27
Anglo-Chinese Junior College
H2 Mathematics 9740
2016 JC 2 Pelim Paper 2 Solutions

3
(i) Sketch y=f(x)
y

h(x) has an inverse if x   12 .


Greatest value of k is  12
y – 1 = e 2 x 1
ln (y – 1) = 2 x  1
2x + 1 =  ln( y  1)
x   1  1 ln( y  1)
2 2

x   1  1 ln( y  1) since x   12
2 2
h (x) =  1  1 ln( x  1)
-1

sgexamguru.com
2 2
h( )  2
1
2

Domain of h-1(x) = Range of h(x) = 2,  

(ii) 1 
y = g(x)
 

0      1 

Composite function exist because


Rg = [0,1]  Dg = [0,1]

g g
(iii) [0, 14 ] [0, 1 ] [0,1]
2

y = gg(x)  

0  1 
g g
[ 1 , 1 ] [ 1 ,1] [0,1]
4 2 2
g g
[ 1 , 3 ] [ 1 ,1] [0,1]
2 4 2
g g 3 
[ 3 ,1] [0, 1 ] [0,1]
4 2
 
ACJC P2 Solutions 28
Anglo-Chinese Junior College
H2 Mathematics 9740
2016 JC 2 Pelim Paper 2 Solutions
g f
(iv) [0,1] [0,1]  e  1, e3  1 

R fg  e  1, e3  1 
4 (i)  1   3 
 1  . 4   0
b  1 
   
3  4  b  0  
b  1

Alternatively,

 1   2   3 
 1    3  =k   4   
 b   6   1 
     

 6  3b   3 
 6  2b  = k   4   
 1  1
   

Equating k component,

k  1.  

sgexamguru.com
Equating i component,

6  3b  3k  

 3b  3(1)  6
b  1 (Shown)

4  2  1
(ii) Line RS: r   4     1  ,   
1  1
   
Substitute this line to p1,
 2     
2
 4    .  3   10
 1  
   6 
8

11
 14 
  11 
  2     52 
OS   4    
 1     11 
   
 3 
 11 


 
ACJC P2 Solutions 29
Anglo-Chinese Junior College
H2 Mathematics 9740
2016 JC 2 Pelim Paper 2 Solutions
4  2
 
(iii)  3
 6  10
p1 : r.   
7 7
 2
 
 3
 6  a
p2 : r.   
7 7

8 10  ( a) 8 (a )  10
distance between two planes :  or 
7 7 7 7
a = -2 or a = -18

4 5   2
(iv)   
p1 :  2  .  3   10
 c   6 
  
10  6  6c  10
c 1

4 Let point A be (5, 0, 0) which is a point on line l.


(v)  0   3   2 
  3     
 2  ×  4   3 
PF  sgexamguru.com
AP ×  4 
1
 
 3 

1   1   6 
 
 3 

 
 3 
 4  4  4
1 1 1
     
7
  1.3728
26
 
 8  16
QF  2   
7 7
1  16 
Area PFQ = (1.3728)    1.5689  1.57
2  7
 
 
 
 
 
 
 
 
 
 


 
ACJC P2 Solutions 30
Anglo-Chinese Junior College
H2 Mathematics 9740
2016 JC 2 Pelim Paper 2 Solutions
Alternative method:
  5  3 
OF   4 
  
 
    3 
FP  OP  OF   2  4 
 1  
 
  3 
Since FP   4  ,
1
 
 3   3 
 2  4  .  4   0
 1    1 
  
9  8  16  1    0
9

26
  9    27 
  3  26    
  
 
26
  9   8 
 FP  2   4        1.3728
  26    13 
  9    17 
 1      26 
  26  

sgexamguru.com
 8  16
QF  2   
 

7 7
1  16 
Area PFQ = (1.3728)    1.5689  1.57
2 7

5 (i) Number the club members in order from 1 to 15000


according to the name list.
(Alphabetical order)
15000
Since k   30 , select a member randomly from the
500
name list.
Thereafter, select every 30th member cycling to the start of
the list if the end of list is reached until we form a sample of
500 members.
5(ii)
The list is not representative since we have selected a group
of all same gender or same age group who may have same
request on the facilities.
6 Let the random variable L be the length of a randomly
chosen glass panel.

Let the random variable B be the breadth of a randomly


chosen glass panel.


 
ACJC P2 Solutions 31
Anglo-Chinese Junior College
H2 Mathematics 9740
2016 JC 2 Pelim Paper 2 Solutions
Let X  2 L1  2 B1  2 L2  2 B2
 X ~ N 1800, 2.32 

L1  L2  L3  ...  Ln
Let L 
n
 0.52 
 L ~ N  300, 
 n 

 0.52 
X  L ~ N 1500, 2.32   
 n 

P  X  L  1501  0.2576
 
 
1501  1500
PZ    0.2576
 0.52 
 2.32  
 n 
 
 
1
PZ    0.7424
 0.52 
 2.32  

1 sgexamguru.com
n 

 0.6507622042
0.52
2.32 
n
0.52
2.32   1.536659618  
n
0.52
2.32   2.361322781
n
0.52
 0.0413227807
n
n  6.049931679
Least n  7.

7 Let the random variable X be the number of guests


(i) checking into the hotel in a given hour. 

P  X  4   0.7064384499  0.706 (3s.f)


 
ACJC P2 Solutions 32
Anglo-Chinese Junior College
H2 Mathematics 9740
2016 JC 2 Pelim Paper 2 Solutions
7 3!
Required probability   P  X  4     P  X  0   
2
(ii) 2!
 0.0015822508
 0.00158 (3s.f)

7 Let the random variable Y be the number of guests


(iii) checking into the hotel in a day.
Y ~ PO  86.4   
Since   10,  
Y ~ N  86.4,86.4  approx.  
c .c
P  85  X  90   P  85.5  X  89.5  0.169 (3s.f)
7 Let the random variable W be the number of one-hour
(iv) blocks in a day, which has not more than 4 guests
checking into the hotel.
W ~ B  24, 0.7064384499   
P W  n   0.124
 
P W  n  1  0.876
Using G.C,
P W  19   0.876  2.797 104
P W  20   0.876  0.7534
 n  1  20 sgexamguru.com
Least n  21.
 

7 This is because the mean number of guests checking


(v) into the hotel per hour is unlikely to be constant
throughout the year. The number of guests checking
into the hotel is likely to vary across different months
in a year due to seasonal fluctuations caused by
factors such as the holiday seasons, etc. Hence a
Poisson distribution may not be a good model.
8 (i) Let random variable X be a randomly chosen Hono’s
lunch waiting time at North Vista outlet.
 
To test H 0 :   60

Against H1 :   60 at 10% level of significance.

Under H 0 , T  X  0 ~ t ( n  1) ,
s/ n

X  60
i.e. T  ~ t (9)
s / 10


 
ACJC P2 Solutions 33
Anglo-Chinese Junior College
H2 Mathematics 9740
2016 JC 2 Pelim Paper 2 Solutions
x  0
Value of test statistic: t 
s/ n

52.1  60
= = 1.6729597
14.9328 / 10

p-value =  0.064332  0.1

 Reject H 0 and conclude that there is sufficient


evidence at 10% level of significance that the average
waiting time during lunch periods at North Vista is less
than one hour.

8 Unbiased estimate of population variance is  


(ii)
 n   56 
s2     sample variance  =   (69.8) = 71.06909
 n 1   55 

To test H 0 :   60

Against H1 :   60 at 5% level of significance.

T  60
Under H 0 , Z  ~ N  0,1 approx. ,
sgexamguru.com
s / 56

t  60 t  60
Value of test statistic: z  
71.06909 / 56 1.12654
Since the null hypothesis is rejected,

t  60 t  60
 1.959964 or  1.959964
1.12654 1.12654

t  57.792 or t  62.208

9  
(a)
(i) p (0.72) + (1 – p) (0.28) = 0.44p + 0.28  

(ii)    
(0.44 p  0.28)(0.6)  1   0.44 p  0.28     
= 0.888 – 0.176p

Since 0  p  1 , 0  0.176 p  0.176   


                         0.888  0.888  0.176 p  0.888  0.176
                         0.888  0.888  0.176 p  0.712
 0.712  0.888  0.176 p  0.888


 
ACJC P2 Solutions 34
Anglo-Chinese Junior College
H2 Mathematics 9740
2016 JC 2 Pelim Paper 2 Solutions
(b) Since n is large, by Central Limit Theorem,  

X1  X 2    X 50  1.44 
X ~ N  4.8,  approx.
50  50 

 
Required probability = P X  5 = 0.8807035 = 0.881 (3s.f)
 

(ci) If Aaron is the goalkeeper: C4 C4 C2  750  


6 5 5

If Aaron is the forward:


6
C4 5C4 5C1  375

Number of ways required = 1125

(cii) Number of ways required = 5!6!8!  = 3483648000   


   

(ciii) Required probability


P  goalkeepers are opposite & defenders with 2 particular midfielders are together 

P  defenders with 2 particular midfielders are together 
8!2!8! 2! 1
  

10
sgexamguru.com
111!8! 10  9 45

(i) Model A
s
10
9.5
9
8.5 h
0 20 40 60 80

Model B
s
10
9.5
9
8.5 h
0 20 40 60 80

10 
 
ACJC P2 Solutions 35
Anglo-Chinese Junior College
H2 Mathematics 9740
2016 JC 2 Pelim Paper 2 Solutions
10
(ii) s
70000 (130,[Y 
VALUE])
60000

50000

40000 (50,[Y 
30000 VALUE])

20000

10000

0 h
0 10 20 30 40 50 60 70 80 90 100 110 120 130 140

10 A Linear model will not be approriate.


(iii) This is because the scatter diagram indicates that as h
increases, s is increasing at a decreasing rate which is
not a linear realtionship.
Furthermore, a linear model will mean that the
product’s monthly sales in Singapore will increase
indefinitely with the increase of the number of
promoters. This is not realistic in the context of the
question as the product’s monthly sales will likely
slow down and perhaps decrease due to market
stauration.
10
(iv) sgexamguru.com
Correlation coefficent of s on ln(h) = 0.981
Correlation coefficent of s on 1/h = -0.998

Since correlation coefficent of s on 1/h is stronger,


hence use least square regression line of s on 1/h.

Least square regression line of s on 1/h:


1896285.284
s  78531.62777 
h
 
1896285.284
s  78531.628   3 d.p 
h
when s  75000,
1896285.284
75000  78531.62777   
h
h  536.9437001  537 (nearest integer).
 
10 The estimate is not reliable as s=75000 does not lie
(v) within 40000  s  63000 . Hence, we are
extrapolating.

11 
 
ACJC P2 Solutions 36
Anglo-Chinese Junior College
H2 Mathematics 9740
2016 JC 2 Pelim Paper 2 Solutions
10 1896285.284
(vi) s  78531.62777 
h
s 78531.62777 1896285.284
 
1.34 1.34 1.34h
s 1415138.2716417
 58605.692365671 
1.34 h
1415138.272
 u  58605.692  (3 dp)
h
s
where u = monthly sales in US dollars (i.e. u  )
1.34
11 The probability that each salesman is successful in
(i) closing a deal is assumed to be constant.
11 Assumption: Deals closed are independent of one
(ii) another

Deals closed may not be independent of one another as


customers may collaborate to buy cars as a group for
better bargaining power.

11 X ~ B  60, 0.2   
(iii) Since np  12  5 and nq  48  5,  
X ~ N 12, 9.6  approx.

sgexamguru.com
Y ~ B  50, 0.3   
Since np  15  5 and nq  35  5,  
Y ~ N 15,10.5  approx.  
X  Y ~ N  27, 20.1 approx.  
c .c
P  X  Y  20   P  X  Y  20.5   0.926  3s.f   
11 P(C  30)  0.03014
(iv)
C30  p 30  1  p   0.03014
60 30

p 2  p  0.2399991029  0
 p  0.6 or 0.3999955
Since p  0.5,
p  0.4 (1 d.p)

11 C  B(60, 0.05)  
(v)
Since n  60  50 such that np  3  5,  

C ~ PO  3  approx.     P(C  4)  0.185 (3s.f)  

12 
 
ACJC P2 Solutions 37
ANDERSON JUNIOR COLLEGE
2016 Preliminary Examination
H2 Mathematics Paper 1 (9740/01)
Duration: 3 hours

1 Poh, Kee and Mong work for GO! taxi firm. The firm pays them different rates for
different periods of time in a day. During off-peak periods, the firm pays $a for each
km driven. For peak and super-peak periods, the firm pays $k and $m more per km
respectively. The table below shows the distance (in km) covered by Poh, Kee and
Mong during the three different periods, together with the amount they were paid, on a
particular day.

Off-peak Peak Super-peak Total


Poh 63 26 10 $165.10
Kee 59 34 11 $176.15
Mong 31 52 28 $205.70

Write down and solve equations to find the values of a, k and m. [4]

2
sgexamguru.com
Without using the calculator, solve the inequality
2

x2
.
x 1 x [4]

n
3 1
3 (i) Prove by induction that  r (3 )  4  4  2n  1 3
r 1
r n 1
. [4]

n
(ii) Hence, find  (2r  1)3
r 2
r
. Give your answer in simplified form. [4]

1
4 Given that f(r )  , where r is a positive integer, express f (r )  f (r  1) as a
r
single fraction.
1 1 1 1 1 1
Hence, find the sum of the series     .....  .
2 6 12 20 30 2n  2n  1
1 1 1 1 1
Deduce that the sum to 2n terms of the series     ... is less than 1. [7]
22 32 42 52 62

Page 1 of 5
AJC 2016 Preliminary Examination 9740/01

Microsoft Word - AJC 2016 H2 Math prelim P1 38


5 y
(0,b)
y = f((x)

x
(˗a,0) 0 a,0)
(a

The diagram shows


s the curve
c C1 w n y  f  x  , where C1 is symmettrical
with equation
0 and  0,bb  .
aboout the y-axxis and passees through tthe points (a, 0), (a, 0)
1
The curve C2 has n given by y 
h equation .
f  x
(i) t curve C2 , showing clearly all the
Sketch the t features of the curvve. [2]]
(ii)) nge of value s of b such that C1 and
Write doown the ran d C2 do not intersect. [1]]

It iis given thatt C1 is part of


o an ellipsee with centrre (0, 0).

sgexamguru.com
clearly all
a the featurres of the cuurve.
quation y  f   x  , sh
(iiii) Sketch, on a diffeerent diagraam, the currve with eq howing
[2]]
(ivv) State a sequence ofo transform mations thaat will transsform C1 iinto a curve with
 x  b  y 2  b2 , y  0 .
2
equationn [3]]

6 Doo not use a calculator


c in
i answerin
ng this queestion.

The complex number


n w iss such that w3  8  0 .

(i) Find exactly


e the possible vaalues of w, giving you m rei
ur answers in the form
where r  0 and      . [3]]
1 8 3
(ii) Hence, solve the equation
e    24 , wheree z  0 .
z 4 z z3 [4]]

Page 2 of 5
AJC 2016 P
Preliminary Examina
ation 9740/01

AJC P1 Questions 39
4 x
7 The function f is defined by f :xe , x  a.

(i) State the greatest value of a for which the function f 1 exists. [1]
Use this value of a for the rest of this question.
(ii) Define the inverse function f 1 in a similar form. [3]

(iii) Solve the inequality f 1 f  x  1. [3]

The function g is defined by g : x  b  3  x, x  3 .

(iv) Find the smallest value of b for the composite function f 1 g to exist. [2]

(v) If b  2 , find the range of f 1 g in exact form. [2]

dy
8 (a) Given that ln(2  y 3 )  x, show that 3 y 2  2  y3.
dx
By further differentiation of this result, find the series expansion of y in
ascending powers of x up to and including the term in x 2 . [4]


In a parallelogram ABCD, AB  2, BC  1 and DAB    . Show that the
sgexamguru.com
(b)
4
length of the diagonal AC is 3  2 cos   2 sin  . Given that θ is small, show
that AC  5 1  a  b 2  , where constants a and b are to be determined. [6]

9 (a) The curve C1 has parametric equations x  t  tan 1 t , y  t 2 , where t  0 .


 4 
Given that C1 meets the line y  2    x at the point where t  1 , find the
 4 
 4 
exact area of the finite region bounded by C1 , the line y  2    x and the
 4 
x-axis. [5]

(b) The curve C2 has parametric equations x  2 t 2 , y t 3 .

(i) Find the equation of the tangent to C2 at point P  2 p 2 , p 3  . [2]

(ii) This tangent meets the y-axis at the point A. The point Q on the line segment
AP is such that AQ : QP  1: 3 . Find the cartesian equation of the locus of Q
as p varies. [4]

Page 3 of 5
AJC 2016 Preliminary Examination 9740/01

AJC P1 Questions 40
10 (i) Using the substitution u  sin x, show that  sin 6 x cos3 x dx   (u 6  u 8 ) du . [2]

O x

The diagram shows part of the curve with equation y  sin 3 x cos3 x . The region
bounded by the curve and the part of the x-axis between 0 and 12  is denoted by R.

(ii) Find the exact volume of revolution when R is rotated completely about the x-axis. [3]

(iii) By differentiating y 2  sin 6 x cos3 x with respect to x, show that the x-coordinate
of the maximum point of the above curve is tan 1 2 , and find the exact
y-coordinate of the maximum point, expressing your answer in the form 2a  3b ,
where a and b are rational numbers to be determined. (You do not need to show
that it is a maximum point.) [5]

sgexamguru.com
11 With reference to the origin O, the point R has position vector 2i  j  6k . The
plane p1 has vector equation r  (2  2  5 ) i  (1  2   ) j  (6    5 ) k ,
where  ,   .
The point Q, which is on OR produced, is such that RQ = 2OR and p2 is the plane
through Q which is parallel to p1 .
(i) Show that R lies in p1 . [1]
(ii) Find a cartesian equation of p2 . [3]
(iii) Find the position vector of the point L in p2 such that OL is perpendicular to p2. [3]
(iv) The point M is on the line segment RQ. Given that h is the perpendicular
distance from M to the line passing through R and L, show that
 
RM  RL
h  . [2]
RL

Page 4 of 5
AJC 2016 Preliminary Examination 9740/01

AJC P1 Questions 41
12 [It is given that the volume of a circular cone with base radius r and height h is
1 2
 r h .]
3

Figure 1 shows a frustum (a truncated cone) with radii 2 cm and 10 cm and height
20 cm. A conical flask (Figure 2) is made up of the frustum and a cylinder. The
cylinder has radius 2 cm and height 6 cm. Water is poured into the empty flask at a
constant rate of  cm3s 1 . At time t seconds, the depth of the water is h cm and the
radius of the water surface is r cm.
2
 

6
2

26
r
20

h
10
10

sgexamguru.com
Figure 1
Figure 2

(i) For 0  h  20 ,
(a) by using similar triangles, find r in terms of h; [3]
2500 4
(b) show that the volume of water in the flask is V    25  h  .
3

3 75
dh [4]
Hence, find in terms of h.
dt

dh [2]
(ii) Find for 20  h  26 .
dt
dh
(iii) Sketch a graph to illustrate how varies with h for 0  h  26 . [2]
dt

End of paper

Page 5 of 5
AJC 2016 Preliminary Examination 9740/01

AJC P1 Questions 42
2016 Prelim H2 Mathematics Paper 1 Solution

1 63a  26(a  k )  10(a  m)  165.10


59a  34(a  k )  11(a  m)  176.15
31a  52(a  k )  28(a  m)  205.70
Simplifying,
99a  26k  10m  165.10
104a  34k  11m  176.15
111a  52k  28m  205.70
Solving,
a  $1.50, k  $0.35, m  $0.75

2 2 x2
  0 x  0, 1
x 1 x
2 x  ( x  2)( x  1)
0  
x  x  1  +
 
+
 x 2  3x  2
0 1 
x( x  1) 3  17   0  3  17  
3 17 2 2
( x  ) 2 
2 4 0
x( x  1)
3  17 3  17
1  x  or 0  x 
2
sgexamguru.com 2

3 n
3 1
Let Pn be the statement  r (3)
r 1
r
   2n  1 3n 1 , n ≥ 1.
4 4
1
When n  1 , LHS =  r (3) r  1(3)  3
r 1

3 1
RHS =  (1)32  3
4 4

Since LHS  RHS, P1 is true

k
3 1
Assume Pk is true for some k  
, i.e  r (3)
r 1
r
   2k  1 3k 1
4 4

To prove that Pk 1 is true,


k 1
3 1 3 1
i.e. to prove :  r (3)
r 1
r
   2(k  1)  1 3( k 1) 1   (2k  1)3k  2
4 4 4 4

k 1 k
LHS   r (3) r   r (3) r  (k  1)3k 1
r 1 r 1

3 1 3 1
  (2k  1)3k 1  (k  1)3k 1   (3) k 1 (2k  1  4k  4)
4 4 4 4
3 1
  (3)( k 1) 1 (2k  1)  RHS . Pk is true  Pk 1 is true
4 4
Since P1 is true, and Pk is true implies Pk 1Page
is1true,
of 10 by Mathematical Induction, Pn is true for
Microsoft Word - AJC 2016 H2 Math prelim P1 solution 43
all n   , n  1.
n n n
(ii)  (2r  1)3r   2r (3)r   3r
r 2 r 2 r 2

n
3 1  9 1
 2r (3)
r 2
   (2n  1)3n 1   6    (2n  1)3n 1
r

2 2  2 2
9(3n 1  1) 9 n 1
  3  1
n


r 2
3r 
3 1 2

n
9 1 9
 (2r  1)3
r 2
r
   (2n  1)3n 1  (3n 1  1)
2 2 2
1 9
 (2n  1)3n 1  (3n 1 )
2 2

1 1
OR  3n 1  (2n  1)    3n 1  n  1
2 2
9 9
OR  3n 1  (2n  1)    3n 1  9n  9 
2 2

4 1 1 1
f (r )  f (r  1) =  = .
r r  1 r (r  1)

2n 2n
1 1 1 1 1 1 1
    .....  =  =  [f (r )  f (r  1)]
2 6 12 20 30

= [ f(1)  f(2)
sgexamguru.com
2n  2n  1 r 1 r (r  1) r 1

+ f(2) – f(3)
+...
+ f(2n-1) – f(2n)
+ f(2n) – f(2n+1) ]

 f (1)  f (2n  1)
1
 1
2n  1
2n
1 1 1 1 1 1
 2  2  2  2 ... (2n) term   th

r 1 ( r  1)
2 2
2 3 4 5 6
1 1 1
Consider  
(r  1) 2
(r  1)(r  1) r (r  1)
2n 2n
1 1
  
r 1 ( r  1) r 1 r ( r  1)
2

2n
1 1
  1
r 1 ( r  1) 2n  1
2

2n
1 1
Since  0,   1 (shown)
2n  1 r 1 ( r  1)
2

Page 2 of 10

AJC P1 Solutions 44
5 (i)
y

C2

(0, 1/b)

x
x=-a 0 x=a

(ii) 0  b 1

(iii)
y  f ' x
y

x
0
sgexamguru.com
x=-a x=a

(iv) The sequence is:


b
(1) Scale parallel to the x-axis by a factor of .
a
(2) Translate by b units in the negative x-direction
(3) Reflect about the x-axis

Or
(1) Reflect about the x-axis
b
(2) Scale parallel to the x-axis by a factor of .
a
(3) Translate by b units in the negative x-direction

Or
(1) Translate by a units in the negative x-direction
b
(2) Scale parallel to the x-axis by a factor of
a
(3) Reflect about the x-axis.
6 (i)
w3  8
 8ei .ei2 k  8ei (1 2 k )

i (1 2 k )
w  2e 3 , k  0,1, 1
 
i i
w  2e , 2e 3 3
, 2ei
Page 3 of 10

AJC P1 Solutions 45
(ii)
1 8 3
   24
z 4 z z3
1 1  1 
 3  8   3 3  8
zz  z 
1  1 
 3  8   3  0
z  z 
1  1 
 3  8  0 or   3  0
z  z 
1
Replace w  from part (i)
z
 
1 i i 1
 2e 3 , 2ei , 2e 3 or 3
z z
1 i  1 1 i 1
z  e 3 , e  i , e 3 or z
2 2 2 3
1 i 3 1 1 i 3 1
z e ,  , e or z
2 2 2 3

7
(i) For the function f 1 to exist, f must be one-one,
y
Greatest value of a = 4.

(ii) Let y  e sgexamguru.com


4 x

Since x  4, y  e4 x
 

ln y  4  x 1
x  4  ln y x
4
f 1 : x  4  ln x , x  1
y
(iii) f 1 f  x   x , x  4
f 1 f | x | | x | , | x |  4 4
For 1
f f  x  1, y =1
4  x  1 or 1  x  4
x

(iv) For the composite function f 1 g to exist,


y
R g  Df 1 , that is [b , )  [1, ) .

Least value of b = 1. (3,b)


x
(v) Given R g  [2, )
f 1 2  4  ln 2
R f 1g  (, 4  ln 2]

Page 4 of 10

AJC P1 Solutions 46
8 (a) ln(2  y 3 )  x --- (1)
Differentiate w.r.t. x,
3 y 2 dy
 1
2  y 3 dx
dy
 3y2  2  y 3 (shown) --- (2)
dx
Differentiate again w.r.t. x,
2
d2 y  dy  dy
3 y2 2
 6 y    3 y2 --- (3)
dx  dx  dx
When x = 0, ln(2  y 3 )  0  y  1, [from (1)]
dy dy 1
3(1)  2  1   , [from (2)]
dx dx 3
2
d2 y 1
2
1 d y 5
3(1) 2  6(1)    3(1)    2  . [from (3)]
dx  3 3 dx 9
1 5
 y  1  x  x 2  
3 18
  3
(b) For a parallelogram, DAB  ABC   . So ABC         .
4  4
D C

sgexamguru.com
A B

 3 
Using cosine rule, AC 2  2  1  2 2(1) cos   
 4 
 3 3 
 3  2 2  cos cos   sin sin  
 4 4 
 1 1 
 3  2 2  cos   sin  
 2 2 
 3  2 cos   2sin 
 AC  3  2 cos   2sin  (shown)
1
Given that  is small, cos   1   2 and sin    .
2
1 1
AC   3  2   2  2  2   5  (2   2 )  2
1
  2     2 2
 5 1   
  5 
 1  2   2  1 ( 1 )  2   2  2 
 5 1    2 2
  
 2  5  
2  5 
 
 
 1 1 1 2
2

 5 1     2     2   
 5 10 85 
 
 1 3  1 3
 5 1     2  [a  , b   ]
 5 25  5 25
Page 5 of 10

AJC P1 Solutions 47
9 
(a) When t  1, x   1 and y  1
4

Line cuts x – axis at x  2
2
Area under graph

  

Area of region


1

= Area of triangle + 4
y dx
0

1 
1   1  0 t 2 1  2  dt
1 1
=
2 4    1 t 
 1 1 1 
8 2 0 
=    t2 1  dt
1 t2 
1

1 t3 


1  4 11 
sgexamguru.com
=     t  tan 1 t 
8 2 3 0

=    = 
8 2 4 3 6 8

dx dy
(b) (i)  4t and  3t 2
dt dt
dy dy dt 3
   t
dx dt dx 4
Equation of Tangent:
3p
y  p3 
4
 x  2 p2 
3p 3
y x  p3  p3
4 2
3p 1
y x  p3
4 2

1 3
(ii) On y – axis, x  0 , y   p
2
 1 
A  0 ,  p 3  and P  2 p 2 , p 3  and Q divide AP in a ratio of 1:3
 2 

Page 6 of 10

AJC P1 Solutions 48
 0 
2p 
2

3 3  3   1 2 
 3OA  OP   2 p   p   2 p 
 
OQ    
4 4   1 p3 
 
 8 

Alternative Method
x – coordinate of Q

1 1
=
4
 2 p2  = p2
2
3
y – coordinate of Q
Q
p3 1  3 p3  p3
1 =    p  = 
2 4 2  8
 

1 2 1
The parametric equation is x  p , y   p3
2 8
1 1
y  x3 2 or y 2  x 3
2 2 8

10
(i) u  sin x 
du
dx
sgexamguru.com
 cos x

 sin x cos x dx   sin x(1  sin x) cos x dx


6 3 6 2

  u 6 (1  u 2 ) du
   u 6  u 8  du (shown)

(ii) When x  0, u  0. When x  12  , u  1.


1

Volume of revolution =   sin 6 x cos3 x dx


2

0
1
   (u 6  u 8 ) du
0
1
 u7 u9 
   
 7 9 0
2
  units3
63

(iii) y 2  sin 6 x cos3 x


dy
 2y  6sin 5 x cos x  cos3 x  sin 6 x  3cos 2 x    sin x 
dx
 6sin 5 x cos 4 x  3sin 7 x cos 2 x

dy
When  0, 3sin 5 x cos 2 x(2 cos 2 x  sin 2 x)  0
dx
Page 7 of 10

AJC P1 Solutions 49
 sin x  0 or cos x  0 or sin 2 x  2 cos 2 x
x  0 or x  12  or tan 2 x  2
From the sketch of the curve, maximum point does not occur at x  0 or x  12  .
 tan 2 x  2  tan x  2 (since 0  x  12  )
 x  tan 1 2 (shown)

y -coordinate  sin 3 x cos3 x


3 x
 2  1 
3

   
 3  3
3
2 2
1
 3
 3
3 2
34
3 9
 3 9
 22  3 4
(a  , b   )
2 4

i) r  (2  2  5 ) i  (1  2   ) j  (6    5 ) k
11
 2  2 5
     
r   1     2     1  , ,  
6 1  5 
     

 2

sgexamguru.com

When   0 and   0, r   1   OR , hence R lies in p1 .
6
 

ii) Cartesian equation of p2 :

6  2   5   9   3
           
OQ  3OR   3  ;  2    1    15  , n   5 
 18   1   5   12   4
         

 3 6  3
     
r  5    3   5   75 , hence p2 : 3x  5 y  4 z  75
 4   18   4
     

 3

iii) Method 1 Let OL    5  ;
 4
 

3
since point L lies in p2 , 3(3 )  5(5 )  4(4 )  75   
2

Page 8 of 10

AJC P1 Solutions 50
 9 
 2 
 3  
 3    15 
 OL   5  = 
2   2
4
   
 6 
 

 3  3
 75 1   3 
Method 2 OL  OL n  5  5
50 50   2  
 4  4

iv)
Q  
h  RM sin  sine ratio
M
R   
 RM  RL
Ɵ  h   RM   Def. of cross product
RM . RL
 
RM  RL
L   
RL

OR From definition:
  1  
RM  RL  RM RL sin  MRL  nˆ

sgexamguru.com
2 
1 
 
 h
 RM RL  .1
2 RM
 
RM  RL
h 
RL

12 Solutions
(i) a)
x x  20
 x
2 10 2 25 - h
10 x  2 x  40
x5 20
r

h
 Height of cone is 25. 10
10

25 25  h

10 r
2
r  10  h
5

b) For 0  h  20 ,
 
Volume of water in flask, V  102  25   r 2  25  h 
3 3
Page 9 of 10

AJC P1 Solutions 51
 
2
 2 
=  10 2
  
25  10  h   25  h  
3  5  
2500 4
 25  h  (Shown)
3
= 
3 75
dV  4 2 dh 
Diff V w.r.t. t,  0  3   25  h     
dt  75   dt 
dh 1 25
   
dt 4 25  h 2 4  25  h 
2
 
25
(ii) For 20  h  26 , V  V frustum  Vcylinder


dV dV f dVc
dt

dt

dt
 0
d
dt

  2  (h  20)  4
2 dh
dt

dh  1
  
dt 4 4

dh 1
(iii) When h  0 , 
dt 100

(cm s-1)

sgexamguru.com
h (cm)
O 20 26

Page 10 of 10

AJC P1 Solutions 52
ANDERSON JUNIOR COLLEGE
2016 Preliminary Examination
H2 Mathematics Paper 2 (9740/02)
Duration: 3 hours

Section A: Pure Mathematics [40 marks]

1 With reference to the origin O, the points A, B, C have position vectors a , b, c


respectively. The mid-points of OA and BC are S and T respectively.
 
(i) Find OS and OT in terms of a , b and c . [2]

OABC is a tetrahedron where angle OBA and angle OCA are right angles.
(ii) Show that a b  b b and a c  c c . [2]
 
(iii) By considering BC BC , show that BC 2  b  c  2 b c .
2 2

1
Hence show that ST  OA2  BC 2 . [4]
2

sgexamguru.com
2 Anson is closing down his chicken farm and he decides to sell exactly k chickens in the
market at the end of every week, where k is a divisor of 1000. Anson has 1000 chickens
and the cost of rearing each chicken per week is $0.50. Show that, when he has sold all
250
his chickens, the total cost of rearing the chickens is $ 1000  k  . [3]
k

At the end of the first week, Anson sells each chicken at a price of $12. At the end of
every subsequent week, he sells the chickens at a price of 5% less than that of the
previous week.
Find the smallest value of k in order for Anson to make a profit when he has sold all his
chickens. [6]

Page 1 of 6
AJC 2016 Preliminary Examination 9740/02

Microsoft Word - AJC 2016 H2 Math prelim P2 53


3 The population of fish in a pond is n thousand at time t years. It is known that n and
t are related by the differential equation

  k  n  n2  ,
dn 1
dt 5
3
where 0  n  and k is a constant. When t  0, the population size is 1250.
2
dn
It is also known that  0.15 when the population size is 1000.
dt
3
(i) Show that k   . [1]
4
(ii) Find the population size at which the population is shrinking at the highest rate.
[2]
(iii) Find an expression for n in terms of t.
[5]
(iv) Find the exact value of t when the fish population size reaches zero.
[1]
(v) Sketch the part of the solution curve which is relevant in this context.
[2]

4
sgexamguru.com
The complex number z satisfies the equation z  z  6  2 3 i .

(i) On an Argand diagram sketch the locus of z. Find the exact values of the
axial intercepts of the locus. [5]

The complex number z also satisfies the equation z  1 2  a , where a  0 .

(ii) Find the range of values of a such that there are two possible values of z . [3]

Another complex number w satisfies the relations

w  w62 3 i , w  1  49 and Im( w)  0 .


2

(iii) Illustrate the region in which the point representing w can lie. [2]
(iv) Given that arg( w  4i)   where      . State, in exact form, the
range of values of  . [2]

Page 2 of 6
AJC 2016 Preliminary Examination 9740/02

AJC P2 Questions 54
Section B: Statistics [60 marks]

5 The management of ABC Country Club would like to conduct a survey to determine if
members are satisfied with the recreational facilities in the club. The club has 10 000
members, where more than 60% are below 30 years old and less than 10% are above
60 years old.
(i) Describe how a sample of 100 members can be chosen using systematic
[2]
sampling.
(ii) State a possible disadvantage of using this sampling method in this context. [1]

6 An experiment is carried out with three coins. Two of the coins are fair. The third
1
coin is biased such that the probability of obtaining a head is . The three coins are
5
tossed. A is the event: All the three coins show the same result.
B is the event: The biased coin shows a tail.
Find (i)

(ii)
sgexamguru.com
P  A  B ,

P  B ' | A ' .
[3]

[3]

7 A group of ten friends, comprising six women and four men, wishes to go out to a
restaurant for lunch. Two of the men own a car each. They decide to go to the
restaurant in two cars driven by their respective owners, with the same number of
passengers in each car.
(i) Find the number of ways in which the remaining eight people may be
allocated to the two cars if two of the women, Helen and Karen, must take the
same car. [The arrangement of people within a particular car is not relevant.] [2]

At the restaurant, the group sits at a round table. Find the number of ways of
arranging the ten people if
[2]
(ii) no two men are seated together,
(iii) neither Helen nor Karen is seated beside a driver. [4]

Page 3 of 6
AJC 2016 Preliminary Examination 9740/02

AJC P2 Questions 55
8 A large number of participants take part in a shooting contest. Each participant is
given 10 attempts at hitting a target. The probability that a participant hits the target
in an attempt is 0.32. It is assumed that the outcome of an attempt is independent of
the outcome of any other attempts.
(i) Find the probability that a participant hits the target more than half of the
time. [2]
(ii) A random sample of 50 participants is taken. Using a suitable approximation,
find the probability that not more than 44 participants hit the target at most 5
[4]
times each.

9 A secretary types letters onto sheets of paper 30 cm long and folds the letters as
shown.
Envelope
Unfolded sheet Folding of sheet

X
sgexamguru.com
Y Y
X
Y

30 cm 11 cm Y

The first fold is X cm from one edge. The second fold, Y cm from the other edge, is
1
exactly in the middle of the remaining part of the paper, so that Y   30  X  .
2
The length X cm is normally distributed with mean 10.2 cm and standard deviation
1.2 cm. The letters have to fit into envelopes 11 cm wide.
(i) Find P 11  Y  15 . [2]

(ii) Find the probability that a randomly chosen letter will fit into the envelope. [3]
(iii) By expressing X  Y in terms of X, verify that
Var  X  Y   Var  X   Var Y  .

Explain why the rule Var  aX  bY   a 2 Var  X   b 2 Var Y  does not apply
[4]
in this case.

Page 4 of 6
AJC 2016 Preliminary Examination 9740/02

AJC P2 Questions 56
10 Travellers arrive at a railway station, to catch a train, either individually or in groups.
On a Saturday morning, the number of travellers who arrive individually during a
one-minute interval may be modelled by a Poisson distribution with mean 7.5. The
number of groups who arrive during a one-minute interval may be modelled by an
independent Poisson distribution with mean 2.0.
(i) The probability that no more than one traveller arrives individually during a
period of t seconds is less than 0.1. Write down an inequality in terms of t to
represent this information, and hence find the range of values of t, giving your
answer correct to one decimal place. [3]
(ii) Find the probability that at most 4 groups arrive during the time interval from
0945 to 0950 with at least 3 of the groups arriving within the first 3 minutes . [3]
(iii) Using a suitable approximation, find the probability that, from 1045 to 1055,
the number of travellers arriving individually exceeds three times the number of
[4]
groups arriving by more than 8.

11 A study of the relationship between the amount of advertising time (in minutes) on

sgexamguru.com
radio for a product and the quantity of the product sold (in thousands) was carried out.
The results of the study are given below:

Advertising time 2 4 6 8 10 12 14 16
(minutes), t
Quantity of the
product sold 1.11 1.62 k 1.91 2.11 2.15 2.20 2.23
(thousands), S

(i) Given that the regression line of S on t is S  0.069940 t  1.2718 , find k,


correct to 2 decimal places. Hence find the product moment correlation
coefficient between S and t. [3]

(ii) It is thought that the relationship between S and t can also be modelled by
S  a  b ln t where a and b are constants. Find the least squares estimates of a
and b and the product moment correlation coefficient r for this new model. [2]
(iii) Use the regression line for the better model to estimate the quantity of the
product sold when the advertising time is 9 minutes, giving your answer to the
nearest integer.
[3]
Give two reasons why this estimate is reliable.

Page 5 of 6
AJC 2016 Preliminary Examination 9740/02

AJC P2 Questions 57
12 Bank A claims that the average monthly salary of a fresh graduate working in the
banking industry is $3000. However, a human resource manager from another bank
suspects that this is not the correct average monthly salary. A random sample of 50
fresh graduates is surveyed. The monthly salaries, $x, of these 50 fresh graduates are
summarised by

  x  3000   505   x  3000   100580


2

(i) Calculate the unbiased estimates of the mean and variance of the monthly
salary of a fresh graduate, giving your answers to 2 decimal places. [2]
(ii) Test at 10% significance level whether the manager’s suspicion is supported. [4]

The manager believes that the average monthly salary of a fresh Mathematics
graduate in the banking industry is more than $3000. He decides to conduct
investigations into this by surveying a random sample of 15 fresh Mathematics
graduates and the mean of this sample is found to be $3050 and the sample variance
is k 2. Find the range of values for k 2 (to the nearest integer) if it is found that, at 5%
level, there is sufficient evidence to justify his belief. State an assumption necessary

sgexamguru.com
to carry out the test. [4]

End of Paper

Page 6 of 6
AJC 2016 Preliminary Examination 9740/02

AJC P2 Questions 58
2016 Preliminary Examination H2 Mathematics Paper 2 (9740/02) Solutions

1  1  1
i) OS  OA  a
2 2
 1
By Ratio Theorem, OT  b  c
2
 
ii) OB  AB  b  (b  a)  0  a  b  b  b
 
OC  AC  c  (c  a)  0  a  c  c  c
 
iii) BC 2  BC  BC   c  b    c  b 
2 2
 b  c  2b c

 1   1


ST   b  c  a  ; ST  ST   b  c  a    b  c  a 
2 4

 1
b  b  b  c  b  a  c  b  c  c  c  a  a  b  a  c  a  a
2
 ST 
4
1
  2
 2b  c  a  b  c
2 2

 sgexamguru.com
4
1 2
4
 2 2 1
 a  2b  c  b  c   OA2  BC 2 
4
1
 ST  OA2  BC 2
2

1000
2 (a) Number of weeks needed to sell off all the chickens n 
k

Total cost =  0.5  1000   1000  k   1000  2k      k  


 1  1000  
=  0.5     1000  k 
2  k  
250
=$ 1000  k  (shown)
k

 1000
1 
Total earnings = 12k 1   0.95    0.95      0.95  k 
2
(b)
 
 1000

 1   0.95  k 
= 12k
 1  0.95 
 

Page 1 of 9
AJC 2016 Preliminary Examination 9740/01

Microsoft Word - AJC 2016 H2 Math prelim P2 Solutions 59


 1000

= 240k  1   0.95  k 
 

For A
Anson to maake a profit,,

Totall earnings > Total costt


 1000
 250
240kk 1   0.95  k   1000  k 
  k
24k 2 
 k   10000  k  0
1000

 
1  0.95
25  
24 x 2  10000

Plot tthe graph off y   1   0.95  x
  1000  x .
25  
Fromm GC, k  38.2
If k iss a positive integer and d 1000 is divvisible by k,
Smalllest k = 40.

3 dn
(i) Siince  00.15 when n  1,
dt
1 3
0.15   k  1  12   k   (shown)
5 4

(ii)
dn 1  2
sgexamguru.com
3  1  1
  n  n     n    1
dt 5  4  5  2
2


dn 1
is the moost negativee when n  .
dt 2
 Populatio on is shrinkiing at the hiighest rate when
w
n
populatioon size is 50
00.

dn 1  
2
1
(iii)   n    1
dt 5  2 

 
1 1
dn  dt
n  2  1
1 2 5

1  n  12   1 1
ln  t c
2  n  12   1 5
1  32  n  1
ln  tc [Givven: 0  n  32  n  32  0 . So n  32  (n  32 ). ]
2  n  12  5

3
n 2
t  2c
2
t
2
 e 5
 Ae 5
, where A  e2c
2

n 2
1

3  1  2t
 n   n   Ae 5
2  2

Page 2 of 9
AJC 2016 P
Preliminary Examina
ation 9740/01

AJC P2 Solutions 60
3 1 52 t 2
 Ae  n 1  Ae 5
t
 
2 2
2
t
3  Ae 5
n

2 1  Ae 5
2
t

1250 5 5 3 A 1
Whhen t = 0, n =  . So   A .
1000 4 
4 2 1 A  7
2
1 t 2
3  e5 t
7 21  e 5
 n 
 1 52 t  2
t
2 1  e  114  2e 5
 7 

2
t
(iv) W
When n  00, 21  e 5  0
2 5
 t  ln 21 t  ln 21
5 2

(v) Skketch of n (in ds) against t :


( thousand

sgexamguru.com

t
O

4 (i) Im
z  z  (6  2 3i))
Locus 
of w
[B1] – Ideentifying the 2

points (0, 0) and 6, 2 3    θ
with the correct
c line. (-4, 0) (-1, 0) O Re

To finnd the interrsection with


h the axes:

M1: F
Finding Carrtesian equaation

Page 3 of 9
AJC 2016 P
Preliminary Examina
ation 9740/01

AJC P2 Solutions 61
 
2
x 2  y 2  ( x  6) 2  y  2 3

y  3x  4 3
Sub x = 0 and y = 0 to find the intersection with axes.
Intersection with x-axis = ( 4 , 0)
Intersection with y-axis = (0, 4 3 )

M2: Using Midpoint


Midpoint = ( 3, 3)
2 3 3
Gradient between (0,0) and (-6, 2 3 ) = 
6 3
3
Gradient of locus =  3
3
Intersection with x-axis = ( 4 , 0)
y
 3  y4 3
4
Intersection with y-axis = (0, 4 3 )

2
(ii) z 1  a  z 1  a

For only one value of z, locus in (i) ≡ tangent of the circle.

To find radius
sgexamguru.com
a :
M1 using gradient of perpendicular bisector = 3,   tan 1 3 

3
 a
sin 
3 3
 3 3
a  3sin 
3 2

M2
3 2 3
 sin  
  
Or right angled triangle , tan  
1 2
1

a
Using another right angled triangle, sin  
3
a 3 3 3
sin     a
3 2 2 3 

27
a for one value of z ,
4
27
 for 2 values of z , a 
4

Page 4 of 9
AJC 2016 Preliminary Examination 9740/01

AJC P2 Solutions 62
(iii) Identifying w  1  7 as the area inside a circle with centre (-1, 0) and radius 7
Shade the correct region

4 3
(iv) angle from (0, 4) to (-4, 0) =   tan 1 =
4 4
 3
hence   or     
2 4

Section B: Statistics [60 marks]

5 (i) Number the members in alphabetical order from 1 to 10, 000.


10 000
Randomly select a member from the first interval of  100 ,
100
eg 5th member was randomly selected. Select every 100th member from every subsequent
interval of 100, ie .105th, 205th etc.. until a sample of 100 members have been selected.

(ii) A possible disadvantage is that members of the age group above 60 years (or below 30
years) may be under (or over) represented and thus the results of the survey may not
truly reflect the opinions of the members of the Club.

6 (a) P  A  B   P  A  P  B   P  A  B 

sgexamguru.com
2 2
1 1 1  4 1
P  A = P(all heads) + P(all tails) =          
 2 5  2 5 4
2
1 4 1
P  A  B  = P(all tails) =     
2 5 5
1 4 1 17
 P  A  B  =     0.85 (or )
4 5 5 20

P  A ' B '
(b) P  B ' | A ' 
P  A '
17
1 P A  B 1
  20  1
1  P  A 1
1 5
4

Alternative:
P  A ' B ' P THH   2  P TTH 

P  A ' P TTH   P THH   2  P THT   2  P  HHT 
2 2
1 1 1 1 3
2        
 2 5  2 5 1
  20 
2 2
1 1 1 1
2 2
1 4 1 4 15 5
2          2         20
 2 5  2 5 2 5 2 5

   

Page 5 of 9
AJC 2016 Preliminary Examination 9740/01

AJC P2 Solutions 63
7 (i) Total number of ways of allocating the remaining 8 people in the 2 cars driven by
its driver with Helen and Karen in the same car
6
= C 2 x 4C 4 x 2 = 30
Comments: H & K have 2 choices of which car to join and there are 6C2 ways of
selecting people to join them
(ii) No of ways of arranging the 10 people if the men must be separated
=  6  1 ! x 6 P4 = 43200

Comments: To separate the men, the easiest way is to arrange the women first then
slot the men into the gaps between the women.
Case 1 :

No of arrangements   6  1 ! 6 P4  43200

Case 2 :

No of arrangements   6  1 ! 6 P3  2  28800

Case 3 :
sgexamguru.com
No of arrangements   6  1 ! 6 P3  2  28800

Case 4 :

No of arrangements   6  1 ! 6 P2  2  2  14400

Total no of arrangements = 43200 + 28800 + 28800 + 14400


= 115200

8 (i) Let X = number of attempts that hit the target out of 10.
X ~ B 10,0.32
P  X  5  1  P  X  5
 1  0.93628  0.063715  0.0637 (to 3 sig. figs)

(ii) Let Y= number of participants who hit the target more than 5 times, out of 50
participants. Then Y ~ B 50, 0.063715
Since n = 50 and np  50  0.063715  3.18575  5
Therefore Y ~ Po(3.18575) approximately
P(not more than 44 participants fire at most 5 successful rounds)
= P Y  6  1  P Y  5  0.10379...  0.104

Page 6 of 9
AJC 2016 Preliminary Examination 9740/01

AJC P2 Solutions 64
9 (ii) X ~ N 10.2,1.22 
 1 
P 11  Y  15   P  11   30  X   15 
 2 
 P  0  X  8 = 0.03376
6 = 0.0334(tto 3 sf)

Alterrnative Meth hod:


1 1
2

Y ~ N   30  10.2  ,   11.22   Y ~ N  9.9, 0.3
36 
2 2 
 
P 11  Y  15  0.03376
0  0.00334 (to 3 sff)

(iii) P(lettter will fit in


nto enveloppe)
 P  X  11 and Y  11
 1 
 P  X  11 and  30  X   11
 2 
 P  8  X  11
= 0.714
1 3
(iiii) X Y  X   300  X   X  15
2 2
3 
Var  X  Y   Vaar  X  15 
2 
2
 3 9
=   Var  X   1.2   3.24
2
3

sgexamguru.com
 2 4
 1 
Var  X   Var Y   1.22  Vaar  15  X 
2 

2
1
 1.2    Var  X  = 1.8
2

2
Thereefore Var  X  Y   Varr  X   Var Y 

Thee rule does not nce X and Y are not ind


n hold sin dependent.

10 (i) Let X = number


n vidually in t seconds.
of trravellers arrriving indiv

 7.5 
X ~ Po  t   X ~ Po  0.125t 
 60 
P  X  1  0.11  e0.125t 1  0.125t   0.1

Frrom GC, t > 31.118  t ≥ 31.2


(ii) Let S = number
n of groups
g arriviing in the fiirst 3minutees
Let T = number
n of groups
g arrivving in the laast 2 minutees
S ~ Poo  6  and T ~ Po  4 
Reqquired probaability = P  S  3  S  T  4 
 P  S  3 P T  1  P  S  4  P T  0 
= 0.0892350.0091578 + 0.133850.0018316
= 0.0106 (to 3 sig.
s figs)

Page 7 of 9
AJC 2016 P
Preliminary Examina
ation 9740/01

AJC P2 Solutions 65
(iii) Let Y = number of travellers arriving individually in 10 minutes Y ~ Po  75 
Since  = 75 > 10, Y ~ N  75, 75  approximately
Let W= number of groups arriving in 10 minutes. W ~ Po  20 
Since  = 20 > 10, W ~ N  20, 20  approximately

Y  3W ~ N  75  3(20),75  32  20    Y  3W ~ N 15,255
P Y  3W  8  
c .c .
P Y  3W  8.5  = 0.658

11 13.33  k
(i) x9 y
8
S  0.069940t  1.1319
13.33  k
 0.069940(9)  1.2718
8
k  1.88008  1.88
r  0.903
(ii) From the G.C., using the formula S  a  b ln t , we get 
S  0.82848  0.53141ln t , r = 0.98479  0.985.  
Thus a  0.828, b  0.531        

(iii) When t = 9,

(iv) sgexamguru.com
S  0.53141ln 9  0.82848  1.9961
Quantity of the product = 1996 ( to nearest integer).
This estimate for S is reliable because:
(1) t = 9 is within the data range of 2 to 16, and
(2) the r value between S and ln t is closer to 1 than the r value between S and t,
hence there is a stronger relationship between S and ln t .

12 (i) unbiased estimate of the mean monthly salary

=
  x  3000   3000  505  3000  2989.90
50 50
Unbiased estimate of the variance of monthly salary, s2

   x  3000 
2
1  
=
49    x  3000 2

50 

 

1  5052  95479.5
=  100580    1948.56
49  50  49

(ii) Let  x be the mean monthly salary

Page 8 of 9
AJC 2016 Preliminary Examination 9740/01

AJC P2 Solutions 66
H 0 :  x  3000
H1 :  x  3000
Since population variance is unknown but sample size (= 50) is large, use a two-tailed Z-test at
10% level, ie reject H0 if p-value < 0.1
X  3000
Under H0 , test statistic, Z  ~ N (0,1) .
1948.56
50
From GC, p-value = 0.106 > 0.1, we do not reject H0 .
There is insufficient evidence at 10% level to support the manager’s suspicion.
(or There is insufficient evidence at 10% level to conclude that the average monthly salary of
a fresh graduate is not $3000).

Let Y be the r.v. denoting the monthly salary of a fresh Mathematics graduate and  y be the
mean monthly salary.

H 0 :  y  3000
H1 :  y  3000
Since population variance is unknown but sample size (=15) is small, use a one-tailed T-test at
5% level, ie reject H0 if p-value < 0.05
15
Given y  3050, n  15, s y 2  k 2
14
Y  y
Test statistic T  t(14)

sgexamguru.com
Sy
15
5%

Since H0 is rejected, tcal  tcrit


1.7613
3050  3000
  1.7613
15 k 2
14
15
50
  1.7613
k2
14
k2 50
 <  k 2 < 11282.40  0  k 2  11282
14 1.7613

Assumption: The monthly salary of a fresh Mathematics graduate follows a Normal


distribution.

Page 9 of 9
AJC 2016 Preliminary Examination 9740/01

AJC P2 Solutions 67
1

CATHOLIC JUNIOR COLLEGE


General Certificate of Education Advanced Level
Higher 2
JC2 Preliminary Examination

MATHEMATICS 9740/01
Paper 1 24 Aug 2016
3 hours
Additional Materials: List of Formulae (MF15)

Name: ___________________________ Class: ________________

READ THESE INSTRUCTIONS FIRST

Write your name and class on all the work you hand in.
Write in dark blue or black pen on both sides of the paper.
You may use a soft pencil for any diagrams or graphs.
Do not use staples, paper clips, highlighters, glue or correction fluid.

Answer all the questions.

sgexamguru.com
Give non-exact numerical answers correct to 3 significant figures, or 1 decimal place in the case of angles
in degrees, unless a different level of accuracy is specified in the question.
You are expected to use a graphing calculator.
Unsupported answers from a graphing calculator are allowed unless a question specifically states
otherwise.
Where unsupported answers from a graphing calculator are not allowed in a question, you are required to
present the mathematical steps using mathematical notations and not calculator commands.
You are reminded of the need for clear presentation in your answers.

At the end of the examination, arrange your answers in NUMERICAL ORDER.

The number of marks is given in brackets [ ] at the end of each question or part question.

Question 1 2 3 4 5 6 7 8 9 10 11 Total

Marks

Total 5 5 5 7 7 8 9 9 13 15 17 100

This document consists of 5 printed pages, including the cover page.

9740/01/Prelims/2016

CJC P1 Questions 68
2

1 [In this question, sketches of the given graphs are not drawn to scale]
The graphs of y  f  x  and y 2  f  x  are given below.

y  f  x y2  f  x

On separate diagrams, draw sketches of the graphs of

(a) y 2  f  2 x  , [2]
(b) y  f  x  , [3]

sgexamguru.com
stating the equations of any asymptotes and the coordinates of any points of intersection
with the axes.

2 The vectors a and b are given by


a  4i  6 pj  8k and b  2i  3j  4 pk , where p  0 .
It is given that a  2 b .
(i) Find p. [2]
1
(ii) Give a geometrical interpretation of b a . [1]
b
1
(iii) Using the value of p found in part (i), find the exact value of b a . [2]
b

3 The cubic equation x3  ax 2  bx  c  0, where a, b and c are constants, has roots 3 + i and 2.
(i) One JC2 student remarked that the third root is 3 − i. State a necessary assumption the
student made in order that the remark is true. [1]
(ii) Given that the assumption in part (i) holds, find the values of a, b and c. [4]

9740/01/Prelims/2016

CJC P1 Questions 69
3

4 A closed container is made up of a cylinder of base radius r cm and height h cm, and a
hemispherical top with the same radius r.

It is inscribed within a fixed right circular cone of base radius 5 cm and height 12 cm, as
shown in the diagram below.

sgexamguru.com 13
(i) By using similar triangles, show that h  12  r.
5
Determine the exact range of possible values of length r. [3]

(ii) Find the total volume V of the closed container in terms of r.


By differentiation, find the exact value of r that produces the maximum container
volume V, as r varies.
4 3
[Volume of a sphere with radius R is πR .]
3 [4]

n
5 A sequence u1 , u2 , u3 , satisfies the recurrence relation un  un 1 , for n  2 .
(n  1) 2
(a) 2n
Given that u1  2 , use the method of mathematical induction to prove that un 
(n  1)! [4]
, for n  1.
(b) Given that u1  a , where a is any constant. Write down u2 , u3 and u4 in terms of a.
Hence or otherwise, find un in terms of a. [3]

9740/01/Prelims/2016

CJC P1 Questions 70
4

6 (i) Given that 1  2r  A(r  1)  Br , find the constants A and B. [1]


n
1  2r
(ii) Use the method of differences to find r 1 3r
. [3]

2  2r
(iii) Hence find the value of 
r 1 3r
. [4]

7 (i) Given that y  1  ln(1  x) , find the exact range of values of x for y to be well defined. [1]
dy 1 y
e .
2
(ii) Show that 2 y [2]
dx
(iii) Hence, find the Maclaurin’s series for 1  ln(1  x) , up to and including the term in
2
x . [3]
(iv) Verify that the same result is obtained using the standard series expansions given in the
List of Formulae (MF15). [3]

8 Do not use a calculator in answering this question.

(i) It is given that complex numbers z1 and z2 are the roots of the equation z 2  6 z  36  0
such that arg(z1) > arg(z2). Find exact expressions of z1 and z2 in the form rei , where

(ii)
sgexamguru.com
r  0 and  π    π .

Find the complex number


z14
in exact polar form.
[4]

[3]
iz 2
(iii) Find the smallest positive integer n such that z2 n is a positive real number. [2]

 x 1
9 (i) By using the substitution u  x  1 , find  dx . [5]
 x 1
x 1
(ii) The region R is bounded by the curve y  and the lines x = 8 and y = 1.
x 1
Find
 B 2 
the exact area of R , simplifying your answer in the form A  2 ln 
 B  2 
(a)
  [5]
where A and B are integers to be determined,
(b) the volume of the solid generated when R is rotated 2π radians about the x-axis,
giving your answer correct to 2 decimal places. [3]

9740/01/Prelims/2016

CJC P1 Questions 71
5

10 The plane p passes through the points A, B and C with coordinates  0, 1,1 ,  2, 1, 4  and
 2, 1,0 respectively.
(i) Show that a cartesian equation of the plane p is 2x – y – 2z = –3. [3]

2 1
 
The line l has equation r   1    0  ,   .
4  2 
   
(ii) Find the acute angle between l and p. [2]

The point Q has position vector 5i  j  2k .


(iii) Show that Q lies on the line l. [1]
(iv) It is given that a variable point R lies on the plane p and is at a distance of 45 from
the point Q. Find the foot of perpendicular from the point Q to the plane p and hence
describe geometrically the locus of R. [6]
(v) Find a vector equation of the line which is a reflection of the line l in plane p. [3]

11 The function f is defined by

sgexamguru.com
2x  k
f :x , x  , x  2,
x2
where k is a positive constant.

(i) Sketch the graph of y  f  x  , stating the equations of any asymptotes and the
coordinates of any points where the curve crosses the x and y axes. [3]
1
(ii) Describe fully a sequence of transformations which would transform the curve y 
x
onto y  f  x  . [4]
(iii) Find f 1 in a similar form and write down the range of f 1 . [3]
(iv) Hence or otherwise, find f 2 .
1
Find the value of f 2017   , leaving your answer in terms of k.
2 [4]

The function g is defined by


g: x a  x  3, x  , x  3,
where a is a real constant.

(v)
Given that fg exists, write down an inequality for a and explain why gf does not exist.
[3]
 THE END 

9740/01/Prelims/2016

CJC P1 Questions 72
6

sgexamguru.com

9740/01/Prelims/2016

CJC P1 Questions 73
1 Topic: Transformation of Curves
Solution
(a) Graph of y 2  f  2 x 

sgexamguru.com
(b) Graph of y  f  x
y

5 x
0

y=-2

x=1 x=3

Page 1 of 26
CJC P1 Solutions 74
1 Topic: Transformation of Curves
Solution

sgexamguru.com

Page 2 of 26
CJC P1 Solutions 75
2 Topic: Vectors
Solution
(i) a 2b
16  36 p 2  64  2 4  9  16 p 2
80  36 p 2  4 13  16 p 2 
28 p 2  28
p  1 or p  1  Reject  p  0 
 p 1
(ii) Length of projection of a on b
. (iii) 2  4
   
 3    6 
 4   8 
1     42
b a  
b 2
22   3  42 29

sgexamguru.com

Page 3 of 26
CJC P1 Solutions 76
3 Topic: Complex Numbers
Solution
(i) The assumption is that a, b and c are all real.
(ii) Let x3  ax2  bx  c  (x  (3 i))(x  (3 i))(x  2)

= (x2  6x 10)(x  2)
= x3 8x2 22x20
By comparing coefficients,
we have a = –8, b = 22 and c = –20.

sgexamguru.com

Page 4 of 26
CJC P1 Solutions 77
4 Topic: Application of Differentiation
Solution
(i) Using the diagram provided,

sgexamguru.com
Identify that  ABC is similar to ADE .
AE AC
 DE  BC

Since AC  52  122  13 , BC  5 , DE  r ,
 AE r
 135  AE  13
5
r
h  AB  AE  12  135 r

Since r and h are lengths,


r  0 and h  0 (i.e. 12  135 r  0, r  13
60
).
60
0r  13

(ii) Volume of inscribed container,


Page 5 of 26
CJC P1 Solutions 78
4 Topic: Application of Differentiation
Solution
V   r 2h  12  4
3
 r3 
 
  r 2 12  135 r  32  r 3
 12 r 2  15
29
 r3
Differentiating this,
dV
dr
 24 r  29
5
 r2 .

dV
Consider dr
 0.
i.e. 24 r  29
5
 r2  0 ,
 r (24  29
5
r)  0 ,
r  0(rejected as r  0) or r  120
29
.

sgexamguru.com
(Check that these stationary points are within
60
the admissible range 0  r  13 ).

First derivative test


dV
dr
 24 r  29
5
 r2
r r 120  r  120 r 120 
29 29 29

e.g. r  119
29 e.g. r  121
29
dV 119 0 121
dr  0   0
145 145

Maximum volume at r  120


29

Second derivative test


dV
dr
 24 r  29
5
 r2

Page 6 of 26
CJC P1 Solutions 79
4 Topic: Application of Differentiation
Solution
d 2V
 24  2  29
5 
r
dr 2

 24  2  29
5 
r
 120 
 24  58
5
 
 29 
  24  0 (maximum volume)

Alternative:
Graphical method
Upon finding the stationary points,
60
inspect the graph of V against r, where 0  r  13
.

sgexamguru.com

Note that the maximum is attained at the stationary point


where r  120
29
 4.1379...

Page 7 of 26
CJC P1 Solutions 80
5 Mathematical Induction
Solution
Let Pn be the statement that u n  2 n for n   , n 1.
( n  1)!

When n 1, LHS= u1  2 (given)

RHS  2  2 = LHS,
(0)!

P1 is true.

Assume that Pk is true for some k, where k  ,k1 i.e.

2k
uk 
(k 1)!

sgexamguru.com
Pk1 is also true ,i.e., 2( k  1)
To prove u k 1 
k!
k 1
LHS = uk 1  uk
(k)2
k  1 2k

(k ) 2 (k  1)!
2(k  1)

k ( k  1)!
2(k  1)
  RHS
k!

Since P1 is true, and Pk is true  Pk1 is true , by


Mathematical Induction, Pn is true for all n    .

Page 8 of 26
CJC P1 Solutions 81
5 Mathematical Induction
Solution
(b) 2 2 2
u2  u  a  2a  a
2 1
(1) 1 1!
3 3 3 3
u3  2 u2  2 2a  a  a
(2) (2) (2) 2!
4 4 3 4 4
u4  2 u3  2 a a a
(3) (3) (2) (3 2) 3!
“Hence” approach
n
un  a by observation from part (ii)
(n 1)!

“Otherwise” approach
n
un  un 1

sgexamguru.com
(n 1)2
n n 1
 un2
(n 1) (n  2)2
2

n n 1 n  2
 un3
(n 1) (n  2)2 (n  3)2
2

n n 1 n  2 3 2
 2 2 2
 2 2 u1
(n 1) (n  2) (n  3) (2) (1)
n 1 1 1 1
 1 1 1
 1 1a
(n 1) (n  2) (n  3) (2) (1)
n
 a
(n 1)!

Page 9 of 26
CJC P1 Solutions 82
6 Sigma Notation
Solution
(i) 1  2r  A(r 1)  Br
 ( A  B)r  A
A1, B3
(ii) n
1  2r n
 r  1  3r

r 1 3 r
 
r 1 3r
n
 r 1 r 
   r  r 1 
r 1  3 3 
2 1
 1  0 
3 3 
 2 2
 2  1 
3 3 

sgexamguru.com

 n n 1 
 n1  n 2  
3 3 
 n 1 n 
 n  n1 
 3 3 
n 1
 n 1
3

Page 10 of 26
CJC P1 Solutions 83
6 Sigma Notation
Solution
(iii) 
2  2 r  1  2r  1
r 1 3r

r 1 3r

 1  2r 1 
  r  r 
r 1  3 3 

 1  2r    1 
  r    r 
r 1  3  r 1  3 
1
 n  1 
 lim  n  1   3
n 
 3  1 1
3
1
 1 
2
1

sgexamguru.com

2

Page 11 of 26
CJC P1 Solutions 84
7 Topic: Maclaurin’s Series
Solution
(i) For y  1  ln(1  x ) to be well-defined,
1  x  0 and 1  ln(1  x)  0
ln(1  x)  1
x  1 and 1  x  e 1
x  e 1  1
 x  e 1  1
(ii) By Implicit Differentiation,
y  1  ln 1  x 
y2 1 ln1 x
 ln 1  x   y 2  1
2
1
 1 x  ey
Differentiate implicitly with respect to x,
2y
dy

1
dx 1  x
1
sgexamguru.com
 y2 1
e
2
 e1 y (shown)
(iii) Differentiate the above results implicitly with respect to x,
2
d2 y  dy  d 2
2y 2
 2   1  y2   e1 y
dx  dx  dx
2
d2 y  dy  dy 2
2 y 2  2   2 y  e1 y
dx  dx  dx

When x0 ,
y  1 ln1 1 ,
dy 1
 ,
dx 2
Page 12 of 26
CJC P1 Solutions 85
7 Topic: Maclaurin’s Series
Solution
d2 y 3
2
 ,
dx 4

3

1
y  1  x  4 x 2  ...
2 2!
1 3
 1  x  x2  ...
2 8
(iv) 1
y  1  ln 1  x   2
1
  x2 2
  1   x   ...  
  2 
1 1

2  2  
2 2
1  x  x2 
 1  x   ...     x   ...   ...

sgexamguru.com
2 2  2!  2 
1 1 1 x 2  x2 
 1 x  x 2   x   x    ...
2 4 8 2  2 
1 1 1
 1  x  x 2  x 2  ...
2 4 8
1 3
 1  x  x 2  ...
2 8

Page 13 of 26
CJC P1 Solutions 86
8 Topic: Complex Numbers
Solution
(i) 6  36  4(1)(36)
z 2  6 z  36  0  z   3  3 3i
2
 
i i
3 3
Thus, z1  6e and z2  6e
(ii)  i 3 
4

 6e  7
i( )
   63 e 6
 i  2  3  
 6e   
 
 
  5   5 
 63 cos    i sin  
  6   6 
(iii)   n 
i i  
3  n n
e  3

sgexamguru.com
z2  6e z2  6
n 
Since z2  ,  n3  2k for some integer k.
n   6k .
n  ... , 12, 6, 0,  6,  12, ...

Smallest positive integer n = 6.

Page 14 of 26
CJC P1 Solutions 87
9 Topic: Techniques of Integration / Definite Integrals
Solution
(i) du 1

dx 2 x  1
1
=
2u

x 1 u
 dx   2 .2udu
x 1 u 2
2
= 2 (1  2 )du
u 2
2 u 2
= 2u + ln c
2 u 2
x 1  2
= 2 x  1  2 ln c

sgexamguru.com
x 1  2

(ii) Area of R
(a) 8 x 1
 5(1)   dx
3 x 1
8
 x 1  2 
= 5   2 x  1  2 ln 
 x  1  2  3
 3 2   2 2 
 5  {2  2 ln    2 ln  }
 3  2   2  2 
  3 2 2  2 
 3   2 ln  .  
 3  2 2  2
  

Page 15 of 26
CJC P1 Solutions 88
9 Topic: Techniques of Integration / Definite Integrals
Solution
  4 2 
 3   2 ln  

  4 2  
 4  2
 3  2 ln  
 4  2 

(ii) Vol. generated


(b) 8 x 1
  (12 ).5    2
dx
3
 x  1
 9.52830
 9.53 units3

sgexamguru.com

Page 16 of 26
CJC P1 Solutions 89
10 Topic: Vectors
Solution
(i)  2   0  2 
      
AB   1   1    2 
 4  1  3 
     
 2   0   2 
      
AC   1    1    2 
 0   1   1 
     
 2   2   8   2
       
AB  AC   2    2    4   4  1 
 3   1   8   2 
       
2
 
Choose normal vector n p for plane p =  1  .

sgexamguru.com
 2 
 
 2  0  2 
p : r   1    1    1   3
 2   1   2 
     
A cartesian equation of the plane p is 2 x  y  2 z  3
(ii) Let the acute angle between l and p be  .

The angle between the normal vector n p (for plane p) and



the direction vector ml (for line l),

 2 1
   
 1    0 
 2   2  6
  cos 1      cos1  26.565
 2 1 3 5
 1   0 
   
 2   2 
   
Page 17 of 26
CJC P1 Solutions 90
10 Topic: Vectors
Solution

  90  26.565  63.4  to 1d.p. or 1.11 rad

Alternative :
Let the acute angle between l and p be  .
 2 1
 1    0 
   
 2   2 
    6
sin   
 2  1 3 5
 1   0 
   
 2   2 
   
6
  sin 1  63.4  to 1d.p. or 1.11 rad (3 s.f.)

sgexamguru.com
3 5

2 (iii) Assume Q lies on the line l.


5 2  1   5 2   3
 1    1    0    1  1  
       
 2   4   2  2  4  2   3
       
Since   3 is consistent throughout, Q lies on the line l.

Alternative 1:

Page 18 of 26
CJC P1 Solutions 91
10 Topic: Vectors
Solution
 5 2  3
      
Since OQ   1    1   0 
 2   4   6 
     
2 1
   
  1  3  0 
 4   2 
   
Q lies on the line l.

Alternative 2:
x2 z4
l:  , y  1
1 2
Q  (5, 1, 2) , i.e. x = 5, y = -1, z = 4.

x 2 52
1

1
sgexamguru.com
3,
z  4 2  4
2

2
 3.
Hence, Q lies on the line l.

(iv) Let F be the foot of perpendicular from the point Q to the


plane p.

5  2
lQF : r   1     1  ,   
 
 2   2 
   
5  2
    
Since F lies on lQF , OF   1     1  , for some    .
 2   2 
   

Page 19 of 26
CJC P1 Solutions 92
10 Topic: Vectors
Solution
2
  
Since F also lies on plane p, OF   1   3 .
 2 
 
 5   2   2 
     
 1     1     1   3
 2   2    2 
   
2  5  2     1     2  2  2    3
15  9   3    2

 5   2  1
      
 OF   1   2  1    1 
 2   2   2 

sgexamguru.com
     
The foot of perpendicular from the point Q to the plane p is
1,1, 2  .
Alternative :
 
QF  (QA  nˆ ) nˆ , where n is a normal vector of p
  
  5   2    2 
  1   1  
   2    1    1 
  3  3  2   3  2 
     
 2
 
 2  1 
 2 
 

Page 20 of 26
CJC P1 Solutions 93
10 Topic: Vectors
Solution
5  2  1
    
 OF  OQ  QF   1   2  1    1 
 2   2   2 
     

The foot of perpendicular from the point Q to the plane p is


1,1, 2  .

 1   5   4 
      
QF   1    1    2 
 2   2   4 
     
 2
QF   4   22  4 2  6

sgexamguru.com
RF  45  36  3
The locus of R is a circle that lies in plane p with centre
1,1, 2  and radius 3.
(v) Let Q  be the image of Q in plane p.
  
OF  12 (OQ  OQ)
  
OQ  2OF  OQ
 1   5   3 
 2  1    1    3 
 2   2   6 
     

Equivalent Methods
  
OQ  OF  QF
  
OQ  OQ  QQ
 
 OQ  2QF
Page 21 of 26
CJC P1 Solutions 94
10 Topic: Vectors
Solution
  
BQ  OQ  OB
 3   2   5 
     
  3    1   4 
6 4  2
     

Alternative:
  
BQ  BQ  QQ
 5   2    4   5 
    
  1    1   2  2    4 
 2   4    4
 
2
 

sgexamguru.com
A vector equation of the line which is a reflection of the line
l in plane p is
2  5 
r   1    4  ,   
 
4 2
   
 3   5 
   
or r   3     4  ,   
6 2
   

Page 22 of 26
CJC P1 Solutions 95
11 Topic: Graphing Techniques & Functions
Solution
(i)
y
2x  k k  4
 2
x2 x2
Vertical Asymptote: x  2
Horizontal Asymptote: y  2
k
x-intercept: x  
2
k
y-intercept: y  
2

sgexamguru.com

Page 23 of 26
CJC P1 Solutions 96
11 Topic: Graphing Techniques & Functions
Solution
1
f  x 
x
 A
1
f  x  2 
x2
 B
 k  4
 k  4 f  x  2 
x2
 C

2   k  4 f  x  2  2 
 k  4
x2
A: Translation in the positive x direction by 2 units

sgexamguru.com
B: Scaling parallel to the x direction (// to y-axis) by a factor
of  k  4 
C: Translate in the positive y direction by 2 units

2x  k
Let y 
(iii) x2
y  x  2  2 x  k
yx2y 2xk
yx2x 2y k
x  y  2  2 y  k
2y  k
x
y2
2y  k
f 1  y  
y2

Page 24 of 26
CJC P1 Solutions 97
11 Topic: Graphing Techniques & Functions
Solution
2x  k
f 1  x  
x2

Df 1  Rf  Df   ,2   2, 
2x  k
f 1 : x  , x  , x  2,
x2
Rf 1  Df  R f   ,2   2, 

(iv) f  x   f 1  x 

 f 2  ff  x   f f 1  x    x
Df 2  Df   ,2  2, 

1
sgexamguru.com
1
 f 2017    f  
2 2
1
2   k
2
  
1
 2
2
1 k

3

2
2  2 k

3
(v) 2x  k
f  x 
x2
Df   ,2  2, 
Rf   ,2  2, 

Page 25 of 26
CJC P1 Solutions 98
11 Topic: Graphing Techniques & Functions
Solution

g  x  a  x  3
Dg   3, 
Rg   a, 
Since fg  x exists, R g  Df
Rg   a, 
Df   ,2  2, 
a  2

For gf  x to exist, R f  D g
Rf   ,2  2, 
Dg   3,  sgexamguru.com
Since R f  D g , gf  x does not exist.

Page 26 of 26
CJC P1 Solutions 99
1

CATHOLIC JUNIOR COLLEGE


General Certificate of Education Advanced Level
Higher 2
JC2 Preliminary Examination

MATHEMATICS 9740/02
Paper 2 30 August 2016
3 hours
Additional Materials: List of Formulae (MF15)

Name: ___________________________ Class: ________________

READ THESE INSTRUCTIONS FIRST

Write your name and class on all the work you hand in.
Write in dark blue or black pen on both sides of the paper.
You may use a soft pencil for any diagrams or graphs.
Do not use staples, paper clips, highlighters, glue or correction fluid.

Answer all the questions.

sgexamguru.com
Give non-exact numerical answers correct to 3 significant figures, or 1 decimal place in the case of angles
in degrees, unless a different level of accuracy is specified in the question.
You are expected to use a graphing calculator.
Unsupported answers from a graphing calculator are allowed unless a question specifically states
otherwise.
Where unsupported answers from a graphing calculator are not allowed in a question, you are required to
present the mathematical steps using mathematical notations and not calculator commands.
You are reminded of the need for clear presentation in your answers.

At the end of the examination, arrange your answers in NUMERICAL ORDER.

The number of marks is given in brackets [ ] at the end of each question or part question.

Question 1 2 3 4 5 6 7 8 9 10 11 12 Total

Marks

Total 9 9 11 11 4 5 6 8 9 9 9 10 100

This document consists of 7 printed pages, including the cover page.

9740/02/Prelims/2016

CJC P2 Questions 100


2

Section A: Pure Mathematics [40 marks]

1 (i) Prove that the substitution u  x 2  y 2 reduces the differential equation


dy
y  x  x2  y 2 to
dx
du
2 u.
dx
dy
Hence, show that the general solution of the differential equation y  x  x2  y 2
dx
[4]
is given by x 2  y 2  x  D , where D is an arbitrary constant.
(ii) The result in part (i) represents a family of curves. On a single diagram, sketch a non-
linear member of the family which passes through the point  2, 0  .
You should state the equation of the graph and axial intercepts clearly on the diagram. [3]
(iii) State an equation of the line of symmetry for the curve in part (ii). [1]

2 The three distinct roots of the equation x3  1  0 are denoted by 1,  and  2 .

(a) sgexamguru.com
Without first finding  explicitly, show that 1     2  0 . [2]

(b)
Given now that 0  arg( )   , sketch, on a single Argand diagram, the loci given by
(i) | z   |  |  | and [3]
(ii) arg( z  1)    arg( 2 ) . [2]
Hence, find the complex number that satisfies both loci, expressing your answer
exactly in the form a + ib, where a and b are real numbers. [2]

9740/02/Prelims/2016

CJC P2 Questions 101


3

3 The diagram shows the graph of curve C represented by y  f ( x) , with oblique asymptotes
y  x and y   x .

(a) On a separate diagram, sketch a graph of y  f ( x) , clearly indicating the equation(s)


of the asymptote(s) and axial-intercept(s). [2]

(b) The above curve C is represented by the parametric equations


 
x  tan  , y  sec , where    .

(i)
sgexamguru.com 2 2

Show that the normal to the curve at point P, with coordinates (tan  , sec ) ,

for 0    , is given by y   x cosec  2sec . [2]
2

(ii) The normal to the curve at point P intersects the x-axis at point N.
Find the coordinates of the mid-point M of PN, in terms of  .
Hence find a cartesian equation of the locus of M, as  varies. [4]

(iii) Taking O as the origin, show that the area of triangle OPN is tan  sec .

Point P moves along the curve such that the rate of change of its parameter 
d
with respect to time t is given by  cos . Find the exact rate of change of
dt

the area of triangle OPN when   . [4]
6

9740/02/Prelims/2016

CJC P2 Questions 102


4

4 Adam and Gregory signed up for a marathon. In preparation for this marathon, Adam and Gregory
each planned a 15-week personalised training programme. Adam runs 2.4 km on the first day of
Week 1, and on the first day of each subsequent week, the distance covered is increased by 20% of
the previous week. Gregory also runs 2.4 km on the first day of Week 1, but on the first day of each
subsequent week, the distance covered is increased by d km, where d is a constant. Assume Adam
and Gregory only run on the first day of each week.

(i) Find, in terms of d, the total distance covered by Gregory in these 15 weeks. [2]
(ii) Adam targets to cover a total distance of 170 km in these 15 weeks. Can Adam achieve
[2]
this target? You must show sufficient working to justify your answer.
(iii) It is given that Adam covers a longer distance than Gregory on the first day of the 15 th
week. Find the maximum value of d, correct to 2 decimal places.
Using this value of d, show that the difference in the distance covered by Adam and
Gregory for their 15th week training is 0.134 km correct to 3 significant figures. [4]
(iv) Due to unforeseen circumstances, Adam has to end his training programme early. In
order for Adam to cover a total distance of 170 km by the end of the 13 weeks, the
distance covered has to be increased by x % of the previous week on each subsequent
week from Week 1. Find x. [3]

Section B: Statistics [60 marks]

5 sgexamguru.com
A bag contains four red and eight blue balls of which two of the red balls and six of the blue balls
have the number “0” printed on them. The remaining balls have the number “1” printed on them.
Three balls are randomly drawn from the bag without replacement.
54
(i) Show that the probability that at least one blue ball is drawn is . [1]
55
Find the probability that
(ii) at least one ball of each colour is drawn, [2]
(iii) the sum of the numbers on the balls drawn is at least two. [3]

6 Packets of a particular brand of potato chips are delivered to a supermarket in boxes of 60. On
average, 1.8 packets in a box are underweight. The number of underweight packets from a randomly
chosen box is the random variable X.

Assume that X has a binomial distribution.


(i) Use a suitable approximation to find the probability that two randomly chosen boxes of
potato chips contain more than 6 packets of underweight potato chips. State the
parameter(s) of the distribution that you use. [4]

A batch of 50 boxes of potato chips is delivered to the supermarket.


(ii) Use a suitable approximation to find the probability that the mean number of [3]
underweight packets per box is more than 2.

9740/02/Prelims/2016

CJC P2 Questions 103


5

7 A group of 9 friends, including Albert and Ben, are having dinner at Albert’s house. They sit in
two groups: a row of 4 on a couch and a group of 5 at a round dining table with 5 identical seats.
Find the number of ways they can sit if
(i) there are no restrictions, [2]
(ii) Albert and Ben sit beside each other, [3]
(iii) Albert and Ben both sit on the couch or both sit at the round table, but they do not sit
beside each other. [3]

8 A factory manufactures a certain product for sale. The following table gives the quantity of
product manufactured, x units in thousands, and its corresponding cost of production, y dollars in
thousands. The data is recorded during different months of a certain year.
Quantity of
2.0 2.4 3.0 3.8 4.8 6.0 7.2 8.2 9.4
product, x
Cost of
10 19 35 47 58 35 78 80 81
production, y

(i) Draw a scatter diagram for the data. [1]


One of the values of y appears to be incorrect.
(ii) Indicate the corresponding point in your diagram by labelling it P. [1]

sgexamguru.com
Remove P from the set of data.
(iii) By using the scatter diagram for the remaining points, explain whether y = a + bx or [1]
y = a + b ln x is the better model for the relationship between x and y.
(iv) Using the better model chosen in part (iii), find the product moment correlation
coefficient and the equation of a suitable regression line.
Explain what happens to the product moment correlation coefficient and the equation
of the regression line if the factory decides to include a fixed cost of M thousand dollars
for purchasing a packing machine to the cost of production, y. [4]
(v) Use the regression line found in part (iv) to estimate the cost of production when the
quantity produced is 6000 units and comment on its reliability. [2]

9740/02/Prelims/2016

CJC P2 Questions 104


6

9 The finishing times in a 10km race with a large number of runners follow a normal distribution.
After 40 minutes, 10% of the runners have completed the race. After one hour, 35% of the runners
have yet to complete the race. The first 20% of runners who finish the race receive a medal.
(i) Show that, correct to 1 decimal place, the runners have running times with mean 55.4 [2]
minutes and standard deviation 12.0 minutes.
(ii) Find the maximum time a runner can take to finish the race in order to receive a medal. [2]

A random sample of 12 runners is selected.


(iii) Find the probability that more than four runners receive a medal. [2]
(iv) Given that none of the runners receives a medal, find the probability that the slowest
runner completes the race in under one hour. [3]

10 (a) A Physical Education teacher wants to plan a volleyball training programme for all students
in a secondary school, where each student has exactly one CCA. In order to check on the
current fitness level of students in the school, he selects a sample of students by choosing
the Captains of every sports team and the Presidents of every Club and Society in the school.

(i) Explain briefly why this may not provide a representative random sample of the
[1]
student population.
(ii) Name a more appropriate sampling method which would provide a

sgexamguru.com
representative random sample and explain how it can be carried out in this
context.
[3]

(b) The vertical jump heights of players from a volleyball team are normally distributed with
mean 40 cm. The coach claims that a particular training regime is effective in improving the
players’ jump heights. After the regime is implemented for a period of time, a random
sample of 7 players is taken and their jump heights are recorded.
The sample mean is 42.1 cm and the sample standard deviation is k cm .
A test is to be carried out at the 10% level of significance to determine whether the training
regime has been effective.

(i) State appropriate hypotheses for the test. [1]


(ii) Find the set of values of k for which the result of the test would be to reject the
[3]
null hypothesis.
(iii) State the conclusion of the test in the case where the sample variance is 15. [2]

9740/02/Prelims/2016

CJC P2 Questions 105


7

11 The average number of calls per hour received by telephone operators at the Call Centre of bank
ECBC is being reviewed.
(i) State, in context, two assumptions that need to be made for the number of calls
received by a telephone operator to be well modelled by a Poisson distribution. [2]

The Call Centre has only three telephone operators at any point in time. One handles calls
pertaining to credit card queries, another handles calls pertaining to business banking queries while
the last operator handles calls pertaining to personal banking queries, with the numbers of calls
received in one hour assumed to have the independent distributions Po( ), Po(6) and Po(7)
respectively.
(ii) It is given that the probability of receiving two calls pertaining to credit card queries
within an hour is eight times that of receiving two calls pertaining to credit card
queries within four hours.
a
Find the exact value of  , expressing your answer in the form ln 2 where a and b
b
are two positive integers to be found. [3]
(iii) On a certain day, the Call Centre receives more than 50 calls from 1200 to 1400 hours.
Find the probability that there are no calls pertaining to credit card queries during this
period. [3]
(iv) Using suitable approximations, find the probability that there are more calls pertaining
to business banking queries than personal banking queries within a two-hour period. [3]

sgexamguru.com  THE END 

9740/02/Prelims/2016

CJC P2 Questions 106


Section A: Pure Mathematics [40 marks]

1 Topic: Differential Equations


Solution
(i) Given u  x2  y 2 , differentiate with respect to x:
du dy
 2x  2 y
dx dx
dy 1 du
x y  (I)
dx 2 dx
Substitute (I) & u  x2  y 2 and into D.E:
1 du
 u:
2 dx
du
 2 u (shown)
dx

sgexamguru.com
Hence,
1 du
2
u dx
1
du 2
u 2
dx
Integrate both sides with respect to x:
1
 du
 2 d x dx   2 d x
u
1
 1
2
u
 2 x  C , where C is an arbitrary constant
1
 1
2

Page 1 of 25
CJC P2 Solutions 107
1 Topic: Differential Equations
Solution
1
2u 2  2 x  C
C
u  x
2
C
x 2  y 2  x  D, where D 
2
(ii) x2  y 2  x  D
2
x2  y2   x  D 
y 2  x 2  x 2  2 Dx  D 2
y 2  2 Dx  D 2
y   2 Dx  D 2

sgexamguru.com
When x  2, y  0
0  4 D  D 2
D  D  4  0
D  0 (rej.) or D  4
y 2  8 x  16
C1 : y 2  8 x  16
y
 0, 4 

 2, 0 O x
 0, 4

(iii) The equation of line of symmetry is y  0 .

Page 2 of 25
CJC P2 Solutions 108
2 Topic: Complex Numbers
Solution
Using Geometric series,
3 1
1   2  0 (as  3  1)
 1

Or
3 1  0
Since  is a root of x3  1  0, (  1)( 2    1)  0
Since   1, 2    1  0
Or
Any acceptable method

sgexamguru.com
Im

M (pt. of
(ii) intersection)
(i)

-1 O Re

Using Pythagoras Theorem, OM  2 2  12  3 .

Page 3 of 25
CJC P2 Solutions 109
2 Topic: Complex Numbers
Solution
 The complex number is 3i .

sgexamguru.com

Page 4 of 25
CJC P2 Solutions 110
3 Topic: Application of Differentiation
Solution
(a)

(b) Given x  tan  , y  sec  , where  2    2 .


(i) dy
dx
d
 sec 2  , d
 sec tan  ,
dy dy dx
dx
 d d

sgexamguru.com
 sec tan
2
  tan 
sec
 sin 
sec 
At point P, gradient of normal =  sin1  .
Equation of the normal to the curve at P :
y  sec    sin1  ( x  tan  ) ,
y  cos1    x sin1   cos1  ,
y   x cosec  2 sec  . (shown)

Page 5 of 25
CJC P2 Solutions 111
3 Topic: Application of Differentiation
Solution
(b)
(ii)

x-intercept of the normal at P :


0   x cosec  2sec ,
sec 
x  2 cosec  2 tan  .

sgexamguru.com

 Point N is (2 tan  , 0) .
Mid-point of PN is M
 xP  xN
2
,
yP  yN
2  3
2
tan  , 1
2
sec  .
Locus of point M is given by the parametric
equations
x  32 tan  , y  12 sec , where  2    2 .

Since sec 2   tan 2   1 ,


and that sec   2 y , tan   23 x ,
 (2 y ) 2  ( 23 x ) 2  1 , a cartesian equation
for the locus of M.

Page 6 of 25
CJC P2 Solutions 112
3 Topic: Application of Differentiation
Solution
(b)
(iii)

A, area of OPN  12 (ON ) (Height of P


w.r.t. x -axis )

 12 (2 tan  ) (sec )  tan  sec

sgexamguru.com
(ON  2 tan  , from (b)(ii)
assuming   0.)

dA
d  
 sec2  sec  tan   sec tan  

 sec3   sec tan 2 

Rate of change of area of OPN ,


dA
dt
 ddA  ddt
 (sec3   sec  tan 2  )  cos 
 sec 2   tan 2 

Alternatively,
Differentiating A  tan  sec implicitly with
respect to time t,

Page 7 of 25
CJC P2 Solutions 113
3 Topic: Application of Differentiation
Solution
dA
dt   
  sec 2  sec  tan   sec  tan     ddt

 (sec3   sec  tan 2  )  cos 
 sec2   tan 2 

When    , sec  1 2 , 1
6 cos 
 tan  
3 3
 dA
dt
 sec 2   tan 2   4
3
 1
3
 35 ,
rate of change of the
area of OPN when    .
6

sgexamguru.com

Page 8 of 25
CJC P2 Solutions 114
4 APGP
Solution
(i) 15
S15 of B   2(2.4)  14d 
2
 36  105d
(ii)
S15 of A 
 15
2.4 1.2   1 
1.2  1
 172.88
 170
Yes, Adam can achieve his target.
(iii) U15 of A > U15 of B
U15 of A  U15 of B > 0
2.4(1.2)14  (2.4  14d )  0
d  2.02957

14 sgexamguru.com
max d  2.02(2 dp)
2.4(1.2)  (2.4  14(2.02))  0.134 (shown)
(iv)  r 
13

2.4  1   1 
  100 
New S13 of A     170
 r 
1   1
 100 
From GC,

Page 9 of 25
CJC P2 Solutions 115
4 APGP
Solution

x = 25.4%

sgexamguru.com

Page 10 of 25
CJC P2 Solutions 116
Section B: Statistics [60 marks]

5 Topic: Probability
Solution
(i) P(at least one blue ball)
= 1 – P(no blue balls)
= 1 – P(3 red balls)
4
C3
= 1 - 12
C3
4
=1-
220
54
 (Shown)
55
Alternative Method
P(at least one blue ball)
= 1 – P(3 red balls)
4 3 2

sgexamguru.com
 1  
12 11 10
54
 (Shown)
55

Alternative Method (Direct Method)


P(at least one blue ball)
= P(1 blue and 2 red) + P(2 blue and 1 red) + P(3
blue)
8
C1 4 C2 8 C 2 4 C1 8 C3
= 12  12  12
C3 C3 C3
48 112 56
=  
220 220 220
54
 (Shown)
55

Page 11 of 25
CJC P2 Solutions 117
5 Topic: Probability
Solution
Alternative Method
P(at least one blue ball)
= P(1 blue and 2 red) + P(2 blue and 1 red) + P(3
blue)
 8 4 3 3!   8 7 4 3!   8 7 6 
           
 12 11 10 2!   12 11 10 2!   12 11 10 
48 112 56
=  
220 220 220
54
 (Shown)
55

(ii) P(at least one of each colour drawn)


= 1 – P(3 red) – P(3 blue)
4
C3 8C3
= 1 – 12 – 12

sgexamguru.com
C3 C3
4 56
=1– –
220 220
8
= or  0.727
11
Alternative Method
P(at least one of each colour drawn)
= P(1 blue and 2 red) + P(2 blue and 1 red)
8
C1 4 C2 8 C 2 4 C1
 12  12
C3 C3
48 112
 
220 220
8
= or  0.727
11

1 (iii) There are 8 balls with “0” and 4 balls with “1”
P(sum is at least two)
= P(1, 1, 0) + P(1, 1, 1)
Page 12 of 25
CJC P2 Solutions 118
5 Topic: Probability
Solution
4
C 8 C1 4 C3
 122  12
C3 C3
48 4
 
220 220
13
= or  0.236
55

Alternative Method
P(sum is at least two)
= P(1, 1, 0) + P(1, 1, 1)
 4 3 8 3!   4 3 2 
       
 12 11 10 2!   12 11 10 
13
= or  0.236
55

sgexamguru.com

Page 13 of 25
CJC P2 Solutions 119
6 Topic: Binomial Distribution
Solution
(i) X ~ B  60,0.03
Since n  60  50 is large, p  0.03  0.1 is
small and np  1.8  5,
X ~ Po 1.8 approximately
X1  X 2 ~ Po  3.6 approximately
P  X1  X 2  6  1 P  X1  X 2  6
 0.073273
 0.0733 (3s.f.)
(ii) Since sample size = 50 is large,
 1.746 
X ~ N 1.8,  approximately by Central
 50 
Limit Theorem

sgexamguru.com
P  X  2   0.14225
 0.142 (3s.f.)

Page 14 of 25
CJC P2 Solutions 120
7 Topic: Permutation and Combination
Solution
(i) No. of ways with no restriction
= (no. of ways to separate 9 people into a group of 4
and a group of 5)
× (no. of ways to arrange the row of 4 people)
× (no. of ways to arrange the circle of 5 people)
= 9C4 × 4! × 5C5 × (5 – 1)!
= 72 576

Alternative Method
No. of ways with no restriction
= (no. of ways to separate 9 people into a group of 5
and a group of 4)
× (no. of ways to arrange the circle of 5 people)
× (no. of ways to arrange the row of 4 people)
= 9C5 × (5 – 1)! × 4C4 × 4! [Note: 9C4 = 9C5]

sgexamguru.com
= 72 576
(ii) No. of ways if Albert and Berta sit together
= (Albert and Berta in the row)
+ (Albert and Berta in the circle)
= (no. of ways to pick 2 remaining people for the
row
× arrange row people × A and B swap × arrange
circle people)
+ (no. of ways to pick 3 remaining people for the
circle × arrange circle people × A and B swap ×
arrange row people)
= (7C2 × 3! × 2 × (5 – 1)! ) + (7C3 × (4 – 1)! × 2! ×
4! )

= 6048 + 10080
= 16128

(iii) No. of ways if Albert and Berta both sit on the


couch or both sit at the table
Page 15 of 25
CJC P2 Solutions 121
7 Topic: Permutation and Combination
Solution
= (Albert and Berta in the row) + (Albert and Berta
in the circle)
= (no. of ways to pick 2 remaining people for the
row × arrange row people × arrange circle people)
+ (no. of ways to pick 3 remaining people for the
circle × arrange row people × arrange circle people)
(7C2 × 4! × (5 – 1)! ) + (7C3 × 4! × (5 – 1)!)
= 32256
No. of ways if Albert and Berta sit separately
= 32256 – 16128
= 16128

Alternative Method
No. of ways if Albert and Berta sit separately
= (Albert and Berta in the row) + (Albert and Berta

sgexamguru.com
in the circle)
= (no. of ways to pick 2 remaining people for the
row × arrangement in row × arrangement in circle)
+ (no. of ways to pick 3 remaining people for the
circle × arrangement in circle × arrangement in
row)
= (no. of ways to pick 2 remaining people for the
row × [arrange remaining 2 people in row × slot in
Albert and Berta] × arrangement in circle)
+ (no. of ways to pick 3 remaining people for the
circle × [arrange remaining 3 people in circle × slot
in Albert and Bert] × arrangement in row)
= (7C2 × [ 2! × 3C2 × 2! ] × (5 – 1)! ) + (7C3 × [ (3 –
1)! × 3C2 × 2! ) × 4! )
= 10080 + 6048
= 16128

Page 16 of 25
CJC P2 Solutions 122
8 Topic: Correlation and Regression
Solution
(i) GC screenshot :

y, Cost of Scatter diagram of y against x


production /
$1,000
(in 81
sgexamguru.com
P

10
x, Quantity
2.0 9.4 produced
/ 1,000

(ii) P = (6.0, 35).


(iii) The better model is y = a + b ln x since the set of
data points in the scatter diagram exhibits a non-
linear trend in which y increases at a decreasing rate
as x increases, rather than a linear trend in which y
increases at a constant rate with x.

Page 17 of 25
CJC P2 Solutions 123
8 Topic: Correlation and Regression
Solution
(iv)

sgexamguru.com
r(ln x ), y  0.991 .
Line of regression y on ln x.
y = (47.73168413…) ln x + (–19.93984362…)
 y = 47.7 ln x – 19.9

If a fixed cost is included in y, the value of


r(ln x ), y  0.991 is unchanged, as there is no change
in the strength of the linear relationship between (ln
x) and y, under translation.

However, as a constant value M is added to the y


value of each data point, the regression line would
be also translated in the direction of the positive y-
axis by M units,
i.e. with resultant regression line equation being
Page 18 of 25
CJC P2 Solutions 124
8 Topic: Correlation and Regression
Solution
y = 47.7 ln x – 19.9 + M

or

y – M = 47.7 ln x – 19.9
(v) When x = 6,
y = (47.73168413…) ln(6) – 19.93984362…
= 65.5838534…
 65.6

Hence, the
estimated cost of production is $ 65,600 (to 3 s.f).

This value is reliable, since r = 0.991 is close to 1


indicating a strong positive linear relationship

sgexamguru.com
between y and ln x, and estimating y at x = 6 is an
interpolation as x = 6 is within the range of values
of x in the data used to construct the regression line
(i.e. 2.0 ≤ x ≤ 9.4).

Page 19 of 25
CJC P2 Solutions 125
9 Topic: Normal Distribution
Solution
(i) Let X minutes be the random variable denoting the
finishing time of a randomly selected runner in the
race.
X ~ N   , 2 
.
 40   
P  X  40  0.1  P  Z   0.1
  
40  
  1.2816

   1.2816  40 (1)
 60   
P  X  60  0.35  P  Z   0.65
  
60  

sgexamguru.com
  0.38532

   0.38532  60 (2)

Solving (1) and (2),   55.377  55.4(3s.f.) and


  11.998  12.0(3s.f.)
(ii) For P  X  a   0.2  a  45.300  45.3(3s.f.)
Maximal timing is 45.3 minutes.
[Accept a  45.279 for 5s.f. intermediate ]
(iii) Let Y be the number of runners, out of 12, who
receive a medal.
Y ~ B 12,0.2
P Y  4  1  P Y  4  0.072555  0.0726
(iv) P  slowest runner finishes within 1 hour | all do not receive medal

Page 20 of 25
CJC P2 Solutions 126
9 Topic: Normal Distribution
Solution
P  slowest runner finishes within 1 hour and all do not receive medal

P  all do not receive medal 

P  all runners finish within 1 hour and do not receive medal


 12
1  0.2 

P  45.279  X 1  60 and 45.279  X 2  60 and ... and 45.279  X 12 


 12
1  0.2 

12


0.65  0.2
12
 0.8
12
 0.45 
 
 0.8 
sgexamguru.com
 0.0010034  0.00100(3s. f .)

Method 2:

Let A be the r.v. denoting the number of runners who


finish in an hour and do not receive medal, out of 12.
A ~ B 12,0.65  0.2
A ~ B 12,0.45

P( A  12)
Required probability 
P(Y  0)

Page 21 of 25
CJC P2 Solutions 127
10 Topic: Sampling Methods Hypothesis Testing
Solution
a(i) This method is quota sampling and is not
representative of the student population as there is
no consideration of the size of each sports team
and each club and society relative to the student
population, and the student from each group is
selected non-randomly i.e. only Captains and
Presidents selected.
a(ii) Stratified sampling.
Divide school population by year/level (strata),
calculate the proportion of each strata relative to
the population and select the respective number
randomly.

b(i) Let X cm be the vertical jump height of a

sgexamguru.com
randomly selected volleyball player.
H 0 :   40 vs H1 :   40

b(ii) X 
Under H 0 , test statistics: T  ~ t  n  1
s/ n
7
Where   40, x  42.1, n  7, s 2  k 2
6
To reject H 0 , p-value < 0.1
 P  T  t   0.1
 42.1  40 
 P T    0.1
 7/6  / 7 
 2.1 
 P T    0.9
 k/ 6
2.1
  1.4398
k/ 6
Page 22 of 25
CJC P2 Solutions 128
10 Topic: Sampling Methods Hypothesis Testing
Solution
2.1 6
k   3.5728
1.4398
Required set is k   : 0  k  3.57 or  0,3.57 
b(iii) Since k 2  15  k  15  3.57, we do not reject
H0 at 10% level of significance and conclude that
there is insufficient evidence to support the claim
that the training regime is effective.

sgexamguru.com

Page 23 of 25
CJC P2 Solutions 129
11 Topic: Poisson Distribution
Solution
(i) Any two of the following 4 assumptions: 1.
(1) The calls are made independently of each other.
(2) The probability of receiving two or more calls
within a very short interval of time is negligible.
OR Calls are received one at a time.
(3) The average number of calls received over any
time interval of the same duration within the day
is constant.
(4) The calls received occur randomly.

(ii) Let Wn , Xn and Yn be the random variables


denoting the number of calls received in n hours
pertaining to credit card queries, business banking
queries and personal banking queries respectively.

sgexamguru.com
Then Wn ~ Po  n  , X n ~ Po  6n
and Yn ~ Po  7n
8P W4  2  P W1  2 where W1 ~ Po    and W4 ~ Po 4

2
8e4   4  e   2
   e 3   1 28
2! 2!
1 7
 ln 128  ln 2
3 3
(iii) Since W2, X2, and Y2 are independent,
P W2  0 and W2  X 2  Y2  50
P W2  0 | W2  X 2  Y2  50  
P W2  X 2  Y2 
P W2  0 and X 2  Y2  50

1  P W2  X 2  Y2  50

Page 24 of 25
CJC P2 Solutions 130
11 Topic: Poisson Distribution
Solution
 14 
where W2 ~ Po  ln 2  and
3 
 14 
W2  X 2  Y2 ~ Po  ln 2  26  ,
3 
X2  Y2 ~ Po  26

P W 2  0   1  P  X 2  Y2  50  

1  P W 2  X 2  Y2  50 
 0.0022359
 0.00224 (3s.f.)

(iv) X 2 ~ Po 12 and Y2 ~ Po 14 .

sgexamguru.com
Since 1 12 10 and 2  14  10,
X2 ~ N12,12 approximately
and Y2 ~ N14,14 approximately

Since X2 and Y2 are independent,


X 2  Y2 ~ N  2, 26  approximately
P X2  Y2   P  X 2  Y2  0
 P  X 2  Y2  0.5 by continuity
correction
 0.31196
 0.312 (3s.f.)

Page 25 of 25
CJC P2 Solutions 131
Nam
me: Index Number: Class:

DUNMAN
D N HIGH SCHOO OL
Prelimina
ary Exa
aminatio
on
Y
Year 6

MATHE
M EMATICS (Higher 2
2) 9740/01
9
Paper
P 1 14 Septem
mber 2016
6
3 hours
Additional
A Materials: An
nswer Pap per
List of Form ulae (MF15)

READ
R THE
ESE INSTRU
UCTIONS FIRST
F

Write
W your NName, Inde ex Number anda Class o on all the wo
ork you han
nd in.
Write
W in darrk blue or bllack pen on
n both sidess of the pape
er.
Y may usse a soft pencil for any diagrams o
You or graphs.
D not use staples, paper clips, highlighters, glue or corrrection fluid
Do d.

Answer
A
Give
G
sgexamguru.com
all tthe questions.
non-exxact numerrical answerrs correct to ant figures, or 1 decim
o 3 significa mal place in the case ofo
angles
a in de
egrees, unle ess a differeent level of accuracy iss specified in the questtion.
You
Y are exp pected to usse an appro oved graphi ng calculato or.
Unsupporte
U ed answers from a gra aphing calc ulator are allowed
a unless a quesstion speciffically statess
otherwise.
o
Where
W unsupported answers
a from a graph hing calcula ator are nott allowed inn a questio
on, you are e
required
r to
o present thet mathem matical steeps using mathematic cal notationns and no ot calculator
commands.
c .
You
Y are rem minded of th he need for clear prese entation in your
y answerrs.

At
A the end o
of the exam
mination, fas
sten all yourr work securely togethe
er.
The
T number of marks is i given in brackets
b [ ] at the end of
o each queestion or parrt question.

For
F teache
ers’ use:
Qn Q1 Q
Q2 Q3 Q4 Q
Q5 Q6 Q7 Q8
8 Q9 Q
Q10 Q11 Total
Score
Max
6 6 7 7 8 9 10 10 11 13 13 100
Score

© DHS 2016 This que


estion pape
er consists o
of 6 printed pages (inclluding this ccover page)).

Microsoft Word - DHS_JC2_H2_Maths_2016_Prelim_Paper_1_Questions 132


2

 f ( x )
n 1

Given that  f '( x)  f ( x) dx   c wh


n
1 (i) G a arbitraryy constant and n  1,
here c is an 1
n 1

 x  (4  x ) dx.
2
find [22]

(ii) H
Hence find the exact volume off revolution when th
he region bbounded by
y the curvve
3 1
y x 2 (4  2 4
x ) , nes x  0, x  2 and y  3, is rotatted complettely about th
the lin he x-axis. [44]

2 The coomplex num mber w is such that kkw2  kww*  iw  iw*  1  0 , whhere w* is the
t compleex
conjuggate of w annd k is a reall and non-zeero constantt.

(i) For w  a  bi where a and b arre real num mbers, obtaain an exprression for b in term
ms
off a and k. Explain
E why
y w is either ppurely real or purely im
maginary. [44]

(ii) U
Using your result in part (i), or otherw
wise, find the real rroots of th
he equatioon
2 w2  2ww*  iw  iw*  1  0. [22]

3 (i) W
Without usinng a calculattor, find thee exact soluttion of the inequality
i

4
4x . [44]
x2

(ii) H
sgexamguru.com
Hence solve 5  x 
4
. [33]
x 1

4 c
P Q

w
S R

To traavel along the River Nile, an adventurer decides to o use a logg with a semi-circula
s ar
cross--section of constant diameter c mmetres to bu
uild a boat. The log iss trimmed such
s that thhe
uniforrm cross-seection of th
he boat is an isoscelees trapezium
m with basse width w metres annd
PS  QR, as show wn in the diiagram abovve.

(i) Shhow that thhe cross-secttional area oof the boat A metres2 iss given by
3 1
A  14 (c  w) 2 (c  w) 2 . [22]

(ii) Find the valuue of w, in


n terms of cc, that givess the station
nary value of A. Hencce determinne
w
whether this stationary value
v is a m
maximum orr a minimum m. [55]

DHS 2016 Ye
ear 6 H2 Math
hematics Preliminary Examination Paper 1

DHS P1 Questions 133


3

5 Given that y  ln(1  tan x),

2
d 2 y  dy  dy
(i) show that 2     2(1  e y )  0, [3]
d x  dx  dx

(ii) find the Maclaurin series for y up to and including the term in x 3 , given that the value of
d3 y
when x  0 is 4. [2]
dx 3

sec 2 x
Hence find the first three terms in the series expansion of . [3]
1  tan x

1
6 (a) Use the substitution x  e t to find
 2e  e
t t
dt. [3]

4 x
(b) (i) Express in partial fractions. [2]
(1  x)(4  x 2 )

4 x  f ( n) 

n
(ii) Evaluate dx, giving your answer in the form 1
2 ln  2
,
2 (1  x )(4  x 2 )  8(n  1) 

sgexamguru.com
where f (n) is a function of n. [2]

4 x
The curve C has equation y  . The diagram below shows the part of C for
(1  x)(4  x 2 )
which x  1. y

x
O

x=1
Find the exact value of the area of the region between C and the positive x-axis for x  2.
[2]

DHS 2016 Year 6 H2 Mathematics Preliminary Examination Paper 1

DHS P1 Questions 134


4

7 A curve C has parametric equations


x , y  sin 1  , for  1    1.
 1  
2
)

dy
(i) Show that  1   2 . What can be said about the tangents to C as   1? [4]
dx

(ii) Sketch C, showing clearly its axial intercept and asymptotes. [2]

(iii) Find the equation of the tangent at the point where C has maximum gradient.
By considering the intersection between C and an appropriate graph, find the set of positive
kx
values of k for which the equation sin 1 x   0 has at most one real root. [4]
 1  x )
2

8 A sequence of real numbers u0 , u1 , u2 ,  satisfy the recurrence relation

 n 
un  un 1  ln  
 n 1

for n  1 and u0  2 .

sgexamguru.com
(i) Use the method of mathematical induction to prove that un  2  ln(n  1) for n  0 . [4]

(ii) By considering ur  ur 1 , show how the result for un in part (i) can be obtained using the
method of differences. [4]

N
(iii) Show that u
n0
n  ( N  1)(2  ln( N  1)). [2]

DHS 2016 Year 6 H2 Mathematics Preliminary Examination Paper 1

DHS P1 Questions 135


5

9 Joseph started a marathon race. After a while, his trainer, Sarah, starts to collect data on
Joseph’s speed and she realises that the rate of change of Joseph’s speed is proportional to the
difference between his speed and a constant a. If the speed of Joseph at time t hours after the
du
start of collection of data is u kilometres per hour, it is found that  1 when u  14.5 and
dt
du
 2 when u  14.
dt

du
(i) Show that  2(u  15). [3]
dt

(ii) Find the general solution of the equation in part (i), expressing u in terms of t. [3]

(iii) Deduce the steady speed of Joseph eventually. [1]

The distance covered by Joseph, s kilometres, at time t hours after the start of collection of data
can be modelled by

ds
 u.
dt

(iv) Find s in terms of t. [2]

sgexamguru.com
(v) The result in part (iv) can be represented by a family of solution curves. Sketch an
appropriate non-linear member of the family of curves that has a linear asymptote that
passes through the origin. [2]

x2  5
10 A curve C has equation y  .
( x  1)2  12

(i) Determine the equations of the three asymptotes of C, giving each answer in an exact form.
[2]

(ii) Prove algebraically that there are no values of x for which 1


2  y  65 . [3]

For parts (iii) and (iv), you do not need to label the point where the graph cuts the y-axis.

(iii) Sketch C. [3]

( x  1) 2  12
(iv) Sketch the graph of y  . [3]
x2  5

(v) Describe a sequence of two transformations which transform C to the graph of


( x  1)2  5
y . [2]
( x  2)2  12

DHS 2016 Year 6 H2 Mathematics Preliminary Examination Paper 1

DHS P1 Questions 136


6

11 The diagram below shows a tetrahedron ABCD. The equation of the plane ABD is
4 x  y  2 z  16.
D

B
C A

(i) Given that A is on the x-axis, find the coordinates of A. [1]

The equation of the plane CBD is 7 x  11 y  5 z  23.

(ii) Find a vector equation of the line that passes through B and D. [2]

(iii) Given that B is on the xy-plane, find the coordinates of B. [2]

4 x y z
The cartesian equation of the line that passes through A and D is   .
2 2 3

(iv) Find the coordinates of D. [3]

The coordinates of C are (−1, 1, 1).

sgexamguru.com
(v) By considering the area of triangle ABC, find the exact volume of the tetrahedron ABCD.
[5]

[Volume of tetrahedron = 13  area of base  perpendicular height ]

DHS 2016 Year 6 H2 Mathematics Preliminary Examination Paper 1

DHS P1 Questions 137


2016 Year 6 Prelim Paper 1 Suggested Solutions
Qn Suggested Solution
1(i) 1
 x  (4  x 2 ) dx  
2  (2 x)  (4  x 2 ) dx

1 2 2 2 
3
   (4  x )   C
2 3 
3
1
  (4  x 2 ) 2  C
3
y
y=3

x
O x=2
Volume required
2
 3 

2 1

sgexamguru.com
 π(3 )(2)  π
2
 x 2 (4  x 2 ) 4
 dx
0 
1
dv

2 1
ux 2
 x(4  x 2 ) 2
 18π  π x3 (4  x ) dx
2 2
dx
0 du
 2x 1
3

  v   (4  x )
2 2


2 1 dx
 18π  π x 2  x(4  x )  dx
2 2
3
0  
 18π
 3 2
 

2 3
 21 1 
 π    x (4  x )  
2 2
 (4  x 2 ) 2 (2 x)dx 
  3  0 0 3 
2
π 2   f ( x) 
5 n 1

 18π   (4  x 2 ) 2  Use f '( x) f ( x) 


 dx  c
n

3  5  0 n 1
π 2 2
5
 18π    (4) 
3  5 
64
 18π  π
15
206
 π
15

DHS P1 Solutions 138


Qn Suggested Solution
2(i) kw2  kww*  iw  iw*  1  0
kw( w  w* )  i( w  w* )  1  0
k (a  bi)(2a)  i(2bi)  1  0

 2ka2  2b   2abki  1    ()


Real part
2ka 2  1
2ka 2  2b  1  b  ---(1)
2

Im part
ab  0 k 0
 b  0 or a  0
ie, w is either purely real or imaginary.

b(ii) Hence
Since w is real, b  0.
Using k  2 and b  0
From part (i):
2(2)a 2  1
2
0
sgexamguru.com
1
4a 2  1  a  
4
1 1
ie, w   or w 
2 2
Otherwise
Since w is real, b  0, ie, w  a
Using k  2 and w  a
eqn becomes:
2 a 2  2 a 2  ia  ia  1  0
1
4a 2  1  a  
4
1 1
ie, w   or w 
2 2

DHS P1 Solutions 139


Qn Suggested Solution
3 (i) 4
4 x
x2
4
 x40
x2
4  ( x  2)  x  4 
0
x2
x2  2 x  4
0
x2
( x  1) 2  5 a 2  b2  (a  b)(a  b)
0
( x  2)
 x  1     x  1     0
x2
– + – +

 x  2 or 1     x  1   

sgexamguru.com

DHS P1 Solutions 140


(ii) 4
4 x 
x2
4   x  1 
4
Since 2  5  0, |x| is
 x  1  2 always greater than
4 2  5 for all real x
5 x 
x 1
Hence, replace x with  x  1 in earlier sol:
 x  1  2 or 1     x  1  1   
 x  1 or 2     x  2   
No solution as x  0 2  x and x  2   
for all real x x as x  0 and  (2    )  x  2   
i.e.  2     x  2   
y
y = |x|

O
x

y = −1

sgexamguru.com
Qn Suggested Solution
4(i)

Height of cross-section of boat = 1 c2


4  14 w2
A 1
2
1 c2
4  14 w2  c  w 

 1
4 c 2  w2  c  w 
 1
4  c  w  c  w   c  w 
1
 (c  w) 2 (c  w) 2  c  w 
1 1

4
1 3 1
 (c  w) 2 (c  w) 2 (shown)
4

DHS P1 Solutions 141


  (c  w) (c  w) 2  14  12  (c  w) 2 (c  w)
(ii) dA 1 1 3 1
 1 3
4 2
2 2
dw
1 3
3(c  w) 2 (c  w)  (c  w) 2
 1
8(c  w) 2
1
(c  w) 2 [3(c  w)  (c  w)]
 1
8(c  w) 2
1
(c  w) 2 [c  2w]
 1
4(c  w) 2
dA
For stationary A  0
dw
1
1 (c  w) 2 (c  2 w) 0
8
w  c (reject as w > 0) or w  12 c

Alternative
1
A2  (c  w)3 (c  w)
16
dA 1
2A  3(c  w) 2 (c  w)  (c  w)3 
dw 16
 16
1 (c  w) 2 (3c  3w  c  w)

dA
 16 sgexamguru.com
1 (c  w) 2 (2c  4 w)

A  16
1 (c  w) 2 (c  2 w)
dw
dA
For stationary A  0
dw
w  c (reject as w > 0) or w  12 c

1 c  ,say 1 c  ,say
2 1c 2
w 2
0.4c 0.6c
dA
+ve 0 +ve
dw
Tangent / – \
c
When w  , A is maximum.
2

DHS P1 Solutions 142


Qn Suggested Solution
5(i) Method 1
y  ln(1  tan x)
e y  1  tan x      (1)
Differentiate wrt x,
dy
ey  sec 2 x      (2)
dx
Differentiate wrt x,
2
d2 y  dy 
ey
 e y    2sec x (sec x tan x) [from (1), tan x  e y  1]
 dx 
2
dx
2
d2 y  dy  dy dy
ey
 e y    2e y (e y  1) [from (2), sec 2 x  e y ]
 dx 
2
dx dx dx
2
d 2 y  dy  dy
 2     2 (1  e y )  0
dx  dx  dx
 e y  0 (shown)

Method 2 (Discouraged)
y  ln(1  tan x)
dy sec2 x

dx 1  tan x
d 2 y (1  tan x)2sec 2 x tan x  (sec 2 x)(sec 2 x)
dx 2

sgexamguru.com
(1  tan x) 2
2sec x tan x  sec 2 x 
2 2

  
1  tan x  1  tan x 
2
 dy   dy 
 2   tan x   
 dx   dx 
2
 dy  y  dy 
2
d y
 2  2   (e  1)    [ e y  1  tan x]
dx  dx   dx 
2
d 2 y  dy   dy 
 2     2   (1  e y )  0 (shown)
dx  dx   dx 
(ii) When x  0, y  ln(1  tan 0)  0
dy dy
e0  sec2 0  1
dx dx
d2 y d2 y
        1
2 0
1 2(1)(1 e ) 0
dx 2 dx 2
d3 y
Given that 4,
dx3
x2 x3 1 2
y  x   1  (4)  ...  x  x 2  x3  ...
2! 3! 2 3

DHS P1 Solutions 143


d d 1 2
ln(1  tan x)  ( x  x 2  x3  ...)
dx dx 2 3
2
sec x
  1  x  2 x 2  ...
1  tan x

Replace x with (-x),


sec 2   x 
 1    x   2   x   ...
2

1  tan   x 
sec 2  x 
  1  x  2 x 2  ...
1  tan  x 
( tan( x)   tan x and cos( x)  cos( x))

Alternative
Replace x with (-x),

ln(1  tan x)  ln(1  tan(  x))


1 2
  x  ( x) 2  ( x)3  ...
2 3
1 2
  x  x 2  x3  ...
2 3

d
dx
sgexamguru.com
d 1 2
ln(1  tan x)  (  x  x 2  x3  ...)
dx 2 3
 sec x
2
 1  x  2 x 2  ...
1  tan x
sec 2 x
 1  x  2 x 2  ...
1  tan x

DHS P1 Solutions 144


Qn Suggested Solution

 x  et
6(a) 1
dt
2e  et
t
dx t
e
1 1


2x 
1  x 
dx dt
1
"dt  dx "
x x


1
 dx
2x 12


1 1
 2
dx
2  1 
x 
2

2
2
 tan 1   2 x   C
2
2
 tan 1   2et   C
2
(b) 4 x A Bx  C
Let   .
(1  x)(4  x ) 2 1  x 4  x2
Then
A(4  x 2 )  ( Bx  C )(1  x)  4  x
x  1: A  1
x2 : A  B  0 B  1

4 x
sgexamguru.com
constant: 4 A  C  4  C  0


1

x
.
(1  x)(4  x ) 2 1  x 4  x2
n 4 x
2 (1  x)(4  x2 ) dx
n 
2  1  x  4  x2  dx
1 x

n


1 
   ln |1  x |  ln 4  x 2 
2 2
 
1
2
1
  ln |1  n |  ln 4  n 2  ln 8
2

1
2
1
2
1
  ln |1  n |2  ln 4  n 2  ln 8
2
 
1  4n  2
 ln  
2  8(n  1) 2 

DHS P1 Solutions 145


The required area A
 1  4  n2 
 lim   ln  
n   2  8( n  1) 2  
  
 1  4  n2 
 lim   ln  
n   2  8( n 2  2n  1)  
  
  4 
 1  2
 1 
lim   ln  n 
n   2  2 1 
  8(1   2 )  
 n n 
1 1 1 1
  ln   [ since and 2  0 as n  ]
2 8 n n
3
 ln2
2

Alternative
The required area A
 1  4  n2 
 lim   ln  
n   2  8( n  1) 2  
  
  4 
 1  2  1 
 lim   ln  n 

sgexamguru.com
n   2  1 2 
  8(1  )  
 n 
1 1 1 4
  ln   [ since and 2  0 as n  ]
2 8 n n
3
 ln2
2

DHS P1 Solutions 146


Qn Suggested Solution
7(i) 
x
1 2
 12
dx (1   2 ) 2   ( 12 )(2 )(1   2 )
1

 
d (1   2 )

(1   2 ) 2 (1   2 )   2 
1


(1   2 )
 32
 (1   2 )
y  sin 1 
dy 1 1
   (1   2 ) 2
d 1 2
1
dy (1   2 ) 2

dx (1   2 )  2
3

 1 2 (shown)
dy
As   1,  0.
dx
The tangents becomes parallel to the x-axis as   1.
(ii) y

sgexamguru.com x
O

Note: as   1, x   and y   12 π.


(iii) dy
Since  2  0,  1   2 is maximum at   0.
dx

Alternative
dy
Let g( )   1 2.
dx
Set g ( )  2  0, so   0.
'

dy
Then g'' ( )  2  0, and thus  1   2 is maximum at   0.
dx

dy
At   0,  1, x  y  0.
dx
Equation of tangent at (0, 0): y  x.

Replacing x with  , and rearranging the given equation, we get

10

DHS P1 Solutions 147



sin 1   k . Since the curve C is described by
 (1   )
2


C:x , y  sin 1  ,
 (1   )
2

the number of real roots of the given equation is equal to the number of intersection
points between the line y  kx and the curve C.

Since the maximum gradient value of C is 1, which is attained at O, thus the required
 
set of positive constants k is given by k   : k  1 , in order for the given equation
to have at most(exactly) one real root.
y

y  kx : k  1 y  kx : 0  k  1.
x
O

Qn Suggested Solution
Let P(n) be the proposition un  2  ln(n  1) for n  0 .
sgexamguru.com
8(i)

When n = 0,
LHS of P(0) = u0  2 (given)
RHS of P(0) = 2  ln(0  1)  2 = LHS of P(0)
 P(0) is true.

Assuming that P(k) is true for some k  0 i.e. uk  2  ln(k  1) , To show that P(k + 1)
is true i.e. uk 1  2  ln(k  2) .

LHS of P(k  1)
 uk 1
 k 1 
 uk  ln  
k 2
 k 1 
 2  ln(k  1)  ln  
k2
 2  ln(k  1)  ln(k  1)  ln(k  2)
 2  ln(k  2)
 RHS
 P(k ) is true  P(k  1) is true.

Since P(0) is true, and P(k ) is true  P(k  1) is true, hence by mathematical
induction, P(n) is true for all n  0.
11

DHS P1 Solutions 148


(ii) Consider
r
ur  ur 1  ln  ln(r )  ln(r  1)
r 1
n n

 u
r 1
r  ur 1     ln(r )  ln(r  1) 
r 1

[ u1  u0  [ln(1)  ln(2)
+ u2  u1  ln(2)  ln(3)
+ u3  u2  ln(3)  ln(4)

+ un 1  un  2  ln(n  1)  ln(n)
+ un  un 1 ]  ln(n)  ln(n  1)]

un  u0  ln(1)  ln(n  1)
un  2  ln(n  1)

Alternative for RHS


n
 r 
  ln
r 1 

1
r 1
2

3
sgexamguru.com
 ln  ln  ln  ...  ln
n
2 3 4 n 1
1 2 3 n 
 ln  ... 
 2 3 4 n 1 
1
 ln
n 1
  ln (n  1)

(iv) N N

 u   2  ln(n  1)
n 0
n
n 0

 ( N  1)(2)  [ln1  ln 2  ln( N  1)]


 ( N  1)(2)  [ln( N  1)  ln( N  1)  ln( N  1)]
 ( N  1)(2)  ( N  1) ln( N  1)
 ( N  1)[2  ln( N  1)]

12

DHS P1 Solutions 149


Qn Suggested Solution
9i) du
 k (u  a) , where k is a constant.
dt
du du
Given  1 when u  14.5 and  2 when u  14,
dt dt
1  k (14.5  a) -----(1)
2  k (14  a) ------(2)
From GC,
k  2 and ak  30
 a  15
du
  2(u  15) (shown)
dt
v(ii) 1
 u  15 du  2 dt
ln | u  15 |  2t  C
| u  15 | e2t  C
u  15  Ae2t where A  eC

2t
(iii) As t  , e  0, u  15
Joseph will eventually reach a steady speed of 15 km/h.

sgexamguru.com
Alternatively, at steady speed,
du
dt
 0, u  15
(iv) s   u dt
  (15  Ae 2t ) dt
A 2t
 15t  e D
2

9(v) For graph to tends towards an asymptote that passes through the origin, D = 0.
A
I.e. s  15t  e2t
2
1
For A = −1, s  15t  e2t
2
s
s = 15t
A = −1

t
O

13

DHS P1 Solutions 150


Qn Suggested Solution
10(i) ( x  1)2  12  0  x  1  12
Asymptotes are: y  1, x  1  12, x  1  12
(ii) x2  5 x2  5
y 
( x  1)2  12 x 2  2 x  11
y ( x 2  2 x  11)  x 2  5
( y  1) x 2  2 yx  5  11y  0
For no values of x, there are no real solutions for the above quadratic equation.
Discriminant  4 y 2  4( y  1)(11 y  5)  0
12 y 2  16 y  5  0
(6 y  5)(2 y  1)  0
 12  y  65 (shown)
(iii) At turning points of C,
When y  12 , x  1;
When y  56 , x  5 .
Coordinates of turning points (1, 12 ) and (5, 65 ).
y
C

sgexamguru.com
y=1
×
×
x
O

(iv) y

×
×
y=1
x
O

14

DHS P1 Solutions 151


(v) (1  x)2  5 ( x  1)2  5
y  .
((1  x)  1)2  12 ( x  2)2  12
Therefore x is replaced with 1 – x.

y  f ( x)  y  f ( x  1)  y  f (( x)  1)  f (1  x)
Sequence of transformation:
C is translated by 1 unit in the negative x-direction and then reflected in the y-axis

OR y  f ( x)  y  f ( x)  y  f (( x 1))  f (1  x)
C is reflected in the y-axis and then translated 1 unit in the positive x-direction.
Qn Suggested Solution
11(i) Plane ABD: 4 x  y  2 z  16
When A is on the x-axis, y = z = 0.
4 x  16  x  4
A(4, 0, 0)

(ii) Plane CBD: 7 x  11y  5z  23


Line BD is the line of intersection between planes ABD and CBD.
From GC,
 3   13 
r   4    '   23 
0  1 
   
 3 1

  sgexamguru.com
lBD : r   4     2  ,  
0  3 
 

(iii) 0
Equation of xy-plane : r   0   0  z  0
1
 
 3 1
Using OB   4     2   3  0    0
0  3 
   
 3
 OB   4 
0
 
B(3, 4, 0)

Alternative
B is the point of intersection between planes ABD, CBD and xy-plane
4 x  y  2 z  16
7 x  11y  5z  23
z0

Using GC, B(3, 4, 0)

(iv)  4  2 
l AD : r   0     2 
0  3
   
D is the point of intersection between lAD and plane CBD :
15

DHS P1 Solutions 152


 4   2    7 
 0     2     11  23
 0   3    5 
   
28   (51)  23
 1
 4   2   2 
OD   0    2    2 
 0  3   3
     
D(2, 2, 3)

Alternative
D is the point of intersection between lAD and lBD.
 4  2   3  1
 0    2    4    2 
0  3  0  3 
       
  2  1
2  2   4
3  3  0     
 1
(v)  1
AB  OB  OA   4 
0
 

sgexamguru.com
AC  OC  OA   1 
 5 
1
 
Area of triangle ABC =
1  1  5  1  4 
2 AB  AC 
1
 4  1    1 
2  0   1  2 19 

Distance from D to plane ABC


 4   2   4 
AD   1   2    1 
19   3  19 
      51
  
4 4 4
1 1 1
19  19  19 
     

Alternative
4
 Plane ABC : r   1   16
19 
 

16

DHS P1 Solutions 153


 2  4 
16   2    1 
 3  19 
    16  67 51
Distance from D to plane ABC   
4 4 4
1 1 1
19  19  19 
     

1 1  4  51 51 17
Volume of tetrahedron OABC =  1  = units3
3 2 19   4  6 2
 
1
19 
 

sgexamguru.com

17

DHS P1 Solutions 154


Inde
ex Number: Class:
Name:

DUNMA AN HIGH H SCHOOOL


Prelimiinary Ex
xaminattion
Year 6

MAT
THEMA
ATICS (Higher
( r 2) 9740/02
9 2
Paper 2 26 Septem
mber 2016
6

3 hours
s
Additio
onal Materials: Answer P Paper
Graph pa aper
List of Fo
ormulae (MF15)

READ T
THESE INS
STRUCTION
NS FIRST

Write yoour Name, Index Numb ber and Claass on all th


he work you hand in.

sgexamguru.com
Write in o black pen on both s ides of the paper.
n dark blue or
You ma ay use a sofft pencil for any diagra ms or graph hs.
Do not use stapless, paper clipps, highlightters, glue orr correction fluid.

Answerr all the queestions.


Give noon-exact nu umerical ansswers corre ect to 3 significant figu
ures, or 1 deecimal plac
ce in the
case off angles in degrees,
d unless a diffe rent level off accuracy is
i specified in the ques
stion.
You aree expected to use an approved
a graaphing calcculator.
Unsupp ported answ wers from a graphing ccalculator area allowed unless a qquestion spe ecifically
states o
otherwise.
Where unsupporte ed answers from a gra n allowed in a question, you
aphing calculator are not
are reqquired to present
p the mathemattical steps using matthematical notations and not
calculattor comman nds.
You aree reminded of the need d for clear p
presentation n in your ansswers.

At the e
end of the examination
e , fasten all yyour work securely
s tog
gether.
The nummber of ma arks is given
n in bracketss [ ] at the end
e of each question oor part questtion.

For teache
ers’ use:

Qn Q1 Q2 Q3 Q4 Q
Q5 Q6 Q7 Q8
8 Q9 Q10 Q11
1 Total
Score
Max
8 9 11 12 3 6 7 10
0 10 12 12 100
Score

© DHS 2016 Thiss question paper


p consiists of 6 prin
nted pages (including tthis cover page).

Microsoft Word - DHS_JC2_H2_Maths_2016_Prelim_Paper_2_Questions 155


2

Section A: Pure Mathematics [40 marks]


1 The function f is defined by

f : x  π sin  12 x  , x  , 0  x  a,

where a is a positive constant.

(i) State the largest exact value of a for which the function f 1 exists. [1]

For the rest of the question, use the value of a found in part (i).

(ii) Write down the equation of the line in which the graph of y  f (x) must be reflected in
order to obtain the graph of y  f 1 ( x) and hence verify that 0 and π are solutions to the
equation f ( x)  f 1 ( x). [2]

(iii) Find the area of the region bounded by the graphs of f and f 1 , giving your answer in terms
of π. [3]

The function g is defined by

g : x  x  1 , x  .

sgexamguru.com
(iv) Find the exact range of the composite function gf. [2]

2 The angle between two unit vectors a and b is cos1 14 . Relative to the origin O, the position

vector of a point P on a line l is given by OP  a   (a  2b),    and the point C has position
vector a  b.

(i) By considering scalar product, show that CP 2  6 2  92   1. [4]

(ii) Deduce the exact shortest distance of C to l and write down the position vector of the point
F, the foot of the perpendicular from C to l, in terms of a and b.
[3]

(iii) Find the equation of the plane that contains l and is perpendicular to CF in the form r.n  d
where n is expressed in terms of a and b and d is a constant. [2]

DHS 2016 Year 6 H2 Mathematics Preliminary Examination [Turn over


DHS P2 Questions 156
3

3 The number of bacteria (in millions) in Pond A at the start of the nth week, before any chemical
treatment, is given by un . Pond A is treated at the start of each week with Chemical A, which
kills 70% of all bacteria instantly. At the end of each week, 6 million new bacteria is reproduced.

(i) Write down a recurrence relation of the form un 1  aun  b, where a and b are constants to
be determined. [1]

(ii) Show that un  0.3n1 u1  607 (1  0.3n1 ). [2]

The number of bacteria (in millions) in Pond B at the start of the nth week, before any chemical
treatment, is given by vn . Pond B is treated at the start of each week with Chemical B. It is
known that vn follows the recurrence relation

vn1  0.01vn 2  6.

It is given that if the sequence v1 , v2 , v3 ,  converges to a limit, it converges to either  or  ,


where    .

(iii) Find  and  . Explain whether vn necessarily converges to  or  . [3]

(iv) If u1  v1  30, determine which chemical would be more effective in killing the bacteria in

sgexamguru.com
the long run. [2]

Pond C is treated with Chemical C. To account for the bacteria’s increasing resistance to the
chemical, the dosage of Chemical C is increased by 5 ml each week. The first dose is 20 ml.

(v) How many weeks does it take to finish the first 3 litres of chemical in the treatment of
Pond C? [3]

4 Do not use a graphic calculator in answering this question.


(a) On a single Argand diagram, sketch the locus of z satisfying both inequalities z  1  2i  2
and 1 π  arg( z  1)  1 π. Hence find the range of arg( z ). [5]
4 2

(b) Solve the equation


w6  64,

giving the roots in the form rei , where r  0 and  π    π. [2]

(i) Hence write down the roots of the equation ( z  1  i  3)6  64 in the form a  rei ,
where a is a complex number in cartesian form, r  0 and  π    π. Show the roots
on an Argand diagram. [3]

(ii) Of the roots found in part (b)(i), find in cartesian form the root with the largest
modulus. [2]

DHS 2016 Year 6 H2 Mathematics Preliminary Examination [Turn over


DHS P2 Questions 157
4

Section B: Statistics [60 marks]

5 The Land Transport Authority (LTA) wishes to gather feedback on the quality of train services at
a new train station.

(i) The LTA decides to station a team of surveyors at the gantries to survey the first 100
commuters passing through the train station. State, with a reason, whether the method
described is quota sampling. [1]

(ii) The LTA decides to obtain a random sample instead to survey 5% of the commuters on a
particular day. Describe how a systematic sample can be carried out in this context. [2]

6 John plays for his school’s soccer team. There is a probability of 0.15 that he scores in a game
and a probability of 0.3 that his parents are present at a game. When he scores in a game, there is
a probability of 0.2 that his parents are present.

(i) Show that the probability that he scores in a game when his parents are present is 0.1 [2]

(ii) State, with justification, whether his parents’ presence at a game will affect his chances of
scoring in the game. [1]

Games are equally likely to be home or away games. In a home game, there is a probability of
0.24 that John does not score and his parents are present.

sgexamguru.com
(iii) Find the least and greatest values of the probability that a game is a home game and his
parents are not present at the game. [3]

7 A committee decides to meet on four days in a span of four weeks. Find the probability that the
committee meets on two Tuesdays and two Saturdays if

(i) committee meetings are equally likely to be held on any day in the four weeks, [2]

(ii) committee meetings are held once a week. The probability of holding a meeting on any day
from Monday to Friday is 19 and the probability of holding a meeting on either Saturday or
Sunday is 92 . [2]

The committee of ten sits in a circle at a meeting.

(iii) Find the probability that the two committee vice-heads are seated together and they are not
seated next to the committee head. [3]

DHS 2016 Year 6 H2 Mathematics Preliminary Examination [Turn over


DHS P2 Questions 158
5

8 A research is being conducted to study the growth of car population over time. The data below
shows the population of the car, y millions after x years of study from the start of the research:
Years (x) 5 7 9 14 18 23 27
Car Population (y millions) 7.2 10.5 11.6 13.0 14.5 15.5 15.7

(i) Draw a scatter diagram for the data, labelling the axes. [1]

(ii) State, with a reason, which of the following models is appropriate:


A : y  a  bx2 , B : y  a  b ln x,
where b is positive. [2]

Based on the appropriate model chosen in part (ii),


(iii) calculate the value of the product moment correlation coefficient. State, with a reason,
whether this value would be different if y is recorded in thousands instead. [2]

(iv) calculate the least square estimates of a and b and write down the corresponding regression
line. Obtain the value of the car population after 20 years of study. [3]

(v) give an interpretation of the value of a in the context of the question. Comment on the
reliability of the value of a. [2]

9
sgexamguru.com
In this question you should state clearly all distributions that you use, together with the values of
the appropriate parameters.

(a) The queuing time, in minutes, for flight passengers at the Economy and Business class
check-in counters have independent normal distributions with means and standard
deviations as shown in the table.

Check-in Counter Mean queuing time Standard deviation


Economy class 11.6 4.2
Business class 3.2 0.9
(i) Find the probability that the queueing time of a randomly chosen Economy class
passenger is within 5 minutes of the total queueing time of 2 randomly chosen
Business class passengers. [4]

(ii) The queueing time of 8 randomly chosen Business class passengers are taken.
Find the probability that the shortest queuing time among all 8 passengers is no less
than 2 minutes. [2]

(b) The probability that a passenger books a flight and does not turn up is 0.05. The airline
decides to allow for over-booking by selling more tickets than the number of seats
available.

For a particular flight with 350 available seats, n tickets were sold, where n  350. By
using a suitable approximation, show that if the flight is to have no more than 1% chance
of having insufficient seats, the number of tickets sold must satisfy the approximate
inequality

DHS 2016 Year 6 H2 Mathematics Preliminary Examination [Turn over


DHS P2 Questions 159
6

        350.5  0.95n  2.3263  (0.0475 n).   [4]


10 A manufacturer claims that ropes with a certain diameter produced by his factory have mean
breaking strength of at least 169.7 kN. Recently, a new material is used to produce the ropes. A
random sample of 8 coils of the rope made with the new material is taken and the breaking
strength of each coil of rope, x kN, is measured as follows.

171.3 168.5 166.5 164.4 170.0 165.1 170.1 167.2

(i) Find the unbiased estimates of the population mean and variance. [2]

(ii) Stating a necessary assumption, test at the 5% significance level whether the manufacturer’s
claim is valid after the change in material. [5]

Instead of using the new material, the manufacturer decides to change the weaving process of the
ropes. The manufacturer claims that the mean breaking strength is now 0 kN. The population
variance is found to be 29.16 (kN) 2 . A random sample of 50 coils of the rope made using the
new process is taken and the mean breaking strength, y kN, is found to be 171 kN.

(iii) Find the set of values of 0 for which the mean breaking strength does not differ from the
claim when tested at the 1% significance level. [4]

(iv) Explain, in the context of the question, the meaning of ‘at the 1% significance level’. [1]

11 (a)
sgexamguru.com
A restaurant has 15 tables consisting of 6 rectangular tables and 9 round tables. During the
restaurant’s opening hours, the rectangular tables are occupied, on average 80 percent of
the time, and the round tables are occupied, on average 65 percent of the time. You may
assume that the tables in the restaurant are occupied independently of each other.

(i) If a customer walks into the restaurant at a randomly selected time, what is the
probability that 4 rectangular tables and 7 round tables are occupied? [2]

(ii) Give a reason in context why the assumption made above may not be valid. [1]

(b) A café sells both coffee and tea. The number of cups of coffee and tea sold in a randomly
chosen 20-minute period have independent Poisson distributions with means 5 and 3.5
respectively.

(i) In a particular 20-minute period, at least 7 cups of beverages are sold. Find the
probability that at least 6 cups of tea are sold during the 20-minute period. [4]

(ii) Let pk denote the probability that k cups of coffee are sold in a 20-minute period.
p 5
Show that k 1  and deduce that pk 1  pk , when k < 4. [3]
pk k 1
Hence find the most probable number(s) of cups of coffee sold in a 20-minute period.[2]

DHS 2016 Year 6 H2 Mathematics Preliminary Examination [Turn over


DHS P2 Questions 160
2016 Year 6 H2 Math Prelim Exam Paper 2 Solutions
Qn Suggested Solution
1(i) Largest a = π y
π
y  f(x)

π x

(ii) The line is y  x.


f(x)  f 1 (x)
Since the points of intersection lies on y  x,
f(x)  x
πsin  12 x   x
When x  0 : LHS  πsin  0   0  RHS
When x  π : LHS  πsin  12 π   π  RHS
 0 and π are solutions to the equation f(x)  f 1 (x)
(iii) y
yx
π

sgexamguru.com
y  f(x)
A y  f 1 (x)
B
C
O
π x

Required area = A + B = 2A (by symmetry)

1 2
Area of B + C = π (area of triangle)
2
π
Area of A  B  C   f(x) dx
0

π 1 
  πsin  x  dx
0
2 
π
  1 
  2πcos  x  
  2 0
 2π
 Area bounded by the graphs of f and f 1
 1 
 2  2π  π 2   4π  π 2
 2 

DHS P2 Solutions 161


Alternative
π
Required area  2  f(x)  x dx
0

π 1 
 2  πsin  x   x dx
0
2 
π
 1  x 
2
 2  2πcos  x   
  2  2 0
 π2 
 2  2π    4π  π 2
 2 

(iv) y

π 1
y  g(x)

O 1 π x

[0,π] f
[0,π] g
[0,π  1]
 R gf  [0,π  1]
Qn
2(i)
Suggested Solution
sgexamguru.com
CP  a   (a  2b)  (a  b)  b   (a  2b)
CP 2  b   (a  2b) .b   (a  2b) 
=b.b   2 (a  2b).(a  2b)  2b.(a  2b)
 b.b   2  a.a  4a.b  4b.b   2  b.a  2b.b 
1  1
 1   2 1  1  4   2   2  as a.a  b.b  1 and a.b 
4  4
9
 6 2   +1 (shown)
2
(ii)  3 
CP 2  6  2     1
 4 
2 2
 3 54  3 5
 6    1  6    
 8 64  8  32
2
 3 5
CP  6     
 8  32
The perpendicular distance from C to l occurs when P is nearest to l, that is when CP is
3
least or    .
8
Least CP is 10 . P is F in this case.
8
3 1
OF  a   a  2b    5a  6b 
8 8
2

DHS P2 Solutions 162


Alternative to find minimum CP:
CP is minimum when CP2 is minimum:
d 9
(CP 2 )  12 
dx 2
d 9
When (CP 2 )  0,12   0
dx 2
3
   .
8
Since CP 2 is quadratic and coefficient of  2  0,
3
CP 2 is minimum at   
8
3
 perpendicular distance from C to l occur when    .
8
1 1
(iii) CF   5a  6b    a  b    3a  2b 
8 8
Equation of plane
2 5
r. 3a  2b   a. 3a  2b   3  
4 2

Qn Suggested Solution
3(i) un1  0.3un  6
(ii) un  0.3un 1  6

sgexamguru.com
 0.3(0.3un  2  6)  6
 0.32 un  2  0.3(6)  6
 0.32 (0.3un 3  6)  0.3(6)  6
 0.33 un 3  0.32 (6)  0.3(6)  6

 0.3n 1 u1  0.3n  2 (6)  0.3n 3 (6)   0.3(6)  6


6(1  0.3n 1 )
 0.3n 1 u1 
1  0.3
n 1
 0.3 u1  7 (1  0.3n 1 )
60

Alternative
u2  0.3u1  6
u3  0.3u2  6
 0.3(0.3u1  6)  6
 0.32 u1  0.3(6)  6

un  0.3n 1 u1  0.3n  2 (6)  0.3n 3 (6)   0.3(6)  6


6(1  0.3n 1 )
 0.3n 1 u1 
1  0.3
 0.3 u1  607 (1  0.3n 1 )
n 1

DHS P2 Solutions 163


(iii) As n  , vn 1  L and vn  L.
 L  0.01L2  6
0.01L2  L  6  0
From G.C., L  6.4110 or 93.588
  6.41,   93.6 (3 s.f.)
vn may not necessarily converge to a limit as we do not know what is the value of its
starting term v1 (or initial number of bacteria).
(iv) As n  ,
un  0u1  607 (1  0)  8.57 (3 s.f.)
v1 = 30, v2  0.01v12  6  15 , v3  0.01v22  6  8.25 , v4  0.01v32  6  6.68 ,
v5  0.01v42  6  6.446 ,…
From GC, vn  6.41 (3 s.f.)
Thus, Chemical B is more effective in the long run.
(v) The amount of Chemical C used per week is an Arithmetic Progression with first term
a  20, common difference d  5.
Want: Sn  3000
n
[2(20)  (n  1)(5)]  3000
2
n 2  7n  1200  0

sgexamguru.com
From G.C.,
n n2  7n  1200
31 22
32 48
33 120
 it takes 32 weeks
Alternative: From GC, n  31.3(to 3 s.f.)  It takes 32 wks
Qn Suggested Solution
4(a) Im

Locus of z

of z
C(-1, 2)
x
π A
π
2
B 4 Re
-1 O 1

z  1  2i  z  (1  2i)
ie, z  (1  2i)  2
2
Min arg  z  occurs at A = tan 1  tan 1 2
1
Max arg  z  occurs at B
Hence, tan 1 2  arg  z   π

DHS P2 Solutions 164


(b) w6  26
 w6  26 e2kπi

i
 w  2e 3 , k  0, 1, 2,3

(i) kπ
i
z  1  i  3  2e 3 , k  0, 1, 2,3

Im
Q


 
  


O Re
\

(ii) z is maximum at Q, where Q is the point representing the root when k = 1 and OQ is
the diameter of the circle
π
i
z  1  i  3  2e 3


sgexamguru.com
π π
 1  i  3  2  cos  i sin 
 3 3
 1 i  3 1 i  3
 2i 23

Qn Suggested Solution
5(i) No, the people are surveyed without consideration of the stratum e.g. age group they
belong to.

(ii) To obtain a systematic sample of 5%, we can first randomly select the first commuter
from the first 20 commuters entering the train station by stationing the surveyors at the
gantries and thereafter select every 20th commuter thereafter entering the train station
from the start to the end of that particular day.

DHS P2 Solutions 165


Qn Suggested Solution
6(i) Let A denote the event that John scores in a game.
Let B denote the event that John’s parents are present at a game.
Given that P( A)  0.15, P( B)  0.3, P( B | A)  0.2,
P( A | B )
P( A B )

P( B )
P( B | A)  P( A)

P( B )
0.2  0.15 0.03
= 
0.3 0.3
 0.1 (shown)
(ii) Since P( A | B)  0.1  P( A)  0.15 his parent’s presence in a game will affect his chances
of scoring in the game.

Alternative
Since P( B | A)  0.2  P( B)  0.3 his parent’s presence in a game will affect his
chances of scoring in the game.

(iii) Let C denote the event that the game is a home game.
P( A ' B C )  P(( A ' B) | C)  P(C )  0.24  0.5  0.12

A
sgexamguru.com Least P( B ' C )
0  0.5  0.03  0.12
0.03  0.35
0.12

C
B

0.03 Max P( B ' C )


0
 0.5  0.12
0.12
 0.38
C
B

DHS P2 Solutions 166


Qn Suggested Solution
7(i) Required probability
 C2 
4 2

 28
C4
4

2275
(ii) Required probability
2 2
1 2 4!
     
 9   9  2!2!
8

2187
(iii) No. of ways to seat the remaining members
 (7  1)!
 720
No. of ways to slot in the committee head and the 2 vice-heads as a pair
 7 P2  42
No. of ways to arrange the 2 vice-heads = 2
Required probability
720  42  2 1
 
(10  1) ! 6

Alternative sgexamguru.com
No. of ways in which the vice-heads are seated together
 (9  1)! 2!
 80640
No of ways in which the vice-heads are seated together and the head is seated next to
one of them
 (8  1) ! 2! 2
 20160
Required probability
80640  20160 1
 
(10  1)! 6
Alternative
No. of ways in which the vice-heads are seated together with the rest of the committee
excluding the committee head
 (8  1) ! 2!
No. of ways to slot in the committee head
6
= C1
Required probability
= (8  1)!(2)(6)  1
(10  1)! 6

DHS P2 Solutions 167


Qn Suggested Solution
8(i) y

15.7

7.2
5 27 x

(ii) Model B: y  a  b ln x is appropriate but not Model A.

From the scatter diagram, as x increases, y increases at a decreasing rate, which is


consistent with Model B but not Model A which predicts an increasing rate of
increase for y.

(iii) Screenshot for reference:

sgexamguru.com
r  0.98047 (5 s.f.)
 0.980 (3 s.f.)

The value of r would not be different as it is unaffected when data is scaled


(iv) a  0.534 , b  4.76

y  0.53445  4.7568ln x (5 s.f.)


(or y  0.534  4.76ln x )

When x  20 ,
y  0.53445  4.7568ln(20)
 14.785
 14.8 (3 s.f.)
Car population is 14.8 millions

(v) Value of a represents the predicted car population after 1 year of study.
[If wrong model chosen:
Value of a represents the predicted car population at the start of the study. ]

The value of a is unreliable (invalid) as it is an extrapolation (outside the data range


Year 5 to Year 27) and the linear relationship may not hold.

DHS P2 Solutions 168


Qn Suggested Solution
9(a) Let A and B denote the queuing times of a randomly chosen passenger at Economy
(i) and Business class counters respectively.
A ~ N(11.6, 4.22 ) B ~ N(3.2,0.92 )
Find P  A  ( B1  B2 )  5

A  ( B1  B2 ) ~ N(11.6  2  3.2, 4.22  2  0.92 )


i.e. A  ( B1  B2 ) ~ N(5.2, 19.26)
 P  A  ( B1  B2 )  5   P  5  A  ( B1  B2 )  5 
 0.47177
 0.472 (3 s.f.)
(ii) Required probability
 P( B1  2)  P( B2  2)  ...  P( B8  2)
  P( B  2) 
8

 0.46526
 0.465 (3 s.f.)

(b) Let X denote the number of passengers who turned up for their flight, out of n
passengers who bought the n tickets.
X ~ B (n,0.95)

sgexamguru.com
Since n > 350 is sufficiently large such that
np = 0.95n > 5 and nq = 0.05n > 5,
X N  0.95n, 0.0475n  approximately

P  Flight is overbooked   0.01


 P  X  350   0.01
 P  X  350.5   0.01 (continuity correction)
 P  X  350.5  0.99
 350.5  0.95n 
 P Z    0.99
 0.0475n 
350.5  0.95n
  2.3263
0.0475n

 350.5  0.95n  2.3263 0.0475n (shown)

DHS P2 Solutions 169


Alternative to show approx inequality:
Let X denote the number of passengers who did not turn up for their flight, out of n
passengers who bought the n tickets.
X ~ B (n,0.05)

Since n > 350 is sufficiently large such that


np = 0.05n > 5 and nq = 0.95n > 5,
X N  0.05n, 0.0475n  approximately

P  Flight is overbooked   0.01


 P  X  n  350   0.01
 P  X  n  351  0.01
 P  X  n  350.5   0.01 (continuity correction)
 n  350.5  0.05n 
 P Z    0.01
 0.0475n 
0.95n  350.5
  2.3263
0.0475n

 0.95n  350.5  2.3263 0.0475n

 350.5  0.95n  2.3263 0.0475n (shown)

sgexamguru.com
Qn Suggested Solution
10(i) Unbiased estimate of population mean, x  167.89  168 (3 s.f.)
Unbiased estimate of population variance,
s 2  2.49882  6.24 (3 s.f.)
(ii) Assume that the breaking strength of each coil of rope is normally distributed.

H 0 :   169.7
H1 :   169.7

Perform 1-tail test at 5% significance level.

X  169.7
Under H 0 , ~ t (8  1).
S
8
Using t -test, p-value  0.039673.
Since p-value  0.05, we reject H 0 and conclude that there is sufficient evidence at
the 5% significance level that the mean breaking strength is less than 169.7 kg, i.e. the
manufacturer’s claim is not valid.

10

DHS P2 Solutions 170


(iii) H 0 :   0
H1 :   0
29.16
Under H0 , Y ~ N ( 0 , ) approximately by Central Limit Theorem.
50

0.005 0.005

-2.5758 0 2.5758 z

Given y  171, and H 0 is not rejected,


171  0
2.5758   2.5758
29.16
50
 169.03  0  172.97
 set of values of 0 is: {0  :169  0  173}

(iv) Testing at the 1% significance level means that there is a probability of 0.01 of

sgexamguru.com
concluding that the mean breaking strength differs from the claim when it is actually
unchanged.

Qn Suggested Solution
11(a) Let X and Y be the number of rectangular tables and round tables that are occupied.
(i) X ~ B(6,0.8) Y ~ B(9,0.65)

Required probability = P( X  4) P(Y  7)


=0.24576  0.21619
 0.0531 (3 s.f.)
(ii)  Customers may arrive as a big group that requires them to be split into two
separate tables next to each other.
OR
 The restaurant may choose to seat the customers at tables in a particular section
first.

(b) Let C and T be the number of cups of coffee and tea sold in 20 minutes, respectively.
(i) C ~ PO (5) T ~ PO (3.5)
C  T ~ PO (8.5)

P({T  6}  { C  T  7})
P(T  6 | C  T  7) 
P(C  T  7)
11

DHS P2 Solutions 171


P(T  6) P(C  1)  P(T  7)P(C  0)

P(C  T  7)
P(T  6) [1  P(C  0)]  [1  P(T  6)]

1  P(C  T  6)
0.077098(0.99326)  0.065288

0.74382
0.14187
= =0.191 (3 s.f.)
0.74382
(ii)  k
Using pk  e for   5,
k!
 5 5k 1 
e 
pk 1  (k  1)!  5k 1 k ! 5
  k  (shown)
pk  5 5 
k
5 (k  1)! k  1
e 
 k!
When k < 4, k + 1 < 5,
5 p
  1  k 1  1  pk 1  pk .
k 1 pk

When k  4, i.e. k  0,1, 2,3


pk 1  pk  p4  p3  p2  p1  p0 .

sgexamguru.com
When k  4, i.e. k  5, 6, 7,...
pk 1  pk  p5  p6  p7 .
When k  4, pk 1  pk  p4  p5 .

From above, p0  p1   p4  p5  p6  p7  p8 
(Thus pk is greatest when k = 4 and 5)

The most probable number of cups of coffee sold (i.e. the mode) are 4 and 5.

12

DHS P2 Solutions 172


  
  
1
 

HWAA CHONG INSTITUTI


I ION
2016 JC2 P
PRELIMINA
ARY EXAMMINATION

MATH
HEMATICS
Highe
er 2
9
9740/0
01
Paperr 1
Wedne
esday 14 Se
eptember 2
2016 3h
hours

als:
Additional materia A
Answer paper
L
List of Form
mula (MF15
5)

READ THESE INSTRUCTIO


ONS FIRST

Write yyour name and CT class on alll the work you hand in, including the Covver Page
which iss found on
n Page 2.
Write in
n dark blue
e or black ppen on both h sides of tthe paper.
You ma ay use a sooft pencil fo
or any diaggrams or grraphs.
Do not use staplees, paper clips, highligghters, gluee or correcction fluid.

sgexamguru.com
Do not write anythhing on the e List of Fo
ormula (MFF15).

Answerr all the qu uestions.


Give no on-exact numerical answers
a orrect to 3 ssignificant figures, orr 1 decimal place in
co
the casse of angle es in degreees, unlesss a differeent level off accuracyy is specifieed in the
questio
on.
You are e expected d to use a graphing
g ca
alculator.
Unsupp ported ansswers from m a graphing calcu ulator are allowed unless a question
specificcally statess otherwise
e.
Where unsupportted answerrs from a g graphing ccalculator aare not alloowed in a question,
you are e required to present the math hematical ssteps using g mathema atical notattions and
not calcculator com mmands.
You are e reminded d of the nee
ed for clea
ar presentation in your answers.

The nuumber of m marks is ggiven in brackets [ ] at the e end of eacch question n or part
questio
on. At the e
end of the examinatioon, place th
he completted cover p page on to op of your
answerr scripts an
nd fasten all your work securely together w
with the striing provide
ed.

stion pape
This ques s of 6 prin
er consists nted pages
s.

   [T
Turn over

Microsoft Word - 2016 HCI JC2 Prelim Paper 1 (Question) 173


2
 

1. A sequence follows the recurrence relation


 2n  1 x x U  sin x
U n 1  U n  2 cos sin , 1 for n  1, 2, 3,... .
2 2
Prove by mathematical induction that U n  sin  nx  for all positive integer n. [4]

2
2. Solve the inequality  1. [2]
4( x  1)2  1
3 2
2
Hence find  1
2
1
4( x  1)2  1
dx, leaving your answer in exact form. [3]

 
3. Referred to the origin O, the points A and B are such that OA  a and OB  b with a not
parallel to b . The point P is on AB produced with AP : AB  3 :1 and the position vector of
point Q is 2a .

(a) Find the position vector of the point of intersection of lines OB and PQ, giving your
answer in terms of b. [4]

(b) sgexamguru.com
It is given that a  b  2i  4 j  2k and the point C  0, 3, 4  does not lie on the
plane OAB. Find the foot of the perpendicular from C to the plane OAB. [4]

2n  1
4. Prove that  n 2  2n  n 2  1. [2]
n  2n  n  1
2 2

N
2n  1
Hence find  . [3]
n 1 n  2n  n 2  1
2

N
2n  1
(a) Deduce the value of  . [3]
n2 n  2n  n 2  1
2

N
2n  1
(b) Show that  2n  1 
n 1
N 2  2N . [1]

[Turn over

HCI P1 Questions 174


3
 

 
5. Sketch on a single Argand diagram, the loci defined by   arg  z  1  2i   and
4 4
2  i w  5  5. [4]
(i) Find the minimum value of arg  w  . [2]
(ii) Find the minimum value of z  w . [2]
(iii) Given that arg  z  w    ,       , state the minimum value of  . [1]

6. A group of boys want to set up a camping tent. They lay down a rectangular tarp OABC on
the horizontal ground with OA = 3 m and AB = 1.5 m and secure the points D and E
vertically above O and B respectively, such that OD  BE  2 m .
E

D 2m
C

2m B

k j
i

sgexamguru.com
1.5 m
O
3m A

Assume that the tent takes the shape as shown above with 6 triangular surfaces and a
rectangular base. The point O is taken as the origin and the unit vectors i, j and k are taken
  
to be in the direction of OA , OC and OD respectively.
(i) Show that the line DE can be expressed as r  2k    2i  j ,    . [2]
(ii) Find the Cartesian equation of the plane ADE. [3]
(iii) Determine the acute angle between the planes ADE and OABC. Hence, or otherwise,
find the acute angle between the planes ADE and CDE. [4]

x2
7. The curve C has equation y  , where k  1 .
kx  x  2
2

(i) Find the equation of the tangent at the point A where C cuts the y-axis. [2]
(ii) Sketch C, giving the equations of asymptotes, the coordinates of turning points and
axial intercepts in terms of k , if any. [4]
(iii) Find the equation of the normal at the point B where C cuts the x-axis. Leave your
answer in terms of k . [2]
(iv) Hence show that the value of the area bounded by the tangent at A, the normal at B
15
and both the x- and y-axes is more than square units. [2]
8

   [Turn over

HCI P1 Questions 175


4
 

8. The n x,   x   .
T curve C (as shownn in the diagrram below) has equatioon y  x 2 sin
               
y

x
  

(i)) Calculatte the exact area of the region R ennclosed by C and the x-aaxis. [4]
(iii) Sketch the curvee with eqquation ( y  1)  4(x  2)  1, shhowing cleearly the
2 2

coordinaates of the tuurning poinnts and the eequation(s) oof any asym
mptote(s). [2]
(iiii) Hence ffind the volume of thee solid gennerated wheen the regioon boundedd by the 2
curves is rotated thrrough 4 righht angles about the x-axxis. [4]

9. A manufacturrer produces cylindricaal containers using sheet metal off negligible thickness.
Thhe cylindricaal containerr has an opeen top, and a base and ccurved sidess made up off the sheet
meetal.

(a)) (i)
sgexamguru.com
IIt is given thhat the voluume of the cyylindrical container is fixed
f at k ccm3. Show
tthat when tthe amount of sheet metalm used ffor the cylinndrical conttainer is a
mminimum, thhe ratio of iits height to its radius iss 1:1 . [5]
(ii) A product ddesigner prooposed a new w design whhere the heiight of the ccylindrical
ccontainer iss always 2.5 times thaat of its raddius. Given that the raadius of a
ccylindrical container
c prroduced usiing the new
w design equuals the raddius of the
ccontainer prroduced in part
p (a)(i) w with minimuum sheet meetal. Find thhe ratio of
tthe amount of sheet meetal used in this new deesign to the minimum amount
a of
ssheet metal used in partt (a)(i). [2]

b)
(b To redduce cost, plastic
p with negligible thickness, iinstead of ssheet metal is used to
manuffacture the new designn cylindricaal containerrs in part (a)(ii) usingg injection
blow moulding
m teechnology. In the injecction blow m moulding pprocess, it iss assumed
that thhe cylindriccal containeers increase in size prooportionatelly with the height to
radiuss ratio remaining constaant at 5:2 thhroughout tthe process. If the voluume of the
cylinddrical contaiiner increasses at a ratte of 80 cm m3 per secoond, find thhe rate of
change of the surfface area off the cylindrrical containner when its height is 500 cm. [4]

[T
Turn over

HCI P1 Questions 176


5
 

10. 
A

x 1
6

B C
y
1 
In the triangle ABC, AB  x , BC  y , AC  , angle ABC  radians and angle
6 6
ACB   radians (see diagram).

x 2 sin 
(a) (i) Show that  . [3]
y cos   3 sin 

x
(ii) Given that  is sufficiently small, express as a cubic polynomial in  . [3]
y

(b) (i) Show that   sin 1  3x  . [1]

(ii) Find the Maclaurin series for  , up to and including the term in x 3 . [5]

sgexamguru.com
11. The functions f and g are defined by
1
f : x  e1 x , x  , x  1 and
2

2
g : x  1  ln x , x  , 0  x  e .
(i) Show that gf exists, and find the range of gf. [4]
(ii) Justify, with a reason, whether f 1 exists. [2]
(iii) The domain of f is restricted to  , b  such that b is the largest value for which the

inverse function f 1 exists. State the value of b and define f 1 clearly. [4]
(iv) The graph of y  h  x  is obtained by transforming the graph of y  g  x  in the

following 2 steps.
Step 1: Scale parallel to the x-axis by a factor of 2.
Step 2: Reflect in the x-axis.

Define h in a similar form. [3]

End of Paper
[Turn over

HCI P1 Questions 177


2016 HCI Prelim Paper 1 Solutions
Qn Solution
1 * Let Pn be statement U n  sin  nx  for all n    .
When n  1 , LHS  U1  sin x , RHS  sin x  P1 is true.
* Assume Pk is true for some k    , i.e. U k  sin  kx  .
Want to prove that Pk + 1 is true, i.e. U k 1  sin  k  1 x .
LHS
 U k 1
 2k  1 x x
 U k  2 cos sin
2 2
 2k  1  1
 sin  kx   2 cos   x sin   x
 2  2
 sin  kx   sin  k  1 x  sin  kx 
 sin  k  1 x  RHS
*Since P1 is true, Pk is true implies Pk + 1 is true, by MI Pn is true for all n    .

2 2
1

sgexamguru.com
4( x  1) 2  1
(2 x  1)(2 x  3)
0
4( x  1) 2  1
Since 4( x  1) 2  1  0  for all x,
(2 x  1)(2 x  3)  0
3 1
  x  
2 2
3
1 2
12 1  4( x  1)2  1 dx

1
 2  3
1  2 
 2
 1  dx   2
 1 dx
4( x  1)  1   
1
1

2 
2  4( x 1) 2
1 
1 3
 1
   x  tan (2 x  2) 
1 2   x  tan (2 x  2) 
1 2
1  
1 
2

1   3 1 
   tan 1 1  1    1  tan 1 3   tan 1 1  
2   2 2 
 3
  1  
6 2

Microsoft Word - 2016 HCI JC2 Prelim Paper 1 (Solution) 178


 
3 OP  a  3 AB  a  3  b  a   3b  2a

       
(a)  PQ  OQ  OP  2a   3b  2a   4a  3b
       
lPQ : r  2a    4a  3b  ,   
   
lOB : r   b,   
 
At point of intersection, 2a    4a  3b   b
   
1 3
Comparing coefficients of  a  and  b ,       ,     
  2 2

3
 position vector of the point of intersection  b   
2

(b)  2 1 1


     
a  b   4   2  2   n   2 
    1  1
 2    

Let F be the foot of perpendicular.


Method 1

lFC 
sgexamguru.com
 0

1
 
1
: r   3   s  2  , s   ,  OAB : r   2   0
   1   
 4   1

 0   1   1 
     
 3   s  2     2   0
 4   1   1 
   

6  4  s 1  4  1  0

1
s
3

 0  1  0 1  1
   1   1
92
1 
 OF  3  2  7
  3  3   3  
 4  1  
     12 1   13 

 1 7 13 
F  , , 
3 3 3 

HCI P1 Solutions 179


Method 2
 

FC  OC  nˆ nˆ
 

 1 1
    
 0  2    2 
   1   1
 
  3     
   1   1
  4   2    
    2 
 1 1
    
1
 
 2 
6  4  1 

6 6
1
1 
   2 
3 
1
  

sgexamguru.com
OF  OC  CF
0  1  0 1  1
  1  1  1 
  3    2    9  2    7 
 4  3  1  3 12  1 3 13 
       

 1 7 13 
F  , , 
3 3 3 

4
Method 1
2n  1 2n  1 n 2  2n  n 2  1
 
n 2  2n  n 2  1 n 2  2n  n 2  1 n 2  2n  n 2  1


 2n  1  n 2  2n  n 2  1 
n 2
 2n    n 2  1
 n 2  2n  n 2  1

HCI P1 Solutions 180


Method 2
 n 2  2n  n 2  1  n 2  2n  n 2  1 

 n 2  2n   n 2  1 
 2n  1
2n  1
  n 2  2n  n 2  1
n  2n  n  1
2 2

N
2n  1

n 1 n 2  2n  n 2  1

 
N
 n 2  2n  n 2  1
n 1

 3 0 
 
  8 3 
 
...
 
 N 2  2 N  N 2  1
 
 N 2  2N
(a) Replace n by n  1 ,
N
2n  1
 n 2

N 1
sgexamguru.com
n2  1  n  n  2
2n  1

n 1 n  2n  n 2  1
2

  N  1  2  N  1
2

 N2 1
(b) Notice that n 2  2n  n and

 
2
n 2  1   n  1  2n  2  0 .
2

 n2  1  n  1
 n 2  2n  n 2  1  2n  1
1 1
 
n  2 n  n  1 2n  1
2 2

N
2n  1 N 2n  1
   N 2  2N
n 1 2 n  1 n 1 n  2n  n  1
2 2

HCI P1 Solutions 181


5 2  i w  5  5
5
2i w  5
2i
w  2  i  1  circle centre  2, 1 , radius 1
Im

Re
O

sgexamguru.com
(i) Im

α
α
Re
O

1 1
  sin 1  sin 1
 2   12
2
5

min arg  w     2  2.2143  2.21 3sf 

HCI P1 Solutions 182


Im
(ii) w

O
Re

–1

min z  w  2 2  1

(iii) sgexamguru.com
Im

z
Re
O



4

HCI P1 Solutions 183


6 0  3 
(i)      
OD   0  , OE  1.5 
 2  2 
   
 3  0  3   2
        
DE  1.5    0   1.5   1.5  1 
 2   2  0  0
       
0  2
   
lDE : r   0     1  ,   
   0
 2  
(ii)  0   3   3 
      
AD   0    0    0 
 2 0  2 
     
 2   3   2  2
         
DE  AD  1.5  1    0   1.5  4   n   4 
0  2   3   3
       
 2  0  2 
     
 ADE : r   4    0    4   6
sgexamguru.com
      
 3   2  3 
 2 x  4 y  3z  6
(iii) 0  2
   
nOABC   0  , nADE   4 
 1   3
   
angle between planes
0  2 
   
 0    4 
1  3 
    3
 cos 1  cos 1
0  2  4  16  9
   
 0   4 
1  3 
   
3
 cos 1  56.1 1dp 
29

3
Angle  180  2 cos 1  67.7 (1 d.p.)
29

HCI P1 Solutions 184


7 dy ( kx 2  x  2)  ( x  2)(2 kx  1)  kx 2  4kx
(i)  
dx (kx 2  x  2) 2 ( kx 2  x  2) 2
dy 0 2
When x  0 ,   0 and y  1
dx (2) 2
2
Hence required equation of tangent is y  1 .

(ii) For axial intercepts, when y  0 , x  2 .


when x  0 , y  1 .

For vertical asymptotes, kx 2  x  2  0


1  1  8k
 x
2k

dy
For turning points, 0
dx
kx 2  4kx  0
kx( x  4)  0
 x  0 or x  4

sgexamguru.com
 

 

  A(0,1)
x

B(2,0)

(iii) dy 4k  8k 4k 1
At x  2 ,   
dx (4k ) 2
16k 2
4k
 gradient of normal  4k

Hence required equation of normal is y  0  4k ( x  2)


y  4kx  8k

   

HCI P1 Solutions 185


(iv)

y  4kx  8k

y=1

8k  1
When y  1 , 1  4kx  8k  x 
W
4k
1  8k  1 
 required area    2  (11)
2  4k 
16k  1

8k
1
2
8k
1
2 (sincee k  1)
8
15

8

8
(i)

sgexamguru.com
Area  2 x 2 sin x dx
A
0

 2    x 2 cos x    2 x cos x dx 
 

 0 0 
 2 2  2( x sin x 0   sin x dx) 
 

 0 
 2  2  2  cos x 0 

 
 2(  4) units
2 2

(ii)
y =  2x  5 y = 2x + 3

(  2, 0 )
(  2,  2 )

HCI P1 Solutions 186


(iii)
y

  

O x

Coordinatess of the poinnts of interseections of thhe 2 curves are (1.537


C 74, 2.3623)) and
(2.7626, 2.8238) .

V
Volume of ssolid generaated
1.5374 1.5374
  (xx 2 sin x)2 dx   (1  1  4(xx  2)2 )2 dx = 26.8 unitss3
2.7626 2.7626

9
(i)
L
sgexamguru.com
Let A cm2 bbe the surfacce area of thhe cylindrical containerr.
Let r cm annd h cm be the
L t radius annd height off the cylindrrical containner respectivvely.

V
Volume   r 2h  k
k
 h 2
r
A  2 rh   r 2  
 k 
 2 r  2    r 2
r   
2k
   r2
r
Can alsoo express r inn terms of
dA 2k
H
Hence   2  2 r h and finnd A in termss of h,
dr r dA
dA then let  0 to obtain h
W
When 0 , dh
dr and subssequently r .

HCI P1 Solutions 187


2k
  2 r  0
r2
k
r3 

k
r 3

k k k
 h  3
r 2
  k  3 
1 2

 
k k
Hence h : r  3 : 3  1:1 (shown)
 
d 2 A 4k
 3  2  0 since p > 0 and k > 0
dr 2 r
k
Hence A is a minimum when r  3

(ii) From (i), h : r  1:1
Hence A  2 rh   r 2  2 r  r    r 2  3 r 2
For new design, h : r  5: 2
5 
Hence new A  2 rh   r 2  2 r  r    r 2  6 r 2
sgexamguru.com
 required ratio is 6 r : 3 r  2 :1
2 2
2 

(b) Method 1
Let V cm3 be the volume of the cylindrical container.
5 5
V   r 2h   r 2 ( r )   r 3
2 2
5 
A  2 rh   r 2  2 r  r    r 2  6 r 2
2 
dV 15 2
 r Can also find
dV
and
dA
dr 2
dh dh
dA
 12 r and use
dr dA d A d h d V
dA dA dr dV   
   d t dh dV d t
dt dr dV dt
2 128
 12 r   80 
15 r 2
r
2
When h  50 , r  (50)  20
5
dA 128
Hence   6.4 cm2 /s
dt 20

HCI P1 Solutions 188


Method 2
A A
A  6 r 2  r  (reject r   since r  0)
6 6
5  5
Hence V   r 2 h   r 2  r    r 3
2  2
3
5  A 
   
2  6 
3
5 A2

2(6) 2  2
3 1

1
dV 15 A 2

dA 4(6) 2  2
3 1

2
When h  50 , r  (50)  20
5
 A  6 (20)  2400
2

dA dA dV
Hence  
dt dV dt
4(6) 2  2
3 1

 1  80

sgexamguru.com
15 A 2
4(6) 2  2
3 1

 1  80
15(2400 ) 2
 6.4 cm 2 /s

10   
sin      
(a) sin   6 
(i) 
x y
x sin 

y  5 
sin   
 6 
x sin 

y sin 5  5
cos   sin  cos
6 6
x sin  2sin 
  (shown)
y 1 3 cos   3 sin 
cos   sin 
2 2

(a) x 2 sin 
(ii) 
y cos   3 sin 

HCI P1 Solutions 189


 3 
2    ... 
x
  
3!
y  2
1  3   ...
2
1
x  3    2 
 2      1   3   
y  3!    2 
  2  
 1    1  3   
x  3   2  
 2   
y 3!   1  2   2 2 
   
  3   
 2!  2  
x  3  2 
 2     1  3   3 2 
y  3!   2 
x 20
 2  2 3 2   3
y 3

(b) Using sine rule,


(i) 
sin
sin 
sgexamguru.com
  sin 1 3x
 6 3
x 1
6
(b) Method 1
(ii) sin   3x
d
cos   3 ---- (1)
dx
d 2  d 
2

cos  2  sin     0 --- (2)


dx  dx 
d 3 d d 2 d d 2  d 
3

cos   sin   2 sin   cos   


dx 3
dx dx 2
dx dx 2
 dx 
 0     3

When x  0 ,
d d 2 d 3
 0 ,  3 , 0 ,  27
dx dx 2 dx 3

27 3 9
  3x  x  ...  3x  x 3  ...
3! 2

HCI P1 Solutions 190


Method 2
  sin 1  3x 
d 1
 3 1  9 x 2  2
3 

dx 1  9x2
d 2  1
3 3

   
2 2 2 2
 3   1  9 x  18 x   27 x 1  9 x
dx 2  2
d 3 5 3
  x  18 x  1  9 x   27 1  9 x 
81 2 2 2 2

dx 3 2
5 3
 729 x 1  9 x
2

2 2
 27 1  9 x 
2 2

When x  0 ,
d d 2 d 3
  0,  3 , 2  0 , 3  27
dx dx dx

27 3 9
  3x  x  ...  3x  x 3  ...
3! 2

11 y
(i)  

sgexamguru.com
x
y=0

 1 
Since Rf   0, e   Dg   0, e , Rf  Dg and gf exists.
 2 

y
x=0

x

Rgf   ln 2,  

HCI P1 Solutions 191


(ii) y

y=1  

x
y=0

Since a horizontal line y = 1 cuts the graph of y = f(x) twice, f is not a one-to-one function and
f -1 does not exist.

(iii) b0

Let y  f  x 

1
y  e1 x
2

2
ln  2 y   1  x 2
x   1  ln  2 y 

sgexamguru.com
Since x  0, x   1  ln  2 y 

1
f 1 : x   1  ln  2 x  , x  , 0  x  e
2

(iv)  x
y  1  ln x 
Step 1
 y  1  ln  
2
 x  Step 2 x
y  1  ln     y   1  ln  
2 2

x
0 xe0 e
2
 0  x  2e

x
h : x   1  ln   , x  , 0  x  2e
2

HCI P1 Solutions 192


1

HWAA CHONG INSTITUTI


I ION
2016 JC2 P
PRELIMINA
ARY EXAMMINATION

MATH
HEMATICS
Highe
er 2
9
9740/0
02
Paperr 2
Tuesda
ay 20 Septtember 20
016 3h
hours

als:
Additional materia A
Answer paper
L
List of Form
mula (MF15
5)

READ THESE INSTRUCTIO


ONS FIRST

Write yyour name and CT cla ass on all th


he work yo ou hand in, including the Cover Page.
Write in
n dark bluee or black p
pen on both h sides of tthe paper.
You ma ay use a so
oft pencil fo
or any diaggrams or grraphs.
Do not use staple es, paper clips, highligghters, gluee or correcction fluid.
Do not write anythhing on the e List of Fo
ormula (MF F15).

Answerr all the qu sgexamguru.com


uestions.
Give no on-exact numerical answers
a orrect to 3 ssignificant figures, orr 1 decimal place in
co
the casse of angle es in degreees, unlesss a differeent level off accuracyy is specifieed in the
questio
on.
You are e expected d to use a graphing
g ca
alculator.
Unsupp ported ansswers from m a graphing calcu ulator are allowed unless a question
specificcally statess otherwise
e.
Where unsupportted answerrs from a g graphing ccalculator aare not alloowed in a question,
you are e required to present the math hematical ssteps using g mathema atical notattions and
not calcculator com mmands.
You are e reminded d of the nee
ed for clea
ar presentation in your answers.

The nuumber of m marks is ggiven in brackets [ ] at the e end of eacch question n or part
questio end of the examinatio
on. At the e on, place th
he completted cover p page on to op of your
answerr scripts an
nd fasten all your work securely together w
with the striing provide
ed.

stion pape
This ques s of 6 prin
er consists nted pages
s.

[T
Turn over

Microsoft Word - 2016 HCI JC2 Prelim Paper 2 (Question) 193


2

Section A: Pure Mathematics [40 marks]

1. On the first day of last month, the temperature of a machine in a manufacturing plant was
found to be 90 C and was deemed as too hot by the supervisor. On each day waste heat
was produced by the machine as a by-product and caused the temperature of the machine to
increase by 2 C . In an attempt to make it cooler, the supervisor decided to adjust the
thermostat and decreased the temperature by 3% at the end of each day.
(i) Show that the temperature of the machine was 87.8 C at the end of the 3rd day. [2]
(ii) At the end of which day would the temperature of the machine first dropped below
70C ? [3]
(iii) Will the temperature continue to drop indefinitely? Justify your answer. If not, what
is the long term temperature of the machine? [3]

i
2. (a) Given that z1   is a root of the equation 2z3  i 8 z2  az 13i  0, find the
2
complex number a and solve the equation, giving your answer in Cartesian form
x  iy. [4]
Hence, find in Cartesian form the roots of the equation
2w3  18i w2  aw13  0.
sgexamguru.com
[2]
(b) Solve the equation z6  729  0, expressing your answers in the form re i , where
r  0 and      . [2]

Given that z1 is a root of the above equation and 0  arg z1  .
2
z1 n
If is a positive real number, find the smallest positive integer n. [2]
z1*

 x
3. (i) Use the substitution x  3 sin   1, 0    to find  dx . [4]
2 9   x  1
2

(ii) A curve has parametric equations

1
x , y  t 2 , 0  t  4.
9   t  1
2

(a) Sketch the curve, indicating the end point and the equation of the asymptote.
[3]
(b) Using the result in part (i), find the exact area of the region bounded by the
25 8
curve, the lines y  1, y  and x  . [4]
4 7
[Turn over

HCI P2 Questions 194


3

4. In harvesting of renewable natural resources, it is desirable that policies are formulated to


allow maximal harvest of the natural resources, and yet not deplete the resources below a
sustainable level. A simple harvesting model devised for the rate of change of the
population of wild salmon in a particular region in the Pacific Ocean is given by
dP
 P 4  P  h ,
dt
where P is the population of wild salmon in millions at time t years and h is the constant
harvest rate in millions.
dP
(i) Sketch a graph of against P, expressing the turning point in terms of h. [2]
dt
(ii) The Maximum Sustainable Yield (MSY) is the largest harvest rate h that allows for a
sustainable harvest of wild salmon without long-term depletion. State the MSY for
the wild salmon. [1]
(iii) It is given that the population of wild salmon in that region was 3.2 million in 2015
and the constant harvest rate is 3 million. Find an expression for P at any time t. [5]

sgexamguru.com
Hence find the population of the wild salmon in that region in 2016. [2]
(iv) State one assumption you made in your calculation. [1]

Section B: Statistics [60 marks]

5. In the last election, there were speculations from unofficial sources before the counting of
votes is completed. For the current election, to prevent unnecessary speculations, the
election office of Sunny Island will be conducting a sample count in each electoral division
after voting is done. Each electoral division has a different number of registered voters and
a sample of 400 votes will to be sampled from each electoral division.

(i) Identify and describe an appropriate method to obtain the sample. [3]

(ii) State an advantage and a disadvantage of the sampling method used in part (i). [2]

[Turn over

HCI P2 Questions 195


4

6. The random variable X has a binomial distribution B  n, p  , where 0  p  1 , and n is an

P X  r  n  r 1 p 
integer. Show that   . [3]
P  X  r  1  r  1 p 
Hence find a condition relating n and p such that X has two values for its mode, and
determine these two values, giving your answer in terms of n and p . [3]

7. A group of ten people consists of four single women, two single men and 2 couples. The
ten people are arranged randomly in a circle.

(i) Find the probability that the four single women are all separated. [2]

(ii) Find the probability that either the four single women are next to one another or the
two single men are next to each other but not both. [3]

One of the ten people left the group and the remaining nine decided to sit at a round table
with ten identical chairs equally spaced around the table. The chairs are decorated such that

sgexamguru.com
every alternate chair is tied with an identical chair sash. Given that the nine people have no
preference to which seat to take, find the number of possible seating arrangements. [2]

8. In an examination, the score, X , for paper 1 of a student is found to follow a normal


distribution with mean 62 and standard deviation  , and the score, Y , for paper 2 of a
student is found to follow a normal distribution with mean 71 and standard deviation 8.
The final score of a student for the examination is the average score of the 2 papers and it
is assumed that X and Y are independent random variables.

(i) Find the probability that for two randomly selected students A and B taking the
examination for paper 2, A has at most 2 marks less than the marks of B . [2]

(ii) Given that 15% of the students have at least a final score of 75, find  . [4]

(iii) Using the value of  found in part (ii), find the probability that a randomly selected
student performs better in her Paper 1 than in her Paper 2.
(iv) Comment on the validity of the answer obtained in part (ii) and (iii). [3]

[Turn over

HCI P2 Questions 196


5

9. To reduce the number of speeding incidents on the road, traffic police in Country S set up
traffic cameras at 3 busy traffic Junctions A , B and C to monitor the speeds of vehicles
passing through these junctions. The average number of speeding vehicles caught by the
camera at Junctions A , B and C are 2 in every 3 hours, 5 in every 4 hours and  in every
hour respectively. It is assumed that the number of speeding vehicles caught by the
cameras at the three junctions followed Poisson distributions.

(a) Find the probability that there are at least 2 speeding vehicles caught at Junctions A
and B in an hour. [2]

(b) Given that there are 2 speeding vehicles caught at the three junctions in an hour, find
the probability that at least one speeding vehicle caught is at Junction C . Leave your
answer in terms of  . [3]

(c) Given that the traffic cameras are in operation 24 hours in a day, using a suitable
approximation, find the probability that there will be more speeding vehicles caught
at Junction A than at Junction B in a day. State an assumption for the calculation to
be valid. [5]

10. sgexamguru.com
A nutritionist claims that the mean number of calories in an energy bar is 350 cal. The
nutritionist collected and measured the number of calories of a random sample of 15
energy bars. The mean and variance of the sample was 347.2 cal and 20.74 cal2
respectively.

(i) The nutritionist wishes to carry out a hypothesis test on his claim. Explain why t -test
instead of z -test is to be used. State an assumption for the test to be valid. [2]

(ii) Test at 5% level of significance, whether the mean number of calories in an energy
bar is 350 cal, defining any symbols that you use. [4]

(iii) Suppose the nutritionist uses a different test in part (ii). Without further calculation,
explain and state whether the conclusion will be different. [2]

The manufacturer of the energy bar refines the manufacturing process and the new energy
bars follow a normal distribution with mean  cal and variance 20.74 cal2. The
manufacturer then provides the nutritionist with another sample of 15 energy bars.

[Turn over

HCI P2 Questions 197


6

(iv) Find the range of mean number of calories, x , of the second sample of 15 energy bar
so that the null hypothesis in part (ii) is not rejected at 5% level of significance.
Leave your answer correct to one decimal place. [3]

11. A group of scientists is interested to find out the correlation between the number of species
and the size of the natural habitat. The scientists sampled non-overlapping lands of
different areas ( x ) in square kilometres, and noted the corresponding number of
species ( y ) found. The results are shown in the table below.

Area ( x ) 300 400 500 600 700 800 900 1000 1100 1200

Number of
species 12 15 18 21 k 25 26 27 27 28
(y)

(i) Given that the equation of the regression line is y  0.01758 x  9.018 , show that the
value of k to the nearest whole number is 23. [3]

sgexamguru.com
Take k to be 23.

(ii) Draw the scatter diagram for the given data, labelling the axes clearly. [2]

(iii) Calculate the product moment correlation coefficient r . With reference to both the
scatter diagram and r , explain why a linear model is not appropriate. [2]

(iv) The following models are suggested for the data.

(A) y  a  bx 2 , (B) y  ax b , where a  0 and 0  b  1 .

Use a graphical approach to determine which model is more appropriate. [2]

(v) Use the more appropriate model to estimate the area of the natural habitat when the
number of species found is 24. Comment on the reliability of your estimation. [3]

End of Paper

[Turn over

HCI P2 Questions 198


2016 Prelim Paper 2 Solutions
Qn Solution
1 1st day: 0.97  92 
(i)
2nd day: 0.97  0.97  92   2 

3rd day: 0.97 0.97  0.97  92   2   2 
 0.973  92   0.972  2   0.97  2 
 87.788
 87.8  C (3 s.f.)
(ii) nth day:  0.97 n  92   0.97n1  2   ...  0.97  2 
 0.97 1  0.97n 1  
 0.97  92   2 
n

 1  0.97 
 
Method 1

Method 2
sgexamguru.com
Using GC,
When n  51 , temperature  70.025C
When n  52 , temperature  69.865C  52 days
(iii) No, temperature will not drop infinitely since r  0.97  1 .
2  0.97
As n   , temperature approaches 0   64.7 C in the long run.
1  0.97

2  i
3
 i
2
 i
(a) 2     (i  8)     a     13i  0  
 2  2  2
1 1 ai
i  2  i   13i  0  
4 4 2
a  26  4i  
 
2 z 3  (i  8) z 2  (26  4i) z  13i  (2 z  i)( z 2  bz  13)
Comparing the coefficient of z 2 ,
i  8  2b  i
 b  4

i
(2 z  i)( z 2  4 z  13)  0  z   or z  2  3i.
2

Microsoft Word - 2016 HCI JC2 Prelim Paper 2 (Solution) 199


Replace z with iw,
2(iw)3  (i  8)(iw) 2  a(iw)  13i  0
2iw3  (8  i) w2  aiw  13i  0
Dividing throughout by i ,
2w3  (1  8i) w2  aw  13  0.
i
iw   or iw  2  3i.
2
1
 w   or w  3  2i
2
(b) z  729  729ei  729e(  2 k )i  
6

  2 k 
  i
z  3e 6 6 
, k  0, 1, 2, 3  
5 5    
 i i  i i  i i
OR  z  3e 6
,3e 6
,3e 2 2
,3e ,3e 6
,3e 6  
 

i
z1  3e 6
zn 
arg  1*   n arg z1  arg z1  (n  1)
 z1  6

Positive real number (n  1)  2k  
6

 (n  1)  2  
6
Minimum n  11.  
sgexamguru.com
3 x  3sin   1
(i) dx
 3cos 
d
x
 9  ( x  1)2
dx,

3sin   1
 (3cos ) d
9  (3sin  )2
  (3sin  1) d
 3cos     C
x 1
  9  ( x  1) 2  sin 1 C
3
   

HCI P2 Solutions 200


(ii)
y
(a)

y = 16

O x
 

(ii)
(b) y

O x
 

 25  8
25
Area   
sgexamguru.com
x dy    1 
4
1
 4  7

5
2t
  2
dt  6
1
9  (t  1)2
5
 t  1 2
 2   9  (t  1)2  sin 1 6
 3 1

 27 1 
 2 (   sin 1 )  (  9  sin 1 0)   6
 4 2 

3 3
3
4(i) dP
 P (4  P)  h     
dt
 P2  4P  h  
        ( P  4 P)  h  
2

 
 ( P  2)2  4  h
  P
  O  2

 
  

HCI P2 Solutions 201


(ii) From graph in (i),   
when h  4 ,
dP P
O 2
 ( P  2) 2  0 at P  2 .
dt
 largest h  4 where the population P remains
constant at P  2 .
Hence MSY  4 million

(iii) dP
 P (4  P )  3   ( P 2  4 P )  3   ( P  2) 2  1
dt
Method 1
1
 ( P  2)2  1 dP   1 dt
1 ( P  2)  1
ln  t  C
2 ( P  2)  1
P 3
  e  2t  2C  Ae  2t where A   e2C
P 1
1
In 2015, let t  0 , P  3.2 ; hence A 
11
1
sgexamguru.com
 P  3  e  2t ( P  1)
11
11P  33  Pe  2 t  e  2 t
33  e 2t 33e2t  1
Hence P  
11  e 2t 11e 2t  1

Method 2
dP
 1  ( P  2) 2
dt
1
 1  ( P  2)2 dP   1 dt
1 1  ( P  2)
ln  t C
2 1  ( P  2)
P 1
  e 2t  2C  Ae 2t where A   e2C
3 P
In 2015, let t  0 , P  3.2 ; hence A  11
P  1  11e 2t (3  P)
P  1  33e 2t  11Pe 2t
1  33e2t 33e2t  1
P or P   
1  11e 2t 11e 2t  1

HCI P2 Solutions 202


Method 3
1
 2 dP   1 dt
P  4P  3
1
 dP   1 dt
( P  3)( P  1)
1 1
 dP   dP   1 dt (using partial fractions)
2( P  3) 2( P  1)
1 P 1
ln  t C
2 P 3
P 1
  e 2 t  2 C  Ae 2 t
P 3
In 2015, let t  0 , P  3.2 ; hence A  11
P  1  11e 2t ( P  3)
P  1  11Pe 2t  33e 2t
1  33e2t 33e2t  1
P or P 
1  11e 2t 11e 2t  1

In 2016, t  1
33e 2  1
Hence P   3.02
11e 2  1
 the population of wild salmon is 3.02 million in 2016.
(iv)
sgexamguru.com
There are no external factors such as marine pollution or climate change that
drastically affect the population of wild salmon in that region.

5 Simple Random Sampling:


(i) Using a random number generator to generate 400 numbers and use select the voting
slips corresponding to these 400 numbers

Systematic Sampling: Consider N registered voter in the electoral division such that
the sampling interval N is an integer. Using a random number generator, select a
400
number from 1 to k and take every kth number thereafter until a sample of 400 is
obtained. Choose the voting slips corresponding to the numbers.

Stratified Sampling:
Use each polling station as the stratum. The number of votes in each stratum is
no. of voters polling at the station
calculated by  400 . The votes is then
total number of voters in the electoral division
obtained from each stratum using simple random sampling.
(ii) Simple random sampling:
Advantage:
The sample obtained is free from bias
The sampling procedures are easy to follow
Disadvantage
The sample obtained might not be a good representation of the electoral division

HCI P2 Solutions 203


Systematic Sampling:
Advantage:
It is easy execute because only the first number needs to be chosen
The electoral division will be evenly sampled as the voting slips is chosen at regular
intervals
Disadvantage:
If there is a periodic trend like every kth voters are of the same gender, systematic
sampling may produce a biased sample

Stratified Sampling:
Advantage:
Stratified sampling will provide a sample of voter that is representative of electoral
division
The results in each polling station can be analysed separately.
Easy to conduct as the sampling frame (registered voters) is known.
Disadvantage:
It is time consuming to carry out stratified sampling
6 P( X  r  1)  P( X  r )
 n  r 1 n  r 1 n
  p (1  p)    p r (1  p) n  r
 r  1 r
n! n!
p r 1 (1  p ) n  r 1  p r (1  p ) n  r
( r  1)!( n  r  1)! ( r )!( n  r )!
r !(n  r )! p r (1  p ) n  r

r

sgexamguru.com
(r  1)!(n  r  1)! p r 1 (1  p ) n  r 1
p
n  r 1 1 p
P( X  r )  n  r 1  p
  (shown)
P( X  r  1)  r 1 p
r (1  p )  p (n  r  1)
r  pr  np  pr  p
r  (n  1) p

X will have two modes when ( n  1) p is a positive integer.


r  (n  1) p , r  ( n  1) p  1
7 No of ways that the single women are all separated
(i) = 6 C4  4!  (6  1)! =43200
43200 5
Probability = =  0.119
9! 42
(ii) Probability that the single women are next to one another
(7  1)!  4! 1
= P( S )  
9! 21
Probability that the single men are next to each other
(9  1)!  2! 2
= P( B )  
9! 9

HCI P2 Solutions 204


Probability that the single women are next to one another and the single men are next
to each other
(6  1)!  2!  4! 1
= P( S  B )  
9! 63
1 2 2 5
Therefore probability = P( S )  P( B )  2P( S  B )      0.238
21 9 63 21
No of ways = 9! 2!  725760
8 A, B ~ N(71,82 )
(i) A  B ~ N(0,128)
P(0  A  B  2)  0.0702 (3.s.f)
(ii) X ~ N(62,  2 ) , Y ~ N(71,82 )
X Y  2  82
Let M  ~ N(66.5, )
2 4
P( M  75)  0.15
P( M  75)  0.85
75  66.5
P( Z  )  0.85
 2  64
4
8.5
 1.03643338
 2  64

sgexamguru.com
4
  14.319
  14.3
(iii) X  Y ~ N(9, 269.0389)
P( X  Y )  P( X  Y  0)
 0.292 (3 s.f.)
(iv) Not valid because X and Y are not be independent for the same student.
9(a) Let X be the total number of speeding incidents caught at Junctions A and B in an
hour.
23
X ~ Po( )
12
P( X  2)  1  P( X  1)  0.571
(b)  23 
A  B  C  ~ Po    
 12 
Required Probability  P  C  1| A  B  C  2 
P  A  B  1 P  C  1  P  A  B  0  P  C  2 

P  A  B  C  2
 23  12
23
   12
23
    2 
   e 2 

 e  e  e
  12    
2
 23 

  12    12
23

e 
  2

HCI P2 Solutions 205


23
 
e 12
 23 2
   

2  6 
23
  2
e 12
 23 
 
2  12 

 23  6 
 6
1
 23  12 
2

144
24  23  6 

 23  12 
2

(c) Let A be the number of speeding incidents caught at Junctions A, and B be the
number of speeding incidents caught at Junction B in a day
A ~ Po(16) , B ~ Po(30)

Since both 16 and 30 are greater than 10,


A ~ N(16,16) and B ~ N(30, 30) approximately

 A  B ~ N( 14, 46)

P( A  B  0) 
c .c
 P( A  B  0.5) = 0.0163

sgexamguru.com
The occurrence of speeding incidents caught at Junction A and Junction B are
independent of each other.

10 Since n is small and population variance unknown, the nutritionist should use t-test. It
(i) is assumed that the calories count of the energy bar follows a normal distribution.
(ii) 15
s2   20.74   22.221
14
Let H 0 be the null hypothesis, H1 be the alternative hypothesis. Let  be the
population mean number of calories in an energy bar and X be the sample mean.
H 0 :   350
H1 :   350
X  350
Under H 0 , Test statistic, T  ~ t14
22.221
15
p value = 0.0373 < 0.05

Since the p – value < 0.05, reject H 0 . There is sufficient evidence at 5% level of
significance to conclude that the mean number of calories in an energy bar is not 350.

(iii) Since H 0 is rejected for t-test, pt  0.05 and since pz  pt  0.05 . Therefore H 0
will be rejected under z test. So the conclusion will not be different.

HCI P2 Solutions 206


A
Alternative method
N(0
0,1)

t(14)

test stat
H0 is rejecteed under t test
 test statiistic is insid
de critical reegion for t-teest
 test statiistic is insid
de critical reegion for Z-test
 H0 is rejected underr Z test
The conclusion would be the samee.

(iv) Since H 0 iss not rejected,


x  350
 1.95996
1.3827
x  350
0
 1.959996   1.959966
1.3827
 2.3047  x  350  2..3047
347.7  x  352.3

sgexamguru.com
11(i) x  750
y  0.01758  750   9.018
 22.203
22.203 10   k  199
 k  23.033  23

(ii)
y

28

12
30
00 1200
x/
(iii) r  0.958 . Though r is close to 1 , the shape of the scatteer plot is cuurvilinear th
herefore
ppropriate
a linear model is not ap
   

HCI P2 Solutions 207


(iv) y

x
y
y  ax b   

x
M
Model B is a more app propriate moodel as grap
ph concave downwardss like the sccatter
diagram
ln y  0.966684  0.61722 ln x
ln y  0.9667  0.617 ln x
ln  24   0.96684
0  0.61722 ln x
0.61722 ln x  ln  24   0.96684
x  825
Since r  0
sgexamguru.com
0.982 is clo
appropriatee
ose to 1 and y = 24 is within
w the daata range, thhe prediction
n is

HCI P2 Solutions 208


INNOVA JUNIOR COLLEGE
JC 2 PRELIMINARY EXAMINATION
in preparation for General Certificate of Education Advanced Level
Higher 2

CANDIDATE
NAME

CIVICS GROUP INDEX NUMBER

Mathematics 9740/01
Paper 1 22 August 2016
3 hours
Additional materials: Answer Paper
Cover Page
List of Formulae (MF 15)

READ THESE INSTRUCTIONS FIRST

Do not open this booklet until you are told to do so.

sgexamguru.com
Write your name, class and index number on all the work you hand in.

Write in dark blue or black pen on both sides of the paper. You may use a soft pencil for any
diagrams or graphs.
Do not use staples, paper clips, highlighters, glue or correction fluid.

Answer all the questions.

Give non-exact numerical answers correct to 3 significant figures, or 1 decimal place in the
case of angles in degrees, unless a different level of accuracy is specified in the question.
You are expected to use a graphic calculator.

Unsupported answers from a graphic calculator are allowed unless a question specifically
states otherwise.

Where unsupported answers from a graphic calculator are not allowed in a question, you
are required to present the mathematical steps using mathematical notations and not
calculator commands.

You are reminded of the need for clear presentation in your answers.

At the end of the examination, fasten all your work securely together.

The number of marks is given in brackets [ ] at the end of each question or part question.

This document consists of 6 printed pages.

Innova Junior College [Turn over

IJC P1 Questions 209


2

1 A theme park sells tickets at different prices according to the age of the customer. The
age categories are senior citizen (ages 60 and above), adult (ages 13 to 59) and child (ages
4 to 12). Four tour groups visited the theme park on the same day. The numbers in each
category for three of the groups, together with the total cost of the tickets for each of these
groups, are given in the following table.

Group Senior Citizen Adult Child Total cost


A 2 19 9 $1982
B 0 10 3 $908
C 1 7 4 $778

Find the total cost of the tickets for Tour Group D, which consists of four senior citizens,
five adults and one child. [4]

2 y

sgexamguru.com
O x

The diagram shows the curve y  f ( x) . The curve passes through the point A  a, 0  and

1
the point B  b, 0  , has a turning point at C  c, 1 and asymptotes y  and x  0 .
2
Sketch, on separate diagrams, the graphs of

(a) y  3  f ( x) , [3]

2
(b) y . [3]
f ( x)
Label the graph in each case clearly and indicate the equations of the asymptotes and the
coordinates of the points corresponding to A, B and C.

IJC/2016/JC2 9740/01/Aug/16

IJC P1 Questions 210


3

3 In the triangle ABC, AB = 1, BC = 4 and angle ABC =  radians. Given that  is a


sufficiently small angle, show that
1
AC   9  4 2 2
  a  b 2 ,

for constants a and b to be determined. [5]

1
4 [It is given that the volume of a pyramid is  (base area)  (height) .]
3
A right pyramid of vertical height h m has a square base with side of length 2x m
8 3
and volume m.
3
(i) Express h in terms of x. [1]

(ii) Show that the surface area S m2 of the pyramid is given by


  4  
S  4 x2  1   1   . [3]
  x6  

(iii) sgexamguru.com
Use differentiation to find the value of x, correct to 2 decimal places, that gives a
stationary value of S. [3]

 
5 Referred to the origin O, the points A and B are such that OA  a and OB  b . The point
C on OA is such OC : OA = 1 : 3. The line l passes through the points A and B. It is given

that angle BOA  60 and a  3 b .

(i) By considering  b  a    b  a  , or otherwise, express b  a in the form k b ,

where k is a constant to be found in exact form. [3]

(ii) Find, in terms of b , the shortest distance from C to l. [5]

IJC/2016/JC2 9740/01/Aug/16 [Turn over

IJC P1 Questions 211


4

6 A curve has parametric equations


 
x  cos2  , y  sin 2 , for    .
2 2
(i) Sketch the curve. [2]

The region enclosed by the curve is denoted by R. The part of R above the x-axis is rotated
through 2 radians about the x-axis.

(ii) Show that the volume of the solid formed is given by

b
  a sin 2 d ,
3

for limits a and b to be determined. [3]

Use the substitution u  cos 2 to find this volume, leaving your answer in exact
form. [4]

7 The equation of a curve C is given by

3 y3  8 y 2  10 y  4  5x .

(i)
sgexamguru.com
Find the equation of the tangent at the point where x 
4
. [5]
5
(ii) Find the Maclaurin series for y , up to and including the term in x 2 . [4]

(iii) State the equation of the tangent to the curve C at the point where x  0 . [1]

8 (a)  
The complex number w is given by    k i , where k  0 .

 
Given that w5 is real, find the possible values of k in the form k    tan  n  ,

where n is a constant to be determined. [4]


(b) (i) If z  cos   i sin  , where 0    , show that
2
1  z 2  2sin   sin   i cos  . [2]

(ii) Hence find 1  z 2 and arg(1  z 2 ) in terms of  . [3]

IJC/2016/JC2 9740/01/Aug/16

IJC P1 Questions 212


5

9 The function f is defined by


1
f :x  2 , x  ℝ, x  2, x  3 .
x  x6
2

(i) Explain why the function f 1 does not exist. [2]


(ii) Find, algebraically, the set of values of x for which f is decreasing. [3]

1
In the rest of the question, the domain of f is further restricted to x  .
2
The function g is defined by
g: x 2  x, x  ℝ.

1  1 
(iii) Find an expression for gf ( x) and hence, or otherwise, find  gf    . [3]
4

1
10 A sequence u1 , u2 , u3 , is such that u1  and
2

sgexamguru.com
 
un1  un 
n2  n  1
n2 !
, for all n  1.

n
(i) Use the method of mathematical induction to prove that un  . [5]
 n  1!
N
n2  n  1
(ii) Hence find  . [3]
n 1  n  2  !


n2  n  1
(iii) Explain why   n  2 ! is a convergent series, and state the value of the sum to
n 1

infinity. [2]

IJC/2016/JC2 9740/01/Aug/16 [Turn over

IJC P1 Questions 213


6

11 y

C1

x
O

ln x
The diagram above shows the curve C1 with equation y  , where x  1 .
x2
 1 
(i) Show that the exact coordinates of the turning point on C1 are   e ,  . [3]
 2e 

 
The curve C2 has equation x   e   2ey   1 , where y  0 . Sketch C1 and C2
2 2
(ii)

on the same diagram, stating the exact coordinates of any points of intersection with
the axes. [3]

(iii) Write down an integral that gives the area of the smaller region bounded by the two

sgexamguru.com
curves, C1 and C2 , and the x-axis. Evaluate this integral numerically. [4]

12 (a) (i) Solve the equation


z 6  2i  0,

giving the roots in the form rei , where r  0 and      . [4]

(ii) Show the roots on an Argand diagram. [2]

(iii) The points A, B, C, D, E and F represent the roots z1 , z2 , z3 , z4 , z5 and z6


respectively in the Argand diagram. Find the perimeter of the polygon
ABCDEF, leaving your answer to 3 decimal places. [2]

(b) The complex number w satisfies the relations



w  5  12i  13 and 0  arg  w  18  12i   .
4
(i) On an Argand diagram, sketch the region in which the points representing w
can lie. [4]
(ii) State the maximum and minimum possible values of w  10 . [2]

IJC/2016/JC2 9740/01/Aug/16

IJC P1 Questions 214


ANNEX B
IJC H2 Preliminary Examination (Paper 1)
Qn/No Topic Set Answers
1 System of linear equations
$570
2 Further Curve Sketching
------
3 Small angle approximation, 2 2
Binomial expansion AC  3   2 , where a  3 and b 
3 3
4 Application of differentiation 2
(Stationary value) (i) h  2
x
(iii) 0.89
5 Vectors (scalar and cross-product) (i) b  a  7 b

3
(ii) b
7
6 Application of Integration (Volume 
of revolution) (ii) a  0, b 
2
2
 units3
3
7
sgexamguru.com
Application of differentiation
(Tangent & Normal),
Maclaurin Series
(i) y  
1
2
2 3
x 
2
5
27 2
 y   x
(ii) x  ...
3 2 20
2 3
(iii) y   x
3 2
8 Complex numbers    2 
(a) k  3 tan    or k  3 tan   
 5  5 

(b)(ii) 2sin  ;  
2
9 Functions (ii)  x  : x  0.5, x  3
1 1
(iii) gf  x    2 , x  ; 1
x  x6 2
10 Mathematical Induction, 1 N 1
(ii) 
Sequence & Series (M.O.D.) 2  N  2 !

n2  n  1 1
(iii)   n  2 !  2 which is a constant,
n 1

hence it is a convergent series.


1
S 
2

ANNEX B 215
 
11 Application of differentiation 2

(Stationary point), e 1 x  e e ln x
Curve Sketching, (iii)  e 1 2e
dx  
1 x2
dx ;
Application of Integration (Area)0.0543
12 Complex numbers (including Loci) (a)(i)
1 11 1 7 1  1  1 5 1 3
i i i i i i
6 12 6 12 6 4 6 12 6 12 6 4
2 e ,2 e ,2 e ,2 e ,2 e , 2 e
(a)(iii) 6.735
(b)(ii) Maximum w  10  26 ;
Minimum w  10  12

sgexamguru.com

IJC P1 Solutions B 216


Innova Junior College
H2 Mathematics
JC2 Preliminary Examinations Paper 1
Solutions

1 Solution
Let $x, $y and $z be the cost of a ticket for a senior citizen, adult and child
respectively.

2 x  19 y  9 z  1982
10 y  3 z  908
x  7 y  4 z  778

Using GC,
x  36
y  74
z  56

Thus, the cost of a ticket for a senior citizen is $36, for an adult is $74 and for a child is
$56.

sgexamguru.com
4(36) + 5(74) + 1(56) = 570
Therefore, the total cost for Group D = $570

IJC P1 Solutions 217


2 Solution
(a) y

(b) y

sgexamguru.com
O x

IJC P1 Solutions 218


3 Solution

Using cosine rule,


AC 2  12  42  2(1)(4) cos
 1  16  8cos
 17  8cos
 2 
 17  8  1  
 2 
 9  4 2
1
AC   9  4 
2 2
(  AC  0)
1
AC   9  4 2 2

1
1
 4 2 2
 92 1  9  
 
 14  
 3 1    2   ... 
 29  
 2 
 3 1   2 

sgexamguru.com
 9 
2
 3  2
3
2
Therefore, a = 3 and b 
3

IJC P1 Solutions 219


4 Solution
(i)
V

h
D C
2x
N

A 2x B
8
Volume of the pyramid =
3
1 8
  2 x 2 h 
3 3
2
h 2
x
(ii)
In triangle VBC, height = VN = h2  x2
1
Area of the triangle VBC =  2 x  h2  x2
2

sgexamguru.com
4
=x 4
 x 2 = x x 2  46  1 
x x 

= x 2 1  4 x 6

Hence total surface area of the pyramid,


S  base area  4  area of triangle VBC
2

=  2 x   4 x 2 1  4 x 6 
  4 
S  4 x2 1   1  6   (shown)
  x 
(iii)
S  4x2 1 (1 4x6)
 
1 1 
dS 

dx 2

 4 x 2  1  4 x 6   24 x 7    8x  1   (1  4 x 6 ) 
2

 
1
5 
=  48 x 
1 4x 6 2
  8 x 1   (1  4 x 6 ) 
 
 1 



=  8 x  6 x 6 1  4 x 6  2  1   (1  4 x 6
)

 
dS
At the stationary value of S,  0.
dx

IJC P1 Solutions 220


 1 



  8x 6 x 6 1  4 x 6  2  1   (1  4 x 6 )   0

 
By G.C.,
x  0.89090  0.89 (to 2dp)

sgexamguru.com

IJC P1 Solutions 221


5 Solution
(i) 2
(b  a)  (b  a)  b  a  2a.b
2

2 2
 b  9 b  2 a b cos 60
2 2 1
b  a  10 b  2  3 b  b
2
b a  7 b
Therefore, k  7 .

(ii) 1
c a
3

 2
CA  a
3

Shortest distance of C to l =
2
a  (b  a)
3
b a
2 2
ab  aa
3 3

sgexamguru.com

ba
2 ab
 a  a  0
3 b a
2 a b sin 60

3 b a
3 2
6b
 2
3 7b
3b

7
3
 b
7

IJC P1 Solutions 222


6 Solution
(i)
 
x  cos2 , y  sin2, for     .
2 2

0 1 x
-1

(ii) x  cos 2 
d x  2 cos  sin  d 
When y  0, sin 2  0
2  0, 

  0,
2

When   0, x  1 ; When   , x0

sgexamguru.com
2
Volume of the solid formed
1
2
 0 y dx
0 2
   sin 2   2cos sin  d 
2
0
  
 sin 2 2   sin 2 d 
2

sin 3 2 d
2
 0 (shown)


 a  0, b  .
2

Let u  cos 2 .
 du  2 sin 2 d
When   0, u  1 ;

When   , u  1
2
Volume of the solid formed

2 3
 0 sin 2 d

IJC P1 Solutions 223



1 2 2
  0 sin 2  2sin 2  d
2

1
0 1  cos 2   2sin 2 d 
2 2
 
2
1 1
1 1  u    du 
2
 
2
1
1  u3 
   u  
2  3 
1

1  1   1 
    1     1   
2  3   3 
1  4
    
2  3
2
  units3
3

sgexamguru.com

IJC P1 Solutions 224


7 Solution
(i) 3 y3  8 y 2  10 y  4  5x
Differentiate wrt x ,
dy
9 y 2  16 y  10
dx

 5

dy 5
 2
dx 9 y  16 y  10
4 3 2
, 3 y  8 y  10 y  0 .
When x 
5
dy 1
 y  0 and  .
dx 2
1 4
Eqn of tangent : y  0    x   ,
2 5
1 2
ie y   x 
2 5
(ii) dy
9 y 2
 16 y  10 
dx
 5
2
d2 y  dy 
Differentiate wrt x ,  9 y  16 y  10  2  18 y  16     0 .
2

dx  dx 
sgexamguru.com
2 dy
3 dx
3 d 2 y 27
When x  0 , y  ,   , 2 
2 dx 10
.

2 3 27 2
 y   x x  ...
3 2 20

(iii) 2 3
y  x
3 2

IJC P1 Solutions 225


8 Solution
(a) arg  w5   5 arg  w   0,  , 2 ...
  2 2
arg  w   0, , , , ,...
5 5 5 5
Since k  0,
 2
arg  w    or  .
5 5
k   k  2 
 tan    or  tan   
3  5 3  5 
   2 
k  3 tan    or k  3 tan   
 5  5 
1 2
n   or 
5 5
(bi) Method 1

2
1  z 2  1   cos   i sin  


 1  cos 2   2i cos  sin    i sin  
2

 1  1  sin 2   2i sin  cos   sin 2  
 1  1  2sin 2   2i sin  cos 
sgexamguru.com
 2sin 2   2i sin  cos 
 2sin   sin   i cos  

Method 2

2
1  z 2  1   cos   i sin  
 1   cos 2  i sin 2 
 1  cos 2  i sin 2
 1  1  2sin 2    2i sin  cos 
 2sin 2   2i sin  cos 
 2sin   sin   i cos  
(bii) Method 1

1  z 2  2 sin   sin   i cos  

 2sin  sin 2   cos 2 


 2sin 


Given that 0   
2

IJC P1 Solutions 226


arg 1  z 2   arg  2 sin   sin   i cos   
 arg  2 sin    arg  sin   i cos  
 cos  
 0  tan 1  
 sin  
  
  tan 1  tan     
 2 
 
   
2 

 
2

Method 2

1  z 2  2sin   sin   i cos  


 2sin   i  cos   i sin  
  2i sin   ei
1  z 2   2i sin   ei
 2sin 

sgexamguru.com
arg 1  z 2   arg   2i sin   ei 

   
 arg 2i sin   arg ei

 
2

Method 3
1  z 2  2sin   sin   i cos  
    
 2sin   cos      i sin     
 2  2 
     
 2sin   cos     +i sin     
  2  2 
1  z 2  2 sin 

arg 1  z 2    
2

IJC P1 Solutions 227


9 Solution
(i)

From graph, the horizontal line y  3 cuts the graph at two points. Hence f is not a
one-one function, hence f 1 does not exist.
(ii) 1
f  x  2 2
x  x6
2
f '  x     x2  x  6  2 x  1
1  2x
 2
x 2
 x  6
For the function to be decreasing, f '  x   0 .
1 2x  0
1  2x
x  0.5
sgexamguru.com
x : x  0.5, x  3
(iii) 1
gf  x   2  2 2
x  x6
1 1
 2 , x
x  x6 2

11
 gf  4  x
 
1
gf  x  
4
1 1
 2 
x  x6 4
x 2  x  6  4
x2  x  2  0
 x  2  x  1  0
x  2 (rejected) or x  1
x  1

IJC P1 Solutions 228


10 Solution
(i) n
Let Pn be the statement un  for n    .
 
n  1 !
1 1
P1 is true since u1   .
2! 2
Assume that Pk is true for some k    ,
k
i.e. uk 
 k 1!
Consider Pk 1 :

i.e. u k 1  k  1
 k  2 !
k k2  k 1
u k 1  
 k  1 !  k  2  !



k k  2  k 2  k 1 
 k  2 !
k 2  2k  k 2  k  1

 k  2 !

sgexamguru.com
k 1

 k  2 !
Thus, Pk is true  Pk 1 is true.

Since P1 is true, and Pk is true  Pk 1 is true, by mathematical induction, Pn is true



for all n   .

(ii) N
n2  n 1 N
  n  2 !    u n  u n 1 
n 1 n 1

[ u1  u 2
 u 2  u3
 u3  u 4

 u N 1  u N
 u N  u N 1 ]
1 N 1
= u1  u N 1  
2  N  2!

IJC P1 Solutions 229


(iii) N 1
As N   , 0
 N  2 !

n2  n 1 1
  n  2 !  2 which is a constant, hence it is a convergent series.
n 1

sgexamguru.com

IJC P1 Solutions 230


11 Solution
(i) ln x
y  2 , x 1
x
1
x 2   ln x  2 x
dy x

dx x4
x  2 x ln x

x4
dy
When  0 and since x  0 ,
dx
1  2ln x  0
2ln x  1
1
ln x 
2
1
x  e2  e
When x e ,
ln e
y 2
 e
sgexamguru.com
1

2e
1
Hence the coordinates of A is  e,  .
 2e 
(ii) B 1, 0  , D  
e  1, 0 and E  e  1, 0 

y A

C1
C2
x
D B E

(iii) Area of the smaller region R


2


e 
1 x  e  dx  
e ln x
dx
e 1 2e 1 x2
 0.14446942  0.09020401
 0.05426541
 0.0543 (correct to 3 s.f.)

IJC P1 Solutions 231


12 Solution
(ai) z 6  2i  0
z 6  2i
 
i  2 k 
6 2 
z  2e , k  0, 1, 2, 3
1 11 1 7 1  1  1 5 1 3
i i i i i i
6 12 6 12 6 4 6 12 6 12 6 4
z2 e ,2 e ,2 e ,2 e ,2 e , 2 e
(aii) Im

Re

(aiii) Since ABCDEF is a regular hexagon, the triangles OAB, OBC … are equilateral
triangles.
Perimeter of the polygon

sgexamguru.com
1
 6  26
 6.735 (to 3 d.p.)
(bi)
Im

12
13
Re
-10 -5 O

(bii) Minimum w10 12


Maximum w10  26 (diameter of circle)

IJC P1 Solutions 232


1

INNOVA JUNIOR COLLEGE


JC 2 PRELIMINARY EXAMINATION
in preparation for General Certificate of Education Advanced Level
Higher 2

CANDIDATE
NAME

CIVICS GROUP INDEX NUMBER

Mathematics 9740/02
Paper 2 24 August 2016
3 hours
Additional materials: Answer Paper
Cover Page
List of Formulae (MF 15)

READ THESE INSTRUCTIONS FIRST

Do not open this booklet until you are told to do so.

sgexamguru.com
Write your name, class and index number on all the work you hand in.
Write in dark blue or black pen on both sides of the paper.
You may use a soft pencil for any diagrams or graphs.
Do not use staples, paper clips, highlighters, glue or correction fluid.

Answer all the questions.


Give non-exact numerical answers correct to 3 significant figures, or 1 decimal place in the
case of angles in degrees, unless a different level of accuracy is specified in the question.
You are expected to use a graphic calculator.
Unsupported answers from a graphic calculator are allowed unless a question specifically
states otherwise.
Where unsupported answers from a graphic calculator are not allowed in a question, you are
required to present the mathematical steps using mathematical notations and not calculator
commands.
You are reminded of the need for clear presentation in your answers.

At the end of the examination, fasten all your work securely together.
The number of marks is given in brackets [ ] at the end of each question or part question.

This document consists of 6 printed pages.

IJC/2016/JC2 Innova Junior College 9740/02/Aug/2016 [Turn over


IJC P2 Questions 233
2

Section A: Pure Mathematics [40 marks]

1 Without using a calculator, solve the inequality


3 x
 . [5]
4x  3 x 1

Hence, or otherwise, solve the inequality

3 ex
 . [2]
4e x  3 ex 1

2 Analysts estimate that when a viral video is posted online, the video attracts comments
in such a way that at the end of every hour, the number of comments added for the video
is thrice the number of comments at the start of that hour.

In a particular instance, a viral video was posted online and there was one comment
immediately after the video was posted. Using the above model proposed by analysts,
there will be 3 additional comments by the end of the first hour, 12 additional comments
by the end of the second hour, and so on.

(i) Find the number of complete hours for the total number of comments posted online

sgexamguru.com
to exceed 200 000.

When the number of these comments posted online reaches 200 000 exactly, Software X
[3]

is immediately activated to remove the comments. Software X works in such a way that
it removes x comments at the start of each day. Once Software X is activated, it is also
known that the number of comments at the end of the day is 2% more than the number of
comments at the start of the day.

(ii) Show that the number of comments at the end of day n is

1.02n  200 000   51x 1.02n  1 ,


where day 1 is the day that the number of comments is exactly 200 000. [3]

(iii) Hence find the range of values of x such that all comments are removed by the end
of day 30. Leave your answer to the nearest integer. [2]

Software Y is able to remove comments at the following rate.


 Day 1: 15 000 comments removed
 Subsequent Day: 90% of the number of comments removed on the preceding
day
Without using Software X, explain whether Software Y alone is able to remove all
200 000 comments eventually. [2]

IJC/2016/JC2 9740/02/Aug/2016

IJC P2 Questions 234


3

3 A team of naturalists is studying the change in population of wild boars on an island. It


is suggested that the population of wild boars, x hundred, at time t years, can be modelled
by the differential equation
dx 1
 x (5  x ) .
dt 10
(i) Find an expression for x in terms of t, given that x  1 when t  0 . [7]
(ii) Find the exact time taken for the population of wild boars to reach 200. [2]
(iii) Explain in simple terms what will eventually happen to the population of wild
boars on the island using this model. [1]

x 1 z7
4 The line l has equation y , and the plane p has equation x  z  2 .
2 4
(i) Find the acute angle between l and p. [3]
(ii) Find the coordinates of the point at which l intersects p. [3]
(iii) The perpendicular to p from the point with coordinates (1,0, 7) meets p at the
point N. Find the position vector of N. [4]
(iv) Find a vector equation of the line which is a reflection of l in p. [3]

sgexamguru.com
Section B: Statistics [60 marks]

5 A company wants to find out the transportation habits of their employees. On one
particular workday, the interviewer selects a sample of employees to interview from those
walking into the company building by
 standing at the entrance of building and choosing at random one of the first 10
employees who walks into the building,
 then choosing every 10th employee after the first employee is chosen.
(i) What is this type of sampling method called? [1]
(ii) State, in this context, a disadvantage of the sampling method stated in part (i). [1]
(iii) Explain briefly how the interviewer could select a sample of 30 employees using
quota sampling. [2]

6 Historical data shows that the number of goals scored per match at European Football
Championships has a mean of 1.93 and a variance of 1.4. A large random sample of n
matches is taken. Find the least value of n such that the probability that the average
number of goals scored per match exceeds 2 goals is less than 0.24. [5]

IJC/2016/JC2 9740/02/Aug/2016 [Turn over


IJC P2 Questions 235
4

7 A class of twenty four pupils consists of 11 girls and 13 boys. To form the class
committee, four of the pupils are chosen at random as “Chairperson”, “Vice Chairperson”,
“Treasurer” and “Secretary”.

(i) Find the probability that the committee will consist of at least one girl and at least
one boy. [3]

(ii) Find the probability that the “Treasurer” and “Secretary” are both girls. [3]

8 Under normal continuous use, the average battery life of a PI-99 calculator is claimed to
be k hours . A random sample of 13 calculators were obtained, and the battery
life, x hours, of each calculator was measured. The results are summarised by

 x  573.39  x  x   42.22 .
2
and

(i) Find unbiased estimates of the population mean and variance. [2]

A test is to be carried out at the 5% level of significance to determine if the claim made
is valid.

(ii) State a necessary assumption to carry out the test. [1]

(iii) State the appropriate hypotheses for the test, defining any symbols that

(iv)
sgexamguru.com
you use. [2]

Find the set of values of k for which the result of the test would be that the null
hypothesis is not rejected. Leave all numerical answers in 2 decimal places. [3]

9 A roller-coaster ride has two separate safety systems to detect faults on the track and on
the roller-coaster train itself. Over a long period of time, it is found that the average
number of faults detected per day by the systems are 0.25 for the track and 0.15 for the
train. Assume that the faults detected on the track are independent of those detected on
the train.

(i) State, in this context, a condition that must be met for a Poisson distribution to be
a suitable model for the number of faults occurring on a randomly chosen day. [1]

(ii) Find the probability that a total of at most 4 faults is detected by the two systems
in a period of 10 days. [2]

(iii) Find the smallest number of days for which the probability that no fault is detected
by the two systems is less than 0.05. [2]

(iv) Find the probability that, in a randomly chosen period of 10 days, there are at least
3 faults detected on the track, given that there are a total of at most 4 faults detected
by the two systems. [3]

IJC/2016/JC2 9740/02/Aug/2016

IJC P2 Questions 236


5

10 Alex and Ben play with each other a set of ten games at table tennis and for each game,
the probability that Ben loses is 0.7.
(i) State, in this context, an assumption needed to use a binomial distribution to model
the number of games that Ben loses. [1]
Assume that the assumption made in part (i) holds.
(ii) Find the probability that Ben loses more than half of the games. [2]

In order to improve his skills at table tennis, Ben attends an intensive training programme.
After completing the training, Ben decides to play another set of n games with Alex.
Assume that the number of games Ben loses, out of these n games, has the distribution
B( n,0.3) .
(iii) Find the greatest value of n such that the probability that Ben loses more than 8
games is at most 0.01. [3]
(iv) Given that n  50 , use a suitable approximation to find the probability that the
number of games Ben loses is between 10 and 20 inclusive. State the parameters
of the distribution that you use. [3]

11 Research is being carried out into how the concentration of a drug in the bloodstream
varies with time, measured from when the drug is given. Observations at successive times

sgexamguru.com
give the data shown in the following table.
Time (t minutes) 20 40 70 100 130 190 250
Concentration (m micrograms per litre) 85 62 51 33 29 14 6

(i) Draw a scatter diagram of these values, labelling the axes. Explain how you know
from your diagram that the relationship between m and t should not be modelled
by an equation of the form m  a  bt . [2]
It is thought that the concentration of the drug in the bloodstream at different times can
be modelled by one of the formulae
m  ct 2  d or m  e ln t  f
where c, d, e and f are constants.
(ii) Find, correct to 4 decimal places, the product moment correlation coefficient
between
(a) t 2 and m ,
(b) ln t and m . [2]

(iii) Explain which of m  ct 2  d or m  e ln t  f is the better model and find the


equation of a suitable regression line for this model. [3]
(iv) Use the equation of your regression line to estimate the concentration of the drug
in the bloodstream when t  150 , correct to 2 decimal places. Comment on the
reliability of the estimate obtained. [2]

IJC/2016/JC2 9740/02/Aug/2016 [Turn over


IJC P2 Questions 237
6

12 Min Ho has just learnt how to use two different methods to mow a piece of lawn in his
house garden.

Method A: This is a two-stage process that involves cutting the grass with a strimmer
and then collecting the grass by raking it up. The time, X minutes, taken to cut the grass
has the distribution N(30, 4.82 ) . Once the grass is cut, the time, Y minutes, taken to
collect the grass has the distribution N(20,3.12 ) .

Method B: This method uses a mower with a rechargeable battery that will cut and
collect the grass at the same time. The time, S minutes, taken to do this has the distribution
N(38, 2.62 ) . In addition to this, the battery has to be recharged once before the cut, and
this time is fixed at 15 minutes.

(i) Find the probability that Min Ho takes more than 45 minutes to mow the lawn using
Method A. [3]

(ii) Find the probability that using Method A to mow the lawn is faster than using
Method B by more than 5 minutes. [4]

Assume that Min Ho mows the piece of lawn in his house garden on a weekly basis. Over
a particular period of ten consecutive weeks, Min Ho uses Method A for the first four

sgexamguru.com
weeks and Method B for the next six weeks. Find the probability the average time taken
to mow the lawn in a week is greater than 50 minutes. [4]

IJC/2016/JC2 9740/02/Aug/2016

IJC P2 Questions 238


ANNEX B
IJC H2 Preliminary Examination (Paper 2)

Qn/No Topic Set Answers


1 Inequalities 3 3 3
x  1 or  x or x  ;
2 4 2
 3
x  ln  
 2 
2 AP and GP (i)9
(iii) x  8755
(last part) Software Y is unable to remove
all the comments because eventually it is
only able to remove 150 000 comments.
3 Differential Equations 1
5e 2 t
(i) x  1
4  e2t
8
(ii) 2ln   years
3
(iii) The population of wild boars will

sgexamguru.com
increase and stabilise at 500 eventually.
4 Vectors (Lines and Planes) (i)   67.8
(ii) ( 1,1, 3)
 2 
(iii)  0 
 4 
 
 1   4
 
(iv) r   1     1  , 
 3   2 
   
5 Sampling Methods (i) Systematic sampling
(ii)(slower, more difficult to collect)
Systematic sampling is a more tedious
process to select the employees, whereas
quota sampling is quick and easy.
6 Sampling distribution
(Central Limit Theorem) 143
7 Probability (i) 0.902
(ii) 0.199
8 Hypothesis Testing (i) 44.1 ; 3.52
(ii) The battery life of a PI-99 calculator is
assumed to be normally distributed.

ANNEX B 239
H0 :   k
(iii)
H1 :   k
(iv) k  : 42.97  k  45.24
9 Poisson Distribution (i) The average number of faults detected by
each system (for the track and the train) is
constant from one day to another.
(ii) 0.629
(iii) 8
(iv) 0.237
10 Binomial Distribution (i) Ben’s performance (i.e. whether he loses
or wins) in a game is independent of any
other games that he plays with Alex.
(ii) 0.850
(iii) 14
(iv) 0.910
11 Correlation & Regression (ii)(a) 0.8454
(ii)(b) 0.9961
(iii) m and ln t is the better model;
m  179  31.2ln t
(iv) 22.61 micrograms per litre ;
The estimate obtained is reliable, because the

sgexamguru.com given value of t = 150 lies within the given


sample data range for t and the product
moment correlation coefficient between m
and ln t is very close to 1 , hence indicating
a strong negative linear correlation between
the variables m and ln t .
12 Normal Distribution (i) 0.809
(ii) 0.375
(last part) 0.916

IJC P2 Solutions B 240


Innova Junior College
H2 Mathematics
JC2 Preliminary Examinations Paper 2
Solutions

1 Solution
3 x

4x  3 x 1
3 x
 0
4x  3 x 1
3 x  3  4 x 2  3x
0
 4 x  3 x  1
4 x 2  3
0    (*)
 4 x  3 x  1
4 x2  3
0
 4 x  3 x  1
 2 x  3  2 x  3   0
 4 x  3 x  1

sgexamguru.com
+
-1

3
+
3

3
+

 
2 4 2

3 3 3
Hence x  1 or   x or x 
2 4 2
3 ex
For  , making use of the result in above part,
4e x  3 ex 1
3 3 3
1 < e x   or  < ex 
2 4 2
(no solns since e x is always positive)
3  3
Hence, e x   x  ln  
2  2 

IJC P2 Solutions 241


2 Solution
(i)
Hour Start of hour End of hour
1 1 1+3=4
2 4 4 + 12 = 16
3 16 16 + 48 = 64
4 … …

n 1
4  4  200 000
n 1
 4  50000
n  1  7.80482
n  8.80842

Number of complete hours = 9

Alternative Solution
n1
4  4  200 000
n Total

sgexamguru.com
8 65536 < 200 000
9 262144 > 200 000
10 11048576 > 200 000

Number of complete hours = 9

(ii)
Day Start of day End of day
1 200000  x 1.02  200000  x 
2 1.02[1.02  200000  x 
1.02  200000  x   x]
x  1.022  200000 
1.02 x  1.022 x
3 … …

At the end of day n, the number comments


 1.02n  200000   1.02 x  1.022 x  ...  1.02n x  --- (*)

IJC P2 Solutions 242


 1.02 1.02n  1 
n
 1.02  200000   x  
 0.02 
 
 1.02n  200000   51x 1.02 n  1
(iii) 1.0230  200 000   51x 1.0230  1  0
1.0230  200 000 
x
511.0230  1
x  8755 (to nearest integer)

Day 1: no. of comments removed  15000


Day 2: no. of comments removed  15000  0.9 
2
Day 3: no. of comments removed  15000  0.9 
As n   , no. of comments removed
15000
  150 000
1  0.9
Software Y is unable to remove all the comments because eventually it is only
able to remove 150 000 comments.

sgexamguru.com

IJC P2 Solutions 243


3 Solution
(i) Method 1:
1 1
 x (5  x )
dx 
10 
dt    (*)

Doing partial fractions


1 A B
 
x (5  x ) x 5  x
A(5  x )  B ( x )

x (5  x )
1
A
5
1
B
5

1 1 1
 +
5 x 5(5  x )
dP 
10 
dt

1 1
 ln x  ln 5  x   t  c
5 10

sgexamguru.com
ln
x
5 x 2
1
 tc

1
x t
 Ae 2 , where A   e c
5 x

1 1
Given x  1 when t  0 ,  Ae 0  A 
5 1 4
1 1
5 t 1 t
x  e 2  xe 2
4 4
1 1
t t
x (4  e 2 )  5e 2
1
5e 2 t
x 1
4  e2t

IJC P2 Solutions 244


(i) Method 2:
1 1
 x(5  x) dx   10 dt    (*)
1 1
  ( x  )  10 dt
25
4
dx 
5 2
2
5
1 2
 ( x  52 ) 1
ln  tc
2  
5
2
5
2
 (x  5
2
) 10
x 1
ln  tc
5 x 2
1
x t
 Ae 2 , where A  e  c
5 x
1 1
Given x  1 when t  0 ,  Ae 0  A 
5 1 4
1 1
5 2t 1 2t
x e  xe
4 4
1 1
t t
x (4  e 2 )  5e 2
1
5e 2 t 5
x or x

sgexamguru.com
1
t  12 t
4e 2
4e 1
1
(ii) 5e 2
t
When x = 2, 1
t
2
4  e2
1 1
t t
8  2e 2  5e 2
1
t
3e 2 8
1
t 8
e2 
3
8
t  2ln  
3
8
It takes t  2 ln   years.
 3
(iii) As t   , x  5 .  The population of wild boars will increase and stabilise at
500 eventually.

IJC P2 Solutions 245


4 Solution
(i)  1  2 
   
l : r   0     1  , where  is a real parameter.
 7   4
   
1
 
p : r  0   2
 1
 
 2   1 
   
 1  0 
 4   1
    6
sin   
 2   1  21 2
   
1 0
 4   1
   
  67.8 (1 dec pl)

(ii) For the point of intersection between l and p,


 1  2   1 

sgexamguru.com
 
   . 0   2
 7  4   1
  
1  2  7  4  2
 1
 1   2   1 
The position vector of point of intersection is  0    1    1  .
 7   4   3 
     

Coordinates of point of intersection are ( 1,1, 3) .

(iii) The line perpendicular to p passing through (1, 0, 7) is


1  1
   
r   0     0 ,   
 7   1 
   
 1    1 
  
 0 . 0   2
 7     1
  
1   7    2

IJC P2 Solutions 246


2   6
  3

  1  3   2 
   
ON   0    0 
 7  3   4 
   

(iv) Method 1:
Let the coordinates of A be (1, 0, 7) .
Let A’ be the reflected point of A in p.
 
 OA  OA '
Using ratio theorem, ON 
2

    2   1   5 
     
 OA '  2ON  OA  2  0    0    0 
 4   7   1 
     

The reflected line contains the point A’ and point of intersection between l and p.

 1   5   4 
The direction vector of the reflected line is  1    0    1 
sgexamguru.com  3   1   2 
     
 1   4
   
r   1     1  ,  
 3   2 
   
(iv) Method 2:
Let the coordinates of A be (1, 0, 7) .Let A’ be the reflected point of A in p. Let
the coordinates of B be ( 1,1, 3) .

  
BA BA '
BN 
2
  
BA '  2 BN  BA
 2   1   1   1   4 
 2  0   2  1    0    1    1 
 4   3   7   3   2 
         
 1   4 
   
 r   1     1  ,   
 3   2
   

IJC P2 Solutions 247


5 Solution
(i) Systematic sampling
(ii) (slower, more difficult to collect) Systematic sampling is a more tedious process
to select the employees, whereas quota sampling is quick and easy.

Another possible reason: might miss out a certain group of people due to different
reporting times.
(iii) The interviewer could consider transport mode of the employees as the stratum. A
possible quota for each stratum is as follows:
By private By public
By walking
transport transport
10 10 10

The interviewer can then stand at the entrance of the building and select the
sample until the above quota is met.

6 Solution
E  X   1.93 Var  X   1.4

sgexamguru.com
Since n is large, by Central Limit Theorem,
 1.4 
X ~ N  1.93,  approximately.
 n 
 
Given that P X  2  0.24 --- (*)

Method 1: Using GC to set up table

 
when n  142 , P X  2  0.24041 (  0.24)

when n  143 , P  X  2   0.23964 ( 0.24)

when n  144 , P  X  2   0.23887 (  0.24)

 least n is 143.

Method 2: Using algebraic method via standardization


 
P X  2  0.76
 
 2  1.93 
P Z   0.76
 1.4 
 n 

IJC P2 Solutions 248


From GC,
2  1.93
 0.70630    (**)
1.4
n
0.70630
n 1.4
0.07
n  11.939
n  142.53
 least n is 143.

sgexamguru.com

IJC P2 Solutions 249


7 Solution
(i) Method 1:
Required probability
13 11
C4 C4
 1 24
 24
 0.902 (3 sig fig)
C4 C4

Method 2:
Required probability
13  12 1110 1110  9  8
 1   0.902 (3 sig fig)
24  23  22  21 24  23  22  21

Method 3:
Required probability
11
C1  13C3  11C2  13C2  11C3  13C1
 24
C4
 0.902 (3 sig fig)
(ii) Method 1:


sgexamguru.com
Required probability
11
C2  2!  22C2  2!
 0.199 (3 sig fig)
24
C4  4!

Method 2:
Required probability
11 10  22  21
  0.199 (3 sig fig)
24  23  22  21

IJC P2 Solutions 250


8 Solution
(i) Unbiased estimate of the population mean
573.39
x  44.10692308  44.1 (3 s.f.)
13
Unbiased estimate of the population variance
42.22
s2   3.518333333  3.52 (3 s.f.)
12
(ii) The battery life of a PI-99 calculator is assumed to be normally distributed.
(iii) Let X be the r.v. denoting the battery life of a randomly chosen PI-99 calculator.
Let  be the population mean battery life of the PI-99 calculators.
H0 :   k
H1 :   k
where H 0 is the null hypothesis and H1 is the alternative hypothesis.
(iv) To test at 5% level of significance.
X k
Under H 0 , the test statistic is T  ~ t12  .
S
13
Since the null hypothesis is not rejected, t-value falls outside critical region.
 2.178812  t  value  2.178812

sgexamguru.com
2.178812 
x k
s
13
 2.178812 --- (*)

 s   s 
x  2.178812    k  x  2.178812  
 13   13 
where x  44.10692 and s  3.51833

 42.97  k  45.24
The required set is k  : 42.97  k  45.24

IJC P2 Solutions 251


9 Solution
(i) The average number of faults detected by each system (for the track and the train)
is constant from one day to another.
(ii) Let X be the r.v. denoting the total number of faults detected by the two systems in
a periods of 10 days.
X ~ Po  (0.25  0.15)  10  , i.e. X ~ Po(4)
 P  X  4   0.6288369  0.629 (3 sig fig)
(iii) Let Y be the r.v. denoting the total number of faults detected by the two systems in
a period of n days.
Y ~ Po(0.4 n )
Given P  Y  0   0.05 ,
Method 1: Algebraic method
e 0.4 n  0.05 
(o.e. e 0.25 n  e 0.15 n
  0.05 )
n  7.489
 the smallest number of days required is 8.

Method 2: GC table
When n  7 , P  Y  0   0.06081 (  0.05)
When n  8 , P  Y  0   0.04076 ( 0.05)
sgexamguru.com
When n  9 , P  Y  0   0.02732 ( 0.05)
 the smallest number of days required is 8.

(iv) Let W and V be the r.v. denoting the number of faults detected on the track and on
the track in a period of 10 days respectively.
W ~ Po(2.5) and V ~ Po(1.5)
Required probability
 P W  3 | V  W  4 
P W  3  V  W  4 

P V  W  4 
P(W  3)P(V  0)  P(W  3)P(V  1)+P(W  4)P(V  0)

P V  W  4 
P(W  3)P(V  1)  P(W  4)P(V  0)

P V  W  4 
 0.237 (3 sig fig)

IJC P2 Solutions 252


10 Solution
(i) Ben’s performance (i.e. whether he loses or wins) in a game is independent of any
other games that he plays with Alex.
(ii) Let X be the r.v. denoting the number of games that Ben loses out of 10 games.
X ~ B 10, 0.7 
P( X  5)  1  P( X  5)
 0.84973
 0.850 (3 sig fig)
(iii) Let Y be the r.v. denoting the number of games that Ben loses out of n games.
Y ~ B  n, 0.3
P(Y  8)  0.01
1  P(Y  8)  0.01
Using GC,
When n  13 , P(Y  8)  0.00403 ( 0.01)
When n  14 , P(Y  8)  0.00829 ( 0.01)
When n  15 , P(Y  8)  0.01524 (  0.01)
 the greatest value of n is 14.
(iv) Let W be the r.v. denoting the number of games that Ben loses out of 50 games.

sgexamguru.com
W ~ B  50, 0.3
As n = 50 is large, np  15 (  5) and nq  35 (  5) ,
 W ~ N 15 , 10.5  approximately

P(10  W  20)  P(9.5  W  20.5)    (*)


 0.910 (3 sig fig)

IJC P2 Solutions 253


11 Solution
(i)
m
85

6
t
20 250
From the scatter diagram, a curvinlinear correlation is observed between m and t
(i.e. as t increases, m decreases at a decreasing rate), and hence a linear model
with equation of the form m  a  bt cannot be used to model the relationship
between m and t.
(ii) Product moment correlation coefficient between m and t 2  0.8454 .
(a)
(b) Product moment correlation coefficient between m and ln t  0.9961 .
(iii) Since the absolute value of the correlation coefficient between m and ln t (i.e.
case (b)) is closer to 1, this indicates that the linear correlation between the

sgexamguru.com
variables m and ln t is stronger as compared to that between the variables for
case (a).

 case (b) is the better model for the relationship between m and t .

m  179.026  31.2175 ln t
 m  179  31.2 ln t (3 sig fig)

(iv) When t  150 ,


m  179.026  31.2175 ln150  22.61 (2 dec pl)

The estimate obtained is reliable, because the given value of t = 150 lies within
the given sample data range for t and the product moment correlation coefficient
between m and ln t is very close to 1 , hence indicating a strong negative linear
correlation between the variables m and ln t .

IJC P2 Solutions 254


12 Solution
(i) X  Y ~ N  50 , 32.65 
P  X  Y  45   0.809224  0.809 (3 sig fig)
(ii) E( X  Y  S )  12
Var( X  Y  S )  39.41
 X  Y  S ~ N 12 , 39.41

P(method A is faster than method B by more than 5 mins)


 P  S  15  ( X  Y )  5 
 P( X  Y  S  10)
 0.375020  0.375 (3 sig fig)
(iii) Let A  X  Y and B  S  15 .

A ~ N  50 , 32.65 and B ~ N 53 , 2.6 2 
A1  A2  A3  A4  B1  ...  B6
Let W 
10
50  4  53  6
 E(W )   51.8
10

& sgexamguru.com
Var(W ) 
32.65  4  2.62  6
102
 1.7116

 W ~ N  51.8 , 1.7116 
Required probability
 P(W  50)
 0.915566  0.916 (3 sig fig)

IJC P2 Solutions 255


 
 
JURONG JUNIOR COLLEGE 
Preliminary Examinations  
 
 
MATHEMATICS 9740 / 1 
Higher 2 30 August 2016 
3 hours 
 
Additional materials: Answer Paper 
Cover Page 
List of Formulae (MF 15) 

READ THESE INSTRUCTIONS FIRST 

Write your name and civics class on all the work you hand in. 
Write in dark blue or black pen on both sides of the paper. 
You may use a HB pencil for any diagrams or graphs. 
Do not use staples, paper clips, glue or correction fluid. 

Answer all the questions. 


Give non-exact numerical answers correct to 3 significant figures, or 1 decimal place in the case of angles

sgexamguru.com
in degrees, unless a different level of accuracy is specified in the question. 
You are expected to use an approved graphing calculator. 
Unsupported answers from a graphing calculator are allowed unless a question specifically states
otherwise. 
Where unsupported answers from a graphing calculator are not allowed in a question, you are required to
present the mathematical steps using mathematical notations and not calculator commands. 
You are reminded of the need for clear presentation in your answers. 

At the end of the examination, fasten all your work securely together, with the cover page in front. 
The number of marks is given in brackets [ ] at the end of each question or part question. 
 
 
 
 
 
 
 
 
 
 
 
This document consists of 6 printed pages. 

[Turn over 

Microsoft Word - 2016 JJC H2 Math P1 Prelim_Qn(updated) 256



 
 

1 In the finals of a General Knowledge Quiz, a team is required to answer 25 questions. Each
question that is correctly answered scores 5 points, while a question that is wrongly answered is
deducted 3 points. If the answer is partially correct, the team scores 2 points. 

After 24 questions, the results are shown in the following table.  


 
Number of Questions   

Correct  Partially Correct  Wrong  Points 

a  b  c  79 
 
If the team answers the last question wrongly, then the total number of questions answered
correctly and partially correct is four times the number of questions answered wrongly. By
forming a system of linear equations, find the values of a, b and c. [4] 
 
 
1
2
sgexamguru.com
A sequence u1, u2, u3,... is such that u1 
3
and  

 
n
(i) Use the method of mathematical induction to prove that u n  . [4] 
4n  1
2

(ii) Hence prove that the sum of the first n terms of the series  

 
3 n3
is  . [3] 
35 4(n  3) 2  1
(iii) Give a reason why the series in part (ii) is convergent and state the sum to infinity. [2] 
 
 
 
 
  [Turn ove
 
 

JJC P1 Questions 257



 
12
3 The diagram shows the curve C with equation y  which has a turning point at
(3 x  2) 2  4

 2 
  ,3  . The region R is bounded by C, the y-axis and the line y = 3.
 3 
 
          y=3
         R
 

                          x
                                                                                          O
 

(i) Find the exact area of R. [5] 


(ii) R is rotated through 2 radians about the y-axis. Find the volume of the solid of

  sgexamguru.com
revolution formed, giving your answer to 4 decimal places. [3] 

 
4 Let y  tan  2 tan 1 x   . 
 4

Show that 1  x 2   
dy
(i)  2 1 y2 . [2]
dx

(ii) Hence find the Maclaurin series for y, up to and including the term in . [4]
 
Denote the answer to part (ii) of the Maclaurin series by g(x) and f(x)= tan  2 tan 1 x   .  
 4
(iii) Find, for 0.4 ≤ x ≤ 0.4, the set of values of x for which the value of g(x) is within 0.5
of the value of f(x). [2] 
 
 
  [Turn over 
 
 
 
 

JJC P1 Questions 258



 
 
5 A curve C has parametric equations 

 
(i) Show that the equation of the normal to C at the point P with parameter  is 


where m is an integer to be determined. [3] 
(ii) The normal to C at the point P cuts the x-axis and y-axis at points A and B respectively.
By finding the mid-point of AB, determine a cartesian equation of the locus of the mid-
point of AB as  varies. [5] 
 

6 A function f is said to be self-inverse if f(x) = f1(x) for all x in the domain of f. 
The function g is defined by 


(i) sgexamguru.com
Sketch the curve y =g(x), stating the equations of the asymptotes clearly. [2] 
(ii) Define g1 in a similar form and show that g is self-inverse. [4] 
(iii) Show that g2 (x) = x and that g3(x) = g(x). Hence find the values of x for which  

. [4] 
 

cos(ln x )
7 (a) Find  x2
dx . [4] 

 
6 x2
(b) Using the substitution u  x  3 , find 
1
x x3
dx , giving your answer in the form  


where a, b, c and d are constants to be determined. [6] 
 

JJC P1 Questions 259



 
8 The complex number z satisfies the following inequalities: 

|z|≤ 4 and   arg( z  3  i)  0   
6

(i) On an Argand diagram, sketch the region R in which the point representing z can lie. 
[4] 
(ii) Find exactly the minimum and maximum possible values of |z  2i|. [3] 
(iii) Determine the number of roots of the equation z100 =2100 that lie in the region R. [3] 
 

m2
9 It is given that f(x) = x  , where 0 < m < 1. 
x2
(i) Sketch the graph of y = f(x) , showing clearly the coordinates of the turning points 
and the equation(s) of any asymptote(s). [5] 
 

sgexamguru.com
(ii) By inserting a suitable graph to your sketch in (i), find the set of values of k, in terms of
m, for which the equation x2 (2+k)x + (m2 + 2k)=0 has two distinct positive roots.
[4] 
(iii) The curve y = f(x) undergoes the transformations A, B and C in succession:  
A: A translation of units in the direction of x-axis, 
1
B: A stretch parallel to the x-axis with scale factor of , and 
2
C: A translation of 2 units in the direction of y-axis. 
1
Given that the resulting curve is y  2 x  , find the value of m. [2] 
8x
 

 
 

JJC P1 Questions 260



 
 

 4   2  1
     
10 The point A has position vector   3  and the line l has equation r =   3     1  , where
 0   1    1
     


(i) Find the position vector of the foot of the perpendicular from A to l. [4] 
(ii) Show that a cartesian equation of the plane 1 which contains A and l is 

. [2] 
The equation of the plane 2 is x + 7z = c, where c is a constant. 
(iii) Given that 1 and 2 intersect in a line L, show that a vector equation of L is  

 c   7
   
r = 1  c     5  , . [2] 
 0   1 
   
Another plane 3 has equation 2x  y + dz = 5, where d is a constant. 

(a)
sgexamguru.com
(iv) Find the values of c and/or d if all three planes 1 , 2 and 3  
meet in the line L, [3] 
(b) have only one point in common. [1] 
 

11 At the beginning of May 2016, Sam borrowed $50 000 from a bank that charges him a special
rate of 0.2% interest at the end of every month. Sam pays back $1 000 for every instalment at
the beginning of every month, starting from June 2016.  

(i) Show that the total amount with interest that Sam still owes the bank at the end of the  
month after the nth instalment is paid is  

. [4]  
(ii) Find the number of instalments required for Sam to settle all the amount owed. [2] 
(iii) How much does he pay on his last instalment? [2]
(iv) If Sam wishes to settle all the amount owed after paying 19 instalments, what is the
minimum amount (to the nearest dollar) he should pay each month? [2] 
 

JJC P1 Questions 261


 
  Jurong Junior
J Co
ollege  
2016 JC22 H2 Mathematics Prelim Paper
P 1 So
olutions 
 
Qn 
Q Soluttion
1  a + b + c = 24 
 
5a + 2b  3c = 79
7  
a + b = 4(c + 1) 
a = 17, b = 3, c = 4 
22(i) 
 
Let be the sttatement thaat for all  
Whenn  

LHS = (given)
(  

RHS =
Hencce is truee.

Assuume is tru
ue for somee i.ee. . 

sgexamguru.com
We want
w to prov
ve that is true, i.e.  
 
 
 


 

Microsoft Word - 2016 JJC H2 Math P1 Prelim_Soln 262


 

Hencce is truee is true. 

Sincee is tru
ue & is ttrue is true, by mathematiccal
inducction, is true for all . 

(ii)  Sum of 1st n term


ms of

=  

sgexamguru.com
 

((iii) 
 
As  

Hencce, the series is converggent and  


 

JJC P1 Solutions 263



(i) 
  Requuired Area  

 
(ii) 
 
 

Sincee ,  

Requuired volum
me =  

sgexamguru.com =  

= 0.51225 
44(i) 
 
 

Alterrnative Method 

 
Diffeerentiate witth respect too x,  


 

JJC P1 Solutions 264


(ii)  Diffeerentiate witth respect too x,  
 

Whenn x = 0,  

 
 

y     

 

y =0.5

-0.359  O  x 
0.2225 

(iii)  sgexamguru.com
Sketch and  
 
 
 

For  


 

JJC P1 Solutions 265


5(i) 
 

 
Equaation of norm
mal at P, : 

(ii)  At thhe x-axis,  


 

 
At thhe y-axis,  

sgexamguru.com
mid-ppoint of AB
B:  

 
Carteesian equation of the loocus of the mid-point
m of AB: 

i.e. 16x2 + 4y2 = 9


 
   


 

JJC P1 Solutions 266


 

Qn  Solution 
     

  
6(i)   
   

 
 
sgexamguru.com  

(ii) 
 
Let  

Then  

 
g is self-iinverse as and


 

JJC P1 Solutions 267


(iii) 
 
 
wn) 
(show
,  

For  

Then  

 
 
 

 
 

Qn  Solution 

(a)     
 

     

=
sgexamguru.com
  

     =-   

 =   

 =  


 

JJC P1 Solutions 268


(b) 
  

Differenttiating w.r.t.. x,  
6 u = 3 
When x = 1, u = 2; When x = 6,

=  

=  

=  

i. e. a = b = 2,
2 c=d=3
 

Qn  Solution 

sgexamguru.com
8  y 
(i)   
    A 
 
 
   
  D
  B 
   
 
 
  
    x

 
  2 
 
  C
 
 
 
 
 
 
 
 
 


 

JJC P1 Solutions 269


(ii) 
  Minimum
m value of  

 
Maximum
m value of  

 
 
(iii) 
 
 
,  

 
Roots aree found in reegion R (alo
ong the minnor arc CD) if

 
 
Numbeer of roots found
f in reg
gion R . 
 

Qn  Solution 
9(i) 
 
 
sgexamguru.com
Let  

 
 

 
The statioonary points are aand  
 
 
   


 

JJC P1 Solutions 270


     

  

  
  

   
 

 
 
 
   
 
 
                                             
                                      
 

(ii) 

sgexamguru.com
 
 

By insertting a horizoontal line C 


on thee graph of C,
by observvation, to haave two disttinct positivve roots, 

then ((ans) 
 
   

10 
 

JJC P1 Solutions 271


 

(iii) 
 
 

After A:  

After B:  

After C:  

Given thaat  

(ans)) 
 

Qn  Solution 
10  Let F be the foot of the dicular. 
t perpend
(i)  1
    
AF   1   0
 1 
 

sgexamguru.com
  
 2   1   4   1 
       
 3     1    3     1   0
 1   1  0    1
     
 

 
 3
  
OF   2    
 0
 

(ii)  Let B be .
 
 1   2   1  1

        
  BAA   1    0    1    1    
 1  1  1  2 
       
 
A cartesiaan equationn of is . 

 
11 
 

JJC P1 Solutions 272


 

(iii) 
 

A directioon vector off L is . 


:  
:  
 
Let . Then and . 
A point oon L is . 

A veector equation of L is , whhere .


(iv) For the 3 planes to meet m in the linel L,
(a)  7 2  c  2
         
   5    1  0 and
a  1  c    1  5  
1 d  0  d
       

and sgexamguru.com
annd
 
 

(b)  For the 3 planes to have h ne point in ccommon, 


only on
  7 2
   
  5     1   0 . 
1 d
   
 
 

   

12 
 

JJC P1 Solutions 273


 

Qn  Solution 
11   
(i)  Instalmeent  Outstaanding amoount Totaal amount with
w interest
  at the beginning of
o d of month 
owedd at the end
monthh 

  500000 

1   
 


 

……..  ………
……………
…… ……
………………
……………

…. 
……… …… 
……
 

nstalments 
Amount oowed at thee end of n in
=

=  

-------------(*) 

(ii) 
 
sgexamguru.com (sshown)

 
i.e. no off instalmentss required = 53 
(iii)  Amount ppaid at the 53th
5 ment  
instalm
  2 instalmennts 
= Amounnt owed at thhe end of 52

=733.12 (to 2 d.p.)  


(iv)  Let the am
mount needd to be paid for each insstalment be k. 
  Then fromm (*) in (i) 

 
 
Least value of k = 26685 (to the nearest
n dolllars)
 
 
 
 
13 
 

JJC P1 Solutions 274


 
JURON
NG JUNIOR
R COLLEG
GE 
Prelimiinary Exam
minations  
 
 
MATHHEMATIICS 97
740 / 2
  Highe
er 2 31 Aug
gust 2016
3 hours
 
Additiona
al materials: Answ wer Paper 
Cover Page 
o Formulae ((MF 15) 
List of

READ TH S FIRST 
HESE INSTRUCTIONS

Write youur name and civics class on all the wo ork you hand d in. 
Write in d
dark blue or black
b pen on both sides oof the paper. 
You may use a HB pe encil for any diagrams or graphs. 
aper clips, glue or correcction fluid. 
Do not usse staples, pa

Answer a

in degree
ons. 
all the questio
Give non--exact nume
es, unless a different
d
sgexamguru.com
erical answerrs correct to 3 significant figures, or 1 decimal plaace in the case of angless
leve d in the question. 
el of accuracyy is specified
You are eexpected to use
u an appro oved graphin ng calculator.. 
Unsupporrted answerrs from a graphing
g callculator are allowed un nless a quesstion speciffically statess
  otherwisee. 
Where un nsupported answers
a from
m a graphing g calculator are
a not allowed in a quesstion, you are e required to
o
present th
he mathematical steps us sing mathem matical notatioons and not calculator coommands. 
You are rreminded of the
t need for clear presen ntation in youur answers.

At the en
nd of the exa
amination, fasten
f all yo
our work sec
curely together, with thee cover pag
ge in front.
The numb
ber of markss is given in brackets
b [ ] a t the end of each
e on or part queestion. 
questio
 
 
 
 
 
 
 
 
 
 
 
ent consists of 7 printed pages and 1 blank page . 
This docume
 
   

[Turn overr 

Microsoft Word - 2016 JJC H2 Math P2 Prelim_Question 275



 
 

Section A: Pure Mathematics [40 marks] 


 
dy
1 (a) Given that x and y are related by  sec2 y   and that y = 0 when x =1 , find x in
dx
terms of y. [4]
 
(b) A medical researcher is investigating the rate of spread of a virus in a group of
people of size n at time t weeks. He suggests that n and t are related by the
d 2n
differential equation 2
 e t /5  . 
dt

(i) Find the general solution of the differential equation, giving your answer in
the form n = f(t). [2] 
(ii) Explain why all solution curves of the differential equation are concave
upwards. [1] 
(iii)
sgexamguru.com
It is given that initially, the number of people infected with the virus is 50.
Sketch on a single diagram, two distinct solution curves for the differential
equation to illustrate the following two cases for large values of t : 
 
I. the population of infected people increases indefinitely, 

II. the population of infected people stabilizes at a certain positive number.  


[3] 
 
 
 
 
2 (a) A parallelogram has two adjacent sides defined by the vectors a and 2a + 3b. Given
that the magnitudes of a and b are 4 and 5 respectively and the angle between a and
b is 30o , find the area of the parallelogram. [4] 
x 1
(b) A point P has coordinates (2, 1, 2) and a line l has equation  1  z , y  3  . 
2
(i) Find the perpendicular distance from P to l. [4] 
(ii) Find the acute angle between l and the line L that is parallel to the z-axis. [2] 
 

JJC P2 Questions 276


[Turn ove

 
3 m3 is made oof cardboard of negligiible thickneess. It has a height of y
A boox with voluume 250 cm
cm aand an equilateral trian
ngular base of side x cm. Its lid has
h depth kyy cm, wheree 0 < k ≤ 1
(see diagram). 
 

 
  k  
ky

 
 
 
x  x 

 
 
  Box  Lid 

(i) Show thatt the total ex


xternal surfaace area of the
t box and
d lid can be expressed as
a  

. [4]] 
(ii) Use differrentiation to
o find, in tterms of k, the value of x that ggives a min
nimum totall

sgexamguru.com
external suurface area of the box aand lid.
y
[3]] 

(iii) Find the raatio   in this


t case, in terms of k, simplifying
g your answ
wer. [2]] 
x
y
(iv) Find the values
v for which
w muust lie. [2]]
x
1
4 Thee complex numbers
n a b are givven by a = (1+3 i) and
a and a b  (11  i) . 
2
(i) Withouut using a caalculator, finnd the valuee of a2b in the form x + iy . [2]] 
(ii) duli and argguments of a and b, find the moddulus and argument
By usinng the mod a off

[3]] 

5 3 1
(iii)) Use your answers to parts (i) and (ii) to show
s that siin  . [2]] 
12 2 2
(iv)) The diaagram below
w shows ann isosceles right
r triangle ABC, wheere the poin
nts A, B andd
C repreesent the com
mplex numb
mbers a, b an
nd c respectiively. Find the exact vaalue of c.  
[2]] 
 
 
 

[Turn overr 
JJC P2 Questions 277

 
  y 
 

 
 
 
 
  x 

 
 
  B 
 
 

 

 

 
sgexamguru.com
Section B: Statistics [60 marks] 

5 A group of 11 people consists of 6 men and 5 women, 3 of whom are sisters. A committee
consisting of six people is to be selected. Find the number of ways the committee can be
formed if  
(i) it consists of exactly two men, [1]
(ii) it includes at least one of the sisters. [2]
 
Given that the chosen committee consists of 2 sisters, Sue and Suzy, together with 3 other
men, Muthu, Mark, Michael and 1 other woman, Wina. They are seated at a round table
meant for six people. Find the number of possible arrangements if  
(iii) one of the men is to be seated between the two sisters, [2]
(iv) the two sisters are sitting directly opposite each other. [2]
 

JJC P2 Questions 278



 
6 Thee table below
w shows the number oof male and female stud
dents studyiing Chemisttry, Physicss
andd Biology att a private school.  

Chemi stry Physics Biology 


Male  2000 130 70
Female  2500 300 50
 
Onee of the studdents is cho
osen at randoom. Eventss C, B and M are definned as follow
ws: 
C : The stu
udent chosenn is studyin
ng Chemistrry. 
B : The stu
udent chosenn is studyin
ng Biology.
M : The stu
udent choseen is a male. 
 
Finnd 
(i) P(C M
M ), [1]
(ii) P(M C), [1]
(iii)) P(M '  B '). [1]
Dettermine whether C and
d M are indeependent. [2]]
 

sgexamguru.com
It iis given thaat 20% of Chemistry
C sstudents, 30
studdents are innternational students.  
0% of Physsics studentts and 5% of Biologyy

(iv)) One off the students selectedd at random nternational student. What
m is an in W is thee
probabbility that this student sttudies Chem
mistry? [2]]
(v) Three students aree chosen att random. Find
F the pro
obability thaat there is exactly
e onee
internaational studeent who studdies Physics. [2]]
 

[Turn overr 

JJC P2 Questions 279



 
7 In oorder to invvestigate whether
w therre is a correelation betw
ween rainfalll and crop yields, thee
totaal rainfall, x mm, and
d the weighhts of a paarticular cro
op per squaare metre, y kg, weree
recorded in a number
n of fields.
f The ddata are shown below.
 
x  36  722  44  74  64  50 
y  2.2  8..4  1.8  7.4  4.3  2.2 
 
(i) Draw a scatter diag
gram to illuustrate the data.
d [2]]
(ii) Calculaate the valu
ue of the prroduct mom
ment correlattion coefficcient, and explain whyy
its valuue does not necessarilyy mean that the best mo
odel for the relationship
p between x

and y is . [2]]
(iii)) By coomparing th
he productt moment correlation
n coefficien
ents, explaiin whetherr

or is a better mod
del. [2]]
(iv)) Using a suitable reegression linne, estimatee the yield of
o crop per ssquare metrre when thee
total raainfall is 55m
mm. Comm
ment on the reliability
r of your estim
mation. [3]]
 

 
sgexamguru.com
8 It is known thhat 8% of th
he populatiion of a larrge city usee a particulaar web brow
wser calledd
Voyyager. A researcher
r wishes to iinterview people
p from
m the city w
who use Voyager andd
seleects people at random, one at a tim
me. 

(i) Find thhe probabilitty that the fi


first person that
t he finds uses Voyaager is the third
t personn  

selectedd. [2]] 

A rrandom sam
mple of n peo
ople is now
w selected. 

(ii) State tw
wo conditions needed ffor the numb
ber of people in the sam
mple who use
u Voyagerr  

to be well
w modelleed by binom
mial distribu
ution. [2]] 

(iii)) Given that


t n = 80
0 , use a suuitable appro
oximation to
t find the pprobability that, fewerr
than 100 people usee Voyager. [3]] 
(iv)) Find thhe least valu
ue of n suchh that the pro
obability off at least 10 people use Voyager iss
more thhan 0.2. [3]] 
 
 

JJC P2 Questions 280



 
9 A ssupermarket sells boxees of a partticular bran
nd of biscuitts in two fllavours, cho
ocolate andd
straawberry. Thhe mean num
mber of boxxes of choco
olate biscuitts sold in a dday is 2.2.  
(i) Find thhe probabilitty that in a dday, no box
xes of choco
olate biscuitts were sold
d. [1]] 
(ii) In a weeek of 7 day
ys, find the eexpected nu
umber of daays that no bboxes of cho
ocolate  
biscuitss were sold.. [2]] 
mber of boxees of strawbberry biscuits sold in a day is denooted by . 
Thee mean num
(iii)) Given that
t the pro
obability of less than 2 boxes of sttrawberry bbiscuits sold
d in a day iss
0.6, wrrite down an ue of , and
n equation for the valu d find  num
umerically, correct
c to 1
decimaal place. [3]] 
(iv)) Find thhe probabilitty that in a week of 7 days, the to
otal numberr of boxes of
o chocolatee
and straawberry bisscuits sold eexceeds 25 boxes.
b [2]] 
(v) Use a suitable
s app
proximationn to find thee probability
y that, in a month of 30
3 days, thee
numberr of boxes of chocolaate biscuits sold is mo
ore than thee number of
o boxes off
strawbeerry biscuitss. [4]] 
 
10 A rresearcher is
i running a trial of a new variety
y of potato. A field coontains 20 rows
r of thee

sgexamguru.com
w variety off potato plaants, with 880 plants in each row. A researchher intends to dig up 8
new
plannts and meaasure the maass of potattoes produceed by each plant.
p  
(i) Describbe how he could
c choos e a systemaatic sample of 8 plants ffrom a sing
gle row of  
80 plannts and statee the advanttage of this sampling method.
m [3]]  
Thee researcherr claims thaat the averaage mass of the new variety
v of ppotato is at least 150g..
y of potato is denoted by X gramss. The massees of a rand
Thee mass of a new variety dom samplee
of 880 new variiety potatoes are summ
marized by 

, . 
(ii) Calculaate the unbiased estimaates of the population
p mean
m and vaariance. [2]] 
(iii)) Test at the 1% sign
nificance leevel, whetheer the researrcher’s claim
m is valid. [4]] 
(iv)) Explainn what you
u understandd by the phrase “at th
he 1% signnificance leevel” in thee
contextt of this queestion. [1]]
 
Another randoom sample of 8 potatooes was ch
hosen with mean masss 148.5g an
nd standardd
devviation k g. Find the raange of valuues that k caan take such
h that at 1%
% level of siignificance,,
thiss sample woould indicatte that the reesearcher’s claim is inv
valid. [3]] 
 

JJC P2 Questions 281


 
  Juron
ng Junior College
2016 JJC2 H2 Mathemat
M tics Prelim
m Paper 2 Solutionss 
 
Qn  Solution
S  
1(a) 
 
 
 

        

 
W
When ,  

1(b) 
(i) 
 

1(b)

sgexamguru.com
(ii) 
for all vallues of .  
S
Solution currves are conncave upwarrds.


 

Microsoft Word - 2016 JJC H2 Math P2 Prelim_Solution 282


1(b)
(iii)  When
W  
C=1
 
C = 0 
 

W
When , . 
 

W
When , . 

50 

25

Qn  Soluttion 
Q
22(a)  Area of paralleloogram 
 
 
sgexamguru.com
 

 
 


 

JJC P2 Solutions 283


(b) 
(i) 
 
A vecctor equatioon of l is . 
Let A be the poinnt oon l. 
 2
  
AP   0 
A
1
 
m P to l= 
Perpeendicular distance from   
 2
 
0
1
 
 

 
(ii)  Acutee angle betw
  sgexamguru.com
ween l and L=
L
 2  0
   
 0 0
 1   1 
   
  coss 1   
 2  0
   
 0  0
 1  1 
   
 

 
 
 

   


 

JJC P2 Solutions 284


 

Qn  Soluttion 
Q
3  
3(i)
 
Area of equilaterral  =  

Givenn that the voolume of thee box is 2500 cm3 

V=  

Surfaace Area A = 3xy + 3kxyy +  

= 3xy(1 + k)) +  

=  

(ii) 
sgexamguru.com
= (shown)) 

For sttationary pooints,  

Thus, gives a m
minimum suurface area..


 

JJC P2 Solutions 285


((iii) 
 
Sincee  

 
(   Sincee 0 < k ≤ 1 
(iv)
 
1 < 1 + k ≤ 2 

i.e.  

Qn  Soluttion 
Q
4  
4(i)
 

sgexamguru.com
 
 
(ii) 
 


((iii)  Considering the imaginary ppart of , we have 
 


 

JJC P2 Solutions 286


(  
(iv) Vecctor BA can bbe obtainedd by rotatingg vector BC through in
  the annticlockwisee direction aabout B. 
 
 

 
 
5   
(i) 
waays 
 
((ii)   Numbber of wayss if at least oone of the siisters are inccluded 
= nummber of wayys without rrestriction – number of ways if nonne of
thee sisters is inncluded 
 
 

Alterrnative Metthod 

sgexamguru.com  

((iii)  Selecct a man to bbe between the 2 sisterss and group the 3 of theem as
  one uunit and arrange 4 unitss round a tabble 
Numbber of wayss =  
 
(  
(iv) Firstt arrange thee other 4 perrsons roundd the table. T
There are 4 w
ways
  to innsert the sistters. 
Num
mber of wayss =  
 
 
   


 

JJC P2 Solutions 287


 

Qn 
Q Soluttion 

(i) 
  P(C M ) =  

=
((ii)  P(M C) = P(M
M)+P(C) P(M
M C)  
 

((iii) 
 
P(M '  B ')  

 
C andd M are not independennt. 
(   No. oof internatioonal studens in the sampple  
(iv)
 
=

sgexamguru.com  

 
 
((v)  Numbber of internnational studdents studyiing Physics
=  

P(exaactly one intternational sstudent studdying Physiccs)

Alterrnative metthod 

Requuired Probabbility =  
 
 
   


 

JJC P2 Solutions 288


 

7   
((i) y 

 
((ii)  (to 3 s.f) 
Thouugh the value of r showss a strong positive lineaar correlatioon,
from the scatter ddiagram, it is possible that x and y may have a
curvilinear relatiionship. 
((iii) 
For ,  

sgexamguru.com
Sincee the value oof r for is clloser to the vvalue of 1,
is a bbetter modeel. 
((iv) 
 

Whenn x = 55,  

Sincee x = 55 is wwithin the raange of dataa given and iis


close to 1, the esstimation is reliable. 
   


 

JJC P2 Solutions 289


 

Qn 
Q Soluttion 
8  P(firsst person thaat uses Voyager is the tthird personn selected)
(i) 
  =  

= 0.067712 

((ii)  1. Whether a person usses Voyagerr is independdent of anotther


  person.
2. The probbability that a person uses Voyager is constant for
every perrson in the sample.
s
((iii)  Let Y be the num
mber of peopple who use Voyager ouut of 80 peoople. 

Sincee , 

 
= 0.899295 
= 0.899 (to 3 s.f.) 
((iv)  Let V be the num
sgexamguru.com
mber of peopple who usee Voyager ouut of n peopple. 

 
Usingg GC, 

   
Leastt value of n = 92 
 

   


 

JJC P2 Solutions 290


 

9 (i)  Let C be the num


mber of boxees of chocollate biscuitss sold in a daay. 

0.110080 
= 0.1111 (to 3 s.f.)) 
((ii)  Let D be the nummber of dayys that no booxes of choccolate biscuuits
were sold out of 7 days. 

 
= 0.775622 
= 0.776 
((iii)  Let S be the num
mber of boxees of strawbberry biscuitts sold in a day.
d  

sgexamguru.com
 

          
Usingg GC,  

   

 
= 11.4 (to 1 d.pp) 
   

10 
 

JJC P2 Solutions 291


 

((iv)  Let T be the totaal number of boxes of cchocolate annd strawberrry


biscuuits sold in 7 days. 

 
= 0.44962 
= 0.450 (to 3 s.f) 
((v)  Let X be numbeer of boxes of chocolatee biscuits soold in 30 dayys. 

Sincee  

Let Y be numbeer of boxes of


o strawberrry biscuits sold
s in 30 daays. 

Sincee  

sgexamguru.com
= 0.99032 
 

= 0.990 (to 3 s.f.) 


   

11 
 

JJC P2 Solutions 292


 

110  Chooose a plant raandomly froom the first 10 plants, ssay the 5th plant.
((i) 
Chooose every 100th plant therreafter until 8 plants aree selected  
i.e. 5tth, 15th, 25th, … 
The 8 plants seleected will bee evenly sprread out acrooss the row of
80 plants.  
((ii)  Unbiaased estimaate of the population meean,  

Unbiaased estimaate of the population varriance,

sgexamguru.com
 

12 
 

JJC P2 Solutions 293


((iii) 

Undeer , sincee n =80 >50, by Centrall Limit Theoorem, 

Test sstatistic appprox. 


From
m GC, p–valuue = 0.0137731  
= 0.01377 (to 3 s.f.)

Sincee p–value = 0.0137 > , we do not rejeect at 1%


%
level of significaance and connclude that tthere is insuufficient
evideence that thee researcherr’s claim is iinvalid. 
((iv)  It meeans that theere is a probaability of 0.01 of concluuding that the
t
popullation meann mass of a nnew variety of potato iss less than 150g
givenn that the poopulation meean mass off a new varieety of potatoo is
in facct 150g.  
sgexamguru.com

13 
 

JJC P2 Solutions 294


 
Unbiaased estimaate of the population varriance  

Undeer , test statistic


s  

 
Reseaarcher’s claim is invalidd at 1% leveel of significance 

at 1% level of signifiicance 

(to 3 ss.f) 

 
sgexamguru.com

14 
 

JJC P2 Solutions 295


MERIDIAN JUNIOR COLLEGE
JC2 Preliminary Examination
Higher 2

___________________________________________________________________

H2 Mathematics 9740/01
Paper 1 13 September 2016

3 Hours
Additional Materials: Writing paper
List of Formulae (MF 15)
___________________________________________________________________
READ THESE INSTRUCTIONS FIRST

Write your name and civics group on all the work you hand in.
Write in dark blue or black pen on both sides of the paper.
You may use a soft pencil for any diagrams or graphs.

sgexamguru.com
Do not use staples, paper clips, highlighters, glue or correction fluid.

Answer all the questions.


Give non-exact numerical answers correct to 3 significant figures, or 1 decimal place in the case of
angles in degrees, unless a different level of accuracy is specified in the question.
You are expected to use a graphing calculator.
Unsupported answers from a graphing calculator are allowed unless a question specifically states
otherwise.
Where unsupported answers from a graphing calculator are not allowed in a question, you are
required to present the mathematical steps using mathematical notations and not calculator
commands.
You are reminded of the need for clear presentation in your answers.

At the end of the examination, fasten all your work securely together.
The number of marks is given in brackets [ ] at the end of each question or part question.

___________________________________________________________________
This document consists of 8 printed pages.

[Turn Over

MJC P1 Questions 296


2

1 A graphic calculator is not to be used for this question.


Show algebraically that x 2  2 x  5 is always positive for x  , and solve the inequality
x x2
 3 . [4]
x  2 x  5 x  2 x2  5x
2

ex ex  2
Hence solve the inequality 2 x  . [2]
e  2e x  5 e3 x  2e2 x  5e x

 ln  x  dx , where p > 1.
p
2 (a) Find, in terms of p, [2]
1

(b)

8
q

1
q x
q

sgexamguru.com
O 1

The diagram shows the curve with the equation y  x3 . The area of the region
bounded by the curve, the lines x  1 , x  q and the x-axis is equal to the area of
the region bounded by the curve, y  1 , y  8 and the y-axis , where q  1 . Find
1
the exact value of q in the form a b , where a and b are integers. [4]

3 Prove by mathematical induction that

1 1 1  1  
1   ...   2 1  , nZ . [5]
1 2 1 2  3 1  2  3  ...  n  n 1 

1 1 1
Hence state the value of the infinite series 1    ...   ... .
1 2 1 2  3 1  2  3  ...  n
[1]

MJC/2016 JC2 Preliminary Examinations/9740/01

MJC P1 Questions 297


3

Let f  x   cos x, where 1  x  1 and 0  f  x    . Show that


1
4

1  x  f   x   x f   x  .
2
[2]

By further differentiation of this result, or otherwise, find the first three non-zero terms

in the expansion of f  x  in ascending powers of x. [3]

The diagram shows a triangle ABC. Given that the lengths of AB and AC are 1 and x units

respectively, show that sin 1 x  cos 1 x  . [1]
2
A

1
x

B C
Hence find the series expansion of sin 1 x in ascending powers of x, up to and including
the term in x3 . [1]

sgexamguru.com
4 3
5 [It is given that a sphere of radius r has surface area 4 r 2 and volume  r .]
3

A rice farmer wants to build a new grain silo to store his rice grains. The cylindrical
section has height h m and the hemispherical roof has radius r m. After building the grain
silo, the farmer will be painting its rooftop and the external curved surface. The time
needed to paint the grain silo will be 20 minutes per square metre for the curved surface
area of the cylinder and 35 minutes per square metre for the hemispherical roof. Given
that a total time of 60 000 minutes is taken to paint the grain silo, find, using
differentiation, the value of r which gives a grain silo of maximum volume. [8]

MJC/2016 JC2 Preliminary Examinations/9740/01 [Turn Over

MJC P1 Questions 298


4

6 The diagram below shows the graph of y  2ln  x  1  4  x .

The two roots of the equation 2ln  x  1  4  x  0 are denoted by  and  , where
  .
y

x
  

(i) Find the values of  and  , correct to 3 decimal places. [2]

A sequence of real numbers x1 , x2 , x3 , ... where xn  1 , satisfies the recurrence relation

xn1  ln( xn  1)2  4 for n  1 .


(ii) sgexamguru.com
Prove algebraically that if the sequence converges, it must converge to either  or
. [2]

(iii) Use a calculator to determine the behaviour of the sequence for each of the cases
x1  3 , x1  12 . [2]

(iv) By considering xn1  xn and the graph above, prove that


xn1  xn if   xn   ,
xn1  xn if 1  xn   or xn   . [2]

MJC/2016 JC2 Preliminary Examinations/9740/01

MJC P1 Questions 299


5

7 The equations of three planes are


x  2 y  z  60
4 x  5 y  10 z  180
2 x  3 y  4 z  100
(i) It is given that all three planes meet in the line l. Find a vector equation of l. [2]

(ii) Find a cartesian equation of the plane which contains l and the origin. [3]

A technology company specialises in manufacturing circuit boards that are used for space
exploration. It manufactures only 3 types of circuit boards (A, B and C). Each circuit board
requires particular amounts of different raw materials for manufacturing. The amounts of
raw material (in units) required for each type of circuit board and the total amounts of raw
material available to the company are shown in the following table.

Copper Lead Fibreglass


Circuit Board A 1 4 2
Circuit Board B 2 5 3
Circuit Board C 1 10 4

sgexamguru.com
Total amount of material
available (in units)
60 180 100

The company is required to use all the materials available to manufacture its circuit
boards.

x
 
The vector r   y  is defined such that variables x, y, and z represent the number of
z
 
circuit boards A, B and C that are manufactured respectively.

(iii) With the aid of your answer in part (i), solve for r. Leave your answer clearly in
the form of a  b and state the possible values for  . [2]
(iv) Explain, in context, why your vector equation in part (i) is not an appropriate answer
for part (iii). [2]

MJC/2016 JC2 Preliminary Examinations/9740/01 [Turn Over

MJC P1 Questions 300


6

8 Functions f and g are defined by


2
 1 
f :x   , x  1,
 x 1 
g: x ln x, x 0.

(i) Show that gf exists and express gf in a similar form. [3]

(ii) Sketch, in a single diagram, the graphs of g and gf, labelling each graph clearly.
Write down the range of gf. [3]

(iii) Describe a sequence of transformations which maps the graph of g onto the graph
of gf. [4]

9 It is given that
x
f  x  , where  1  x  1.

 1  x2 
(i) Show by differentiation that f is strictly increasing. [3]

(ii) Sketch the graph of y  f  x  , stating the equations of any asymptotes and the

sgexamguru.com
coordinates of any points of intersection with the axes. [3]

The diagram below shows the graph of y  g  x  , which is continuous and differentiable

on  1, 1 . It has a minimum turning point at  0, 3 .

x
O

(iii) It is given that w  x   g  x  f  x  , where 1  x  1 . By finding w '  x  and using

your earlier results in (i) and (ii), determine the number of stationary points on the
graph of w. [4]

MJC/2016 JC2 Preliminary Examinations/9740/01

MJC P1 Questions 301


7

10 (a) Solve the simultaneous equations


5
z  w  2i 1 and z 2  iw  0,
2
giving z and w in the form x  yi where x and y are real. [5]

where w  2  cos   isin   ,      , express the


1
(b) (i) Given that z  w 
w
real and imaginary parts of z in terms of  . [3]

(ii) Hence show that locus of z on an Argand diagram lies on the curve with

x2 y 2
cartesian equation 2  2  1 where a and b are constants. [3]
a b
(iii) Sketch this locus on an Argand diagram, indicating clearly the points of
intersection with the axes. [2]

sgexamguru.com

MJC/2016 JC2 Preliminary Examinations/9740/01 [Turn Over

MJC P1 Questions 302


8

11
y

x
O Q

It is given that curve C has parametric equations


x  t3, y   1  t 2  for 0  t  1 .

The diagram shows the curve C and the tangent to C at P . The tangent at P meets the
x -axis at Q.

(i) The point P on the curve has parameter p . Show that the equation of the tangent

sgexamguru.com
at P is 3 p 1  p 2   3 py  1  p 2   x  p3 . [3]

(ii) Given further that the line y      x meets the curve at point P , find the exact

coordinates of P . [3]

(iii) Hence find the exact coordinates of Q . [2]

(iv) Show that the area of the region bounded by C , the tangent to C at P , and the
  1 2
x-axis is given by  1 3t   t 2  dt . [3]
32 2

Show that the substitution t  sin u transforms the above integral to


9   3 2
32 8  6
  1  cos 4u du . Hence, evaluate this area exactly. [6]

END OF PAPER

MJC/2016 JC2 Preliminary Examinations/9740/01

MJC P1 Questions 303


2016 H2 MATH (9740/01) JC 2 PRELIM EXAMINATION SOLUTIONS

Qn Solution
1 Inequalities
x 2  2 x  5   x  1  1  5
(i) 2

  x  1  4  0 for all real x


2

(ii) x x2
 3 , x0
x  2 x  5 x  2 x2  5x
2

x x2
 0
x  2 x  5 x  x  2 x  5
2 2

x2  x  2
0
x  x 2  2 x  5

Since x 2  2 x  5  0 for all real x,


 x  2  x  1  0
x
x  1 or 0  x  2

ex  2
sgexamguru.com
(iii) ex

e 2 x  2e x  5 e3 x  2e 2 x  5e x
Replace x with e x .
1  e x  0 (rej.  e x  0) or e x  2
x  ln 2

MJC/2016
Microsoft JC2 Prelim Examination Marking Scheme/H2 Math (9740/01)/Math Dept
Word - MJC_JC2_H2_Maths_Prelim_P1_Solutions Page 1 of 12 304
Qn Solution
2 Definite Integrals
(a) p 1 
ln(x )dx   x ln x 1     x dx
p

p
1 1
 x
  p ln p  0    p  1 
 p ln p  p  1
(b) q 8

1
x 3 dx  
1
3 y dy
8
 43 
q
x  4
3y 
 4   4 
 1
 1
q 4 1  3   43 
     8  1
4 4  4  
4
q 1 3
    16  1
4 4 4
q 4  46
1
q  46 4
 a  46, b  4

sgexamguru.com
Qn Solution
3 Mathematical Induction + APGP
(i) 1 1 1  1  
Let Pn be the statement 1    ...   2 1  ,n .
1 2 1 2  3 1  2  3  ...  n  n 1 
When n  1,
LHS = 1.
 1 
RHS  2  1    1  LHS
 11 
 P1 is true.

Assume that Pk is true for some k 
1 1 1  1 
i.e. 1    ...   2 1  
1 2 1 2  3 1  2  3  ...  k  k 1 
To show that Pk 1 is also true
1 1 1  1 
i.e. 1    ...   2 1  
1 2 1 2  3 1  2  3  ...  k  1   k  1  1 
When n  k  1,
LHS

MJC/2016
MJC P1 SolutionsJC2 Prelim Examination Marking Scheme/H2 Math (9740/01)/Math Dept Page 2 of 12 305
1 1 1
 1   ... 
1 2 1 2  3 1  2  3  ...  k  1
 1  1
 2 1  
 k  1  1  2  3  ...  k  1
 1  1
 2 1    k 1
 k 1  1  k  1
2
 1  2
 2 1  
 k  1   k  1 k  2 
2  k  2   2
 2
 k  1 k  2 
2k  2
 2
 k  1 k  2 
2  k  1
 2
 k  1 k  2 
2
 2
 k  2
 1 
 2 1  
   k  1  1 
 

 Pk is true  Pk 1 is true

sgexamguru.com
Since P1 is true and Pk is true  Pk 1 is true, by Mathematical Induction, Pn is true for
all n   .
1 1 1  1 
1   ...   2 1  
1 2 1 2  3 1  2  3  ...  n  n 1 
1  1 
As n  ,  0  2 1  2
n 1  n 1
1 1 1
1    ...   ...  2
1 2 1 2  3 1  2  3  ...  n

Qn Solution
4 Maclaurin Series
f ( x)  cos 1 x
1
f ( x)  
1  x2
3
1
1  x 2  2  2 x 

f ( x) 
2
1

1  x  f ( x)   x 1  x 
2 2 2

1  x  f ( x)  x f ( x) (shown)
2

MJC/2016
MJC P1 SolutionsJC2 Prelim Examination Marking Scheme/H2 Math (9740/01)/Math Dept Page 3 of 12 306
1  x  f ( x)  x f ( x)
2

1  x  f ( x)  2 x f ( x)  x f ( x)  f ( x)


2

1  x  f ( x)  3x f ( x)  f ( x)
2


f (0)  , f (0)  1, f (0)  0, f (0)  1
2
 x3
cos 1 x  x  ...
2 6
A
Let sin 1 x  
Let cos 1 x   1
x

  
2
 B C
sin 1 x  cos 1 x  (shown)
2

sin 1 x  cos 1 x 
2
   x3 
sin 1 x   cos 1 x     x   ... 
2 2 2 6 
x3
sin 1 x  x   ...
6

sgexamguru.com
Qn Solution
5 Maximum/Minimum Problem
Let volume of silo be V
2
V   r 2h   r 3
3
Time needed to paint the silo  20  2 rh   35  2 r 2 
60000  40 rh  70 r 2
60000  70 r 2
h
40 r
 60000  70 r 2  2 3
V   r2   r
 40 r  3
7 2
 1500r   r 3   r 3
4 3
13
 1500r   r 3
12

dV 13
 1500   r 2
dr 4
dV
For maximum V, 0
dr

MJC/2016
MJC P1 SolutionsJC2 Prelim Examination Marking Scheme/H2 Math (9740/01)/Math Dept Page 4 of 12 307
13 2
 1500  r  0
4
13 2
 r  1500
4
6000
r2 
13
6000
r
13
6000
Since r  0 ,  r   12.1207  12.1  3 s.f. .
13
d 2V 13
2
   r  0 , for r  12.1207 .
dr 2

Alternative:
 
r  6000  6000  6000 
  13  
 13   13 
dV + 0 −
dr

Qn Solution
6 Recurrence Relations
(i) Using GC, roots of equation are   1.253,   7.848 .

(ii) sgexamguru.com
As n  , xn  L, xn1  L
 L  ln  L  1  4  2 ln  L  1  4  L  0
2

Since equation is identical to 2 ln  x  1  4  x  0


 L  1.253   or L  7.848  
Hence the sequence converges to either  or  .

(iii) Using GC, it can be observed that


when x1  3 , the sequence increases and converges to 7.848   .
when x1  12 , the sequence decreases and converges to 7.848   .

(iv) xn 1  xn  ln( xn  1)2  4  xn


From graph,

if   x n   , 2 ln( xn  1)  4  xn  0  ln( xn  1)2  4  xn  xn 1  xn


if 1  xn   or xn   , 2 ln( xn  1)  4  xn  0  ln( xn  1) 2  4  xn  xn 1  xn .

MJC/2016
MJC P1 SolutionsJC2 Prelim Examination Marking Scheme/H2 Math (9740/01)/Math Dept Page 5 of 12 308
Qn Solution
7 Vectors
(i) Using GC,

x  20  5 z
y  20  2 z
zz

 20   5 
   
l : r   20     2  ,  
 0 1
   
(ii) Normal Vector
 20   5   20 
     
 20    2    20 
 0   1  140 
     

 1  0 1 
     
r  1    0   1   0
 7  0  7 
     

Cartesian Equation: x  y  7 z  0 or equivalent

(iii)  20 
 
sgexamguru.com
 5 
 
r   20     2  where   0,1, 2,3, 4
 0 1
   
(iv) The vector equation in (i) allows for x, y and z to be real numbers. But the circuit
boards produced is a physical quantity and must minimally be an integer.

Qn Solution
8 Functions & Transformation of Graphs
(i) Since R f   0,    Dg   0,   , gf exists.

2
 1 
gf : x  ln   , x  1.
 x 1

MJC/2016
MJC P1 SolutionsJC2 Prelim Examination Marking Scheme/H2 Math (9740/01)/Math Dept Page 6 of 12 309
(ii)
y

1, 0  x
O
 0.466, 0.764 

R gf   ,   
(iii) Since x  1, x  1  0 , gf  x   2 ln  x  1

From y  ln x to y  2 ln  x  1 :
1. Translation of 1 unit in the negative x-direction

sgexamguru.com
2. Reflection in the x-axis
3. Scaling parallel to y-axis by a factor of 2

[Accept any other possible correct sequence such as 1-3-2]

Qn Solution
9 Curve sketching and differentiation
(i) x
f ( x) 

 1  x2 
1

  1
 

 1  x 1  x   1  x 2
2 2   2 x 
f '( x)  2
1  x2 
1 x2
 

 1  x2  1  x  3
2 2

 3 

1
3
 0  1  x  1, 1  x 2
   2 0 

1  
x 2 2  

Since f '  x   0 for 1  x  1 , f is strictly increasing.

MJC/2016
MJC P1 SolutionsJC2 Prelim Examination Marking Scheme/H2 Math (9740/01)/Math Dept Page 7 of 12 310
(ii) y

y  f  x

(iii) w'  x   g  x  f '  x   g'  x  f  x 


From (i), f '  x   0 for  1  x  1.
 0 for  1  x  0

From (ii), f  x     0 for x  0
  0 for 0  x  1

 0 for  1  x  0

From the given graph, g  x   0 for  1  x  1 and g'  x    0 for x  0
 0 for 0  x  1

Therefore,
When 1  x  0, w'  x   0. .
When x  0, w'  x   0.
When 0  x  1, w'  x   0.

sgexamguru.com
Since w'  x   0  0 for  1  x  1, there are no stationary points in the graph of w.

Qn Solution
10 Complex Numbers
(a) z  w  2i  1 --- (1)
5
z 2  iw   0 --- (2)
2
Method 1
From (1): w  z  2i  1 --- (3)
Substitute (3) into (2):
5
z 2  i  z  2i  1  0
2
1
z 2  iz  i  0
2

 1
  i    i   4 1  i  
2

 2
z
2(1)
i  3  4i

2

MJC/2016
MJC P1 SolutionsJC2 Prelim Examination Marking Scheme/H2 Math (9740/01)/Math Dept Page 8 of 12 311
i  1  2i 

2

1 3 3 1 1 1 1 5
z   i, w   i , or z    i, w   i ,
2 2 2 2 2 2 2 2

----
Method 2
Substitute (1) into (2):
5
 w  2i 1
2
 iw   0
2
5
w2   2i  1  2  2i 1 w  iw   0
2

2
1
w2  w  3i  2   4i  0
2
 1 
  3i  2   3i  2  4 1    4i 
2

 2 
w
2(1)
  3i  2  1  2i 
w
2
3 1 1 3 1 5 1 1
w  i , z   i or w   i , z    i

sgexamguru.com
2 2 2 2 2 2 2 2

(i) 1 1 1  3 5
z  w  2 cos   2i sin    cos   i sin   = cos   i sin 
w 2 2  2 2
3 5
Re( z )  cos  , Im( z )  sin 
2 2

3 5
(ii) x cos  , y  sin 
2 2
2 2
 cos   x, sin   y
3 5
Since cos   sin   1
2 2

2 2
2  2  Im
 x  y 1
3  5 
x2 y2
2
 2
1 Locus of z
3 5
   
2 2 O Re
3 5
 a  , b=
2 2

MJC/2016
MJC P1 SolutionsJC2 Prelim Examination Marking Scheme/H2 Math (9740/01)/Math Dept Page 9 of 12 312
Qn Solution
11 Parametric Equations + Applications of Differentiation and Integration
(i) dx dy t
 3t 2 , 
dt dt 1 t2

dy dy dx
 
dx dt dt
t 1
  2
1  t 2 3t
1

3t 1  t 2

At point P , x  p 3 , y  1  p 3 .

1
y  1  p2   x  p 
3

Equation of tangent at P : 3 p 1 p 2

3 p 1  p 2   3 py  1  p 2   x  p 3 (Shown)

(ii) Substitute x  p 3 , y  1  p 2 into y  4 3 x

1  p 2  4 3 p3

sgexamguru.com 1  p 2  48 p 6
48 p 6  p 2  1  0

1 1
Using GC, p   or p
2 2
(N.A. since 0  p  1 )

1 3
 Exact coordinates of p are  ,  .
 8 2 

(iii) Equation of tangent to C at P :


 1   1   3
2 2 3
1 1
3   1      y 1     x   
 2    2   2 2 2
9 3y 3 1
  x
8 2 4 8

When y  0 ,
9 1
 x
8 8
5
x
4

MJC/2016
MJC P1 SolutionsJC2 Prelim Examination Marking Scheme/H2 Math (9740/01)/Math Dept Page 10 of 12 313
5 
 Exact coordinates of Q are  , 0  .
4 

(iv)
y

x
O

1 1
When x  , t  .
8 2
When x  1 , t  1.

Area of shaded region = Area of required region

sgexamguru.com

1  3  5 1 
     
2  2   4 8 
1
8
1
yC dx

9 3 1  dx 
  1 1  t 2   dt
32 2  dt 
9 3 1 2
32 2
  1 3t 1  t 2 dt

(Shown)

dt
Let t  sin u,  cos u.
du
1 
When t  , u  .
2 6

When t  1, u  .
2

MJC/2016
MJC P1 SolutionsJC2 Prelim Examination Marking Scheme/H2 Math (9740/01)/Math Dept Page 11 of 12 314

9 3 1 2 9 3
 1 3t 1  t 2 dt   2 3sin 2 u 1  sin 2 u cos u du
32 2 32 6

9 3
  2 3sin 2 u cos 2 u du
32 6

9 3 3
  2sin u cos u 
2
  2
du
32 4 6

9 3 3
 
32 4 
6
2
sin 2 2u du

9 3 3 2
32 8  6
   1  cos 4u du


9 3 3 2 9 3 3 1 2
  1  cos 4u du   u  sin 4u  du
32 8 6 32 8  4 
6

9 3 3   1   1 2 
    sin 2     sin 
32 8  2 4  6 4 3 
9 3 3    1  3 
      
32 8  2 6 4  2  

9 3 3  3
    

sgexamguru.com
32 8  3 8 
9 3 3 3 
  
32 64 8
15 3 
 
64 8

MJC/2016
MJC P1 SolutionsJC2 Prelim Examination Marking Scheme/H2 Math (9740/01)/Math Dept Page 12 of 12 315
MERIDIAN JUNIOR COLLEGE
JC2 Preliminary Examination
Higher 2

___________________________________________________________________

H2 Mathematics 9740/02
Paper 2 21 September 2016

3 Hours
Additional Materials: Writing paper
List of Formulae (MF 15)
___________________________________________________________________
READ THESE INSTRUCTIONS FIRST

Write your name and civics group on all the work you hand in.
Write in dark blue or black pen on both sides of the paper.
You may use a soft pencil for any diagrams or graphs.

sgexamguru.com
Do not use staples, paper clips, highlighters, glue or correction fluid.

Answer all the questions.


Give non-exact numerical answers correct to 3 significant figures, or 1 decimal place in the case of
angles in degrees, unless a different level of accuracy is specified in the question.
You are expected to use a graphing calculator.
Unsupported answers from a graphing calculator are allowed unless a question specifically states
otherwise.
Where unsupported answers from a graphing calculator are not allowed in a question, you are
required to present the mathematical steps using mathematical notations and not calculator
commands.
You are reminded of the need for clear presentation in your answers.

At the end of the examination, fasten all your work securely together.
The number of marks is given in brackets [ ] at the end of each question or part question.

___________________________________________________________________
This document consists of 7 printed pages.

[Turn Over

MJC P2 Questions 316


2

Section A: Pure Mathematics [40 marks]

1 Referred to the origin O, points A and B have position vectors a and b respectively, where
a and b are non-zero vectors that are neither perpendicular nor parallel to each other.
The length of projection of a onto b and the length of projection of b onto a are equal.
Show that a  b . [2]

Hence state the geometrical interpretation of a  b . [1]

It is further given that a  i  j  3k and b  2i  j  pk , where p  0 .

(i) Find the exact value of p. [2]

(ii) A circle with centre O passes through A and B. Find the area of the minor sector
OAB. [4]

2 (a) The roots of the equation z 3  z 2  z  15  0 are denoted by z1 , z2 and z3 where

sgexamguru.com
arg  z1   0 , and arg  z2   arg  z3  . Find z1 , z2 and z3 and show these roots on an

Argand diagram. [3]

Explain why the locus of all points z such that z  1  2 passes through the roots

represented by z2 and z3 Draw this locus on the same Argand diagram. [3]


i 
(b) (i) Show that 1  ei  2e 2 cos . [2]
2

(ii) Hence find, in trigonometric form, the imaginary part of the complex number
e i
w . [2]
1+ei

MJC/2016 JC2 Preliminary Examinations/9740/02

MJC P2 Questions 317


3

3 Newton’s law of cooling states that the rate of decrease of temperature of a hot body is
proportional to the difference in temperature between the body and its surroundings.
Using t for time in minutes,  for temperature of the body in C and  for the
temperature of the surroundings (assumed constant), express the law in the form of a
differential equation. [1]

(i) Show that the general solution of the differential equation may be expressed in the
form     Ae kt where A and k are constants. [3]

(ii) Given that   9 when t  0 and that   5 when t  T , find, in terms of T, the
value of t when   2 . [3]

(iii) State what happens to  for large values of t and sketch the solution curve of 
against t. [3]

4 (a) Judith is making a pattern consisting of rows of matchstick triangles as shown. She
uses three matchsticks to complete a triangle. She adds two more triangles in the
second row, three more triangles in the third row and four more triangles in the

sgexamguru.com
fourth row.

Judith has completed n  1 rows in the pattern. How many matchsticks does she
need in order to form the nth row? [1]

Show that the total number of matchsticks used in making a pattern with n rows is
3n  n  1
. Hence find the maximum number of complete rows she is able to make
2
with two thousand matchsticks. [4]

(b) A geometric progression has first term a and second term b, where a and b are non-
zero constants. Given that the sum to infinity of the series is a  2b , find the
common ratio. [2]
The sum of the first n terms is denoted by Gn . Find Gn in terms of a and n. Hence
N
show that G
n 1
n  2aN  GN . [4]

MJC/2016 JC2 Preliminary Examinations/9740/02 [Turn Over

MJC P2 Questions 318


4

Section B: Statistics [60 marks]

5 (i) Describe what is meant by ‘systematic sampling’. [2]

(ii) A bakery wishes to gather feedback on what residents in the neighbourhood think
of its new salted egg lava buns. A surveyor is hired to survey a sample of 150
residents who visit the bakery during the evening rush hour using systematic
sampling. State, in this context, one advantage and one disadvantage of this
procedure. [2]

6 A box consists of a very large number of balls, of which 20% are red and 80% are white.

A game consists of a player drawing n balls at random from the box and counting the
number of red balls drawn. If at most one red ball is drawn, the player wins. If more than
two red balls are drawn, the player loses. If exactly two red balls are drawn, the player
draws another n balls and if none of these n balls drawn are red, the player wins.
Otherwise, the player loses.

sgexamguru.com
Show that the probability that a randomly chosen player wins is P where
n
P   0.8  0.2n  0.8     0.2   0.8 .
n 1 2 2n2
[3]
 2

(i) Given that the probability that a randomly chosen player wins is less than 0.1, write
down an inequality in terms of n to represent this information. Hence find the least
possible value of n. [3]

(ii) Given instead that P  0.3 , find the probability that out of 100 games played, at
least 40 games are won. [2]

MJC/2016 JC2 Preliminary Examinations/9740/02

MJC P2 Questions 319


5

7 An overseas study revealed that school children sleep an average of 6.5 hours each night.
Ms Patricia believes that the children in her school sleep even fewer than that. She took a
random sample of 8 children from her school. The number of hours of sleep each child
gets at night was reported as:

5.9 6 6.1 6.2 6.3 6.5 6.7 6.9


Test, at the 8% level of significance, whether this evidence supports Ms Patricia’s belief,
stating clearly any assumption made. [5]

Ms Patricia conducted a further study involving a random sample of 15 children from


another school and the number of hours of sleep each child gets at night is recorded. The
sample mean is x and the sample variance is 0.849. Find the set of values of x for which
the null hypothesis would be rejected at the 8% level of significance. [3]

8 The mass of a randomly chosen bar of body soap manufactured by a factory has a normal
distribution with mean 110 grams and standard deviation 1.5 grams.

sgexamguru.com
(i) Find the probability that the difference in sample means between any two random
samples of 20 bars of body soap each, is within 0.5 grams. [4]

(ii) Five randomly chosen bars of body soap are liquefied and separated into four equal
portions, which are each placed into a bottle. Find the probability that the mass of
liquid body soap in a randomly chosen bottle exceeds 140 grams. [4]

The factory ventured into the manufacturing of coconut oil soap as its new product and
the mass of a randomly chosen bar of coconut oil soap has a normal distribution. A
random sample of 15 bars of coconut oil soap is taken and the mass, u grams, of each bar

 u  1590 ,  u
2
is measured. The results are summarised by  169046 .

(iii) Find unbiased estimates of population mean and variance. [2]

MJC/2016 JC2 Preliminary Examinations/9740/02 [Turn Over

MJC P2 Questions 320


6

1 1
9 (a) For events A and B, it is given that P  A  and P  B   .
4 2
3
(i) Given that P  A ' | B   , determine whether events A and B are independent
4
and calculate P  A  B  . [3]

2
(ii) For a third event C, it is given that P  C | A  . Find the value of P  A  C  .
3
[2]
(b) Find the number of ways in which the word EVERYDAY can be arranged if

(i) all the vowels (A, E) must be together and the two ‘Y’s must be separated,
[3]
(ii) the repeated letters E and Y must appear symmetrical about the centre of the
word (e.g. EVRYYDAE, YVERDEAY). [2]

10 (i) A bakery sells cookies in tins and keeps track of the number of tins sold per week.
State two conditions under which a Poisson distribution would be a suitable

sgexamguru.com
probability model for the number of tins sold in a week. [2]

(ii) Two types of cookies, chocolate and raisin, are sold. The mean number of tins for
chocolate cookies sold in a week is 2.4. The mean number of tins for raisin cookies
sold in a week is 1.8. Use a Poisson distribution to find the probability that in a
given week, the total number of tins sold is more than 9. [3]

(iii) Use a normal approximation to the Poisson distribution to find the probability that
the total number of tins sold in 4 weeks is at least 15 but not more than 25. [4]

(iv) Explain why the Poisson distribution may not be a good model for the number of
cookies sold in a year. [1]

MJC/2016 JC2 Preliminary Examinations/9740/02

MJC P2 Questions 321


7

11 The table gives the population y, in thousands, for a particular species of mammal over
10 years.

Year, x 1 2 3 4 5 6 7 8 9 10
Population, y
10.8 8.7 6.9 5.5 4.4 3.5 2.8 2.3 1.8 1.4
(in thousands)

(i) Find the equation of the regression line of y on x, giving your answer to 3 decimal
places. [1]

(ii) Let Y be the value obtained by substituting a value of x into the equation of the

 y Y 
2
regression line of y on x found in (i). Find . [1]

(iii) For each of the values of x, Y ' is given by Y '  A  Bx , where A and B are any

  y  Y '
2
constants. What can you say about the value of ? [1]

(iv) Draw a scatter diagram to illustrate the data. [2]

An animal conservationist suggested the model ln y  c  dx for this set of data.

sgexamguru.com
(v) Find, correct to 4 decimal places, the value of the product moment correlation
coefficient between

(a) x and y,

(b) x and ln y. [2]

(vi) Use your answers to parts (iv) and (v) to explain which of y  a  bx or
ln y  c  dx is the better model. [1]

(vii) Using the better model found in (vi), predict the population of this species in the
20th year. [2]

END OF PAPER

MJC/2016 JC2 Preliminary Examinations/9740/02

MJC P2 Questions 322


2016 H2 MATH (9740/02) JC 2 PRELIM EXAMINATION SOLUTIONS

Qn Solution
1 Vectors
a b b  a

b a
a  b as a  b  0 (as a and b are not perpendicular)

a  b is the area of rhombus with adjacent sides OA and OB.

(i) a  i  j  3k  11 b  2 i  j  p k  5  p 2
As a  b ,
 11  5  p 2
 p2  6
Since p  0, p   6
(ii) Let  denote angle AOB.
a  b
cos  
a b
1  2 
   
 1    1 
 3  6 
   

sgexamguru.com
  
11 11

3  3 6

11
 3  3 6 
  cos 1    2.79569 rad or 160.18126 degrees
 11 

Area of minor sector OAB


  
  r 
1 2  2
 r
2  360 
1  160.18126 
 11 2.79569  OR   11 
2  360 
 15.4 units 2  15.4 units 2

 2.79569 
   r 
2

 2 
 2.79569 
OR   11 
 2 
 15.4 units 2

MJC/2016
Microsoft JC2 Prelim Examination Marking Scheme/H2 Math (9740/02)/Math Dept
Word - MJC_JC2_H2_Maths_Prelim_P2_Solutions Page 1 of 10 323
Qn Solution
2 Complex Numbers
(a) Using GC, z1  3, z2  1  2i, z3  1  2i.

Im

2
3
Re
(1,0) O

2

The locus of points given by z  1  2 passes through the roots represented by z2


and z3 since z2  1  1  2i  1  2i  2 and z3  1  1  2i  1  2i  2 .


(b) i i2 
i  i
(i) 1 e  e  e  e 2  2

sgexamguru.com
 

i     
 e 2  cos  i sin  cos  i sin 
 2 2 2 2
 
i   i 
 e 2  2 cos   2e 2 cos
 2 2
i
(ii) e
w
1  ei

i
ei e2
 
i

 
2e 2 cos 2cos
2 2
 
cos  isin
 2 2  1  1 i tan 
 2 2 2
2cos
2
1 
 Im( w)  tan
2 2

Qn Solution
3 Differential Equations
dθ  k θ  α , k is a positive constant
dt
 

MJC/2016
MJC P2 SolutionsJC2 Prelim Examination Marking Scheme/H2 Math (9740/02)/Math Dept Page 2 of 10 324
dθ  k θ  α , k  0
(i)
dt
 
 θ  α dθ   k dt
1

ln  θ  α   kt  c since θ > α


θ  α  ekt c
θ  α  Aekt, A  ec
θ  α  Aekt (shown)
(ii) When t  0,  9
 9    A  A  8
When t  T ,  5
5    8 e  kT
1
e  kT 
2
kT  ln 2
ln 2
k
T
 ln 2
t
    8 e T

  ln 2
t 
   1  8e T

 

When   2
sgexamguru.com

2   1  8e T 
 ln 2
t 

 
 ln 2
t 1
e T 
8
ln 2
 t   ln 8
T
ln 8
t T  3T
ln 2
(iii)  ln 2
t
For large values of t, e T  0,    ,
 decreases and approaches to  .


 ln2
t
   1 8e T 
9
 


t
O

MJC/2016
MJC P2 SolutionsJC2 Prelim Examination Marking Scheme/H2 Math (9740/02)/Math Dept Page 3 of 10 325
Qn Solution
4 APGP + Summation
(a) 1st row: number of matches = 3
2nd row: number of matches = 6
3rd row: number of matches = 9

nth row: number of matches = 3   n  1 3   3n


1 row: total number of matches = 3
2 rows: total number of matches = 3 + 6
3 rows: total number of matches = 3 + 6 + 9

n rows: total number of matches


n n
  3  3n    2  3   n  1 3 
2 2
OR
3n  n  1 3n  n  1
 (shown)  (shown)
2 2
3n  n  1
 2000
2
Using GC,
3n  n  1
When n  36,  1998  2000
2
3n  n  1
When n  37,  2109  2000
2

(b) b
sgexamguru.com
Maximum number of complete rows  36.

Let r 
a
a
 a  2b
1 r
1
 1  2r
1 r
1  1  r  2r 2
2r 2  r  0
r  2r  1  0
b 1
r  0 (rejected
 0) or r 
a 2
1
 common ratio =
2

  1 n 
a 1    
 2 
Gn   
1
1
2
  1 n 
Gn  2a  1    
 2 
 

MJC/2016
MJC P2 SolutionsJC2 Prelim Examination Marking Scheme/H2 Math (9740/02)/Math Dept Page 4 of 10 326
N 
1 
N n


n 1
Gn  2 a 
n 1 
1 
   
2 
 N
1 
n

 2a  N     
 n 1  2   
 1   1  
N

 1     
 2   2   
 2a  N  
1
 1 
 2 
 
   1  N

 2a  N   1     
   2  
  
  1 N 
 2aN  2a 1    
 2 
 
 2aN  GN

Qn Solution
5 Sampling Methods
(i) Systematic sampling is a sampling method in which the entire population is listed
in some order. The population is divided into sampling intervals of k members.

sgexamguru.com
After obtaining a random starting point from the first k members, every kth
member is chosen from the list until the required number is achieved.
(ii) Possible Advantages:
 It is easy to conduct the survey as the members of the sample are easily
accessible.
 It is easy to conduct as the surveyor does not need the list of all the residents in
the neighbourhood.

Possible Disadvantages:
 It is a biased sample as only residents who visit the bakery during the evening
rush hour is surveyed. Hence the sample may not be representative.
 It is a biased sample as some people may visit the bakery multiple times during
the evening rush hours increasing their chances to be selected.
 It may not be easy to get residents to visit the bakery in sequence so selection
of every kth resident in this case may be difficult.

MJC/2016
MJC P2 SolutionsJC2 Prelim Examination Marking Scheme/H2 Math (9740/02)/Math Dept Page 5 of 10 327
Qn Solution
6 Binomial Distribution
Let X be the number of red balls drawn, out of n balls.
X ~ B  n, p 

Player wins

Player wins

Player draws
another
balls
Player loses

Player loses

P  P  player wins 
 P  X  1  P  X  2  P  X  0 

n
sgexamguru.com
 P  X  0   P  X  1  P  X  2  P  X  0 
n  n  n  2   n  n
    0.2   0.8      0.2   0.8      0.2   0.8      0.2   0.8  
0 n 1 n 1 2 0

0 1  2    0  
 n  n2 
  0.8   n  0.2  0.8      0.2   0.8    0.8 
n n 1 2 n

 2  
n
  0.8  0.2n  0.8      0.2   0.8 
n 1 2 2 n 2

2
(Shown)
(i) n
P   0.8  0.2n  0.8     0.2   0.8
n 1 2n2
 0.1
2

 2

Using G.C.,

When n  18, P  0.10218  0.1


When n  19, P  0.08509  0.1

 least n  19

(ii) Let Y be the number of games won, out of 100 games played.
Y ~ B 100, 0.3 .

MJC/2016
MJC P2 SolutionsJC2 Prelim Examination Marking Scheme/H2 Math (9740/02)/Math Dept Page 6 of 10 328
Required probability  P Y  40 
 1  P  Y  39 
 0.020988
 0.0210 (3 s.f.)

Qn Solution
7 Hypothesis Testing
Let X denote the number of hours of sleep each child gets at night.
Let  denote the population mean hours of sleep each child gets at night.
 2 
Assumption: X ~ N   ,  2
  X ~ N  ,  .
 n 

H0:   6.5
H1:   6.5
X 
Test statistic : T  ~ tn 1
S
n
Level of Significance: 8%
Reject H0 if p-value < 0.08
Under H0, using GC, p-value = 0.0998

Since p-value = 0.0998 > 0.08, we do not reject H0 and conclude that there is

sgexamguru.com
insufficient evidence at 8% level of significance, that supports Ms Patricia’s claim.

15
s2   0.849   0.90964 (5 s.f)
14
Level of Significance: 8%
Reject H 0 if t-value  1.48389
x  6.5
 1.48389
0.90964
15
x  6.13458
 set of values required 0  x  6.13

Qn Solution
8 Normal Distribution
(i) Let X be the mass of a randomly chosen bar of body soap in grams.
Let X be the sample mean mass of 20 randomly chosen bars of body soaps in
grams.

X ~ N 110,1.52 
 1.52 
X ~ N  110, 
 20 
  1.52  
X 1  X 2 ~ N  0, 2  
  20  

MJC/2016
MJC P2 SolutionsJC2 Prelim Examination Marking Scheme/H2 Math (9740/02)/Math Dept Page 7 of 10 329
 
P X 1  X 2  0.5  P  0.5  X 1  X 2  0.5 
 0.708  3 s.f.

(ii) X1  X 2  X 3  X 4  X 5
Let W be the mass of a portion of liquefied soap. W 
4
  5 110   5  1.52  
W ~ N , 
 4 42 
 
 275 45 
W ~ N , 
 2 64 
P W  140   0.00143  3 s.f.

(iii)
Unbiased estimate of population mean, u 
 u  1590 =106
15 n
1    u 
2

Unbiased estimate of population variance, s 


2

n 1 
 u  n 
2

 
1  1590  
2

  169046 
15  1  15 
253
 36.1  3 s.f. OR
7

Qn
9
(ai)
Solution
Probability sgexamguru.com
3
Since P( A ' | B ) 
 P( A ') , A ' and B are independent events  A and B are
4
independent events

P( A  B)  P( A)  P( B)  P( A  B)
 P( A)  P( B)  P( A) P( B) (Since A and B are independent)
1 1 1 1
   
4 2 4 2
5

8
(ii) P(C  A) 2
P(C | A)  
P( A) 3
2
 P( A  C )  P( A)
3
2 1 1
  
3 4 6
(bi) 3! 5
Required number of ways  4!   C2
2! Choose 2 out of 5 slots to put ‘Y’s
 720 Arrange
vowels. AEE V R D

Group vowels together, and arrange these 4 groups.

MJC/2016
MJC P2 SolutionsJC2 Prelim Examination Marking Scheme/H2 Math (9740/02)/Math Dept Page 8 of 10 330
(ii) Required number of ways  4C2  2!  4! 3. The 4 remaining letters can
permute themselves within the 4
 288 remaining blanks.

_ _E Y_ _ _ _Y E_ _
1. Choose 2 out of the 1st 4 blanks 2. Correspondingly, the remaining
to put E & Y. Because E & E & Y would take their respective
Y can switch positions. positions in the next 4 blanks.

Qn Solution
10 Poisson Distribution
(i) The average number of tins sold per week is constant.

The sale of one tin is independent of another throughout the week.


(ii) Let X be the number of tins for chocolate cookies sold in a week. X  Po  2.4 
Let Y be the number of tins for raisin cookies sold in a week. Y  Po 1.8
X  Y  Po  4.2 
P  X  Y  9   1  P  X  Y  9   0.0111 (3 s.f.)
(iii) Let W be the total number of tins sold in 4 weeks. W  Po 16.8
Since   16.8  10, W N 16.8,16.8 approximately.
P 15  W  25   P 14.5  W  25.5  after continuity correction
 0.69576  0.696 (3 s.f.)

(iv)
sgexamguru.com
The mean number of tins sold per week might not be constant from one week to
another because of seasonal fluctuations such as sales and holidays.

Qn Solution
11 Correlation and Regression
(i) Using GC,
y  10.30667  0.99939x
y  10.307  0.999x

 y  Y 
(ii) 2
Using GC,  6.3689  6.37 (to 3 s.f.)
[In GC, define L3=10.30667 – 0.99939L1 and L4 = (L2 – L3)2 and use 1-var Stat
to find the summation]

 y  Y '
(iii) 2
 6.37

MJC/2016
MJC P2 SolutionsJC2 Prelim Examination Marking Scheme/H2 Math (9740/02)/Math Dept Page 9 of 10 331
y

100.8

1.44

1 10

(v) For x and y, r = – 0.9635

For x and lny, r = – 0.9999

(vi) Since the scatter diag


gram showss that the po
opulation decreases at a decreasin
ng rate
as the yeaars pass and the r valuee for the model ln y  c  dx is closser to –1 thaan that
of y  a  bx , ln y  c  dx is thee better mod
sgexamguru.com
del.
(vii) C, when x  20,ln y  1.89587  y  0.15018
Using GC 87 .

The popullation in 20tht year will be 150.

MJC/2016
MJC P2 SolutionsJC
C2 Prelim Exaamination Maarking Schemee/H2 Math (97
740/02)/Math Dept Page
P 10 of 10 332
  NATIONAL JUNIOR COLLEGE
SENIOR HIGH 2 PRELIMINARY EXAMINATION
Higher 2

MATHEMATICS 9740/01
Paper 1 25 August 2016

3 hours

Additional Materials: Answer Paper


List of Formulae (MF15)
Cover Sheet

READ THESE INSTRUCTIONS FIRST

Write your name, registration number, subject tutorial group, on all the work you hand in.
Write in dark blue or black pen on both sides of the paper.
You may use an HB pencil for diagrams or graphs.
Do not use staples, paper clips, glue or correction fluid.

Answer all the questions.

sgexamguru.com
Give non-exact numerical answers correct to 3 significant figures, or 1 decimal place in the case of angles in
degrees, unless a different level of accuracy is specified in the question.
You are expected to use an approved graphing calculator.
Unsupported answers from a graphing calculator are allowed unless a question specifically states otherwise.
Where unsupported answers from a graphing calculator are not allowed in a question, you are required to
present the mathematical steps using mathematical notations and not calculator commands.
You are reminded of the need for clear presentation in your answers.

At the end of the examination, fasten all your work securely together.
The number of marks is given in the brackets [ ] at the end of each question or part question.

This document consists of 6 printed pages.

National Junior College

[Turn_over
 
Microsoft Word - 2016 NJC SH2 H2 Mathematics Paper 1 Question Paper 333
2
 
1 A local café, Toast Rox, sells its coffee in three sizes (regular, medium and large). Toast Rox
customers get a 12.5% discount on their total bill if they buy at least 12 cups of coffee,
regardless of size. The number of cups of coffee bought by three particular customers and the
total amount they paid are shown in the following table.

Customer Regular Medium Large Amount paid


A 5 3 2 $20.90
B 3 4 1 $17.10
C 2 8 4 $28.00

Find the original price of each of the 3 sizes of coffee drink. [3]

2 (i) By using an algebraic method, solve the inequality

3 x 2  14
 2. [4]
 x  1 x  2 
(ii) Hence, showing all your working clearly, solve the inequality

3 x 2  14
 2. [2]
 x  1 x  2 

3
sgexamguru.com
Referred to the origin O, points A and B have position vectors a and b respectively. Point C
lies on the line segment AB, such that AC: CB = 2 : 1. Find, in terms of a and b, the position
vector of C. [1]
 
If the angle between a and b is 60 , show that the length of projection of OC on OA is

1
3
 a  b . [4]

4 (a) Two complex numbers z and w are such that

13
2 w  z  6i and wz  .
2

Find w and z, giving each answer in the form a + bi, where a and b are real numbers. [4]

(b) The points P and Q represent the fixed complex numbers p and q respectively. It is

given that 0 < arg p  arg q  , p  1, q  2, and arg q  2arg p.
2

In a single Argand diagram, sketch and label the points P, Q, and the points R and S
representing q* and q*  2 p 2 respectively, showing clearly any geometrical relationships.
Identify the shape of the quadrilateral OQSR, where O is the origin. [4]

[Turn_over
NJC P1 Questions 334
3
 
1
5 [It is given that volume of pyramid   base area  height .]
3

x x
2   2
 

A model of a house is made up of the following parts.


x
 The roof is modelled by a pyramid with a square base of sides x cm and height cm.
2
For each triangular side of the prism, the length of the perpendicular from the vertex to
x
the base is cm.
2
 The walls are modelled by rectangles with sides x cm and h cm as shown in the diagram.
 The base is a square with sides x cm.

sgexamguru.com
All the parts are joined together as shown in the diagram. The model is made of material of
negligible thickness. It is given that the volume of the model is a fixed value V cm3 and the
external surface area is at a minimum value, A cm2. Use differentiation to find
1
(i) x, in the form pV 3 , and
2
(ii) A, in the form qV 3 ,
leaving the values of p and q correct to 3 decimal places. [6]

6 A curve C has parametric equations

x  t 3  kt , y  3t 2  k  ,

where k is a positive constant and t is a real parameter.

(i) Sketch C, labelling clearly the coordinates of any points of intersection with the axes. [2]
dy
(ii) Find in terms of t and k. [2]
dx
k
(iii) Find, in terms of k, the exact equation of the tangent to C at the point where t   .
3
[3]
2 
(iv) Given that the tangent found in part (iii) intersects C again at the point  k , k  , find
3 
the value of k. [2]

[Turn_over
NJC P1 Questions 335
4
 

d3 y  d 2 y   dy 
7 (i) Given that y  ln  sec x  , show that  2  2   . [2]
dx 3  dx   dx 

(ii) Hence, by further differentiation, find the first two non-zero terms in the Maclaurin’s
series for y. [3]

1 2
(iii) The equation x  ln  sec x   cos 2 x has a positive root  close to zero. Use the
12
result in part (ii) and the first three terms of the Maclaurin series for cos 2x to obtain an
approximation to  , leaving your answer in surd form. [3]

8 The diagram below shows the curve with equation y  f ( x) . The curve passes through the
origin O, crosses the x-axis at the points A and B, and has a turning point at C. The
 b
coordinates of A, B and C are ( 4, 0), (4, 0) and  a,   respectively, where a and b are
 2
positive constants such that a > 1. The curve also has asymptotes x  2 and y  c , where
c  1.

sgexamguru.com

On separate diagrams, sketch the following curves, labelling clearly any asymptotes, axial
intercepts and turning points in terms of a,  b and c whenever necessary.

(a) y  f (1  2 x) [3]

(b) y 2  f ( x) [3]

1
(c) y [4]
f ( x)

[Turn_over
NJC P1 Questions 336
5
 

9 The gradient of a curve at the point (x, y) is given by the differential equation

1 dy x2
1  .
y dx y

(i) By using the substitution y  z  x , find the equation of the curve such that it has a
minimum point at (1,1). [6]

(ii) Sketch the curve, indicating clearly the axial intercept(s) and the minimum point. [2]

 5x2  2 x  7
10 (a) Using partial fractions, find  dx. [6]
 1  x   2 x  3 
2

(b) (i) Differentiate sin  e  x  with respect to x. [1]

e 2 x cos  e  x  dx in terms of n, where n > 0.


n
(ii) Obtain a formula for 
0
[3]

e 2 x cos  e  x  dx exactly.

(iii) Hence find  [2]

sgexamguru.com
0

11 (a) Prove by the method of mathematical induction that

2 3 n 1 5 1 1
 2 2   2     [5]
1 3 2 4
2 2
n (n  2) 16 4(n  1) 4(n  2) 2
2 2

4n  5
(b) (i) By expressing in partial fractions, show that
n(n  1)

N
 4n  5  1   b
  n(n  1)  5
n 1 
n 1  

a
( N  1)5N 1
,

for some real constants a and b to be determined exactly. [3]

(ii) State the sum to infinity of the series in part (b)(i). [1]

N 2
 4n  1  1  
(iii) Use your answer to part (b)(i) to find   n(n  1)  5
n2 
n 

in terms of N. [2]

[Turn_over
NJC P1 Questions 337
6
 

12 The curves C1 and C 2 have equations x 2  16( y  1)2  16 and x 2  16( y  1)2  16
respectively.

(i) Verify that the point (4,1) lies on both C1 and C2 . [1]

(ii) Sketch C1 and C 2 on the same diagram, labelling clearly any points of intersection with
the axes and the equations of any asymptotes. [4]

(iii) The region R is bounded by the two curves C1 , C 2 and the positive x-axis. Find the
numerical value of the volume of revolution formed when R is rotated completely about
the x-axis. [3]

S is the region bounded by C1 .

2
(iv) Using the substitution y  1  cos  , where      , evaluate 
0
1  ( y  1) 2 d y
exactly. [4]

(v) Hence find the exact area of S. [2]

– END OF PAPER –

sgexamguru.com

*
NJC P1 Questions 338
National Junior College Mathematics Department 2016

2016 SH2 H2 Mathematics Preliminary Examination Paper 1


Suggested Solutions
Qn No. Solution
1 Let $x be the price of a regular cup of coffee.
$y be the price of a medium cup of coffee.
$z be the price of a large cup of coffee.

5x + 3y +2z = 20.90 (1)

3x + 4y + z = 17.10 (2)

There is a 12.5% discount given to customer C since she has bought more than 12 cups.
Hence
100
2x + 8y + 4z =  28  32 (3)
87.5

Using GC, x = 1.80 , y = 2.30 , z = 2.50


Qn No. Solution
2 (i) 3x  14
2
2
 x  1 x  2 
3 x 2  14
  2  0, x  1, 2
 x  1 x  2 

sgexamguru.com

  

3 x 2  14  2 x 2  6 x  4
x 1 x  2
0

x 2  6 x  10
 0
 x  1 x  2 
Method 1: Completing the square
 x  3 2  1
    0, x  1, 2.
 x  1 x  2 
Since  x  3   1  0 for all values of x,
2

  x  1 x  2   0 and x  1, 2 .


 x  2 or x  1.
Method 2: Using both the discriminant and coefficient of x 2
For x 2  6 x  10, discriminant  62  4(1)(10)  4  0 and the coefficient of x 2 is
positive. Hence the graph of y  x 2  6 x  10 lies entirely above the x-axis, which
implies that x 2  6 x  10  0 for all values of x.
2 (ii) 3x 2  14
2
 x  1 x  2 
3   x   14
2

2
  x  1  x  2 

Page 1 of 16
Microsoft Word - 2016 NJC SH2 H2 Mathematics Paper 1 Solutions 339
National Junior College Mathematics Department 2016

Therefore  x  2 or  x  1
 x  2 or x  1
 x  2 or  1  x  1 or x  2
Qn No. Solution
3
B
1
C
2
A

O
 2b  a
By ratio theorem, OC  .
3
1
a  b = a b cos 60  a b
2
 
Length of projection of OC on OA

sgexamguru.com

 OA
OF  OC 
OA

1  2b  a   a

3 a
1 2b  a  a  a

3 a

1 2b  a  a
2


3 a

1  2 a b 
   a 
3 a 
1  2 a b cos 60 

   a 
3 a 
1 2 a b 
   a 
3 2 a 
 a  b
1
3

Page 2 of 16
NJC P1 Solutions 340
National Junior College Mathematics Department 2016

Qn No. Solution
4 (a) Method 1A
2w  z  6i  (1)
13 13
wz   (2)  w   (3)
2 2z
 13 
Substituting (3) into (1), 2    z  6i
 2z 
13
 z  6i
z
13  z 2  6iz
z 2  6iz  13  0
method 1: Using formula
6i  (6i) 2  4( 13) 6i  4
z   2  3i
2 2
method 2: Using completing square
z 2  6iz  (3i) 2  (3i) 2  13  0
( z  3i) 2  4
z  2  3i or  2  3i
13 3
When z = 2 – 3i, w   1 i
2(2  3i) 2

sgexamguru.com
13 3
When z = –2 – 3i, w   1  i
2( 2  3i) 2
3 3
 w  1  i, z  2  3i or w  1  i, z  2  3i
2 2
Method 1B
2w  z  6i  (1)
13 13
wz   (2)  z   (3)
2 2w
13
Substituting (3) into (1), 2w   6i
2w
4 w2  13  12iw
4 w2  12iw  13  0
 9
4  w2  3iw    9  13  0
 4
2
 3i 
4 w    4  0
 2
2
 3i  3i 3i
 w    1  w  1  or  1 
 2 2 2

3 13
When w  1  i, z   2  3i
2  3 
2 1  i 
 2 

Page 3 of 16
NJC P1 Solutions 341
National Junior College Mathematics Department 2016

3 13
When w  1  i, z   2  3i
2  3 
2  1  i 
 2 
3 3
 w  1  i, z  2  3i or w  1  i, z  2  3i
2 2
Qn No. Solution
4 (b)

sgexamguru.com
Let p = e i . Then q = 2ei(2 ) .
Let S represent
q *  2 p 2  2e i( 2 )  2e i(2 )  2 Re  e i(2 )   2 cos 2 ,
i.e. S represents the point whose real coordinate is twice that of Q, and imaginary
coordinate zero.
Qn No. Solution
5 1 1   1 
V  x2h  x2  x   x2  h  x 
3 2   6 
V 1
h 2  x
x 6

1  1 
S  4 xh  x 2  4   x  x
2  2 

 4 xh  1  2 x 2 
V 1
Substituting h   x:
x2 6

Page 4 of 16
NJC P1 Solutions 342
National Junior College Mathematics Department 2016

V 1 
S  4 x  2  x   1  2 x2
x 6 
 
4V  1 
    2  x2
x 3 

For minimum surface area:


dS 4V 1 
  2  2  2  x  0 .
dx x 3 
4V 1 
 2  2  x
x 2
3 
1 
2V    2  x 3
3 

6V  1  3 2 x 3 
6V
x3 
1 3 2
1

 6V  3
x   1.0460049 V 
1
3

1 3 2 
 1.046 V 3  (3d.p)
1

Min S  A

sgexamguru.com
 
4V 1
1.0460049 V 3   3
1

   2  1.0460049 V 3 

1
2

 1  2
   2  1.0460049   V 3 
4 2

 1.0460049  3  
 5.736110 V 3 
2

 5.736 V 3  (3d.p)
2

Remark:
1st or 2nd derivative test is not necessary as it is given in the question that the surface
area had a minimum value.

Page 5 of 16
NJC P1 Solutions 343
National Junior College Mathematics Department 2016

Qn No. Solution
6 (i)

sgexamguru.com
To find y-intercept, To find x-intercept,
let x = 0: let y = 0:
t 3  kt  0 3 t 2  k   0
t t 2  k   0 t k
t 0 or t k x0
y  3k y0   0, 0 
  0, 3k   0, 0 
6 (ii) dx
x  t 3  kt   3t 2  k
dt
y  3t 2  k  
dy
 6t
dt
dy
dy dt 6t
  2
dx dx 3t  k
dt

Page 6 of 16
NJC P1 Solutions 344
National Junior College Mathematics Department 2016

Qn No. Solution
6 (iii) k
When t   ,
3
dy 6t k
 2 is undefined since 3t 2  k  3    k  0 .
dx 3t  k 3

Hence, the tangent is a vertical line with equation


x  t 3  kt
 t t 2  k 
 k  k 
      k 
 3  3 
2 k
 k
3 3

6 (iv) 2 
The tangent passes through the point  k , k  , therefore
3 
2 k
x k
3 3
2 2 k
k k
3 3 3

k 1 

k
3
sgexamguru.com
  0

k
k 0 or 1 0
3
k
(NA since k  0) 1
3
k 3
Qn No. Solution
7 (i) y  ln  sec x 
dy 1
  sec x tan x   tan x
dx sec x
d2 y
 sec 2 x
dx 2
d3 y
 2 sec x  sec x tan x 
dx 3
 d 2 y   dy 
d3 y
 
 2 sec x  tan x   2  2    (shown)
2
dx3  dx   dx 
7 (ii) d4 y  d3 y   dy   d 2 y 
2

 2  3    2 2 
dx 4  dx   dx   dx 
When x  0 ,

Page 7 of 16
NJC P1 Solutions 345
National Junior College Mathematics Department 2016

dy
y  ln  sec 0   ln1  0  tan 0  0
dx
d2 y d3 y
2
 sec2 0  1  2(1)(0)  0
dx dx3
d4 y
 2  0  0   2 1  2
2
3
dx
Thus, Maclaurin series for y is
 x2   x4 
y  0 x  1    0 x 3  2  
 2!   4! 
1 2 1 4
 x  x
2 12
7 (iii) 1 2
x  ln  sec x   cos 2 x
12
Using the above results and the first three terms of Maclaurin Series of cos2x ,
1 2 1 2 1 4 1 1
x  x  x  1  2x   2x
2 4

12 2 12 2! 4!
7 2 1 4 2
x  x  1  2x2  x4
12 12 3
7 4 31 2
x  x 1  0
12 12
7 x  31x 2  12  0
4

7x  3 x 2  4   0
sgexamguru.com
2

3
x2  or x 2  4
7
3
x or  2
7
Since  is positive and close to zero,
3

7

Page 8 of 16
NJC P1 Solutions 346
National Junior College Mathematics Department 2016

Qn No. Solution
8 (a)

sgexamguru.com
5   3   1 a b 
Coordinates of A¢ , B¢ and C¢ are  , 0  ,   , 0  and  , 
2   2   2 2

Page 9 of 16
NJC P1 Solutions 347
National Junior College Mathematics Department 2016

Qn No. Solution
8 (b)

sgexamguru.com

8 (c)

 2
Coordinates of C are  a,   .
 b

Page 10 of 16
NJC P1 Solutions 348
National Junior College Mathematics Department 2016

Qn No. Solution
9 (i) dy dz
y zx  1
dx dx
1 dy x2
The 1  becomes
y dx y
1  dz  x2 dz
  1  1   1   z  x   x  2
z  x  dx  zx dx
dz
  z 1
dx

1
 dz   1 dx
 z 1
 ln z  1  x  c
 z  1  e x c  Ae x
 z  1   Ae x  Be x
 y  1  x  Be x
Since the solution curve passes through (1,1), then 1  11 Be  B  e 1 .
Hence the equation of curve is y  1 x  e x1.

9 (ii)

sgexamguru.com

Page 11 of 16
NJC P1 Solutions 349
National Junior College Mathematics Department 2016

Qn No. Solution
10 (a) 5x  2 x  7
2
A Bx  C
  2 , where
1  x   2 x  3 1  x 2 x  3
2

527
A  2,
23
5 x 2  2 x  7  2  2 x 2  3   Bx  C 1  x 

Comparing the coefficients of x2 and x0 , we have


4  B  5  B  1, and
6  C  7  C  1.

5x2  2 x  7 2 x 1
Hence we get   2
1  x   2 x  3 1  x 2 x  3
2

Therefore,
 5x2  2 x  7
 dx
 1  x   2 x  3
2

x 1
 
2
  2 dx
 1  x 2x  3
x 1
 dx  
2
  2 dx

sgexamguru.com
 1 x  2x  3
 dx   dx  
2 x 1
  2  2 dx
 1 x  2x  3  2x  3
1 1
 2 dx  
1 4x 1
  2 dx   dx
 1 x 4  2x  3 2
 
2
x 
2 3
2

1 1 1  x 
 2 ln 1  x  ln 2 x 2  3   tan 1  c
4 2 32  3
 2
1 1  2 
 2 ln 1  x  ln 2 x 2  3  tan 1  x   c
4 6  3 

Page 12 of 16
NJC P1 Solutions 350
National Junior College Mathematics Department 2016

Qn No. Solution
sin  e x    e x cos  e x 
10 (b) (i) d
dx  

10 (b) (ii) e2 x cos  e x  dx


n
 0


  e x e  x cos  e  x  dx 
n

   
 e  x  sin  e x      sin  e x   e x  dx
n n

 0 0
 e  n   sin  e   e   sin  e     e x sin  e  x  dx
n
n 0 0
 0

 e  n sin  e  n   sin1    cos  e x  


n

 e  n sin  e  n   sin1    cos  e n   cos  e0  

 e  n sin  e  n   cos  e n   sin1  cos1

e 2 x cos  e  x  dx


10 (b)
(iii) 0

 lim  e 2 x cos  e  x  dx
n

n  0

 lim  e  n sin  e  n   cos  e  n   sin1  cos1


n 

 sin1  cos1  1

Qn No.
sgexamguru.com Solution
11 (a) Let Pn be the statement
2 3 n 1 5 1 1
 2   2   
1 3
2 2
2 4 2
n ( n  2) 2
16 4( n  1) 2
4( n  2) 2
for all n    .

When n = 1,
2 2
LHS  2 2 
1 3 9
5 1 1
RHS   
16 4(1  1) 4(1  2)2
2

5 1 1 2

    LHS
16 16 36 9
Thus, P1 is true.

Assume Pk is true for some k  , i.e.


2 3 k 1 5 1 1
 2 2   2    
1 3
2 2
2 4 k ( k  2) 2
16 4( k  1) 2
4( k  2) 2
Consider Pk1 : To show
2 3 k2 5 1 1
 2      
1 3
2 2
2 4 2
( k  1) ( k  3)
2 2
16 4( k  2) 2
4( k  3) 2

Page 13 of 16
NJC P1 Solutions 351
National Junior College Mathematics Department 2016

2 3 k 1 k 2
LHS of Pk 1   2 2   2 
1 3 2 4
2 2
k (k  2) (k  1) 2 (k  3) 2
2

5 1 1 k 2
   
16 4(k  1) 4(k  2) (k  1) 2 (k  3) 2
2 2

5 1 k 2 1
   
16 4(k  2) (k  1) (k  3) 4(k  1) 2
2 2 2

5 1 4(k  2)  (k  3) 2
  
16 4( k  2) 2 4(k  1) 2 (k  3) 2
5 1 4k  8  k 2  6k  9
  
16 4(k  2) 2 4(k  1) 2 (k  3) 2
5 1  k 2  2k  1
  
16 4( k  2) 2 4(k  1) 2 (k  3) 2
5 1 (k  1) 2
  
16 4( k  2) 2 4(k  1) 2 (k  3) 2
5 1 1
  
16 4( k  2) 4(k  3) 2
2

 RHS of Pk 1
Pk is true  Pk1 is true .
Since P1 is true, and Pk is true  Pk1 is true, by mathematical induction, Pn is true for

sgexamguru.com
all n    .
Qn No. Solution
11 (b) (i) 4n  5 5 1
 
n(n  1) n n  1
N
4n  5  1  N  5 1  1 
  n 1     
n 1 n( n  1)  5

 n 1  n n  1   5n 1 

N
 1 1 
  n  n 1 
n 1  n5 (n  1)5 
1 1

1  5 2  52
1

1 1
 
2  5 3  53
2

 
1 1
 N 1

( N  1)5 N 5N
1 1
 
N5 N
( N  1)5 N 1
1 1
  (shown)
5 ( N  1)5 N 1
1
a  , b  1
5

Page 14 of 16
NJC P1 Solutions 352
National Junior College Mathematics Department 2016

11 (b) 
4n  5 1  1
 n(n  1)  5
1  1
(ii) n 1 
 lim     
 N   5 ( N  1)5  5
N 1
n 1

11 (b) Method 1: Substitution of Indices


(iii) N 2
 4n  1  1   n 1 N 2  4(n  1)  1  1  
   n    
n  2  n( n  1)  5  
 n1  
n 1 2  ( n  1)(( n  1)  1)  5 
N 3
 4n  5  1  
   n1  
n 1  n( n  1)  5 
1 1
 
5 ( N  2)5N 2

Method 2: Listing of Terms


N 2
 4n  1  1  
   n 
n  2  n( n  1)  5  

9  1  13  1  4N  7  1 
  2   3     
2 1  5  3  2  5  ( N  2)( N  3)  5N 2 
4 1  5  1  4  2  5  1  4( N  3)  5  1 
  2   3     
1 2  5  23  5  ( N  3)( N  2)  5N  2 
N 3
 4n  5  1  
   n 1  
n 1  n( n  1)  5 
1
  sgexamguru.com
1
5 ( N  2)5N 2
Qn No. Solution
12 (i) By substituting (4,1) into the equations of C1 and C2 ,
C1 : LHS  (4) 2  16(1  1) 2  16  RHS
C2 : LHS  (4) 2  16(1  1) 2  16  RHS (justified)

12 (ii)

Page 15 of 16
NJC P1 Solutions 353
National Junior College Mathematics Department 2016

12 (iii) Volume of revolution of R about x-axis


2 2
 16  x 2 
4 4 2  x 2  16 
   1   dx   4 1   dx
0
 16   16 
 2.17 units3 (to 3 s.f.)
12 (iv) 2
 0
1  ( x  1) 2 dx


0

  1  cos 2     sin   d
 sin   d

 2
0

1 
1  cos 2  d
2 0


1 sin 2 
  
2 2  0


2
12 (v) Area of S  2 x dy
2

0
2
 2 16  16( y  1) 2 dy
0
2
 8 1  ( y  1) 2 dy
sgexamguru.com
0

 
 8 
2
 4 units 2

Page 16 of 16
NJC P1 Solutions 354
  NATIONAL JUNIOR COLLEGE
   
SENIOR HIGH 2 PRELIMINARY EXAMINATION
Higher 2

MATHEMATICS 9740/02
Paper 2 14 September 2016

3 hours

Additional Materials: Answer Paper


List of Formulae (MF15)
Cover Sheet

READ THESE INSTRUCTIONS FIRST

Write your name, registration number, subject tutorial group, on all the work you hand in.
Write in dark blue or black pen on both sides of the paper.
You may use an HB pencil for diagrams or graphs.
Do not use staples, paper clips, glue or correction fluid.

Answer all the questions.

sgexamguru.com
Give non-exact numerical answers correct to 3 significant figures, or 1 decimal place in the case of angles in
degrees, unless a different level of accuracy is specified in the question.
You are expected to use an approved graphing calculator.
Unsupported answers from a graphing calculator are allowed unless a question specifically states otherwise.
Where unsupported answers from a graphing calculator are not allowed in a question, you are required to
present the mathematical steps using mathematical notations and not calculator commands.
You are reminded of the need for clear presentation in your answers.

At the end of the examination, fasten all your work securely together.
The number of marks is given in the brackets [ ] at the end of each question or part question.

This document consists of 6 printed pages.

National Junior College

Microsoft Word - 2016 NJC SH2 H2 Mathematics Paper 2 Question Paper 355
2
 
Section A: Pure Mathematics [40 marks]

1 (a) Kenny took a loan of $9600 from a friend, and arranged to pay his loan fully in a period
of exactly 48 months. To fulfil this arrangement, he paid $a on the last day of the first
month, and on the last day of each subsequent month, he paid $d more than in the
previous month. However, due to financial difficulties, Kenny stopped his payments
after his 40th payment, and as a result he still had exactly $2400 left unpaid.

In which month did Kenny first pay at least $130 on the last day of that month? [5]

(b) (i) Explain why the series 1  e 2 x  e 4 x   converges for any positive real number x,
and express the sum to infinity in terms of x. [2]

(ii) Given that x = 10, find the least value of n such that S  Sn  S (10100 ), where S
and Sn represent the sum to infinity and the sum of the first n terms of the series
respectively. [3]

2 The functions f and g are defined by

f : x  x 2  4 x  3 , for x  a and

g : x  tan 1 (2 x  1) , for x  2 ,

sgexamguru.com
  where a is a constant.

(a) If a  2 , solve the equation f ( x )  x exactly. [2]

(b) If a  3 ,

(i) give a reason why f has no inverse. [2]

(ii) Prove that the composite function gf exists and state the rule, domain and exact
range of the composite function. [6]

3 The point A has coordinates (2 q, 0, 2), where q is a constant, and the planes p1 , p2 have
equations x  y  4 and 3 x  2 y  5 z  7 respectively.

(i) Find the coordinates of the foot of perpendicular from A to p1. Express your answer in
terms of q. [3]
(ii) The point B is the mirror image of A in p1. If B lies in p2 , find the value of q. [4]
(iii) p1 and p2 intersect in a line l. Find a vector equation of l. [1]

Another plane p3 has equation  x  z   , where  and  are constants.

(iv) Given that the three planes have no point in common, what can be said about the values
of  and  ? [2]

[Turn_over
NJC P2 Questions 356
3
 
4 The complex number z satisfies the relation z  3  5.

(i) Illustrate this relation in an Argand diagram. [2]

(ii) Find the largest possible value of arg( z  3  3i). [3]

It is further given that z also satisfies the relation z  4i  z  6  4i .  

(iii) Illustrate this relation in the same diagram as your sketch in part (i). Find the possible
values of z exactly. [5]

Section B: Statistics [60 marks]

5 A school comprises a large number of students. A sample comprising 2% of the student


population is to be selected to take part in a survey on their opinions about the school
facilities.

(a) Describe briefly how this sample can be obtained via systematic sampling. [2]

(b) Give one advantage and one disadvantage of quota sampling in this context. [2]

6
sgexamguru.com
The continuous random variable X has the distribution N( ,  2 ). It is known that
P  X  17.7   0.15 and P  X  21.9   0.2 . Calculate the values of  and  . [4]

7 A group of 15 student councillors comprises 6 from the House Committee, 5 from the Liaison
Committee and 4 from the Welfare Committee. Two particular student councillors, Louis and
Lionel, are from the House Committee and the Liaison Committee respectively.

The group stand in a circle to have a meeting. Find the number of possible arrangements if

(i) no two student councillors from the House Committee stand next to each other. [2]

(ii) student councillors from the same committee must stand next to one another and Louis
and Lionel must stand next to each other. [2]

The group is to form a Task Force of 10 student councillors to organise a school activity. Find
the number of possible ways the Task Force may be formed if the Task Force must include at
least 1 student councillor from each of the 3 committees. [3]

[Turn_over
NJC P2 Questions 357
4
 
8 The table below shows the ages of teak trees, x years, with trunk diameters, y inches. It can be
assumed that the diameters of teak trees depend on their ages.
Age x (years) 11 15 28 45 52 57 75 81 88 97
Diameter y (inches) 7.5 11.5 16 19 20.5 21 21.5 21.9 22.2 22.22

(i) Draw a scatter diagram for these values, labelling the axes.             [2]

(ii) It is desired to predict the diameters of very old trees (of over hundred years old).
Explain why, in this context, neither a linear nor a quadratic model is likely to be
appropriate. [2]

b
(iii) Fit a model of the form y  a  to the data, and calculate the least squares estimates
x
of a and b. Find the product moment correlation coefficient for this model. Use the
equation that you have obtained to estimate the diameter of a 40 year-old teak tree, and
comment on the reliability of your answer. [4]

9 It has been estimated that only 8% of the world’s population has blue eyes. A group of 60
people are randomly selected from all over the world. The number of people in this group
who have blue eyes is the random variable Y.

(i) State, in the context of this question, one assumption needed to model Y by a binomial

sgexamguru.com
distribution.

Assume now that Y indeed follows a binomial distribution.


[1]

(ii) Find the probability that at least 5 but less than 21 people in the group will have blue
eyes. [2]

(iii) Use a suitable approximation to find the probability that more than 9 people in the
group have blue eyes. You should state the parameters of the distribution you have used.
[3]

[Turn_over
NJC P2 Questions 358
5
 
10 (i) Suppose a fair die is tossed twice. Calculate the probabilities that

(a) the sum of the scores of the two tosses is at least 8, and [1]

(b) the absolute difference between the scores of the two tosses is at least 4. [1]

In one round of a game, a player is to draw a ball, without replacement, from a box that
contains 3 red balls and 4 white balls. If a red ball is drawn, the player will add the scores
obtained from tossing a fair die twice. If a white ball is drawn, the player will take the
absolute difference of the scores obtained from tossing a die twice.

The game ends if the sum of the scores is at least 8 or the absolute difference of the scores is
at least 4. Else, the player will proceed to the second round of the game where the process of
picking a ball from the box and tossing the die twice repeats.

(ii) Find the probability that the game ends at the first round. [2]

(iii) Suppose the game ends at the first round. Find the probability that a red ball is drawn.[2]

(iv) Find the probability that there are a total of 3 rounds of game played and exactly 2 white
balls are selected. [3]

11 An accountant believes that the figures provided by a particular company for the amount of

sgexamguru.com
loans borrowed by its clients, $x, are too low. He carries out an online survey for clients of
this company. The responses from a random sample of 20 clients are summarised by

 x  21350 ,  ( x  x ) 2
 345900 .

(i) Calculate unbiased estimates of the population mean and variance of the amount of
loans borrowed by each client, correct to 1 decimal place. [2]

The company claims that its clients will borrow $1000 on average.

(ii) Stating a necessary assumption, carry out a test at the 5% level of significance to
determine whether the company has understated the mean amount of loans received by
its clients. [5]

(iii) Explain, in the context of the question, the meaning of ‘at the 5% level of significance’.
[1]

The responses from another random sample of n clients are collected. The sample mean value
for this sample is the same as the sample mean value for the previously collected sample.

(iv) Given that the standard deviation of X is 250, and that the assumption you have made in
part (ii) holds, calculate the range of values of n for which the null hypothesis would not
be rejected at the 5% level of significance. [3]

[Turn_over
NJC P2 Questions 359
6
 
12 Cars join an immigration checkpoint queue in a 1-hour period, such that no two cars join the
queue at the same instant in time.

(i) State, in the context of this question, an assumption needed for the number of cars
joining an immigration checkpoint queue in a 1-hour period to be well modelled by a
Poisson distribution. [1]

Assume now that the number of cars joining an immigration checkpoint queue in a 1-hour
period is a random variable with the distribution Po(23). It is further given that the number of
cars leaving the same immigration checkpoint queue in a 1-hour period is a random variable
with the distribution Po(27).

(ii) It is given that in a period of n minutes, the probability that at least one car leaves the
queue exceeds 0.9. Write down an inequality in n. Hence find the least integer value of
n. [4]

(iii) At 0900 on a certain morning there are 19 cars in the queue. Use appropriate
approximations to find the probability that by 1100 there are at most 12 cars in the
queue, stating the parameters of any distributions that you use. (You may assume that
the queue does not become empty during this period.) [5]

(iv) Explain why a Poisson model for the number of cars joining an immigration checkpoint
queue would probably not be valid if applied to a time period of several hours. [1]

sgexamguru.com – END OF PAPER –

*
NJC P2 Questions 360
National Junior College Mathematics Department 2016

2016 SH2 H2 Mathematics Preliminary Examination Paper 2


Suggested Solutions
Qn No. Solution
1 (a) Since Kenny would have paid up his loan in full exactly in the 48th month,
48
S 48   2a  (48  1)d 
2
9600  24(2a  47 d )
400  2a  47 d  (1)

Since Kenny had an outstanding payment of $2400 after the 40th month, total amount
paid by the 40th month = $9600 – 2400 = $7200. Therefore,
40
S40   2a  (40  1)d 
2
7200  20(2a  39d )
360  2a  39d  (2)
Solving (1) & (2), a = 82.5, d = 5.

On the last day of the nth month (for 1 ≤ n ≤ 40), the amount paid by Kenny = $82.5 +
(n – 1)(5).

sgexamguru.com

Therefore,
amount paid on last day of 10th month = $127.5 < $130,
amount paid on last day of 11th month = $132.5 > $130.

Therefore Kenny first paid at least $130 on the last day of the 11th month.

Page 1 of 14
Microsoft Word - 2016 NJC H2 Mathematics Paper 2 Solutions 361
National Junior College Mathematics Department 2016

Qn No. Solution
2 x 4 x
1 (b) (i) Common ratio of 1  e e   is r  e2 x .

For x > 0, 0  e2 x  1 (see above sketch).


Therefore, the geometric series converges (since e2 x is the common ratio).

OR

As n  , e2nx  0 (for x  0). Therefore


1  e2 nx 1
Sn  2 x
 , i.e. the series is convergent.
1 e 1  e2 x

1 (b) (ii)
sgexamguru.com
Sum to infinity 
1
1  e 2 x
1  e20 n 1
For x = 10, Sn  20
,S .
1 e 1  e20
S
S  Sn  100
10
S
S n  S  100
10
 1 
S n  S 1  100 
 10 
1  e 20 n 1  1 
20
> 20 
1  100 
1 e 1  e  10 
1
1  e 20 n  1  100
10
1
e 20 n   100
10
1
e 20 n  100
10
1
20n  ln 100  100 ln10
10
n  5ln10  11.513
Therefore, least value of n = 12

Page 2 of 14
NJC P2 Solutions 362
National Junior College Mathematics Department 2016

Qn No. Solution
2(a) f ( x)  x
x2  4 x  3  x
x2  5x  3  0
  5   (5) 2  4(1)(3)
x
2
5  13

2
5  13
Since x  2 , we have x 
2
2(b)(i) y

x
1 3

 2, 1

sgexamguru.com
There exist a horizontal line y  k where k  (1,0] cuts the graph of y  f ( x) twice.
Thus f is not one-one and hence its inverse does not exist.

OR

f (3)  f (1)  0 but 3  1.


Thus f is not one-one and its inverse does not exist.
2(b) (ii) From the graph, R f  [1, ).
Moreover, Dg  (2, ).
Since R f  Dg , gf exists.

gf ( x )  g  x 2  4 x  3
 tan 1  2 x 2  8 x  7 
Note that D gf  D f . Thus,
gf : x  tan 1  2 x 2  8 x  7  , x  3.
  
R gf    , 
 4 2

Page 3 of 14
NJC P2 Solutions 363
National Junior College Mathematics Department 2016

Qn No. Solution
3 (i) Let the foot of perpendicular be N.
Equation of the line that passes through A and perpendicular to p1 is
 2q  1
   
lA : r   0     1  ,    .
 2 0
   
   1
2q
 
Since N lies on l A , ON   0     1  for some   .
 2 0
   
 2q   1   1 
     
 0     1     1   4  2q  2  4
 2   0   0 
   
  2q
 2q   2  q   2  q 
      
 ON   0    2  q    2  q 
 2  0   2 
     
Hence, N is the point  2  q, 2  q, 2  .

3 (ii) Let b be the position vector of point B.

sgexamguru.com
 2q  2 q
By Ratio Theorem, 0  b  2  2  q 
 
   
 2  0 
   
 4 
 
b   4  2q 
 2 
 

 4   3
Since B lies in p2 ,  4  2q    2   7
   
 2   5 
   
12  8  4q  10  7
3
q  0.75 or
4
3 (iii)  1 5
   
Using GC, l : r   5     5  ,   .
0 1
   
3 (iv)    5 
    1
0  
    5  0     .
1  1  5
   

Page 4 of 14
NJC P2 Solutions 364
National Junior College Mathematics Department 2016

 1   
    1
 5  0       5 .
 0  1
   
Qn No. Solution
4 (i),
(iii) (1st
part)

4 (ii) sgexamguru.com
CD  (3  (3)) 2  (0  3) 2
 45

Largest value of arg( z  3  3i)


 
 AC  1  BC 
 sin 1    tan  
 CD   BD 
 5  1  1 
 sin 1    tan  2 
 45   
 0.37742
 0.377 rad (to 3 s.f.)

Page 5 of 14
NJC P2 Solutions 365
National Junior College Mathematics Department 2016

4 (iii) Method 1: Applying relationship between gradient of a line and the angle it makes
(2nd with the positive horizontal axis
part) 1 3
Gradient of perpendicular bisector   4 
 3  4
1  3 
Angle that perpendicular bisector makes with positive real axis, θ  tan  
4
Hence,
GJ 3
 sin    GJ  3
5 5
CJ 4
 cos    CJ  4
5 5
So G and H represent the complex numbers
z = (3 + 4) + (0 + 3)i = 7 + 3i (corresponding to G), and
z = (3 – 4) + (0 – 3)i = –1 – 3i (corresponding to H) resp.
Qn No. Solution
nd
4 (iii) (2 Method 2: Using Similar Triangles
part)
GCE  90  OCE  90  GCJ  180
 OCE  90  GCJ

Also, OEC  90  OCE

sgexamguru.com
 90   90  GCJ 
 GCJ

Furthermore, COE  GJC  90.

Therefore, COE ~ GJC. Hence,


CO GJ 3 GJ
    GJ  3 , and
CE GC 5 5
OE CJ 4 CJ
    CJ  4
CE GC 5 5

So coordinates of G are (3 + 4, 0 + 3), i.e. (7, 3) and similarly, coordinates of H are


(3 – 4, 0 – 3), i.e. (–1, –3). Therefore, possible values of z are 7 + 3i and –1 –3i.

Method 3: Using Cartesian Equations


Equation of circle:  x  3   y 2  52
2

Equation of perpendicular bisector:


1 3
y  0   8 ( x  3)  y  ( x  3)
 6  4
2
3 
Substituting,  x  3   ( x  3)   52
2

4 
 3  2

1      x  3  5
2 2

 4 

Page 6 of 14
NJC P2 Solutions 366
National Junior College Mathematics Department 2016

25
 x  3  25
2

16
 x  3  16
2

x  3  4
x  7 or  1.
3
When x  7, y  (7  3)  3
4
3
When x   1, y  ( 1  3)  3
4
Therefore, possible values of z are 7 + 3i and –1 –3i.
Qn No. Solution
5 (a) Assign a number from 1 to N to each of the students, where N represents the student
population size OR obtain a list of the students from the administration office in order
of their identification numbers or registration numbers.

1
Next, determine the sampling interval size k   50.
0.02
Randomly select any student from the list, say the 1st student. Select every 50th student
thereafter (i.e. 51st, 101th,…) until the required sample is obtained.

5 (b) Advantages:

 Representativeness of Sample
Quota sampling allows the survey to capture the responses that represent various

sgexamguru.com
groups of students (e.g. different PM classes, or 1st CCAs); this may be preferred as
certain homeroom or sports facilities may not be in as good a condition as others, and
the representation of each group will ensure that the results will not be biased towards
those who are often using these less functional facilities or towards those who are often
using the more functional facilities.

 Efficiency of Collecting the Sample


Quota sampling may be more efficient as systematic sampling in this case requires the
surveyor to identify the selected respondents and to contact them, which can be time
consuming (e.g. student selected may be on MC on day of survey, selected students do
not respond to online survey etc).

Disadvantages:

 Non-randomness/Selection Bias
Quota sampling is non-random and may contain selection bias, where the surveyor
chooses people who may appear friendlier or choose students in the canteen only at a
selected time period. This results in certain students having no chance of being selected
at all, which may affect the validity of the survey results.

 Non-representativeness of Sample
Quota sampling may result in a group (e.g. one entire cohort, or people coming later to
the canteen etc.) being excluded entirely from the selection, which may result in the
data collected being an inaccurate representation of the entire school population.

Page 7 of 14
NJC P2 Solutions 367
National Junior College Mathematics Department 2016

Qn No. Solution
2
6 X~ N(,  )
P  X  21.9   0.2
 17.7  μ  P  X  21.9   0.8
P  X  17.7   0.15 P  Z    0.15
 σ   21.9  μ 
17.7  μ PZ    0.8
 1.03643 --- (1)  σ 
σ 21.9  μ
 0.841621 --- (2)
σ

Solving simultaneous equations (1) and (2):


From (1): μ  1.03643σ  17.7
From (2): μ  0.841621σ  21.9
Using GC, μ  20.0  3s.f  and σ  2.24  3s.f 
Qn No. Solution
7 (i)  9!   9 
No. of ways      6!  2 438 553 600
 9  6
7 (ii) No. of ways
 3! 
    5! 4! 4! (1)
3

sgexamguru.com
 138 240

OR

Case 1
No. of ways W HHH
W H
  5! 4! 4! W H
 138 240 / 2 W H
L L L LL
Case 2 HH
No. of ways LL H
L H
  5! 4! 4! L H
 138 240 / 2 L H
W WW W

7 (last Case 1 : None from Liaison Committee


part) No of ways  1

Case 2 : None from Welfare Committee


 6  5   6  5 
No of ways          11
 6  4   5  5 

or
 11
No of ways     11
10 

Page 8 of 14
NJC P2 Solutions 368
National Junior College Mathematics Department 2016

Total Number of ways to select at least 3 men and 3 women


 15 
    1  11
10 
 2991
Qn No. Solution
8 (i)
y

22.222 ××
× ×
×
×
×

×
×
7.5 ×
x
O 11 97

8 (ii) Unsuitability of a Linear Model

A linear model predicts the average diameter will keep increasing indefinitely without

sgexamguru.com
any limit. Therefore a linear model is not appropriate.

Unsuitability of a Quadratic Model

A quadratic model predicts that the average diameter will eventually attain a maximum
value, and thereafter decrease as the age increases, till it eventually takes on negative
values. This is not possible, and therefore a quadratic model is not appropriate.

8 (iii) Using the suggested model, the least square regression line is
185.346 185
y  23.886   23.9  (to 3 s.f.)
x x

r-value = –0.994 (to 3 s.f.)

185.346
When x = 40, y  23.886   19.3 (to 3 s.f.)
40

Since x = 40 is within the range of values of x , [11, 97] and the product moment
correlation coefficient, –0.994, has an absolute value that is close to 1, suggesting a
strong linear correlation between the variables y and 1/x , therefore the estimate is
reliable.

Page 9 of 14
NJC P2 Solutions 369
National Junior College Mathematics Department 2016

Qn No. Solution
9 (i) The eye colour of a person is independent of that of another person.

OR

Every person is equally likely to have blue eyes.

9 (ii) Y ~ B  60, 0.08 


Then P(5  Y  21)
 P(Y  20)  P(Y  4)
 0.530 (to 3 s.f.)
9 (iii) Since n = 60 (> 50) is large and np = 4.8 < 5, Y ~ Po(4.8) approximately.

P(Y  9)  1  P(Y  9)  0.0251 (to 3 s.f.)


Qn No. Solution
10 (i) (a) Let E and F denote the event that the sum of scores is at least 8 and the event that the
absolute difference between the scores is at most 4 respectively.

Sum of Scores

1st die \ 2nd die 1 2 3 4 5 6

sgexamguru.com
1 2 3 4 5 6 7
2 3 4 5 6 7 8
3 4 5 6 7 8 9
4 5 6 7 8 9 10
5 6 7 8 9 10 11
6 7 8 9 10 11 12
1 1 5
P  E     15 
6 6 12

10 (i) (b) Absolute Difference between Scores

1st die \ 2nd die 1 2 3 4 5 6


1 0 1 2 3 4 5
2 1 0 1 2 3 4
3 2 1 0 1 2 3
4 3 2 1 0 1 2
5 4 3 2 1 0 1
6 5 4 3 2 1 0

1 1 1
PF   6 
6 6 6

Page 10 of 14
NJC P2 Solutions 370
National Junior College Mathematics Department 2016

Qn No. Solution
10 (ii)
5
12 E

3 Red
7 7
E
12

1 F
4 6
7
White
5
F
6
P  game ends at 1st round 
 P  red and game ends at 1st round 
 P  white and game ends at 1st round 
 3   5   4  1 
        
 7   12   7  6 

sgexamguru.com
23

84
10 (iii) P(red ball selected | game ends at the first round)
P(red ball selected  game ends at the first round)
=
P(game ends at the first round)
 3  5 
   15
    
7 12
23 23
84
10 (iv) P(total of 3 rounds of game &
exactly 2 white balls selected)
 P(WWR)  P(WRW)  P(RWW)
 4  5  3  5  3  5   4  5  3  7  3  1 
=               
 7  6  6  6  5  12   7  6  6  12  5  6 
 3  7   4  5  3  1 
        
 7  12   6  6  5  6 
25 1 1
  
504 72 72
13

168

Page 11 of 14
NJC P2 Solutions 371
National Junior College Mathematics Department 2016

Qn No. Solution

Unbiased estimate of  , x   
11 (i) x 21350
 1067.5
n 20
1
Unbiased estimate of  2 , s 2  (345900)
19
 18205.3 (to 1 d.p.)
11 (ii) H0:   1000
H1:   1000

Assume that the amounts of loans borrowed by the bank’s clients follow a normal
distribution.

OR

Assume that the amount of loans borrowed by each client follows a normal
distribution.

Level of Significance: 5% (upper-tailed)

X  1000
Under H0, T  ~ t (19 )
S
20

sgexamguru.com
x  1000
Test Statistic: t 
s
20

Method 1: Using critical region and observed test statistic, t

Critical region: t  2.015

1067.5  1000
t
s / 20
 2.237 s  18205.3 
0.05
Since t  2.237  2.015 , we reject H0.

Method 2: Using p-value

p-value = 0.0187 2.015


0
Since p-value = 0.0187 < 0.05, we reject H0. 2.237

We conclude that there is sufficient evidence at 5% level of significance that the


company has understated the mean amount of loans borrowed by its clients.

11 (iii) The meaning of ‘at the 5% significance level’ is that there is a probability of 0.05 that it
was wrongly concluded that the company had understated the mean amount of loans
borrowed by its clients.

Page 12 of 14
NJC P2 Solutions 372
National Junior College Mathematics Department 2016

Qn No. Solution
11 (iv) x  1000
Test Statistic: z 
250
n

To not reject H 0 , z  1.6449


1067.5  1000
 1.6449
250
n
n  6.092
n  37.1

Since n   , n  37.
Qn No. Solution
12 (i) Any one of the following:

[Constant mean rate]


The mean number of cars joining the immigration checkpoint queue for any subinterval
of the same length of time within 1 hour (e.g. minute) is constant.

OR

[Independence of occurrence of event]


Cars join the immigration queue independently of one another, throughout the entire

12 (ii)
hour.
sgexamguru.com
Let X denote the random variable for the number of cars leaving an immigration
 27 
checkpoint queue in a period of n minutes. X Po  n  i.e. X Po  0.45n 
 60 
P  X  1  0.9
1  P  X  0   0.9
P  X  0   0.1
e 0.45 n  0.45n 
0

 0.1
0!
e 0.45 n  0.1
0.45n  ln  0.1
ln  0.1
n  5.11
0.45
Therefore, least integer n is 6.

Page 13 of 14
NJC P2 Solutions 373
National Junior College Mathematics Department 2016

Qn No. Solution
12 (iii) Let J and L denote the random variables for the number of cars joining and leaving an
immigration checkpoint queue respectively in a 2-hour period. Then

J~Po(46) and L~Po(54)

Since 46 > 10, J ~ N(46, 46) approximately.


Since 54 > 10, L ~ N(54,54) approximately.

Let W denote the number of people in the queue at 1100. Then W  19  J  L.

E(W )  E(19  J  L)
 19  E( J )  E( L)  19  46  54  11, and

Var(W )  Var(19  J  L)
 Var( J )  Var( L)  46  54  100

Therefore, W  19  J  L ~ N(11,100) approximately.

P 19  J  L  12   P 19  J  L  12.5  (by c.c.)


 0.560 (to 3.s.f)

sgexamguru.com
Alternatively, J  L ~ N(8,100) approximately.

P  J  L  7   P  J  L  6.5
 0.560 (to 3.s.f)

Or equivalently, L  J ~ N(8,100) approximately.

P  L  J  7   P  L  J  6.5
 0.560 (to 3.s.f)

12 (iv) The mean number of cars joining the immigration checkpoint queue every hour may not
be constant due to peak periods as there may be more cars heading to or returning from
work.

Page 14 of 14
NJC P2 Solutions 374
  NANYANG JUNIOR COLLEGE
  JC2 PRELIMINARY EXAMINATION

Higher 2

MATHEMATICS 9740/01

Paper 1 15th September 2016

3 Hours

Additional Materials: Cover Sheet


Answer Papers
List of Formulae (MF15)

READ THESE INSTRUCTIONS FIRST

Write your name and class on all the work you hand in.
Write in dark blue or black pen on both sides of the paper.
You may use a soft pencil for any diagrams or graphs.
Do not use staples, paper clips, highlighters, glue or correction fluid.

sgexamguru.com
Answer all the questions.
Give non-exact numerical answers correct to 3 significant figures, or 1 decimal place in the case of
angles in degrees, unless a different level of accuracy is specified in the question.
You are expected to use a graphic calculator.
Unsupported answers from a graphic calculator are allowed unless a question specifically states
otherwise.
Where unsupported answers from a graphic calculator are not allowed in a question, you are
required to present the mathematical steps using mathematical notations and not calculator
commands.
You are reminded of the need for clear presentation in your answers.

At the end of the examination, fasten all your work securely together.
The number of marks is given in brackets [ ] at the end of each question or part question.

This document consists of 6 printed pages.

Microsoft Word - NYJC 2016 H2 Mathematics P1 375


4n n 2
1 The nth term of a sequence is given by un  , for n  1.
 n  1 n  2 
The sum of the first n terms is denoted by S n . Use the method of mathematical induction
n 1 
 n 1   4 2
to show that S n       for all positive integers n. [5]
 n  2  3  3 

2
 17 x 2  23x  12
2 Using partial fractions, find  dx , leaving your answer in exact form.
2 (3x  4)( x  4)
2

[6]

3 A curve C has parametric equations


1 1 1 
              x   sin t cos t  t  , y  t  sin 2t , for  t 0.
2 2 4 2
The tangent to the curve at the point P has gradient 1. Find the equation of the normal at P.
The region bounded by this normal, the curve C and the x-axis is rotated through 2 radians
about the x-axis. Find, to 5 decimal places, the volume of the solid obtained.

sgexamguru.com
What can be said about the tangents to the curve as t approaches 0? [7]
 

4 Referred to the origin, the points A and B have position vectors a and b respectively. A
point C is such that OACB forms a parallelogram. Given that M is the mid-point of AC, find
the position vector of point N if M lies on ON produced such that OM : ON is in the ratio
3:2. Hence show that A, B and N are collinear. [4]
Point P is on AB is such that MP is perpendicular to AB. Given that angle AOB is 60o ,
a  2 and b  3 , find the position vector of P in terms of a and b. [4]

2016 H2 Math Prelim 9740/01 2


 
NYJC P1 Questions 376
5 y

r
O x

A particle P moves along the curve with equation x 2  y 2  r 2 , where x  0, y  0, and r is a

 y dm
constant. By letting m  tan  sin 1  , find an expression for  in terms of y and r.
 r dy
Given that the rate of change of y with respect to time t is 0.1% of r, show that
3
dm  r 
  . 
dt  10 r 2  y 2 

dm
State the geometrical meaning of . [7]
dt

sgexamguru.com
r2  r  1 A B C
6 (i) Show that =   , where A, B and C are constants
(r  2)! r !  r  1 ! (r  2)!
to be determined. [2]
n
(ii) Hence find 
r 1
r2  r  1
(r  2)!
in terms of n. (There is no need to express your answer

as a single algebraic function.) [3]


n
(iii) Explain why 
r 1
r2  1 1
 .
(r  2)! 2
[2]

n
(iv) Use your answer to part (ii) to find 
r 4
r 2  3r  1
r!
in terms of n. [3]

2016 H2 Math Prelim 9740/01 3 [Turn Over]


 
NYJC P1 Questions 377
7

10

12

A 10 feet tall statue is mounted on a 12 feet tall pedestal. A boy is standing x feet away from
the pedestal. His eyes are 5 feet above ground level, and the angle subtended by the statue
from the boy’s eyes is  radians (see diagram).
Prove that
10 x
tan   .
119  x 2
Hence, or otherwise, find the exact value of x for which   is maximum and justify that this

sgexamguru.com
value of x gives the maximum value of  .
Deduce, to the nearest degree, the maximum angle subtended by the statue from the boy’s
eyes.   [9]

8 (i) Find the fourth roots of 1  3 i , giving the roots in the form rei , where r  0 and
     . [3]

Hence, or otherwise, write down the roots of the equation 1  z   1  i 3  0 and


4
(ii)

show the roots Z i , i  1, 2,3, 4 on an Argand diagram. [3]

(iii) Illustrate, using the same Argand diagram, the locus of a point Q representing the

complex number v, where v  1  4 3  4i  2 .

Hence find the exact greatest and least possible values of ZiQ . [4]

2016 H2 Math Prelim 9740/01 4


 
NYJC P1 Questions 378
9 Two biologists are investigating the growth of a certain bacteria of size x hundred thousand at
time t days. It is known that the number of bacteria initially is 20% of a, where a is a positive
constant.
(i) One biologist believes that x and t are related by the differential equation
dx
 x  a  x  . Given that the number of bacteria increases to 50% of a when
dt
2
t  ln 2 days, show that x  2t
. [7]
4e 1
(ii) Another biologist believes that x and t are related by the differential equation
d2 x
 10  9t 2 . Find the general solution of this differential equation and sketch
dt 2
three members of the family of solution curves. [5]

10 (a) (i) Express sin x  3 cos x in the form R sin  x    where R and 
are exact positive constants to be found. [1]

The function f is defined by f : x  sin x  3 cos x,  x  k.

(ii) sgexamguru.com 6
Find the largest exact value of k such that f has an inverse. Hence define
f 1 in similar form and write down the set of values of x for which
ff 1  x   f 1f  x  . [5]

5x
(b) The function g is defined by g : x  2  , x  .
1  x2
(i) Use an algebraic method to find the range of g. [3]
(ii) State a sequence of transformations which transform the graph of y  g( x ) to
10 x
the graph of y  . [3]
4  x2

2016 H2 Math Prelim 9740/01 5 [Turn Over]


 
NYJC P1 Questions 379
11 The line l1 passes through the point A with coordinates (1, 2, 1) and is parallel to the
y z 5
vector i +aj +2k, where a   . The line l2 has equation x  3   . It is given
2 3
that l1 and l2 intersect at point B.
(i) Find the value of a. [4]
(ii) The plane p1 contains the point A and is perpendicular to l2. Find the exact

shortest distance from point B to p1 . Hence find the acute angle between l1

and p1 . [5]

(iii) Find a cartesian equation of plane p2 that is perpendicular to p1  and contains


l1 . [3]
(iv) Find the acute angle between p2 and the xy-plane. [2]

sgexamguru.com

2016 H2 Math Prelim 9740/01 6


 
NYJC P1 Questions 380
Solutions to P1 Prelim 2016
n 1 
 n 1   4
1 2
Let Pn denote the proposition Sn      for n=1,2,3,….
 n  2   3  3

41 (1) 2 2
When n  1 , LHS  S1  u1   .
(2)(3) 3

 1 1   4  2 2
2
RHS         LHS.
 1  2   3  3 3

 P1 is true.
k 1 
 k 1   4 2
Assume Pk is true for some k=1,2,3.., i.e. Sk     .
 k  2   3  3

k 2 
 k  4 2
To prove Pk 1 is also true, i.e. Sk 1     .
 k  3   3  3

LHS  Sk 1
 Sk  uk 1
k 1
 k 1   4
k 1 
2 4  k  1
2

sgexamguru.com
  
 k  2   3  3  k  2  k  3

 4k 1   3  k  1  2
2
 
 k 1  
 3 k  2   k  3  3
  

 4k 1    k  1 k  3  3  k  12  2
  
 3 k  2   k 3  3
  
 4k 1   k 2  2k  3  3k 2  6k  3  2
  
 3 k  2   k  3  3
 
 4k 1   4k 2  8k  2
  
 3 k  2   k  3  3
  
 4k  2   k  k  2   2  k   4 k  2  2
        RHS.
 3  k  2    k  3  3  k  3   3  3
   
Since P1 is true and Pk is true  Pk 1 is true, by Mathematical Induction, Pn is true
for all n =1, 2, 3,….

Microsoft Word - NYJC 2016 H2 Mathematics P1 Solutions 381


2 17 x 2  23 x  12 A Bx  C
  2  
(3 x  4)( x  4) 3 x  4 x  4
2

17x2 + 23x + 12 = A(x2 + 4)+ (Bx + C)(3x+ 4)


          Solving, A = 2, B = 5 and C = 1
17 x 2  23 x  12 5x  1
 
2 2
2
dx   2 dx
2 (3 x  4)( x  4) 2 3 x  4 x 4
2

5  2x 

2
2 1
=   2  2 dx
2 3 x  4 2 x  4 x  4

2
2 2 5 2 1  x 
          = ln 3 x  4  2  ln( x 2  4)    tan 1      
3 2  2
2  2   2
2 
          =  ln 5    
3 4

3 1 1 1 1 1
x  sin t cos t  t   t  sin 2t and y  t  sin 2t
2 2 4 2 4
dx 1 1 dy 1 1
 cos 2t  and   cos 2t
dt 2 2 dt 2 2
dy 1  cos 2t 1  (1  2sin 2 t )
   tan 2 t
dx 1  cos 2t (2cos t  1)  1
2

When
dy
dx sgexamguru.com
 1  tan 2 t  1  t  

4

 1  1 
Since t  0, t   , and  x     and y    
4 4 8 4 8
1     1  
Equation of normal is y        x     
4 8   4 8 

y  x 
4

When y  0 , x   .
4
1 1    1
2 2
1  1 1

0
1
        
t  sin 2t   cos 2t   dt
Volume required is 3  4 8   8 4   2
4
4  2 2
 0.00759 (5 d.p.)

dy
 tan 2 t  t 2  0  as t approaches 0.
dx
Therefore the tangents are parallel to the x-axis.

 1   1


4

OM  OA  OC   2a  b 
2 2

 2  2 1 1
ON  OM    2a  b    2a  b 
3 3 2 3

NYJC P1 Solutions 382


   1 1 1 
AN  ON  OA   2a  b   a=  b  a   AB
 3 3 3

Since AN is parallel to AB and A is the common point, hence A, B and N are
collinear. B1 
Since P is on AB, OP  a   (b  a), where   
 
MP AB  0
 1 
a   (b  a)  2  2a  b   (b  a)  0
 1 
   2  b  a  (b  a)  0
  
 1 2  1
    b      ab  ab   a  0
2

 2  2
But ab  a b cos AOB  2  3cos 60o  3
 1  1
Hence, 9      3      3  4  0
 2  2
3

7
 3 1
OP  a  (b  a)   4a  3b 
7 7

sgexamguru.com
Alternative method:
ab  a b cos AOB  2  3cos 60o  3

Using cosine formula b-a  a + b -2 a b cos 60  7


2 2

 
   AB  AB
AP   AM    
 AB  AB
 
 1 (b  a)  b  a
  b  
2 b  a  b  a

1  b  b-a  b 
   (b  a)  
2  b  a 2 
1 6
=  (b  a)
2 7
3
= (b  a)
7
 3 1
OP  a  (b  a)   4a  3b 
7 7

NYJC P1 Solutions 383


5 y y y2
1. Let   sin 1  sin    cos   1  2 ,
r r r
d 1 d 1
Diff wrt y : cos    
dy r dy r cos 

dm d 1
 m  tan    sec2  
dy dy r cos3 
dm 1
ie,  3
dy
 r 2  y2 2
r 2 
 r 
r2
= 3

r 2
 y2 2

dm dm dy dm r2 r r3
Using   , we have   =
 
3

r  y2 2
3
dt dy dt dt 2 1000 103 r2  y2

3
 r 
=   
 10 r 2  y 2 
 
dm
dt
sgexamguru.com
is the rate of change of the gradient of the line OP 

Alternate method 1:
 y
m  tan  sin 1 
 r
 
 
dm 2  1 y   1  1 
 sec  sin   
dy  r   
2
 r 
 1 y
   
 r 
1  1 
  
2  1 y   r  y 
2 2
cos  sin  
 r
1  1  y y
   where   sin 1  sin  
cos   r  y 
2 2 2 r r
 
1  1  r2  1 
     
   r y 
  
 r  y 
2 2
x / r 2 2 x 2 2

NYJC P1 Solutions 384


r2  1 
  
r  y  r 2  y2
2 2


2
r

r  y2 
2 3/ 2

dm dm dy
Using  
dt dy dt
r2 r
 
 r 2  y 2  1000
3/ 2

r3

103  r 2  y 2 
3/2

3
 r 
 
 10 r 2  y 2 
 
dm
is the rate of change of the gradient of the line OP. 
dt

Alternate method 2:

m  tan  sin 1 
 sgexamguru.com
y
r
y
tan 1 m  sin 1
r
 1  dm 1 1
 2 
  
 1  m  dy  y
2
r
1  
r
dm 1  m2

dy r 2  y2
1  tan 2  y y
 where   sin 1  sin  
r 2  y2 r r
1   y / x
2


r 2  y2
y2
1
 x2
r 2  y2
y2
1 2
r  y2

r 2  y2

NYJC P1 Solutions 385



r 2
 y2   y2

r 2
 y2  r 2  y2
r2

r  y2 
2 3/ 2

Second part is similar to the above.

Alternate method 3:
 y y
m  tan  sin 1   tan  , where   sin 1
 r r
y y
m 
x r2  y2
1
r 2  y 2  y    r 2  y 2   2 y 
1/2

dm
 2
dy  r 2  y2 
r 2  y2  y2 r2  y2 
1/2


r 2
 y2 


r 2
 y2   y2

r  y2  r  y2 
1/2

sgexamguru.com
2 2

r2

r  y2 
2 3/ 2

Second part is similar to the above.


6(i) r2  r  1 A B C
=  
(r  2)! r !  r  1 ! ( r  2)!

r 2  r  1 A  r  1 r  2   B  r  2   C
=
(r  2)! ( r  2)!

By comparing coefficients,
A  1, B  2, C  1

r2  r  1 1 2 1
   
(r  2)! r !  r  1 ! ( r  2)!

NYJC P1 Solutions 386


(ii) n n
 

r 1
r2  r  1
(r  2)!
   r1!   r 21!  (r 1 2)! 
r 1

1 2 1
  
1! 2! 3!
1 2 1
  
2! 3! 4!
1 2 1
  
3! 4! 5!
1 2 1
  
4! 5! 6!

1 2 1
  
(n  2)! ( n  1)! n !
1 2 1
  
(n  1)! n ! (n  1)!
1 2 1
   ]
n ! (n  1)! (n  2)!

sgexamguru.com

1

1

1
2 ( n  1)! (n  2)!
(iii) Since r 2  1  r 2  r  1 for r  0, so we have
n n


r 1
r2  1
( r  2)!
 
r 1
r2  r  1 1
(r  2)!
 
1

1
2 (n  1)! (n  2)!

1  n 1  1
  2
2  ( n  2)! 
n 1
(  0 as n    )
( n  2)!

(iv) n


r 4
r 2  3r  1
r!

Replace r with ( k  2)
n2
 k  2  3 k  2  1
2
 
k 2
 k  2 !

NYJC P1 Solutions 387


n2
 
k 2
k 2  k 1
 k  2 !
n2 2
  k kk2!1   16 
k 1

1 1 1 1
   
2 (n  1)! n ! 6
1 1 1
  
3 (n  1)! n !
n
Alternatively, consider 
r 1
r 2  r 1
 r  2 !
and sub. r  k  2 . So we have

n n2 2


r 1
r 2  r 1
 r  2 !
 k
k3
 3k  1
k!
.

n2 2
 k  3k  1 1
 
1

1
2 (n  1)! ( n  2)!
sgexamguru.com
k!
k3

n2 2
 32  3(3)  1 
 
k4
k  3k  1 1
k!
 
1

1

2 (n  1)! (n  2)!  3!


1 1 1
  
3 ( n  1)! (n  2)!
n
 
k4
k 2  3k  1 1
k!
 
1

1
3 (n  1)! n !

   

NYJC P1 Solutions 388


7

10

12

7 x
From diagram, tan  
x
17
and tan     
x
tan   tan((   )   )
tan(   )  tan 

1  tan(   ) tan 
17 7

 x x
1 ( )

10
sgexamguru.com
17 7
x x

10 x
 x  2 (shown)
119 x  119
1 2
x

Differentiating wrt x,

d (119  x 2 )(10)  10 x(2 x)


sec 
2

dx (119  x 2 )2
1190  10 x 2
=
(119  x 2 )2
For maximum angle,
d
 0  10 x 2  1190
dx
Since x  0 , x  119
Since sec 2   0 ,

NYJC P1 Solutions 389


Using first derivative test at vicinity of x  119 , it can be shown that the angle is
maximum.

10 119 5 119
Therefore, tan       25o
119  119 119
 119 

 119   119 

d
Sign of + 0 -
dx
Slope

8 (i) w 4  1  3 i
2
i
w  2e
4 3

1 1  2 
i   2 k 
w2 e 4 4 3 
k  2, 1, 0, 1,
1  5  1   1   1  2 
i   i   i  i 
w2 e 4  6 
, 2 e 4  3
, 2 e 4 6 4
, 2 e  3 

(ii) 1  z   1  i  3  0  1  z   1  i  3
4 4

z  w 1

z2 e sgexamguru.com
 5 
1
4
 5 
i  
 6 
 1, 2 e
1
4
 
i  
 3

 
 1, 2 e
1
4
 
i 
6
 1, 2 e
 
1
4
 2 
i
 3 

1
 2 
1 i   1 i   1 i  1 i 
z1  2 e 4  6 
 1, z2  2 e 4  3
 1, z3  2 e 4 6
 1, z4  2 e 4  3 
1

Im 

x   
2

Z4 
x  x  Z3 
x  Re
Z1 x 
Z2 

(iii)
Least possible ZiQ =Z3Q= 8  2  21/ 4  6  21/ 4
Greatest possible ZiQ =Z1Q= 8  2  21/4  10  21/4
   

NYJC P1 Solutions 390


9 dx
(i)  x(a  x)
dt
dx
 x(a  x)   dt
1 1

x a

ax
a
dx   dt
1 1
ln x  ln a  x  t  C
a a
1 x
ln t C
a ax
x
ln  at  aC
ax
x
 Aeat , where A  eaC
ax
When t = 0, x = 0.2a
0.2a
A
0.8a
1
A
4
When t = ln 2, x = 0.5a
0.5a 1 a ln 2
 e
0.5a 4
4  ea ln 2 sgexamguru.com
2a  4
a2
x 1
Subst. values of A and a ,  e 2t
2 x 4
4 x  2e 2t  xe 2t
x  4  e 2t   2e 2t
2e 2t 2
x  2t (shown)
 4  e  4e  1
2t

d2 x
(ii) 2
 10  9t 2
dt
dx
 10t  3t 3  A
dt
3
x  5t 2  t 4  At  B
4
When t = 0, x = 0.2a
B  0.2a
3
x  5t 2  t 4  At  0.2a
4

NYJC P1 Solutions 391


x

A= 0 A>0
0.2a
A<0

10  
sin x  3 ccos x  2sin
nx 
(a)(i)  3

(ii) y 


sgexamguru.com  

‐2 
 

7
ne. Hence largest k 
For f to havve an inversse, f must bee one-to-on .
6
   y 
Consider y  2 sin  x    x  sin 1   
 3 2 3

 x 
So f 1 : x  sin 1    ,  2  x  2.
 2 3

For ff 1  x   f 1f  x  , we must have Df  Df 1 .

 
So the soluution set is x   , 2  .
6 

NYJC P1 Solutions 392


(b)(i) 5x
Consider y  2 
1  x2
5x
 2 y 
1  x2


  2  y  1  x2  5x 
  2  y  x2  5x   2  y   0

D   5   4  2  y  2  y   0
2

25  4  2  y   0
2

5  2  2  y  5  2  2  y   0
1  2 y  9  2 y   0
1 9
  y
2 2
 1 9
So range of g    , 
 2 2
(ii)
g( x)  2  sgexamguru.com
5x
1  x2
 x
5  
 x
g   2 
2 10 x
 2
 2  x
2
4  x2
1   
 2
 x 10 x
g  2 
 2 4  x2
Scale the graph of g by factor 2 parallel to the x-axis followed by a reflection in the
y-axis followed by a translation of -2 units in the direction of y-axis.
Or
 x 
 5  
 x
 g      2      2 
5x 2 10 x
g( x)  2 
1  x2 2  2  4  x 2 Scale the graph of g
 1   x  
  2  
 x 10 x
 g    2 
2 4  x2

NYJC P1 Solutions 393


by factor 2 parallel to the x-axis followed by a reflection in the x-axis followed by a
translation of 2 units in the direction of y-axis.

11 1 1
   
(i) l1 : r   2     a  ,   
1  2
   
 3 1
   
l2 : r   0     2  ,   
5  3
   
Equating the equations of the 2 lines,
1   3  
2  a  2 
1  2  5  3
  0,   2
Solving,
 a  1
(ii) Coordinates of B is (3, 0, 5)
1
  
AB  2 
 3
 
sgexamguru.com
Shortest distance from B to p1 
14
 3   1    1   2   1 
          
 0    2    2   2  2 
 5   1    3   4   3  10
  
14 14 14
Alternative solution 1:
1 1 1
    
 2   2  2 
 3  2  3
Equation of plane p1 : r         
8
14 14 14
Equation of plane parallel to p1 and containing point B is

 1  3  1
    
 2   0  2 
 3   5   3  18
r        
14 14 14
18 8 10
Shortest distance from B to p1 is  
14 14 14
Alternative solution 2:
Let F be the foot of perpendicular from point B to plane p1
Line l2 meets plane p1 at point F

NYJC P1 Solutions 394


 3 1
    
OF   0     2  for some   
5  3
   
 3   1   1 
     
 0     2    2   8
 5   3   3 
   
3  15   (1  4  9)  8
5
 
7
1
 5  5 5
Shortest distance from B to p1 is BF    2   1 4  9  14
7  7 7
 3


AB  22  ( 2) 2  42  24
10 1
sin  
14 24
  33.10

1  1   7 

sgexamguru.com      
(iii)A normal vector to p2 is  2    1   1 
     
 3   2   3 
 7  1  7 
    
Equation of p2 is r  1    2  1   6
 3   1   3 
    
Cartesian equation of p2 is 7 x  y  3 z  6
(iv) Equation of x-y plane is z = 0
Let  be the acute angle between p2 and x-y plane.
 7  0
  
 1  0 
 3   1 
   3
cos   
59 59
 = 67.0 o

NYJC P1 Solutions 395


NANYANG JUNIOR COLLEGE
JC2 PRELIMINARY EXAMINATION

Higher 2

MATHEMATICS 9740/02

Paper 2 22nd September 2016

3 Hours

Additional Materials: Cover Sheet


Answer Papers
List of Formulae (MF15)

READ THESE INSTRUCTIONS FIRST

Write your name and class on all the work you hand in.
Write in dark blue or black pen on both sides of the paper.
You may use a soft pencil for any diagrams or graphs.
Do not use staples, paper clips, highlighters, glue or correction fluid.

Answer all the questions.

sgexamguru.com
Give non-exact numerical answers correct to 3 significant figures, or 1 decimal place in the case of
angles in degrees, unless a different level of accuracy is specified in the question.
You are expected to use a graphic calculator.
Unsupported answers from a graphic calculator are allowed unless a question specifically states
otherwise.
Where unsupported answers from a graphic calculator are not allowed in a question, you are
required to present the mathematical steps using mathematical notations and not calculator
commands.
You are reminded of the need for clear presentation in your answers.

At the end of the examination, fasten all your work securely together.
The number of marks is given in brackets [ ] at the end of each question or part question.

This document consists of 6 printed pages.

Microsoft Word - NYJC 2016 H2 Mathematics P2 396


Section A: Pure Mathematics [40 marks]

1 The first four terms of a sequence of numbers are 10, 6, 5 and 7. Sn is the sum of the first n

terms of this sequence. Given that Sn is a cubic polynomial in n, find Sn in terms of n.

[4]

3 2 17
Show that U n  n  n  17 , where U n denotes the nth term of the sequence. [2]
2 2

Find the set of values of n for which Sn  3U n . [2]

2 On separate diagrams, draw sketches of the graphs of


x 2 (3  x)
(i) y ,
1 x
x2 (3  x)
(ii) y2  ,
1 x
including the coordinates of the points where the graphs cross the axes and the equations of
any asymptotes. You should show the features of the graphs at the points where it crosses
the x-axis clearly.
sgexamguru.com
Show that the area of the region enclosed by the graph in (ii) may be expressed in the form
3

2
3x  x 2
 dx .
0 (4  ( x  1) 2 )

By using the substitution x  1  2 sin  , evaluate this area exactly. [10]

3 (a) Solve z 3  2  2  i  z 2   8  3i  z  5  i  0 , given that one of the three roots is real.[5]


(b) The complex number u is given by u  cos   i sin  , where 0    .
2

(i) Show that 1  u 2  2iu sin  and hence find the modulus and argument of
1  u 2 in terms of  . [4]

Given that 1  u 2  is real and negative, find the possible values of  in terms
10
(ii)

of  . [3]

H2 Math Prelim 2016 9740/02 2


NYJC P2 Questions 397
4 [In this question, you may use the result that for a circle with radius r , a sector with angle
1 2
 has arc length r and area r  .]
2
(a) A circle of radius r is divided into 16 sectors of decreasing arc length. Let Ln and An

be the arc length and the area of the n th sector respectively. Suppose Ln is an

arithmetic sequence with first term r and common difference d .


 8
(i) Show that d   r . [2]
 60 
(ii) Show that An is an arithmetic sequence. [3]

(b) Let Gn be the area of a sector of a circle with radius a . Suppose that Gn is a

geometric sequence with first term a and common ratio r , where 0  r  1 .


(i) If N sectors are needed to form the circle, show that r satisfies the equation
r N   ar  ( a  1)  0 . [3]
(ii) If an infinite number of sectors are needed to form the circle, find r in terms of
a. [2]

sgexamguru.com
Section B: Statistics [60 marks]

5 A company sells a certain brand of baby milk powder and would like to gather feedback on
their product. Explain why quota sampling is appropriate in this situation and describe
briefly how a sample of 50 could be chosen using quota sampling. [3]
The company wishes to randomly reward 5 customers with free milk vouchers through a
lucky draw. Suppose that 2000 customers qualify for the draw, show that there will be equal
probability of a particular customer being the first to be selected or the third to be selected
for the free milk vouchers. [2]

H2 Math Prelim 2016 9740/02 3 [Turn Over]


NYJC  P2 Questions 398
6 The mass, in grams, of an ice-cube has the distribution N(  ,  2 ) .  The mean mass of a

random sample of n ice-cubes is denoted by X . It is given that P( X  35.0)  0.97725 and

P( X  20.0)  0.84134 .
(i) Obtain an expression for  in terms of n. [3]
(ii) Find P( X  32) . [2]
Assume now that the mass of an ice-cube has the distribution N(25, 50) .  
An ice dispenser discharges 15 ice cubes each time into a cup. State the distribution of the
mass of a discharge of 15 ice cubes. [1]
(iii) Find the mass exceeded by 10% of these discharges, correct to 1 decimal place.
[2]
(iv) Find the probability that the mass of the first discharge of ice-cubes is more than the
second discharge. [2]

7 A team of 5 men and 5 women is to be picked from 8 men and 9 women such that two of the
9 women, Ann and Lucy, must both be selected or not at all. Find the number of ways in
which this can be done. [2]

(i)
sgexamguru.com
Assume now the team is selected and Ann, Carrie and Lucy are included.
The selected team is to form a queue. Find the number of possible arrangements
if Ann and Lucy are to occupy both the second and the sixth positions and no two people of
the same gender are to stand next to each other. [3]
(ii) On another occasion, the selected team is required to be seated at a round table
with 10 chairs of different colours. If only Carrie can be seated between Ann
and Lucy, find the number of possible arrangements. [3]

H2 Math Prelim 2016 9740/02 4


NYJC P2 Questions 399
8 Two teams, the Ramblers and the Strollers, meet annually for a quiz which always has a
winner. If the Ramblers wins the quiz, the probability of them winning the following year is
0.7. If the Strollers wins the quiz, the probability of them winning the following year is 0.5.
The Ramblers won the quiz in 2015.
(i) Find the probability that the Strollers will win in 2018. [2]
(ii) If the Strollers were to win in 2018, what is the probability that it will be their first
win for at least three years since 2015?
[2]
(iii) Assuming that the Strollers wins in 2018, find the smallest value of n such that the
probability of the Ramblers winning the quiz for n consecutive years after 2018 is
less than 5%. [3]

9 It is believed that the probability p of a randomly chosen pregnant woman giving birth to a
Down Syndrome child is related to the woman’s age x, in years. The table gives observed
values of p for 6 different values of x.

x 20 25 30 35 40 45

sgexamguru.com
p 0.00023 0.00067 0.00125 0.00333 0.01000 0.03330

(i) Sketch the scatter diagram for the given data. [1]
(ii) Find, correct to 4 decimal places, the product moment correlation coefficient
between
(a) p and x ,
(b) ln p  and x ,
(c) p and x2 . [2]
(iii) Using the most appropriate case from part (ii), find the equation which best models
the probability of a pregnant woman giving birth to a Down Syndrome child at
different ages. [2]
(iv) Hence, estimate the expected number of children with Down Syndrome that will be
born to 5000 randomly chosen pregnant women of age 32. [2]

H2 Math Prelim 2016 9740/02 5 [Turn Over]


NYJC  P2 Questions 400
10 At an early stage in analysing the marks, x, scored by a large number of candidates in
an examination paper, the Examination Board takes the scores from a random sample
of 250 candidates. The results are summarised as follows:

 x  11872 and x 2
 646193

(i) Calculate unbiased estimates of the population mean and variance to 3 decimal
places. [2]
(ii) In a 1-tail test of the null hypothesis   49.5 , the alternative hypothesis is
accepted. State the alternative hypothesis and find an inequality satisfied by the
significance level of the test. [4]
(iii) It is subsequently found that the population mean and standard deviation for
the examination paper are 45.292 and 18.761 respectively. Find the probability
that in a random sample of size 250, the sample mean is at least as high as the
one found in the sample above. [2]

11 On a typical weekday morning, customers arrive at the post office independently and
at a rate of 3 per 10 minute period.
(i) State, in context, a condition needed for the number of customers who arrived at the

distribution.
sgexamguru.com
post office during a randomly chosen period of 30 minutes to be well modelled by a Poisson
[1]
(ii) Find the probability that no more than 4 customers arrive between 11.00 a.m. and
11.30 a.m. [2]
(iii) The period from 11.00 a.m. to 11.30 a.m. on a Tuesday morning is divided into
6 periods of 5 minutes each. Find the probability that no customers arrive in at
most one of these periods. [2]
The post office opens for 3.5 hours each in the morning and afternoon and it is noted
that on a typical weekday afternoon, customers arrive at the post office independently
and at a rate of 1 per 10 minute period. Arrivals of customers take place independently
at random times.
(iv) Show that the probability that the number of customers who arrived in the afternoon
is within one standard deviation from the mean is 0.675, correct to 3 decimal places.
[3]
(v) Find the probability that more than 38 customers arrived in a morning given
that a total of 40 customers arrived in a day. [4]
(vi) Using a suitable approximation, estimate the probability that more than 100 customers
arrive at the post office in a day. [3]

H2 Math Prelim 2016 9740/02 6  


NYJC P2 Questions 401
Solutions to P2 Prelim 2016:
1. S n  an3  bn 2  cn  d

10  a  b  c  d
16  8a  4b  2c  d
21  27 a  9b  3c  d
28  64a  16b  4c  d
1 7
a , b   , c  13, d  0
2 2
1 3 7 2
Sn  n  n  13n
2 2

U n  S n  S n 1
1 3 7 2 1 7 
 n  n  13n    n  1   n  1  13  n  1 
3 2

2 2  2 2 
3 17
 n 2  n  17
2 2

Sn  3U n

1 3 7 2 3 17 
n  n  13n  3  n 2  n  17 
2
sgexamguru.com

2 2 2
1 3 77
n  8n 2  n  51  0
2 2
From GC
n 1 3 77
n  8n 2  n  51
2 2
1 20
2 2
3 6
4 7
5 4
6 0
7 2
8 1

From GC, n : n    , n  1, 2, 7

   

Microsoft Word - NYJC 2016 H2 Mathematics P2 Solutions 402


2  
y  3 x 
y  x2   
 1 x 

x
-1 3

 
 
 

sgexamguru.com
 3 x 
y 2  x2  
 1 x 

3 x
 x 1  x ddx
3
b loop = 2
Areaa enclosed by
0

(3  x))(3  x)
 (1  x))(3  x) dx
3
= 2 x
0

x(3  x )
 3  2 x  x dx
3
                                             = 2
0 2

3x  x 2

 (4  ( x  1) ) dx
3
= 2
0 2

dx
Usinng x  1  2ssin  , we haave  2 ccos 
d
1 
Also, when x  0 , sin       
2 6

And when x  3 , sin   1   
2
3x  x 2


3
Therrefore area = 2 dx
0 (4  ( x  1) 2 )

NYJC P2 Solutions 403



3(2sin   1)  (2sin   1) 2
 (2cos )d
2
=2 

6 (4  (2sin  ) 2 )

=2
 
2

 2sin   2  4sin  d 2

6

 sin   1  2sin  d


2 2
=4 

6

  sin   cos 2 d


2
=4 

6

 sin 2  2
= 4   cos  
 2   
6

  3 3 
= 4 0       3 3
  2 4  
3(a) z 3  2  2  i  z 2   8  3i  z  5  i  0

Let z  x be the real root.


x3  2  2  i  x 2   8  3i  x  5  i  0
x3  4 x 2  2ix 2  8 x  3ix  5  i  0
x 3
 4 x 2  8 x  5    2 x 2  3x  1 i  0

Since z  x is a root,
sgexamguru.com
x3  4 x 2  8 x  5  0 and 2 x2  3x  1  0
From GC: x  1

Therefore, the real root is z  1


z 3  2  2  i  z 2   8  3i  z  5  i  0
 z  1  z 2  Az   5  i    0
 z  1  z 2   3  2i  z   5  i    0
z  1 or z 2   3  2i  z   5  i   0

  3  2i    3  2i   4 5  i 
2

z
2
  3  2i   1  4i 

2
 2  3i or 1+i
Roots: 1, 2  3i , 1+i

NYJC P2 Solutions 404


3(b) 1  u 2  1   cos   i sin  
2

 1  cos 2   sin 2   2i sin  cos 


 2sin 2   2i sin  cos 
 2sin   sin   2i cos  
 2i sin   cos   i sin  
 2iu sin 
Alternative
u  cos   i sin   e i
1  u 2  1  e 2 i
 e i  e  i  e i 
 u  cos   i sin   i sin   cos  
 u  2i sin  
 2iu sin 
1  u 2  2iu sin   2sin  i u
 2sin 
arg 1  u 2   arg  2iu sin  
 arg  2i sin    arg  u 

 

sgexamguru.com
2

1  u 
2 10
is real and negative: arg 1  u 2   10 arg 1  u 2    2k  1  , k  
10

  
10        2k  1 
 2 
5  10   2k  1 
 2k  6  

10
 1 2
0   :   , 
2 5 5
Alternative
 10
   
1  u 
2 10
  2sin  e 2 
 
  210 sin10    cos  5  10   i sin  5  10  

Since 1  u 2  is real and negative, and 210 sin10   0 ,


10

sin  5  10   0 and cos  5  10   0

5  10  k , k  
 k  5 

10

NYJC P2 Solutions 405


 1 1 3 2
0   : , , , 
2 10 5 10 5
1 2
Only when    ,  will cos  5  10   0 .
5 5
1 2
Therefore,    ,  .
5 5
4(a)(i) Since S16  2 r , thus
16
(2r  15d )  2 r
2
r
 2r  15d 
4
 8 
 15d   r
 4 
 8 
d  r
 60 
(a)(ii) 1 2 1
Since Ln  r n and An  r  n , thus An  rLn .
2 2
Hence
1 1
An 1  An  rLn 1  rLn
2 2
1
 r  Ln 1  Ln 
2
1
sgexamguru.com
 rd  constant
2
for all n  2, ,15
Thus An is an arithmetic sequence.

(b)(i) Since S N   a 2 , we have


a(r N  1)
  a2
r 1
 r N  1   ar   a
 r N   ar  ( a  1)  0

(b)(ii) Since 0  r  1 , we have r N  0 as N   .


Thus
  ar  ( a  1)  0
 a 1
r
a

Alternative:
Since S   a 2 , we have
a
  a2
1 r
1
 1 r 
a
 a 1
r
a
NYJC P2 Solutions 406
5 No sampling frame or entire list of people consuming this brand is not available
Station an interviewer at the exits of a local supermarket store during peak hours and he is free
to choose 25 male and 25 female customers who buy the products.
1
P( a particular consumer is the first to be selected) =
2000

P(a particular consumer is the third to be selected)


1999 1998 1 1
=  (shown)
2000 1999 1998 2000

(i) P( X  35.0)  0.97725


6

35  
P( Z  )  0.97725
/ n
35  
 2      [1]
/ n
P( X  20.0)  0.15866

20  
P( Z  )  0.15866
/ n
20  
 1    [2]
/ n
3
 15
Eqn [1] –[2]: n

(ii)
sgexamguru.com
 5 n


  25 , X  N (25,52 ) since 5
n

P( X  32) = 0.0808

let M be the mass of a randomly chosen discharge of 15 ice cubes. M  N (375, 750)

(iii)
P( M  a )  0.1
P( M  a )  0.9
a  410.1
(iv)
M 1  M 2  N (0,1500)
P( M 1  M 2 )  P( M 1  M 2  0)  0.5
7 5M, [3W, A, L]:
No. of selections =  8
C 5  7 C 3  = 1960
5M, 5W (exclude A and L):
No. of selections =  8 C 5  7 C 5  = 1176
Total number of selections
=  8 C 5  7 C 3    8 C 5  7 C 5 
= 1960 + 1176= 3136
NYJC P2 Solutions 407
(i) M1 A M2 M3 L M4 M5

W W W 
Arrange 5M: No. of ways = 5! = 120
Arrange A and L: No. of ways = 2! = 2
Arrange 3W: No. of ways = 1(3!) = 6

Total no. of arrangements =  5!  2!   3! 

= 1440
(ii) No of arrangements (8  1)!(2)(10) =100800
8(i) P(Strollers will win in 2018)
  0.7  0.7  0.3   0.7  0.3 0.5
  0.3 0.5 0.3   0.3 0.5 0.5
 0.372
(ii) Let event A denotes “Strollers first win for at least three
years” and event B denotes “Strollers win in 2018”
P  A  B
P  A | B 
P  B

 0.7  0.7  0.3



0.372
 0.39516
sgexamguru.com
 0.395  3 s.f.
 0.5 0.7   0.05
(iii) n 1

 0.7   0.1
n 1

ln  0.1
n 1   6.4557
ln  0.7 
n  7.4557
Hence, the smallest value is n = 8.
Alternative method:

 0.5 0.7   0.05


n1

Using GC:
n probability
7 0.05582
8 0.04418 < 0.05
9 0.02882
Hence, the smallest value is n = 8.
Or

NYJC P2 Solutions 408


 0.7   0.1
n1

n probabiility
7 0.117655
8 0.082355 < 0.01
9 0.054655
Hencce, the smalllest value iss n = 8.
9 (i)
p
  0.03
3330
 
 
 
 
 
 
  0.000023 x
  20
 
45
5

(ii)(aa) r = 0.81300 (b) r = 0.9960 (c) r  0.8667  

(iii)S
Since |r| is cllosest to 1 for
f model (bb), equation
n would be
ln p  0.194409 x  12.32 22

When x = 322,
(iv)W
sgexamguru.com
p  00.0022181
Expeected numbeer = 5000(00.0022181) = 11.1
10
0 (i) U
Unbiased estiimate of pop mean =

x
 x  118772  47.4888
n 2500
1 118722
nce, s2 =
Unbiiased estimaate of popullation varian (646193  )  330.986
24
49 250

(ii) To teest
H o :   49.5
H1 :   49.5
At  % significance level
330.986
Central limit Theorem, X  N(49.55,
Sincee n= 250 is large, by C ) approx.
250
Sincee H0 is rejeccted,

p-vallue = P( X  47.488)  0.040179 <
100
  44.02

(iii) Sincee n =250 iss large, by C


Central Lim
mit Theorem
m,
2
18.761
X  N (45.292, ) approximattely
250
P( X  47.488)  0.0321

NYJC P2 Solutions 409


11(i) The mean number of customers who arrived at the village post office during a random chosen
30 minutes period must be a constant.

(ii) Let X be the random variable denoting the number of customers who arrive at the village post
office between 11.00 a.m. and 11.30 a.m.
i.e. X ~ Po  9 

P  X  4   0.054964  0.0550  3 s.f.


(iii) Let Y be the random variable denoting the number of customers who arrive at the village post
office in 5 minutes
i.e. Y ~ Po 1.5 

P Y  0   0.22313

Let W be the random variable denoting the number of periods (of 5 minutes each) out of 6
where Y = 0
i.e. W ~ B(6, 0.22313)
P W  1  0.59867 = 0.599 (3 s.f.)

(iv) Let U be the random variable denoting the number of customers who arrive at the village post
office in 3.5 hours in the afternoon
i.e. U~ Po  21 sgexamguru.com
P (     U     )  P (16.4  U  25.6)
 P (U  25)  P (U  16)
 0.675
(v) Let T be the random variable denoting the number of customers who arrive at the village post
office in 3.5 hours in the morning
T  U  Po(84)
P(T  38 and T  U  40)
P(T  38 | T  U  40) 
P(T  U  40)
P(T  39)P(U  1)  P(T  40)P(U  0)

P(T  U  40)
 1.44 104
(vi) T  U ~ Po  84 

Since   84  10 , hence use the normal distribution for approximation

i.e. T  U ~ N 126,   126   approximately


2

c .c .
P T  U  100   P T  U  100.5   0.0359 (3 s.f.)

NYJC P2 Solutions 410


1 The police wish to crack a 3-digit passcode. The sum of the digits is 14. When the
digits in the number are reversed, the new number becomes 495 more than the
original number. The digit in the tens position is 3 more than the digit in the hundreds
position. What is the passcode? [4]

2 C
A

B C

A
Fig. 1 Fig. 2

Fig. 1 shows a circular card with centre C. A sector CAB is removed from the card,
and the remaining card is folded such that AC and BC meet without overlapping to
form a cone, as shown in Fig. 2 (A will meet B). Use differentiation to find the angle

sgexamguru.com
ACB exactly such that the volume of the cone is as large as possible.
1 2
[6]

[It is given that a cone with radius r and height h has volume  r h and curve surface
3
area  rl where l is the slant height.]

4 A B
3 (i) Show that can be expressed as  , where A and B are
4r  12r  5
2
2r  1 2r  5
constants to be determined. [2]
n 1
2
(ii) Hence, find an expression for  4r
r 1
2
 12r  5
in terms of n. [3]

n 1
2
(iii) Hence, find the smallest value of n for which  4r
r 1
2
 12r  5
is at least 99%

of its sum to infinity. [3]

Microsoft Word - Prelim Paper 1 questions and solutions for PJC 411
2

4 A curve C has parametric equations


x  2a cos3  , y  a sin 3  ,
 
where    and a is a positive constant.
2 2

A point P lies on C.

(i) Find, in terms of a, the exact coordinates of P, whose tangent is parallel to the
line 2 y   x . [4]

(ii) The tangent at P meets the y-axis at a point Q. Find the cartesian equation of
the locus of the mid-point of PQ as  varies. [4]

(i) A point P lies on C. Find, in terms of a, the exact coordinates of P, whose


tangent is parallel to the line 2 y   x . [4]


(ii)  
The tangent to C at the point Q 2a cos3 t , a sin 3 t , where 0  t 
2
, meets the

x- and y-axes at R and S respectively. Find a cartesian equation of the locus of

sgexamguru.com
the mid-point of RS as t varies. [4]

5 The sum, Sn , of the first n terms of a sequence is given by

1 2 3 n
Sn     ...  .
2! 3! 4! (n  1)!
(i) Find the values of S1 , S2 , S3 and S4 . [2]

(ii) By expressing Sn in the form 1  f (n) for n  1, 2,3, 4 , find a conjecture for

Sn in terms of n . [2]

(iii) Hence prove by mathematical induction the result of Sn for all positive

integers n . [4]
[It is given that n !  n  n  1 n  2   3  2 1 ]

@PJC 2016 [Turn Over]


PJC P1 412
3

6 Referred to the origin O, points A and B have position vectors a and b respectively.
Point C lies on OA produced such that OA : OC  2 : 5 . Point D is on AB, between A
and B such that AD : DB  4 :1 .
 
(i) Find the position vectors OC and OD , giving your answers in terms of a and
b. [2]
(ii) Find a vector equation of line CD. [2]

(iii) Point E lies on CD produced, and it is also on OB, between O and B. Find OE
and the ratio OE: EB. [5]

7 Newton’s law of cooling states that the rate of cooling in t minutes is proportional to
the difference between the body temperature T oC and its immediate surrounding
temperature To oC . Show that T  To  Ae kt , where A and k are positive constants.
[3]

Nurul is the chef of a dessert shop and she leaves her work place at 9pm daily. Before
she leaves, she is required to cook a big pot of dessert and leave it to cool, before

sgexamguru.com
placing it in the refrigerator for the next business day. She takes 30 minutes to cook
the pot of dessert to 100 oC , and then leaves it to cool. After 15 minutes, the pot of
dessert cools to 70 oC .

The room temperature of the kitchen is 30 oC , and the refrigerator can only
accommodate items with temperature of at most 35 oC . By what time, correct to the
nearest minute, must Nurul start to cook the pot of dessert so that she will be able to
leave her work place on time? [5]

8 A lion eyes its prey which is k m away. The lion starts his chase of its prey with a leap
of 2.5 m. Each subsequent leap of the lion is shorter than his preceding leap by
0.05 m. Its prey notices the lion’s chase and runs away with a first leap of 1.5 m, with
each subsequent leap 5% less than the previous leap. You may assume that the lion
and the prey start running at the same moment and they complete the same number of
leaps after the first leap.

(i) Find the total distance covered by the lion after n leaps. [2]

@PJC 2016 [Turn Over]


PJC P1 413
4

(ii) Find the total distance covered by the prey after n leaps. Deduce that the
distance covered by the prey can never be greater than 30 m. [3]

(iii) Given k  25 , find the least number of leaps the lion needs to take to catch its
prey. [3]

(iv) Assuming that the lion can cover a maximum of 30 leaps, find the least integer
k, so that the prey will survive the hunt. [3]

 2t
9 (a) (i) If t  tan , show that sin  . [2]
2 1 t2

(ii) Use the substitution t  tan to find the exact value of
2
  

 tan  1 
02  sin 2  1  d . [5]
 
 

(b) Find  e 2 v cos 3v dv . [4]

10
sgexamguru.com
The point A has coordinates 18, 2, 0  . The plane p1 has the equation x  3 y  z  a ,

where a is a constant. It is given that p1 contains the line l1 with equation

x 1 z 1
y .
2 5
(i) Show that a  2 . [2]
(ii) Find the coordinates of the foot of perpendicular from the point A to p1 .
[3]
(iii) B is given to be a general point on l1 . Find an expression for the distance
between the point A and B. Hence find the position vector of B that is nearest
to A. [4]

The planes p2 and p3 have the equations x  z  1 and 2 x  by  z  4 respectively,


where b is a constant.

@PJC 2016 [Turn Over]


PJC P1 414
5

(iv) Given that p2 and p3 intersect at l2 , show that l2 is parallel to the vector

 b 
  . By finding a point that lies on both planes, find a vector equation of l .
 1  2
 b 
 

[3]


11 (a) The complex number w is such that w  rei , where r  0 and 0    . The
2
w2
complex conjugate of w is denoted by w * . Given that  3 , find the exact
w*
values of r and  . Hence find the three smallest positive integer n for which
wn is a real number. [5]

(b) The complex number z is such that z 5  1  i  0 .

(i) Find the modulus and argument of each of the possible values of z. [5]

(ii) sgexamguru.com
Two of these values are z1 and z2, where

2
 arg z1   and


  arg z2   . Find the exact value of arg  z1  z2  in terms of 
2
and illustrate the locus arg  z  z1   arg  z1  z2  on an Argand

diagram. [5]

@PJC 2016 [Turn Over]


PJC P1 415
6

2016 JC 2 Preliminary Examination Paper 1 Solution


1
Let the passcode be xyz.
x  y  z  14........(1)
100 z  10 y  x  100 x  10 y  z  495
99 z  99 x  495......(2)
y  x  3..................(3)
Using the GC, x  2, y  5, z  7
 the passcode is 257
2
Let the radius and height of the cone be r and h respectively.
Let the radius of the circular card be x and angle ACB be  .
By Pythagoras Theorem,
x 2  r 2  h2  r 2  x 2  h2
1 1 1
V   r 2 h    x 2  h 2  h    x 2 h  h3 
3 3 3
dV 1
   x 2  3h 2   0
dh 3
x2 2
h2   r 2  x2
3 3
Consider the circumference of the circle without sector:
2 r 
2  
2 sgexamguru.com
 2 x 
2
2 x  (2   )( x)
3
 2
  2 1  
 3 

Alternatively, consider the curve surface area of the cone,


 2   
 x2     rx
 2 
   2 2
 x2   x2    x x
 2  3
   2
1  
 2  3
 2
  2 1  
 3 
2
dV 1
   6h   2 h  0 (Max)
dh 2 3

@PJC 2016 [Turn Over]


PJC P1 416
7

4 4 A B
(i)   
4r  12r  5  2r  1 2r  5  2r  1 2r  5
2

4  A  2r  5   B  2r  1
5   5 
when r   : 4  B  2     1  B  1
2   2 
1   1 
when r   : 4  A  2     5  A 1
2   2 
4 1 1
 2  
4r  12r  5 2r  1 2r  5
n 1
2 1 n 1 4
(ii) 
r 1 4 r  12 r  5
2
 
2 r 1 4r  12r  5
2

1 1 1
  
2 3 7
1 1
+ 
5 9
1 1
+ 
7 11
1 1
+ 
9 13
+ ..........
1 1
+ 
2n  5 2n  1

sgexamguru.com
+
1

1
2n  3 2n  1
1 1 
+  
2n  1 2 n  3 
11 1 1 1  1 8 4n  4  4 2(n  1)
        2   2
2  3 5 2n  1 2n  3  2  15 4n  8n  3  15 4n  8n  3
(iii) Sn 1  0.99S
2n  2  1  4 
  
4n  8n  3  100   15 
2

1500(2n  2)  4(4n 2  8n  3)
0
(1500)(4n 2  8n  3)
4n 2  742n  747
0
(1500)(4n 2  8n  3)
(n  186.501)(n  1.001)
0
1500(2n  1)(2n  3)
 n  186.501  0 since (2n  1)  0, (2n  3)  0, (n  1.001)  0
n  186.501

Alternatively

@PJC 2016 [Turn Over]


PJC P1 417
8

4 2n  2  4
 2  0.99  
15 4n  8n  3  15 
2n  2  1  4 
  
4n  8n  3  100   15 
2

4n 2  8n  3  750n  750 (Since n is positive integer)


4n 2  742n  747  0
n  186.5
minimum n = 187 (Alternative solution)
4 (i)
x  2a cos3  y  a sin 3 
dx dy
 2a(2 cos 2  )( sin  )  a (3sin 2  )(cos  )
dt dt
dy d y d x
 
d x d d
3a cos  sin 2 

6a cos 2  sin 
1
  tan 
2
dy 1

dx 2

sgexamguru.com


4
       a 2 a 2 
Point P:  2a cos3   , a sin 3      , 
 4  4   2 4 
(ii) The equation of tangent at Q is
1
y  a sin 3 t   tan t ( x  2a cos3 t )
2
1 
y    tan t  x  a sin t cos 2 t  a sin 3 t
2 
1 
y    tan t  x  a sin t
2 

@PJC 2016 [Turn Over]


PJC P1 418
9

R(2a cos t , 0), S (0, a sin t )


 1 
Midpoint of RS   a cos t , a sin t 
 2 
x
x  a cos t  cos t 
a
1 2y
y  a sin t  sin t 
2 a
cos t  sin t  1
2 2

2 2
 x   2y 
    1
a  a 
x2  4 y 2  a2
 a
Since 0  t  , 0  x  a or 0  y 
2 2
1 1
5 (i) S1  
2! 2
1 2 5
S2   
2! 3! 6
1 2 3 23
S3    
2! 3! 4! 24
1 2 3 4 119

sgexamguru.com
S4     
2! 3! 4! 5! 120
(ii)
1 1
S1   1
2 2
5 1
S2   1 
6 6
23 1
S3   1
24 24
119 1
S4   1
120 120
1
Sn  1 
(n  1)!
(iii)
1
Let Pn be the statement Sn  1  for n  1, 2,3,...
(n  1)!
when n  1
1
LHS  S1 
2
1 1
RHS  1  
(1  1)! 2
 P1 is true

@PJC 2016 [Turn Over]


PJC P1 419
10

Assume Pk is true for some k  1,2,3,...


1
Sk  1 
(k  1)!
We want to prove that Pk 1 is also true
1
Sk 1  1 
(k  2)!
LHS  Sk 1
k 1
 Sk 
( k  2)!
1 k 1
 1 
( k  1)! ( k  2)!
 ( k  2)  ( k  1) 
 1  
 ( k  2)! 
1
 1
( k  2)!
 RHS
 Pk 1 is true
Since P1 is true and Pk is true  Pk 1 is true ,
by mathematical induction Pn is true for all n  1, 2,3,...

sgexamguru.com

@PJC 2016 [Turn Over]


PJC P1 420
11

6
OA 2
(i) 
OC 5
OC 5

OA 2
 5  5
OC  OA  a
2 2
By ratio theorem,
 a  4b
OD 
5 E
 1 4
OD  a  b
5 5
 1 4 5 23 4
(ii) CD  a  b  a   a  b
5 5 2 10 5
5  23 4 
lCD : r  a     a  b    R
2  10 5 
(iii) Since E is a point on CD produced,
 5  23 4 
OE  a     a  b    R
2  10 5 
Since E is a point on OB,

OE   b   R

sgexamguru.com
 23
5 4 
a     a  b   b
2  10 5 
 5 23  4
    a  b   b
 2 10  5
5 23 25
  0 
2 10 23
4 20
   
5 23
 20
 OE  b
23
OE: EB = 20: 3

@PJC 2016 [Turn Over]


PJC P1 421
12

dT
  k T  To  , k  0
dt
1
 T  To dT   kdt Note that no modulus required
since T > T0
ln T  To   kt  C , where C is an arbitary constant
T  To  e  kt C
T  To  e  kt eC
T  To  Ae  kt , where A  eC
T  To  Ae  kt (shown)
To  30 oC
At t  0 : 100  30  Ae  k (0)
A  70
At t  15 : 70  30  70e 15 k
40  70e 15 k
4
e 15 k 
7
1 4
k   ln  0.0373077
15 7
To find time taken for pot of dessert to cool to at most 35C :
30  70e  kt  35
70e  kt
5
sgexamguru.com
5
e  kt 
70
5
 kt  ln
70
ln(5 / 70)
t
1
 ln(4 / 7)
15
t  70.74
t  71 minutes
It takes at least 71 minutes for the pot of dessert to cool to 35 oC and 30 minutes to cook.
Hence Nurul must start preparing the pot of dessert at 7.19pm the latest.

8
(i) Let L be the distance covered by the lion.

@PJC 2016 [Turn Over]


PJC P1 422
13

a  2.5 and d  0.05


n
L   2a   n  1 d 
2
n
  2  2.5   (n  1)  0.05  
2
1 101
  n2  n
40 40

(ii) Let P be the distance covered by the prey.


a  1.5 and r  0.95
1.5 1  0.95n 
P
1  0.95
 30(1  0.95n )
When n  , P  30
So the distance covered by the prey can never exceed 30m

(iii) In order for the lion to catch its prey,


L  P+25
1 101
 n2  n  30(1  0.95n )  25
40 40
1 101
 n2  n  30(0.95n )  55
40 40
1 sgexamguru.com
n  24,  n 2 
101
n  30(0.95n )  54.96  55
40 40
1 101
n  25,  n 2  n  30(0.95n )  55.822  55
40 40
1 101
n  26,  n 2  n  30(0.95n )  56.556  55
40 40
least n  25
Hence, the lion will need at least 25 leaps to catch its prey.

(iv) Let the initial distance be k


In order for the prey to escape the hunt,
P+k  L
1 101
30(1  0.9530 )  k   (302 )  (30)
40 40
23.561  k  53.25
k  29.689
 the shortest distance is 30 m.

@PJC 2016 [Turn Over]


PJC P1 423
14


(a) (i) t  tan
2
t 2 1

2 tan t
tan   2  2t
 1 t2 
1  tan 2 2
2 1
by triangle rule:
2t
sin  (shown)
1 t2
  
2 tan 2 tan 2sin
Alternatively RHS =
2t
= 2  2  2  cos 2   sin  =LHS
1 t 2
   2
1  tan 2 sec 2 cos
2 2 2
Alternatively
 
sin 2 tan
  2 cos 2   2  2t
Use double angle formula: sin  2sin cos  2
2 2  2 sec 2  1 t2
cos
2 2

(ii)

 tan 1 
2
 2
d t  tan

sgexamguru.com
sin   1 2
0

t1  2
1    2
  dt  when   : t  tan =1
2t
1 1 t
2
0
 2 2
1 t 2
0
t1  2 when   0 : t  tan =0
1  2
 dt 
0 2t  1  t 2  1  t 2
  
tan 1 t 
1 t 2
2
1 2  t  1 1 dt 1
 dt 
0 2t  1  t 2
1  t 2 d 2
1 2t  2 1 2
dt 1  t 2
 2 dt   dt 
0 t  2t  1 0 t 1
d 2
 2  ln(t  1) 0 d
1
2

 2 ln 2 dt 1  t 2

@PJC 2016 [Turn Over]


PJC P1 424
15

(b)
dy
u  e2v  cos(3v)
 e cos 3vdv
2v
dv
1 2 du 1
 e 2 v sin 3v   e 2 v sin 3vdv  2e2v y  sin(3v)
3 3 dv 3
1 2 1 2 
= e 2 v sin 3v    e 2 v cos(3v)   e 2 v cos(3v)dv 
3 3 3 3 
dy
u  e2v  sin(3v)
1 2 4 dv
= e 2 v sin 3v  e 2 v cos(3v)   e 2 v cos(3v)dv du 1
3 9 9  2e2v y   cos(3v)
13 2 v 1 2 dv 3
9  e cos 3vdv  e 2 v sin 3v  e 2 v cos(3v)
3 9
3 2v 2 2v
 e cos 3vdv  13 e sin 3v  13 e cos(3v)  c
2v

Alternatively
dy
 e cos 3vdv
2v
u  cos(3v)  e2v
dv
1 2v 3
 e cos 3v   e 2 v sin 3vdv du 1
2 2  3sin(3v) y  e2v
dv 2
1 3 1 3 
= e 2 v cos 3v   e 2 v sin(3v)   e 2 v cos(3v)dv 
2 2 2 2 
dy
u  sin(3v)  e2v
dv
sgexamguru.com
1 2v 3 2v 9 2v
= e cos 3v  e sin(3v)   e cos(3v)dv du 1
2 4 4  3cos(3v) y  e2 v
13 2 v 1 2v 3 2v dv 2
4
e cos 3vdv  e cos 3v  e sin(3v)
2 4
3 2v 2 2v
 e cos 3vdv  13 e sin 3v  13 e cos(3v)  c
2v

@PJC 2016 [Turn Over]


PJC P1 425
16

10
1  2
   
(i) 1 : r   0     1  ,   R
1  5 
   
Since (1, 0, 1) is on  1 and p1
 1 1
  
 0  3   a
 1 1
  
1 0 1  a
a  2 (shown)
(ii) Let N be the foot of perpendicular from A to p1
18  1
   
 AN : r   2     3  ,   R
0 1
   
 18   
  
let ON   2  3  for some value of 
  
 
Since N is a point on p1
 18     1 
   
sgexamguru.com
 2  3    3   2
   1
There is a typo in Q11, so the
correct
   
18    6  9    2 1  2 
  
OB    
24  11  2  1  5 
 
11  22
1  2  18   17  2 
  2       
AB       2    2   
18  2   16   1  5   0   1  5 
          
ON   2  6    4   N 16, 4, 2 
 2   2   2
    AB  294  72  30 2
(iii) Since B is on  1  2
AB must be minimum
1  2 
    2
OB      AB  30 2  72  294
 1  5   2
 
 d AB
1  2  18   17  2  2 AB  60  72
       d
AB       2    2     2
 1  5   0   1  5  d AB
      0
 d
AB   17  2    2     1  5 
2 2 2
60  72  0

AB   289  68  4 2    4  4   2   1  10  25 2  6
 5
AB  294  62  30 2
17 
 1  2   5   17 
   6  1 
OB      or  6 
 5 5 
@PJC 2016  1  5   [Turn Over]
   7   35 
 
PJC P1 426
17

 2
AB  294  62  30 2
For shortest distance from A to 1
 2
AB must be minimum
 2
 AB  30 2  62  294
 2
 d AB
2 AB  60  62
d
 2
d AB
0
d
60  62  0
31

30
 46 
1  2   15   92 
    31  1  
OB      or  31 
 30  30
  185 
 1  5   37 / 6   
 

 1   2   b   b 
       
(iv) direction vector of  2   0    b     1  2     1 
1 1  b   b 
       

sgexamguru.com
To find a common point between p2 and p3 by letting y  0 :
x  z  1 --- (1)
2 x  z  4 --- (2)
Solve (1) and (2):
x  3, z  2
 3  b 
   
Hence  2 : r   0     1  ,   R (shown)
 2  b 
   

11

@PJC 2016 [Turn Over]


PJC P1 427
18

(i) w  rei
w*  re  i

w 2  re 
i 2


w * re  i
r 2 ei2
  i
re
 rei3  3  3ei
 1
3      (0     )
3 2
r 3  n
i i
w  3e 3 , wn  3n e 3
n
wn is real ,   0,  , 2 ,... , so n = 3,6, 9,…
3

(b) (i) z5  1  i
 
 2 k   i
 2e  4

1  2 k  
  i
z2 e 10  5 20 
, k  0 , 1 , 2
1  1 9 1 7 1 17 1 3
i i  i i  i
z  2 e 20 , 2 e 20 , 210 e
10 10 20
, 210 e 20 , 210 e 4

So |z| = 2 sgexamguru.com
for all z
1
10

 9
7 17 3
arg(z) = , , , ,
20 20 20 20 4
1 17
(ii) i
z1  210 e 20
1 15
 i
z2  210 e 20
Let Point A and B represent z1 and z 2 respectively.
z1  z2  OAB is an isosceles triangle.
2 Im(z)
AOB 
5
1
OAB  OBA    AOB 
2
1 2  3
   
2 5  10
arg  z1  z2     OBA Re(z)
O
 15  3
   
 20  10 α
11

20

@PJC 2016 [Turn Over]


PJC P1 428
19

Im(z)

11
20

Re(z)
O

sgexamguru.com

@PJC 2016 [Turn Over]


PJC P1 429
Section A: Pure Mathematics [40 marks]

1 Two complex numbers a and b are given by 2  3i and 4  5i respectively.


(i) On a single Argand diagram, sketch the loci
(a) 2z  a  b  a  b ,

(b) 0 ≤ arg(z – b) ≤ arg(a – b). [4]

(ii) Find range of arg(z) where z is the complex number satisfies the relations in
part (i). [4]

ax
2 A curve C has equation y  where a  0 .
x 1
B
(i) By writing the equation of C as y  A , state a sequence of
x 1
1
transformations which transform the graph of y  to C. [3]
x
(ii) Sketch C, giving the equations of any asymptotes and the coordinates of any

(iii)
sgexamguru.com
points of intersection with the axes.
The region R is bounded by C, the lines x  2 , x  4 and y  a . Find the
[2]

exact volume in terms of a when R is rotated through 2 radians about the x-


axis. [3]

a
(iv) The region S is bounded by y  , the lines x  1 , x  3 and y  0 . State the
x
exact volume in terms of a when S is rotated through 2 radians about the
line y   a . [1]

Microsoft Word - Prelim Paper 2 questions and solutions for PJC 430
2

x
 if x  0,
3 The function f is defined by f( x)   2
2sin x if 0  x  4.
(i) Sketch the graph of y  f( x ) . [2]
(ii) If the domain of f is restricted to x  k , state the largest value of k , in exact
form, for which the function f 1 exist. [1]
(iii) Using the domain from part (ii), define f 1 in a similar form. [4]
(iv) Solve f 1 ( x)  f ( x) . [2]

In the rest of the question, the domain of f is as originally defined.


The function g is defined by g : x   x 3 , x  , x  0 .
(v) Find an expression for fg( x ) . [2]

 3 
4 (i) Differentiate tan 1  x  with respect to x. [2]
 2 

(ii)
sgexamguru.com
Find the binomial expansion for
1
3x  4
2
up to and including the term in x 6 ,

giving the coefficients as exact fractions in their simplest form. Find the set of
values of x for which the expansion is valid. [5]

(iii) Hence, find the first four non-zero terms of the Maclaurin series for
 3 
tan 1  x  . Give the coefficients as exact fractions in their simplest form.
 2 
[5]

@PJC 2016 [Turn Over]


PJC P2 431
3

Section B: Statistics [60 marks]

5 A pharmaceutical company has invented a new drug for diabetic patients and wishes
to carry out a trial of the new drug involving 5% of the patients from a local hospital.
(i) Explain how a systematic sample could be carried out. [2]
(ii) State one disadvantage of systematic sampling in this context and name a more
appropriate sampling method. [2]

6 Given that P  A | B '   3P  A | B  and P  B '  4P  B  .

4
(i) Show that P  B '  . [2]
5
(ii) Using P  A  B '  P  A  P  A  B  , find P  B ' | A  . [3]

7 The sales department of a company consists of 3 teams led by Mrs Wong, Miss Tan
and Mr Lim. Each team is made up of 1 team leader and 5 sales executives. The
sgexamguru.com
number of male and female sales executives within each team is given in the table
below:
Team A Team B Team C
Team Leader Mrs Wong Miss Tan Mr Lim
Number of Male Executive(s) 3 4 0
Number of Female Executive(s) 2 1 5

A taskforce is to be formed by selecting 7 representatives from the 18 members of the


department. Find the number of different taskforces that can be formed if the taskforce
must include
(i) Miss Tan and 1 other team leader, [2]
(ii) more females than males, [2]
(iii) at least 1 representative from each team. [3]

@PJC 2016 [Turn Over]


PJC P2 432
4

8 In this question you should state clearly the values of the parameters of any normal
distribution you use.

The mass in kilograms of an Atlantic salmon is a normally distributed continuous


random variable X with mean  and standard deviation  .

(i) It is known that P  X  22   0.159 and P  X  31  0.106 . Show that

  26.0 and   4.01 . [3]


(ii) In a random sample of 40 Atlantic salmon, estimate the probability that at least
35 of them have a mass of at most 31 kilograms. [3]

It is also known that the mass in kilograms of the Bluefin tuna has the distribution
N  380,10 2  .

(iii) Find the probability that the average mass of 2 randomly chosen Bluefin tuna

sgexamguru.com
and 3 randomly chosen Atlantic salmon is at most 170 kg. [2]

@PJC 2016 [Turn Over]


PJC P2 433
5

9 The table gives the world record time, in seconds, for the 100 metres free style
swimming event at the Olympic Games in the past years.

Year, x 1908 1920 1956 1968 1976 2000


Time, t 65.60 60.40 55.40 52.20 49.99 48.18

(i) Draw a scatter diagram to illustrate the data. [1]


(ii) Comment on whether a linear model would be appropriate, referring both to
the scatter diagram and the context of the question. [2]
(iii) It is thought that the time can be modelled by one of the formulae ln t  a  bx
1
or  a  bx . Find, correct to 4 decimal places, the value of the product
t
moment correlation coefficient between
(a) ln t and x,
1
(b) and x. [2]
t
1
(iv) Use your answers to part (iii) to explain which of ln t  a  bx or  a  bx is

sgexamguru.com
t
the better model. [1]
(v) The time corresponding to 1964 was added to obtain the equation with
appropriate model chosen in part (iv) where a  0.09836 and
b  5.96846  10 5 . Find the time in 1964. [2]

@PJC 2016 [Turn Over]


PJC P2 434
6

10 A car manufacturer launches a new car model “Green Leaf” that is marketed to be
environmentally friendly. It is claimed that the carbon emission of the “Green Leaf” is
at most 80 g/km. The transport authority suspects that the figure is understated, and
requests the manufacturer to submit test data from 20 units of the “Green Leaf”. The
test data submitted is as follows.

Carbon Emission (g/km) 78 79 80 81 82


No. of units 2 3 6 4 5

(i) Calculate unbiased estimates of the population mean and variance. [2]
(ii) Stating a necessary assumption, test at the 10% level of significance whether
there is any evidence to doubt the manufacturer’s claim. [4]

The transport authority subsequently decides to conduct their own test, and invites 10
owners of the “Green Leaf” to form a sample. The mean and variance of this sample
is found to be 80.6 g/km and m2 g2/km2 respectively.

(iii)
sgexamguru.com
Find the set of values of m for which the result of the test would be to reject
the manufacturer’s claim, at the 1% significance level. [3]

@PJC 2016 [Turn Over]


PJC P2 435
7

11 There are 2 main types of T-cells in the human body. T4-cells are “helper” cells that
lead attacks against infections in the human body, while T8-cells are “suppressor”
cells that kill cancer and virus infected cells in the human body. It is to be assumed
that the number of T4-cells per 0.01 mm3 of blood can be modelled by the distribution
Po(5) and the number of T8-cells per 0.01 mm3 of blood can be modelled by the
independent distribution Po(1.5).

A patient is considered healthy if he or she has at least 4 T4-cells and at least 1 T8-
cells in 0.01 mm3 of blood.

(i) Find the probability that a randomly selected patient is healthy. [3]
(ii) Find the probability that only 1 out of 3 randomly selected patients is healthy.
[2]

A patient is susceptible to infections if his or her T4-cells count falls below 3 per 0.01
mm3 of blood.

(iii)
sgexamguru.com
Use a suitable approximation, which should be stated, to find the probability
that, in 100 randomly selected patients, the number of patients susceptible to
infections is between 20 and 50 inclusive. [4]

12 Mr Ouyang, a car manufacturer, finds that on average, 2% of his cars have faulty
gearboxes. On a particular occasion, he selects n cars randomly for inspection, and the
number of cars with faulty gearbox is denoted by the random variable C.
(i) State in context of this question, what must be assumed for C to be well
modelled by a binomial distribution. [2]
(ii) Given that n  20 , find the probability that C is between 2 and 6. [2]
(iii) The probability that there are less than 2 cars with faulty gearbox in a sample
of n cars is at most 0.95. Write down an inequality in terms of n, and find the
least possible value of n. [3]
(iv) Mr Ouyang selects 100 batches of 20 cars. Estimate the probability that the
average number of cars with faulty gearbox per batch is at least 0.3. [3]

@PJC 2016 [Turn Over]


PJC P2 436
8

Pioneer Junior College


H2 Mathematics
JC 2 Preliminary Examination Paper 2 Solution
1
(i) 2z  a  b  a  b
(2  3i)  (4  5i)
Centre of circle C =  1  i
2
(2  4) 2  (3  5) 2
Radius of circle C = 5
2

Im(z)

(1,1 Re(z)
5 )

B
sgexamguru.com M

(ii)AM is the common region satisfies both (i) and (ii).


AMB = 90o since AB is diameter and angle in semicircle is a right angle
Max arg(z) = arg(a) = tan1(3/2) = 0.983 radians
Min arg(z) = arg(2 – 5i) = tan1(5/2) = 1.19 radians
So range required is 1.19 ≤ arg(z) ≤ 0.983

@PJC 2016 [Turn Over]


PJC P2 437
9

2
axx a
(i) y a
x 1 x 1
1
y  is i translated d 1 unit in thhe directionn of x-axis, followed
f byy a scaling of
o a
x
units paarallel to thee y axis andd is translateed a units in
n the directioon of y-axiss
(ii) The equuations of asymptotes aare x = 1 an nd y = a
The inteercepts are (0,0)

y=a

x=1

(iii)) sgexamguru.com 2


4
The vollume requirred =    a 
a 
2

 dx   (a) (2))
x 1 
2

4 2 1 
= a 2  1   2 
dx  2 a 2
2
 x  1 ( x  1) 
4
 1 
= a   x  2 ln | x  1| 
2
  2 a 2
 x  1 2
2 
=   2 ln 3   a 2
3 
(iv) Using part
p (i), the area
a S is thee same as th he area R found in (iii).. To rotate S about
the linee y = a is th
he same as to rotate R about the x--axis. So thhe volume obtained
o
2 
is   2 ln 3   a 2
3 

@PJC 22016 [Turrn Over]


PJC P2 438
10

3
(i)


( , 2)
2

(4,  1.51)

(ii) 
Frrom the graaph, take k 
2

(iii) x
 2 if x  0
f( x )  
 2 sin x if 0  x  
 2
x

sgexamguru.com
Let y1 
2
x  2 y1
Let y2  2ssin x
y 
x  sinn 1  2 
 2 
 2x if x  0
1 
f ( x )   1  x 
ssin  2  iff 0  x  2

(iv) f 1 ( x)  f ( x) is the sam


me as solvinng f( x) = x
x
= x  x  0 if x  0
2

22sin x = x, x = 1.90 > , so only solution is x = 0
2
(v)  x3
Using R g   , 0  , fg(f x) 
2

@PJC 22016 [Turrn Over]


PJC P2 439
11

4
(i)
d  1  3   1  3
 tan  x  2 
dx   2 
  3   2 
1  x
 2 
 3 1
 
 2  1  3 x2
4
 3 4
  2
 2  3x  4
2 3

3x 2  4
(ii)
1
1 2 1 1  3x 2 
 (3 x  4)  4 
 1  
3x 2  4  4 

1  
2 3
 3 x 2  1 2   3 x 2  1 2  3  3 x 2 
 1   1           ...
4  4  2  4  6  4  
 
1 3 9 27 6 
4 4
1 3
  x2 
sgexamguru.com
 1  x 2  x 4 
16
9 4 27 6
x 
64
x  ... 

x

4 16 64 256
3x 2 4 2 2
Range of validity:  1  x2    x
4 3 3 3
(iii)
 3  1
y  tan 1  x   2 3  2 dx
 2  3 x  4
1 3 9 27 6 
 2 3    x2  x4  x  dx
 4 16 64 256 
1 1 9 5 27 7 
 2 3  x  x3  x  x C
4 16 320 1792 
when x  0 , y  0 , C  0
 3  3 3 3 9 3 5 27 3 7
 tan 1  x   x x  x  x
 2  2 8 160 896
5 (i) Number the list of patients from 1 to N. k = N/0.05N =20 (Randomly select a number
from 1 to 20, and let every 20th patient after first patient chosen try the new drug. For
example, if a number 5 is chosen, then survey every 5th, 25th, 45th patient and so on, until the
sample size of 5% patients is obtained.

@PJC 2016 [Turn Over]


PJC P2 440
12

(ii) Disadvantage: The sample is not representative of the population of diabetic


patients as age and gender may affect the drug. More appropriate method is
stratified sampling.
6
(i) P  B'   4P  B  Alternatively,
1  P  B   4P  B 
P  B   4 1  P  B '  
'
5P  B   1
P  B   4  4P  B 
' '
1
P  B 
5P  B   4
'
5
4
P  B'  
4
(shown) P  B '   (shown)
5 5
(ii) P  A | B '   3P  A | B 
P  A  B'  3P  A  B 

PB  '
P  B

P  A  B'  3P  A  B 

PB  '
1  P  B' 
P  A  B '  3P  A  B 

4 1
5 5
P  A  B '   12 P  A  B     (*)

sgexamguru.com
P  A  P  A  B   12 P  A  B 
P  A  13P  A  B     (**)
P  A  B' 
P  B ' | A 
P  A
12 P  A  B 

13P  A  B 
12
 or 0.923 (3 s.f.)
13
7 (i)Number of teams  2C1 15C5  6006
(i) 4 cases: 4F3M, 5F2M , 6F1M and 7F
Number of teams  10C4 8C3  10C5 8C2  10C6 8C1  10C7  20616
(ii) Total  teams from A and B  teams from B and C  teams from A and C

 18C7  12C7  3
 29448

@PJC 2016 [Turn Over]


PJC P2 441
13

8 (i) X – mass in kilograms of an Atlantic salmon


X  N   , 2 
P  X  22   0.159
 22   
P Z   0.159
  
22  
 0.99858

  0.99858  22 ---(1)
P  X  31  0.106
 31   
PZ   0.106
  
31  
 1.2481

  1.2481  31 ---(2)
Solving (1) and (2):
  26.00022  26.0 (shown)
  4.00591  4.01 (shown)
(ii) Let W be the number of Atlantic salmon with more than 31 kg, out of 40
W~B(40, 0.106)
n = 40 large, np = 40(0.106) = 4.24 < 5

sgexamguru.com
so W~ Po(4.24) approx.
Required prob = P(W ≤ 5) = 0.74659  0.747
(iii) Y – mass in kilograms of an Bluefin tuna.
Y  N  380,102 
Let T be the mass of 2 Bluefin tuna and 3 Atlantic salmon
T  X 1  X 2  X 3  Y1  Y2  N  838, 248.2403
X 1  X 2  X 3  Y1  Y  838 248.2403 
 T  N ,  exactly
5  5 25 
 
P T  170  0.777 (3 s.f.)
Alternatively
P(T  170  5)  0.777

@PJC 2016 [Turn Over]


PJC P2 442
14

9
(i) t

(ii) Thee time for sw wimming caannot decreaase forever as there is a limit on hoow fast a sw wimmer
can swiim and from m the scatterr diagram, aas x increasees, t decreasses with deccreasing am
mount, so
linear m model is nott appropriatee.
(iii)
lnt = a + bx : r = 0.9851

1
 a  bx : r = 0.99877
t
1 1
(iv) Sinnce |r| for  a  bx is higher thann that of lnt = a + bx,  a  bx iss the preferrred
t t
model.
(v) Let the timing be b t
1
 0.09836   5.96846 105  x

sgexamguru.com
t
 1
quation is  x,    .
Only vaalue that sattisfies the eq
  t 
 
1
 0.09836   5.96846 105  x  00.09836  0.000059684 46(1956)  00.018383
t
1 1 1 1 1 1 1 1
So (       )  0.018383 3 =
7 665.6 60.4 55.4 52.2 49.99 48.18 t
t  552.87  52.99
So the ttiming at 19964 is 52.9 second
s
10
(i) Let X be thhe carbon em
mission of ““Green Leaf”.
From GC, unbiased
u estimate of poopulation mean
m = x  80.35 ,
vvariance = s  1.3089 9   1.71322
2 2
Unbiased estimate
e of population
p
(ii) Since n is small
s and  2 is unknow wn, we use the
t t-test.
Assumptionn: The carbon emissionn of the “Grreen Leaf” is i normally distributed.
H 0 :   800 vs H1 :   800
Test Statisttic, t  1.195
59
From GC, p- p value = 0.12323 > 0. 1

Since the p-value is more


m than thee level of siignificance, we do not reject H 0 . There is
insufficientt evidence, at the 10% level, to ind dicate that the
t manufaccturer’s claiim is not
true.
(iii) Since n is small
s and  2 is unknow wn, we use thet t-test.

@PJC 22016 [Turrn Over]


PJC P2 443
15

For H0 to be rejected, Test Statistic  2.8214


10 2
Unbiased estimate of population variance s 2  m
9
80.6  80
Test Statistic, t   2.8214
m/3
m  0.638
11 (i) X – number of T4-cells in 0.01 mm3 of blood
X  Po  5 
Y – number of T8-cells in 0.01 mm3 of blood
Y  Po 1.5 
P  healthy   P  X  4  P Y  1
 1  P  X  3  1  P Y  0  
 0.57098  0.571 (3 s.f.)
3!
=  P  healthy    P  unhealthy   
2
(ii) Req prob
2!
3!
=  0.570981  0.57098    0.315 (3 s.f.)
2

2!
Alternatively,
A – number of patients who are healthy out of 3 patients
A  B  3, 0.57098 
P  A  1  0.31528  0.315 (3 s.f.)
(iii) sgexamguru.com
P  susceptible 
   
P X 3 P X 2
 0.12465
W – number of patients who are susceptible to infection out of 100 patients
W  B 100, 0.12465 
Since n is large and
np  (100)(0.12465)  12.465  5 and n(1  p )  (100)(1  0.12465)  87.535  5
W  N 12.465,10.911 approx
P  20  W  50 
 P 19.5  W  50.5 (c.c)
 0.016595  0.0166 (3 s.f.)
12 (i) The occurrences of faulty gearbox must be independent of one another
The probability of a faulty gearbox is constant
(ii) C – number of cars that has gearbox issues out of 20 cars
C  B  20, 0.02 
P  2  C  6   P  C  5  P  C  2 
 0.0070667  0.00707 (3 s.f.)
Alternatively,
P  2  C  6   P  C  3  P  C  4   P  C  5 
 0.0070667  0.00707 (3 s.f.)

@PJC 2016 [Turn Over]


PJC P2 444
16

(iii) C  B  n, 0.02 
P  C  2   0.95
P  C  1  0.95
P  C  0   P  C  1  0.95
C0  0.02   0.98   nC1  0.02   0.98 
0 n 0 1 n 1
n
 0.95
 1 
 0.98  n    0.98   0.95
n n

 49 

 0.98 1    0.95


n n
 49 
 n 
 0.98 1    0.95  0
n

 49 
n  18.0977
Hence the least number of cars Mr Ouyang has to sample is 19
Alternatively, By GC using table
n  0.98 1  
n n
 
 49 
17 0.95541
18 0.95049
19 0.94538

(iv) sgexamguru.com
Hence the least number of cars Mr Ouyang has to sample is 19.
C – number of cars that has gearbox issues out of 20 cars
C  B  20, 0.02 
E(C) = 0.4 Var(C) = 0.392
 0.392 
Since n = 100 large, by CLT, C ~ N  0.4,  approx.
 100 
P(C  0.3)  0.945

@PJC 2016 [Turn Over]


PJC P2 445
 

RA
AFFLES INSTITUTION
  
 
  201
16 YEAR
R 6 PRE
ELIMINA
ARY EX
XAMINA
ATION 
 
         

MATHEMATICS PAPER
R1 9740/0
01

Higherr 2 MBER 201


19 SEPTEM 16
Total M
Marks: 100 3 hourrs

Additional materialss: Answer Paper


P
List of Fo
ormulae (MF
F15)

READ T
THESE INST
TRUCTION
NS FIRST

Write your name an nd CT group p on all the work you hand in.
Write in dark blue or
o black pen n on both sid
des of the paper.
p
You mayy use an HB B pencil for any diagramms or graph hs.
Do not uuse staples, paper clips ers, glue or correction fluid.
s, highlighte f

n-exact num
sgexamguru.com
Answer all the quesstions.
Give non merical answ wers correcct to 3 signifficant figure
es, or 1 deccimal place in the case of
angles in
n degrees, unless
u a diffferent levell of accurac
cy is specifie
ed in the quuestion.
You are expected too use an appproved gra aphing calcuulator.
Unsuppo orted answeers from a graphing
g ca
alculator aree allowed unless
u a queestion spec
cifically states
otherwisse.
Where u unsupportedd answers from a gra aphing calcu ulator are not
n allowedd in a ques stion, you are
a
required to presennt the math hematical ssteps using g mathema atical notatiions and not
n calculattor
comman nds.
You are reminded of
o the need for clear prresentation in your answers.

At the en
nd of the tesst, fasten all your workk securely to
ogether.
The nummber of marks is given in brackets [ ] at the en nd of each question
q or part question.
 
 
 
 
 
 
 
 
 
 
 

This
T docum
ment consists of 6 printe
ed pages.
 
RAF
FFLES INSTIT TUTION
partment 
Mat hematics Dep
 RI2016 [Turn over

Microsoft Word - RI_JC2_H2Maths_2016_Prelim_P1 446


    2
 

1 It is given that f  x   ax3  bx 2  cx  d , where a, b, c and d are constants.


The graph of y  f  x  passes through the point with coordinates  1,  27  and has a
turning point at  2, 27  . Given also that f ''  0  0, find f  x  . [4]

2 In a laboratory experiment, an empty 10-litre tank is transported back and forth between
station A and station B by a machine.

Starting at station A, 1000ml of water is added to the tank and on subsequent visits, 90%
of the amount of water added previously is added to the tank, that is, 900ml on the 2nd
visit, 810 ml on the 3rd visit and so on.

At station B, the machine removes 100ml of water from the tank and on subsequent
visits, it removes 50ml more water than the previous visit, that is, 150ml on the 2nd visit,
200ml on the 3rd visit and so on.

(i) Show that the amount of water in the tank after the 3rd visit to station B is 2260ml.
[1]

The machine stops when the amount of water to be removed exceeds the amount of water
present in the tank.

(ii)
sgexamguru.com
Determine the amount of water in the tank when the machine stops.
Leave your answer to the nearest millilitre. [3]

3 The even positive integers, starting at 2, are grouped into sets containing 1, 3, 5, 7, ...
integers, as indicated below, so that the number of integers in each set is two more than
the number of integers in the previous set.

{2}, {4,6,8}, {10,12,14,16,18}, {20,22,24,26,28,30,32}, …

Find, in terms of r, an expression for

(i) the number of integers in the rth set, [1]

(ii) the last integer in the rth set. [2]

Given that the nth set contains the integer 2016, find n. [2]

RI P1 Questions 447
    3
 

4 (i) Use integration by parts to show that for any real constant a,

e x  sin ax  a cos ax 
e sin ax dx   c,
x

1  a2

where c is an arbitrary constant. [4]

(ii) By expressing sin 2 x cos x in the form A  sin Px  sin Qx  , for real constants A, P
and Q, find  e x sin 2 x cos x dx. [2]

5 [A right circular cone with base radius r , height h and slant height l has curved surface
area  rl. ]

A right circular cone of base radius r is


designed to contain a sphere of fixed
radius a. The sphere touches both the
curved surface and the base of the cone.
(See diagram for a cross-sectional view.)
O
The point O is the centre of the sphere,
a

sgexamguru.com
the point B is on the circumference of
the base of the cone, the point P is the
B

centre of the circular base of the cone r P
and  is the angle OB makes with the
base.
r 2  a2
(i) Show that cos 2  . [2]
r 2  a2

(ii) Use differentiation to find, in terms of a, the minimum total surface area of the
cone (consisting of the curved surface area and the base area), proving that it is a
minimum. [6]

6 Do not use a calculator in answering this question.

(i)
2
3

For y  2cos  cos1 x  , show that 1  x 2

d2 y
dx 2
xdy
dx
4

  y . Hence find the
9
Maclaurin series for y, up to and including the term in x 2 . [4]

(ii) Given that the first three terms found in part (i) are equal to the first three terms in
the series expansion of 1  bx  , find the values of the constants b and n .[4]
n

[Turn over

RI P1 Questions 448
    4
 

7 The complex numbers z and w are such that

z  1  i  3 and w    2  ic

z
where c is real and positive. It is given that  1.
w
(i) Find the exact value of c. [2]

 z  11
(ii) Show that arg    . [2]
 w  12

z
(iii) Express in the form x  iy , where x and y are real, giving the exact values of
w
x and y in non-trigonometrical form. [2]

z
(iv) Hence, by considering the complex number on an Argand diagram, show that
w
 5 
tan    2   3. [2]
 12 

8
sgexamguru.com
The function f is defined by

f : x  x2   x  7 , x  , x  3,

where  is a constant.

(i) State the range of values that  can take if f 1 exists. [1]

It is given that   6.  


(ii) Find f 1  x  and state the domain of f 1 . [3]

(iii) Sketch on the same diagram the graphs of y = f  x  and y = f 1  x  . [3]

(iv) Write down the equation of the line in which the graph of y = f  x  must be
reflected in order to obtain the graph of y = f 1  x  . Show algebraically that the
solution to f  x   f 1  x  satisfies the equation x 2  7 x  7  0.
Hence find the exact value of x that satisfies the equation f  x   f 1  x  . [3]

RI P1 Questions 449
    5
 

9 A curve C has parametric equations

x  t2, y  1  2t for t  0.

(i) Sketch C. [2]

(ii) Find the equations of the tangent and the normal to C at the point P( p2 ,1  2 p).
[4]

(iii) The tangent and normal at P meet the y-axis at T and N respectively.
PT 2
Show that  p. [4]
TN

a( x  1)( x  2)
10 The curve C has equation y  .
2  3x

(i) The curve C is scaled by a factor of 3 parallel to the x-axis to get the curve C '.
Given that the point (4,1) lies on C ' show that a  9. [2]

For the rest of the question, use a  9.


(ii) Obtain the equations of the two asymptotes of C. [2]

(iii) Sketch C, stating the coordinates of any turning points and of the points where the

(iv)
sgexamguru.com
curve crosses the axes. [4]

Without using a calculator, find the range of values of  for which the line
y  9 x   and C have at least one point in common. [3]

11 The line l1 passes through the point A, whose position vector is i  2 j , and is parallel to
the vector i  k. The line l2 passes through the point B, whose position vector is
i  j  3k , and is parallel to the vector j  k .

(i) Show that the lines l1 and l2 are skew. [2]

(ii) Find the position vector of the point N on l2 such that AN is perpendicular to l2.
[3]

The plane  contains l2 and is perpendicular to AN.

(iii) Find a vector equation for  in the form r  u   v   w , where v and w are
perpendicular vectors. [3]

(iv) The point X varies in such a way that the mid-point of AX is always in  . Find a
vector equation for the locus of X. Describe this locus and state its geometrical
relationship with the plane . [4]

[Turn over

RI P1 Questions 450
    6
 

12 (a) By using the substitution y  2ux 2 , find the general solution of the differential
dy
equation 2 x 2  4 xy  y 2  0, where x  0. [4]
dx

(b) A glass of water is taken from a refrigerator and placed in a room where the
temperature is a constant 32oC. As the water warms up, the rate of increase of its
temperature  oC after t minutes is proportional to the temperature difference
 32   oC. Initially the temperature of the water is 4oC and the rate of increase of
the temperature is 2oC per minute.
1
 t
By setting up and solving a differential equation, show that   32  28e 14
.
[6]

(i) Find the time, to the nearest minute, it takes the water to reach a
temperature of 20oC. [1]

(ii) State what happens to  for large values of t. [1]

(iii) Sketch a graph of  against t. [2]

sgexamguru.com
******* End of Paper *******

RI P1 Questions 451
Prelim Paper 1 Soolutions

Qn. Soluttion
1 f  x   ax  bx  cx  d
3 2

f '  x   3ax 2  2bx  c


f ''  x   6 ax  2b
f ''  0   0  2b  0  b  0
f  1  277   a  c  d   27
f  2   27  8a  2c  d  227
f '  2  0  12a  c 0
Use GC to obtain a  2 , c  24 aand d  5 .
U
So f  x   2 x3  24 x  5 .
S
Qn. Solu
ution
2(i) 1000  100   9000 150   8810  200  2260
(ii) Solution 1
Find thee least n whhen
1000 1  0.9  n
n

  2 100    n  1 50 
1  0.9 2

sgexamguru.com
10000 1  0.9   25n  n  3
n

Least n=
=17,
Amountt of water iss 8332.3  77600  732  nearest ml
OR Sollution 2
After ntth visit to Station
S A, ((so n  1 vissits to Station B) the aamount of water
w in
tank is
1000 1  0.9n  n  1
1  0.9

2
 2 100    n  2  500 
 100000 1  0.9n   25  n  1 n  2 
Amountt that iss to be removed at nth visit to Station B is
100   n  1 50  50
0  n  1

Microsoft Word - RI_JC2_H2Maths_2016_Prelim_P1_Solutions (updated) 452


Amountt of water in
n the tank w
when machin
ne stops is 732
7 ml.

OR Solution 3
Let un be unt of waterr after nth visit
b the amou v to Statio
on B
un  un1  1000  0.9n1  50(n  1), u0  0
(i) u1  900, u2  1650, u3  22260
(ii) Froom GC, u16  546.98, u117  167.7
Thee required amount
a water  u16  1000  0.9171  732 mll
of w
Qn. Sollution
3(a)(i)) Numberr of integerss in rth set  1  ( r  1)2
2  2r  1
1  2r  1
(ii) Total nuumber of integers in r ssets  (r )  r 2
2
s  2  (r  1)2  2r 2
Last integer in rth set 2

(iii)
sgexamguru.com
2  n  1  2016  2n 2
2

 n  1  1008  n2
2

n  32
Qn. Soolution
4(i)
 e sin ax dx  e siin ax  a  e cos ax dx
x x x


 e xsiin ax  a e x cos ax  a  e x sin ax dx   
 e xsiin ax  ae x coos ax  a 2  e x sin ax dx

1  a   e
2 x
sin ax dx  e x sin ax  ae x cos ax
a
e x sin ax  a coos ax   
 e sin ax dx  
x
c shown)
(s
1  a2
ex 1
OR:  e sin ax dx   cos aax   e x co
x
os ax dx
a a
ex 1  ex 1 
  cos aax   siin ax   ex sin ax dx 
a a a a 
x x
e e 1
  cos aax  2 sin axx  2  e x sin ax dx
a a a
a  e sin ax dx   ae cos ax  e sin ax   e sin ax dx
2 x x x x

RI P1 Solutions 453
1  a   e
2 x
sin ax dx  e x sin ax  ae x cos ax
e x  sin ax  a cos ax 
 e sin ax dx   c (shown)
x

1  a2
(ii) 1 1
sin 2 x cos x   sin 3 x  sin x  i.e. A  , P  3, Q  1
2 2
1
 e sin 2 x cos x dx  2  e  sin 3x  sin x  dx
x x

   e x sin 3 x  e x sin x  dx
1
2
1  e  sin 3 x  3cos 3x  e  sin x  cos x  
x x

   c
2 1  32 1  12 
e x  sin 3x  3cos 3x sin x  cos x 
    +c
2 10 2 
ex
  sin 3x  3cos 3x  5sin x  5cos x   c
20

Qn. Solution

sgexamguru.com
O

B

5(i) cos 2  2 cos 2   1


2
 r  2r 2 r 2  a2 r 2  a2
 2  1    [ shown]
 2  2
 2 2
 2 2
 2
 r a
2 r a r a r a

OR
2 2
 r   a  r 2  a2
 
cos 2  cos   sin   
2 2     [ shown]
 2 2   2  2
 2
 r a   r a
2 r a

(ii) Let T be the total surface area of the cone.

RI P1 Solutions 454
T   rl   r 2
 r 
r r
2
 cos 2 
 r 2  a 2  2 r 4
  r2   1 
 r 2  a2  r 2  a2
 

dT 8 r

 r 2  a2   2 r 4  2r 
3

dr
 r 2
 a 
2 2

4 r 3  r 2  2a 2  4 r 3
2
 or r  2a  r  2a 
r  a  r  a 
2
2 2 2 2

dT
For  0 , r  2a since r  0
dr
4 r 3
By the First Derivative test, since  0 , we have 
r 2
a 
2 2

r
 2a 

 2a   2a 

r2
 2a2 

 2a2   2a2 

sgexamguru.com
r 2  a2 0 0 0
dT -ve 0 +ve
 
dr

Alternatively, by the Second Derivative Test,


d 2T (r 2  a 2 ) 2 (5r 4  6a 2 r 2 )  r 3 (r 2  2a 2 )[2(r 2  a 2 )(2r )]
 4
dr 2 r  2 a (r 2  a 2 )4 r 2a

(a ) (20a  12a )  0
2 2 4 4
 4  32  0
(a 2 ) 4

   8 a2 .
4
2 2a  
Hence r  2a gives minimum T 
 2a   a 2
2

Qn. Solution
6(i) 2 
y  2 cos  cos 1 x 
3 
dy 2 2 1 
 2sin  cos 1 x    
dx 3  3  1  x 2 

dy 4  2 
1  x2  sin  cos 1 x 
dx 3  3 

RI P1 Solutions 455
d2 y 1 dy 4 2 2 1 
1  x2   2 x   cos  cos 1 x    
dx 2 2 1  x2 dx 3 3  3  1  x 2 


1  x2  dx2  x ddyx   94 y
d2 y

OR “implicit differentiation”
y 2
cos 1  cos 1 x
2 3
1 1 dy 2  1 
  
2 2 dx 3  
 1 x
2
 y 
1  
2  
16   y  
2 2
 
2  dy 
1  x    1    
 dx  9  2 
 
  2
 
2
2 d y  dy   dy  16  y  dy
2 1 x    2x      
 dx 2   dx   dx  9  2  dx
    
 2   dy 
1 x  
2 d y

 x
 dx 2   dx 
4
 y
9
 

sgexamguru.com
 
dy 2 3 d 2 y 4
At x  0 , y  1,  , 
dx 3 2 9
dx
2 3 2  
The series is y  1  x  x 2 
3 9
(ii) n  n  1 2 2
1  bx n  1  nbx  b x 
2
2 3
x: nb     (1)
3
n  n  1 2 2
x2 : b     (2)
2 9
(2) n 1 3
: b    (3)
(1) 2 9
(1) 2n 3
:  6  n 
(3) n 1 4
2 3 4 8 3
(1) : b  
3 3 9
   

RI P1 Solutions 456
Qn. Solution
7(i) z
Given  1,
w
2
1
2  c2
c 2  2  c  2 (  c is positive)
(ii) z
arg    arg( z )  arg( w)  2
 w
 3
   2
3 4
11
 (shown)
12
(iii) z 1 i 3 ( 1  i) 1  3  i( 3  1)
  
w 2( 1  i) ( 1  i) 2 2
( 3  1) ( 3  1)
 i
2 2 2 2
(iv) From the diagram,

sgexamguru.com
z
Re( )
 5  w
tan   
 12  Im( z )
w
3 1 3 1
 
2 2 2 2
3 1 3 1
 
3 1 3 1
42 3

2
 2  3 (shown)
Qn. Solution
8(i) f ( x)  x   x  7
2


f '( x)  2x    0  x   is the x-coodinate of the min. point
2

Hence, f 1 exists when   3    6
2

RI P1 Solutions 457
Alternative solution:
 
2

2
f ( x)  x 2   x  7   x    7
 2 4  

Inverse of f exists when   3    6.
2
(ii) Given that   6 ,
f : x  x2  6 x  7 , x  , x  3
R f   2,  

For f 1  x  , let y  x 2  6 x  7   x  3  2
2

 y  2   x  3  x  3  y  2
2

Since Df   ,3  Rf -1
f 1  x   3  x  2
 
(iii) y     

sgexamguru.com
 

       

7
x

   

(iv) y  x 
To solve f  x   f 1  x  , we can use f  x   x with x  3
 x 2  6 x  7  x  x 2  7 x  7  0 [AG] [shown]
7  49  28
x
2
       
7  21
 x  3, x 
2

RI P1 Solutions 458
Qn. So
olution
9(i)

(ii) dx dy
x  t2   2t y  1  2t  2
dt dt
dy dy dx 2 1
 /  
dx dt dt 2t t
Equatiion of tangeent at P( p ,,1  2 p) ,
2

1
y  (1  2 p)  (x  p 2 )
p
1
y 1  2 p  x  p
p
1
y  x 1 p
p

sgexamguru.com
mal at P( p ,,1  2 p) :
Equatiion of norm
2

y  (1  2 p)   p( x  p 2 )
y  1  2 p   px  p3
y   ppx  p 3  1  2 p
(iii) 1
Subs. x  0 into y  x  1  p , we haave y  1  p
p
 T is (0, 1  p )
Subs. x  0 into y   px  p 1  2 p , y  p 1 2p
3 3

 N is (0, p 1 2 p)
3

Given that P is ( p ,1  2 p)
2

PT 2  ( p 2  0) 2  (1  2 p  1  p ) 2
 p4  p2
TN  ( p 3  1  2 p )  (1  p )
 p3  p
 p3  p ( p  0)

PT 2 p 4  p 2 p 2 ( p 2  1)
 3  p (shown
n)
TN p p p ( p 2  1)

RI P1 Solutions 459
Qn. Solution
10(i) If the point (4,1) lies on the curve which is C transformed by a stretch with scale
4 
factor 3 parallel to the x-axis, then  ,1 is on C.
3 
4
Subs. x  and y  1 into C,
3
 4  4   1  2 
a   1  2  a    
1 
3  3   1   3  3 
4 2
2  3 
3
a9
(ii) 9 x 2  27 x  18 4
y  3 x  7 
2  3x 2  3x  
2
Vertical asymptote, x 
3
Oblique asymptote, y  3 x  7
(iii) y

sgexamguru.com
x

(iv) 9( x  1)( x  2)
 9x  
2  3x
9 x 2  27 x  18  18 x  2  27 x 2  3 x
36 x 2  3(  15) x  2(9   )  0
Since the line y  9 x   and C have at least one point in common,
b 2  4ac  0
[3(  15)]2  4(36)(2(9   ))  0
 2  30  225  288  32  0
 2  2  63  0
(  9)(  7)  0
  9 or   7

RI P1 Solutions 460
Qn. Solution
11(i)  1   1  1  0
       
l1 : r   2   s  0  , s   l2 : r   1   t  1  , t  
 0  1  3  1 
       
If l1 and l2 intersect,

1  s  1  (1)
2  1  t  (2)
s  3  t  (3)
from (1), s = 2
from (2), t = 1
but for (3), LHS = 2  3 + 1 = 4 = RHS
Hence l1 and l2 are non-intersecting.
1 0
Since  0   k  1  for any k   , l1 and l2 are non-parallel.
 
1 1
   

Thus the lines l1 and l2 are skew.


(ii) 1  0
    
Since N is on l2, ON   1   t  1  for some t   .
 3  1 
   
sgexamguru.com
 1   0   1   2 
        
AN   1   t  1    2    1  t 
 3  1   0   3  t 
       
0
  
AN .  1   0
1
 
 2  0
  
  1  t  .  1   0
 3 t  1
  
 1  t  3  t  0
 t  1
1  0  1 
      
 ON   1    1    0 
 3  1   2 
     
(iii) 1 0
Let u   1  , v   1  .
 
 3 1
   

RI P1 Solutions 461
0  2
    
w is perpendicular to v   1  and also to AN   2  .
1  2
   
0  1   2 
Consider  1    1   1  .
 1   1   1 
     
2
Let w   1  .
 1 
 
1  0 2
     
 : r   1     1     1  ,  ,   
 3 1  1
     

(iv) Let M be the mid-point of AX.


 1  
By ratio theorem, OM  OA  OX .
2
 
1 0 2
  
Since M lies on  , OM   1     1     1  for some  ,    .
 

sgexamguru.com
 3 1  1 
     

  1   1 0 2


1          
  2   OX    1     1     1 
2  
 0    3  1
 
 1 
 
1 0  2   1
        
OX  2  1   2  1   2   1    2   
 3 1  1  0 
       
 3 0 2
     
 0  h1  k  1 
6 1  1
     
 
 3 0 2
The equation of the locus of X is r   0   h  1   k  1  , h, k  
   
6 1  1 
     

The locus of X is a plane parallel to  .


 

   

RI P1 Solutions 462
Qn. Solution
12(a) y  2ux          (1)
2

dy du
 4ux  2 x 2      (2)
dx dx
dy
Substituting (1) and (2) into 2 x 2  4 xy  y 2  0, we have
dx
 du 
  4 x  2ux    2ux   0
2 2
2 x 2  4ux  2 x 2 2

 dx 
du
                      8ux3  4 x 4  8ux3  4u 2 x 4  0
dx
du
 u 2
dx
1
  u 2 du   dx
1
 xc
u
1
                   u 
xc
y 1

2x 2
xc

sgexamguru.com
2x2
y
xc
(b) d
 k  32    , k  0
dt
1
 32   d   k dt
 ln  32     kt  A since   32
ln  32     kt  A
 32     e kt  A
32    e kt e A  Ce  kt where C  e A
  32  Ce kt
d
Given that   4,  2 when t  0
dt
  32  Ce kt  4  32  C  C  28
d 1
 k  32     2  k  32  4   k 
dt 14
1
 t
  32  28e 14
(shown)  
 
 

RI P1 Solutions 463
(i) 
1
t
  32  28e 14

At   20,
1
 t
20  32  28e 14

1
 t
28e 14
 12
1 3
 t  ln
14 7
3
t  14 ln  11.86  12 min
7
(ii) 
1
t
As t   , e 14  0 .   32 .
i.e. the temperature of the water increases and approaches the room temperature, i.e.
32 o C , for large values of t.

(iii)
 

32  

4 sgexamguru.com
O t

RI P1 Solutions 464
 

RA
AFFLES INSTITUTION
  
 
  201
16 YEAR
R 6 PRE
ELIMINA
ARY EX
XAMINA
ATION 
 
         

MATHEMATICS PAPER
R2 9740/0
02
Higherr 2 MBER 201
21 SEPTEM 16
Total M
Marks: 100 3 hourrs

Additional materialss: Answer Paper


P
List of Fo
ormulae (MF
F15)

READ T
THESE INST
TRUCTION
NS FIRST

Write your name an nd CT group p on all the work you hand in.
Write in dark blue or
o black pen n on both sid
des of the paper.
p
You mayy use an HB B pencil for any diagramms or graph hs.
Do not uuse staples, paper clips ers, glue or correction fluid.
s, highlighte f

n-exact num
sgexamguru.com
Answer all the quesstions.
Give non merical answ wers correcct to 3 signifficant figure
es, or 1 deccimal place in the case of
angles in
n degrees, unless
u a diffferent levell of accurac
cy is specifie
ed in the quuestion.
You are expected too use an appproved gra aphing calcuulator.
Unsuppo orted answeers from a graphing
g ca
alculator aree allowed unless
u cifically states
a queestion spec
otherwisse.
Where u unsupportedd answers from a gra aphing calcu ulator are not
n allowedd in a ques stion, you are
a
required to presennt the math hematical ssteps using g mathema atical notatiions and not
n calculattor
comman nds.
You are reminded of
o the need for clear prresentation in your answers.

At the en
nd of the tesst, fasten all your workk securely to
ogether.
The nummber of marks is given in brackets [ ] at the en nd of each question
q or part question.
 
 
 
 
 
 
 
 
 
 
 

This
T docum
ment consists of 7 printe
ed pages.
 
RAF
FFLES INSTIT TUTION
partment 
Mat hematics Dep
 RI2016 [Turn over

Microsoft Word - RI_JC2_H2Maths_2016_Prelim_P2 465


    2
 

Section A: Pure Mathematics [40 Marks]

 4r   n  n  1  .
3 2
1 (a) Use the method of mathematical induction to prove that
r 1

[4]

(b) It is given that f  r   r 4  2r 3  2r 2  r.

Show that f  r   f  r  1  ar 3  2r , where a is a real constant to be

 r  2r  1.
n
2
determined. Hence find a formula for [4]
r 1

2 Referred to the origin O, points A and B have position vectors a and b respectively.
Point C lies on OA, between O and A, such that OC : CA = 3 : 2. Point D lies on OB,
between O and B, such that OD : DB = 1 :  .

(i) It is given that the area of triangle ABD is twice the area of triangle ABC.
Find . [4]

(ii) Show that the vector equation of the line BC can be written as

sgexamguru.com
3
r  s a  1  s  b , where s is a parameter. By writing down the vector
5
equation of the line AD in a similar form, in terms of a parameter t, find, in
terms of a and b , the position vector of the point E where the lines BC and
AD meet. [3]

It is further given that the angle AOB is 45 and O lies on the perpendicular bisector
of the line segment AB.

(iii) Find the length of projection of a on b , giving your answer in terms of b .


Hence find the position vector of the foot of the perpendicular from A to OB.
[3]

RI P2 Questions 466
    3
 

24
2
3 The diagram th equation y  2 
m shows the curve with , for 0  x  8. Th
he curve
x( x  8)
crosses thee x-axis at x  2 and x  6, has a maximumm turning ppoint at x  4, and
asymptotess at x  0 annd x  8.

x
O 2 4 6 8

244
For 0  x  8, the region boundedd by the curves with equations y  2  and
x( x  8)

y  2
4
sgexamguru.com
x 8
is denoteed by S .

(i) On the same diiagram, for 0  x  8 , sketch


s the curves with equations

24 4
y  2 and y  2  .
x( x  8)
8 x 8

y on your diiagram the region S , the coordinaates of the points


Indiicate clearly p of
inteersection of the two currves and thee equation(s) of any asyymptote(s). [3]

(ii) Findd the exact area of S.


Exppress your answer in the form A  B ln 2  C ln 3  D lnn 7 where A, B, C
andd D are connstants to bee determined
d. [5]

(iii) me of the sollid of revolution formeed when S is rotated about


Findd the volum a the
x-axxis through 360. [2]

[Tu
urn over

RI P2 Questions 467
    4
 

4 (a) Solve the equation


z 4  4i  0,

giving the roots in the form rei , where r  0 and      . [3]

These roots correspond to four points on an Argand diagram. Identify the


quadrilateral that has these four points as vertices. [1]

(b) The point A represents the fixed complex number a , which has modulus r

and argument  , where 0    .
2
(i) On a single Argand diagram, sketch the loci

(a) z  a  2r ,

(b) arg  z  a    ,

making clear the relationship between the loci and the point A. [3]

(ii) Hence, or otherwise, find exactly the complex number(s) z that satisfy
both equations in part (i), giving your answer(s) in terms of r and  .
[2]

(iii) sgexamguru.com
 
Given instead that z  a  2r and   arg z  a   , find exactly
the minimum and maximum possible values of z  ia , giving your
answers in terms of r. [3]

Sections B: Statistics [60 Marks]

5 The recreational committee of a large company is organizing a family day and would
like to conduct a survey with 5% of its employees about their preferences for an
outdoor or indoor based carnival as well as the activities involved.

Describe how the committee could obtain a sample using

(i) systematic sampling, [2]

(ii) quota sampling. [2]

RI P2 Questions 468
    5
 

6 A market stall sells rice in packets which have masses that are normally distributed.
The stall owner claims that the mean mass of the packet of rice is at least 5 kg. Jane
buys a random selection of 10 packets of rice from the stall. The 10 packets have
masses, in kg, as follows:

4.9 4.7 5.1 4.8 4.5 5.3 5.0 4.8 4.6 5.2

Find unbiased estimates of the population mean and population variance of the mass
of rice packets. [2]

A test, at  % significance level, shows that there is insufficient evidence for Jane to
doubt the stall owner. Find the set of possible values for  . [4]

7 Simon owns a diecast car display case which has 4 shelves and 8 individual
compartments on each shelf. Each of these compartments can only hold one diecast
car. He arranged his collection of 32 different diecast cars in the display case.

Find the number of different selections that can be made by

(i) taking two cars, both from the same shelf, [2]

(ii) taking a total of six cars from the display case, [1]

sgexamguru.com
(iii) taking a total of six cars from the display case with at least one from each shelf.
[3]

8 (a) S and W are independent random variables with the distributions N(20, 25)
and N(  ,  2 ) respectively. It is known that P(W  10)  P(W  13) and
P( S  2W )  0.43. Calculate the values of  and  correct to three
significant figures. [4]

(b) A small hair salon has two hairstylists Joe and Joan attending to customers
wanting an express haircut. For Joe, the time taken to attend to a customer
follows a normal distribution with mean 10 minutes and standard deviation 42
seconds. For Joan, the time taken to attend to a customer follows a normal
distribution with mean 10.2 minutes and standard deviation 45 seconds.

(i) Find the probability that among three randomly chosen customers
attended to by Joe, one took more than 10.5 minutes while the other
two each took less than 10 minutes. [2]

(ii) Joe and Joan each attended to two customers. Find the probability that
the difference in the total time taken by Joe and Joan to attend to their
two customers respectively is more than 3 minutes. State any
assumption(s) that you have used in your calculation. [4]

[Turn over

RI P2 Questions 469
    6
 

9 The following table shows the marks (x) obtained in a mid-year examination and the
marks (y) obtained in the year-end examination by a group of eight students. The
year-end mark of the eighth student was accidentally deleted from the records after
the marks were analyzed, and this is indicated by m below.

Mid-year mark (x) 70 31 68 73 46 78 79 55


Year-end mark (y) 80 39 70 80 48 94 98 m

It is given that the equation of the regression line of y on x is y  1.2 x  4 .


Show that m  59. [2]

(i) Draw the scatter diagram for these values, labelling the axes clearly. Find the
value of the product moment correlation coefficient between x and y. [2]

(ii) It is thought that a model of the form ln y  a  bx may also be a suitable fit to
the data. Calculate least square estimates of a and b and find the value of the
product moment correlation coefficient between x and ln y. [3]

(iii) Use your answers to parts (i) and (ii) to explain which of

y  1.2 x  4 or ln y  a  bx

is the better model.

sgexamguru.com
Hence, estimate the mark that a student who obtained a mark of 75 in the mid-
year examination but was absent from the year-end examination would have
obtained in the year-end examination. [3]

5 2
10 For events A and B, it is given that P( A)  and P( B )  .
8 3
(i) Find the greatest and least possible values of P( A  B ). [2]

3
It is given in addition that P( A ' | B ')  .
8

(ii) Find P( A ' B '). [1]

(iii) Find P( A  B ). [2]

(iv) Determine if A and B are independent events. [3]

3 1
(v) Given another event C such that P(C )  , P( A  C )  P( B  C )  and
8 4
11
P( A  B  C )  , find P( A  B  C ). [2]
12

RI P2 Questions 470
    7
 

11 A chocolate shop puts gift vouchers at random into 7% of all their packets of mini
chocolates produced. A customer must collect 3 vouchers to exchange for a gift.

(i) Adeline buys 8 packets of the mini chocolates. Find the probability that she
gets exactly 2 gift vouchers. [2]

(ii) Aileen buys 31 packets of the mini chocolates. Find the probability that she is
able to exchange for at least one gift. [2]

(iii) Angelina and Angeline buy 60 packets of the mini chocolates altogether. Use
a suitable approximation to estimate the probability of them being able to
exchange for exactly two gifts. [4]

(iv) Ashley buys n packets of the mini chocolates. Given that she already has 2
unused vouchers from her previous purchase, find the value of n for which the
probability of her being able to exchange for exactly one gift is greatest. [3]

(v) The shopkeeper observes that the number of gifts exchanged in a day has a
mean of 10 and variance of 25. Estimate the number of gifts the shop needs to
stock if there is to be no more than a 5% chance of running out of gifts in a
40-day period. [3]

  sgexamguru.com
******* End of Paper *******

RI P2 Questions 471
Prelim Paper 2 Solutions

Pure Mathematics [40 Marks]

Qn. Solution
1(a) n

 4r   n  n  1  for n    .
3 2
Let Pn be the statement
r 1

When n  1 , LHS = 4 and RHS= 1 2  4


2

 P1 is true.
Assume Pk is true for some k    .
k

 4r   k  k  1  .
3 2
i.e.
r 1

To prove that Pk 1 is true,


k 1

 4r   k  1 k  2   .
3 2
i.e.
r 1
k 1 k

 4r 3   4r 3  4  k  1
3

r 1 r 1

  k  k  1   4  k  1
2 3

  k  1  k 2  4  k  1 
2

  k  1  k 2  4k  4 
sgexamguru.com
2

   
2 2
 k 1 k2
  k  1 k  2  
2

 Pk 1 is true.
Hence Pk is true  Pk 1 is true, and since P1 is true, by Mathematical Induction, Pn
is true for all n    .
(b) f  r   f  r  1   r 4  2r 3  2r 2  r 

  r  1  2  r  1  2  r  1   r  1 
4 3 2
 
 r  2r  2 r  r
4 3 2

 r 4  4r 3  6r 2  4r  1
 2r 3  6r 2  6r  2
 2r 2  4 r  2
 r 1
 4r 3  2r , a  4
1 n
r  2r 2  1    4r 3  2r 
n


r 1 2 r 1

Microsoft Word - RI_JC2_H2Maths_2016_Prelim_P2_Solutions(updated) 472


1 n
   f  r   f  r  1 
2 r 1
 f 1  f  0  
 
  f  2   f 1 
 f 3  f 2 
     
1 
 
2 
 
  f  n  1  f  n  2  
 
  f  n   f  n  1 
 
1
 f  n  f  0
2
n 4  2n3  2n 2  n n  n  1  n 2  n  1
 OR
2 2
Qn. Solution
2(i) 1   1 2
Area of triangle ABC = CA  AB  a  b  a
2 2 5
1
 ab  aa
5
1

sgexamguru.com  ab
5
1   1 
Area of triangle ABD = DB  AB  b  b  a
2 2 1 

 bb  ba since   0
2 1   

 ab
2 1   
Area of triangle ABD = 2 (area of triangle ABC)
 2
ab  ab
2 1    5
5  4 1   
4
 
(ii) Line BC : r  OB  sBC , s  
 

 b  s OC  OB , s   
3 
    b  s  a  b  , s  
5 
3
 s a  1  s  b , s  
5  

Line AD : r  OA  t AD , t  

RI P2 Solutions 473
 
 
 a  t OD  OA , t  
1 
 a  t  b a , t 
5 
1
 t b  1  t  a , t  
5
3 1
At point of intersection E, s a  1  s  b  1  t  a  t b
5 5
3
s  1  t ...(1)
5
1
1  s  t ...(2)
5
10 5  3  10   10  6 1
Solving, s  , t  and OE    a   1   b  a  b
11 11 5  11   11  11 11
(iii) Length of projection of a on b = a . bˆ

 a bˆ cos 45
 a cos 45
1
 b
2
  1  1

Qn.
OF  
 2  sgexamguru.com
b  bˆ 
2
b

Solution

3(i)
[3] y

x
2 4 (6,0)

 
                           x =8
   

RI P2 Solutions 474
(ii) Area of S
2  4    24  
  2      2    dx
1
 x 8   x( x  8)  
6  4   24 
  2    2  dx
2
 x 8   x( x  8) 
2  4    3 3 
  2     2     dx
1
 x 8   x  8 x 
6  4   3 3
  2    2    dx
2
 x 8   x 8 x
2  7 3 6  1 3
  4   dx      dx
1
 x 8 x  2
 x 8 x
  4 x  7 ln | x  8 | 3ln | x |1   ln | x  8 | 3ln | x |2
2 6

  4  7 ln 6  7 ln 7  3ln 2   ln 2  ln 6  3ln 6  3ln 2   4  4 ln 2  9 ln 3  7 ln 7


 A  4, B  4, C  9, D  7
(iii) Required Volume
2 2

6 4   24 
  2  2  dx
1
 x 8   x( x  8) 
 4.63989  14.577 units 2 (5s.f)=14.6 units 2 (3s.f)

sgexamguru.com
Qn. Solution
4(a) 
 i
] z 4  4i  4e 2  e2 k i where k  
By De Moivre’s Theorem,
1
  4 k21  i  4
z  4e  
 
 4 k 1 
  i
 2e  8 
, k  0, 1, 2

The quadrilateral is a square.

RI P2 Solutions 475
(b)(i)

(ii) At point of intersection of the loci, P, z  3rei or 3r  cos   i sin  


(iii)

sgexamguru.com

Minimum z  ia  BO  r
Maximum z  ia  BC  BA  AC  r 2  r 2  2r   
2 2 r
Qn. Solution
5(i) Note: For the interval number, k
total population total population 1 1
k     20
Sample Size 5% of total population 5% 5
100
Use a computer to generate a random number from 1 to 20, say 5. Select the 5th
employee to arrive at the office, thereafter every 20th employee that arrives, ie 5th,
25th, 45th, etc.
(ii) Any reasonable choice of strata: Age/Gender/Marital status etc.
Strata 1 2 3 total
Sample Size (n) x y z x+y+z = 5% of population

RI P2 Solutions 476
Choose the individuals from each strata according to some convenient/non-random
scheme such as the first x, y, z individuals committee encounters belong to strata 1,2,3
respectively such that sample size, x+y+z is 5% of the population.

OR

Choose the first “n” employee arrived at the company on a particular morning from
each stratum as shown in the table.
Qn. Solution
6 From GC, unbiased estimate of population mean, x  4.89
[ unbiased estimate of population variance,
 203 
s 2  0.26012817352  or 
 3000 

H0 :   5 H1 :   5

Perform 1-tail t-test at  % significance level

Under H0,
X  0
Test statistic , T  ~ t (n  1) where  0  5 and n  10
S/ n

For the sample, x  4.89, s  0.2601281735 .


sgexamguru.com
Using a t -test, p -value  0.1069800566

Since there is insufficient evidence for Jane to doubt the stall owner, we Do Not
reject H0,

p -value   %     10.7 (3sf)
100
Qn. Solution
7(i) No. of selections = C1  C2 = 112
4 8

(ii) No. of selections = 32C6 = 906192

No. of selections = 6   8C1  C  4   8C1  C


2 2 3
(iii) 8 8
2 3

= 301056 + 114688
= 415744
Alternative Solution
(I) By complement
No. of selections if
All from 1 shelf = 8C4  4  112
All from 2 shelves = 4C2 [ 8C3   ( 8C2  8C4  2)  ( 8C1  8C5  2)]
3

All from 3 shelves = 4C3 [ 8C2   (( 8C1 ) 2  8C4  3)  ( 8C1  8C2  8C3  3!)]
3

RI P2 Solutions 477
Answer = 906192  112  47712  442624  415744

(II) By Inclusion-Exclusion Principle


No. of selections if
No cars are taken from 1 of the 4 shelves = 4C1  24C6  538384
No cars are taken from 2 of the 4 shelves = 4C2  16C6  48048
No cars are taken from 3 of the 4 shelves = 112
Answer = 906192  538384  48048  112  415744

Qn. Solution
8(a) 10  13
Since P(W  10)  P(W  13) ,    11.5 .
2
2W  S ~ N(3, 25  4 2 )
P( S  2W )  0.43
P(2W  S  0)  0.43
03
P( Z  )  0.43
25  4 2
3
From GC,  0.176374
25  4 2
Solving,   8.13 .
8(b) Let X be the time taken by Joe to attend to a customer.

sgexamguru.com
(i) X ~ N(10, 0.72 )
Required probability = 3C2   P( X  10)  P( X  10.5)  0.178
2

(ii) Let Y be the time taken by Joan to attend to a customer.


Y ~ N(10.2, 0.752 )
( X1  X 2 )  (Y1  Y2 ) ~ N(0.4, 2.105)
P( ( X 1  X 2 )  (Y1  Y2 )  3)
 1  P(3  ( X 1  X 2 )  (Y1  Y2 )  3)
= 0.0461
The following answers are accepted:
(i) The time taken by Joe to attend to a customer is independent of the time
taken by Joan to attend to a customer.
(ii) The time taken by Joe to attend to one customer is independent of the
time taken by Joe to attend to another customer.
Qn. Solution
9  x  500,  y  509  m, n  8
 
Since x, y lies on the regression line of y on x,
y  1.2x  4
509  m  500 
 1.2  4
8  8 
m  59

RI P2 Solutions 478
(i)
(79,98)

(31,39)

From GC, r  0.96826  0.968


(ii) From GC, ln y  3.0351  0.018945x
a  3.0351  3.04
b  0.018945  0.0189
r  0.990499  0.990
(iii) The scatter plot of x and y shows a non-linear relationship as when x increases, y
appears to be increasing at an increasing rate.

sgexamguru.com
Since the product moment correlation coefficient between x and ln y of 0.990 is
closer to 1 than the product moment correlation coefficient between x and y of
0.968, ln y  a  bx is the better model.
When x = 75, ln y  3.0351  0.018945(75)  4.455975
y  86.140
The student would have obtained 86.1 marks in the year-end examination.
Qn. Solution
10(i) P( A)  P( B )  1  P( A  B )  min P( A), P( B )
7 5
 P( A  B ) 
24 8
5
Greatest value of P( A  B ) is .
8
7
Least value of P( A  B ) is .
24
(ii) 3 1 1
P( A ' B ')  P( A ' | B ')P( B ')   
8 3 8
(iii) 7
P( A  B )  1  P( A ' B ') 
8

RI P2 Solutions 479
(iv) P( A  B )  P( A)  P( B)  P( A  B)
7 5 2
   P( A  B)
8 8 3
5
P( A  B ) 
12
5 2
   P( A)P( B )
8 3
Hence, A and B are independent events.
3 5
OR  P( A ' | B ')   1   P( A ')
8 8
A ' and B ' are independent events
 
 A and B are independent events
(v) P( A  B  C )  P( A)  P( B )  P(C )
 P( A  B )  P( A  C )  P( B  C )
 P( A  B  C )
11 5 2 3 5 1 1
       P( A  B  C )
12 8 3 8 12 4 4
1
P( A  B  C ) 
6
Qn. Solution

sgexamguru.com
[
11 Let X denote the number of gift vouchers obtained (out of 8).
(i) X  B (8, 0.07)
P (X = 2) = 0.0888 (3 sf )
(ii) Let Y denote the number of gift vouchers obtained (out of 31).
Y  B (31, 0.07)
P (Y  3) = 1  P (Y ≤ 2) = 0.371 (3 sf )
(iii) Let V denote the number of gift vouchers obtained (out of 60).
V  B (60, 0.07)
Since n = 60 is large, p = 0.07 is small such that np = 4.2  5,
V  Po (4.2) approximately
P (6  V  8) = P (V ≤ 8)  P (V ≤ 5) = 0.219 (3 sf )
(iv) Let W denote the number of gift vouchers obtained (out of n).
W  B (n, 0.07)
For the condition of the question to be satisfied, the probability is
p = P (1  W  3)
From GC,
when n = 25, p = 0.74343
when n = 26, p = 0.74365
when n = 27, p = 0.74250

 Value of n for the probability p to be maximum = 26.


   

RI P2 Solutions 480
(v) Let T denote the number of gifts exchanged in a day.
E T   10, Var T   25
Let T1 , T2 ,..., T40 be a random sample from distribution X
n  40 large, by the Central Limit Theorem,
T1  T2    T40 ~ N  40 10  , 40  25   approximately
P T1  T2    T40  k   0.05
From GC, k  452.01
Least k = 453.
The shop needs to stock at least 453 gifts.

 
sgexamguru.com
 

RI P2 Solutions 481
1

River Valley High School 2016 H2 Maths Preliminary Examination - Paper 1

1. (i) The first three terms of a sequence are given by u1  4.4 , u2  4.1 and

u3  2.6 . Given that un is a quadratic polynomial in n, find un in terms of n. [3]

(ii) Find the set of values of n for which un is more than 15. [2]

3x
2. Let f ( x)  .
( x  4)( x  1)

(i) Express f(x) in partial fractions. [1]

(ii) Find the series expansion, in ascending powers of x, of f(x) up to and including
the term x 3 . State the values of x for which this expansion is valid. [3]

(iii) The first two terms of the expansion of f(x) are identical to that of the expansion
1

of ax (1  bx ) 2
. Find the exact values of a and b. [2]

3. (a)
sgexamguru.com 2
Differentiate e  x with respect to x. Hence find  x e dx .
2
3 x
[4]


2
(b) Find the exact value of π
sin x sin x dx . [3]

4

4. On the first of January 2011, John opened a new special savings account and deposited
$x into his account. On the first day of each subsequent month, he would deposit
another $x into the account. A compound interest of 0.3% per month would be paid at
the end of each month.

(i) Given that x  600 , find the total amount of money in the savings account on first
of January 2013 just before John deposited money into his account. [3]

On first January 2013, instead of going to the bank to make a deposit into his savings
account, John went to a yatch club and decided to purchase a yatch. For the yatch that
he was interested to buy, he had to make a down payment of $23000.

(ii) What should his minimum monthly deposit be (in multiples of $10) so that his
savings account would have sufficient amount for him to make the down payment?
[2]

©RIVER VALLEY HIGH SCHOOL 9740/01/2016

Microsoft Word - RVHS_JC2_H2_Maths_2016_Prelim_P1_Question 482


2

(iii) Assume that upon making the withdrawal for the down payment, there was $190
left in his savings account. John transferred this amount from the savings account
to a checking account which charges no interest. With the $190 as the initial
savings, he decided to make a monthly contribution of 90% of the amount he
contributed in the previous month at the start of every subsequent month to the
checking account. He claimed that he would be able to save $2000 eventually. Do
you agree with him? Explain. [3]

5. With reference to the origin O, the points A and B have position vectors a and b
respectively and b is a unit vector.

(i) Give a geometric description of b a . [1]

Given that S denotes the area of triangle OAB , show that 4S 2  a   a b  .


2 2
(ii)
[3]
(iii) Given that C is a point on the line AB with position vector c, explain why
(c  a)  (c  b)  0 . [2]

sgexamguru.com
(iv) Suppose a is perpendicular to b and that the angle between c and b is 30 , find in
terms of |a| and |c|, the ratio of the area of triangle OAB to that of triangle OCB. [3]

3a  5i
6. (a) The complex numbers z and w are such that z  and w  1  13bi , where
1  2i
a and b are real numbers. Given that z *  w , find the exact values of a and b. [4]

(b) Without using a graphic calculator, find the modulus and argument of the
(1  i)2
complex number z  , giving your answers in exact form. Hence
(1  3i)4

evaluate z 6 exactly. [5]

2
7. A sequence u1 , u2 , u3 , … is such that u0  1 and un 1  un  , for n  0 .
 2n  1 2n  3
1
(i) Prove by induction that un  . [4]
2n  1
N
1
(ii) Find   2n  1 2n  3
n 1
in terms of N. [3]

©RIVER VALLEY HIGH SCHOOL 9740/01/2016

RVHS P1 Questions 483


3

N
1
(iii) Hence find   2n  3 2n  5 .
n 0
[2]


1
(iv) State the value of   2n  1 2n  3 .
n 1
[1]

8. In a certain fish farm, the growth of the population of garoupa is studied. The
population of garoupa at time t days is denoted by x (in thousands). It was found that
the rate of birth per day is twice of x, and the rate of death per day is proportional to x 2 .

(i) Given that there is no change in the population of garoupa when its population
dx
hits 10000, write down a differential equation relating and x. [2]
dt
(ii) Its owner decides to sell away 1800 garoupa daily. Modify the differential
equation in part (i) and show that the resulting differential equation can be written
dx 1
as   ( x  5) 2  a 2  , where a is a constant to be determined. [2]
dt 5
Given that the initial population of garoupa is 13000, solve this modified

sgexamguru.com
differential equation, expressing x in terms of t.

Deduce the long term implication on the population of garoupa in the farm, and
[4]

sketch the curve of x against t. [2]

9. The parametric equations of a curve C, are

x  4cos , y  sin 2 , where 0    π .

(i) Sketch C, stating the exact coordinates of any points of intersection with the axes.
[2]
(ii) The point P on the curve has parameter p. Show that the equation of the tangent
 cos 2 p 
at P is y     x  2cot p cos 2 p  sin 2 p . [4]
 2sin p 
π
(iii) The region S is the area bounded by C, the tangent at   and the line x  1 .
2
d
Show that the area of S can be expressed in the form a  b  sin 2  cos  d ,
c

where a, b, c and d are exact constants to be determined. Hence evaluate the area
of S numerically. [4]

©RIVER VALLEY HIGH SCHOOL 9740/01/2016

RVHS P1 Questions 484


4

(iv) C is transformed to give the curve C ' given by

x  2cos  , y  sin 2  1 , where 0    π .

Describe the sequence of transformations from C to C ' . [2]

10. The diagram shows the graph of y  2g( x  1) . The graph intersects the axes at  0,3

and  2, 0  and has a turning point at (3, 5) . The asymptotes of the graph are at x  1

and y  2 .
y

sgexamguru.com 0 x

On separate diagrams, sketch the following graphs indicating the points corresponding
to the axial intercepts, turning point and asymptotes where necessary.

(i) y  g  x [3]

1
(ii) y [3]
2g( x  1)

(iii) y   2g( x  1) [3]

Suppose the above diagram shows the graph for y  h  x  . Sketch the graph of

y  h'  x  . [3]

©RIVER VALLEY HIGH SCHOOL 9740/01/2016

RVHS P1 Questions 485


5

11. The functionn f is defined


d by
f : x  2 x 2  12 x  19 , x  .
(ii) Sketchh the graph of y  f  x  , indicatin
ng clearly th
he coordinaates of any
y turning

point(ss) and axial intercept(s)). [1]


(iii) Give a reason why
y f does nott have an inv
verse. [1]
(iiii) If the domain
d t x  k , state the greaatest value of k for
of f is further restricted to
which the n f 1 exists..
t function [1]
(iiv) Find f 1 ( x) , statin
ng the domaain of f 1 . [3]

Foor the rest of main of f iss as found in


o the questiion, the dom n part (iii).
(vv) On the same diagrram, sketchh the graphs of y  f  x  , y  f 1  x  and y  ff 1 ( x) ,

showinng clearly th
he geometriccal relationship among
g the graphss. [3]

The functionn g is defined by


2
g : x  ex 3
, x .

(vvi) Explainn why the composite


c fuunction gf 1 exists. [1]

(vvii) Find thhe range of gf 1 , givingg your answ


sgexamguru.com
wer in the ex
xact form. [2]

EN
ND OF PAP
PER

©RIVER
R VALLEY HIGH
H SCHOO
OL 9740/01/2
2016

RVHS P1 Questions 486


Solutions to 2016 RVHS Y6 H2 Maths Preliminary Exam II (Paper 1)

Question 1 [5 Marks]
(i) Let un  an 2  bn  c
u1  4.4
a  b  c  4.4 -------------- (1)
u2  4.1
4a  2b  c  4.1 ----------- (2)
u3  2.6
9a  3b  c  2.6 ----------- (3)

Solving (1), (2) and (3)


a  0.6 , b  1.5 and c  3.5
un  0.6n 2  1.5n  3.5
(ii) un  15
0.6n 2  1.5n  3.5  15
0.6n 2  1.5n  18.5  0

sgexamguru.com
Since n is a positive integer, the set of values of n is n  7 .

Question 2 [6 Marks]
(i) 3x
f ( x) 
( x  4)( x  1)
A B
 
( x  4) ( x  1)
3x  A( x  1)  B( x  4)
When x  1 ,  3  3B  B  1
When x  4 ,  12  3 A  A4
4 1
f ( x)  
( x  4) ( x  1)
(ii) f ( x)  4(4  x) 1  (1  x) 1
 4(4  x) 1  (1  x) 1
 1  x  1 
 4 4 1     (1  x) 1
  4  
2 3
 x  (1)(2)  x  (1)(2)(3)  x 
 1  (1)         ...
4 2!  4  3! 4
 (1)(2) 2 (1)(2)(3) 3 
 1  (1)( x)  x  x  ...
 2! 3! 

Pg 1
Microsoft Word - RVHS_JC2_H2_Maths_2016_Prelim_P1_Solutions 487
x x 2 x3
1    1  x  x 2  x 3  ...
4 16 64
3 15 63 3
 x  x2  x  ...
4 16 64
1
 x
Range of validity for 1   is  4  x  4
 4
Range of validity for (1  x) 1 is  1  x  1
Therefore, overall range of validity is  1  x  1 .
(iii) 
1
ax (1  bx ) 2

  1 
 ax 1     bx   ...
  2 
ab 2
 ax  x  ...
2
Comparing the coefficient of x terms:
3
a
4
Comparing the coefficients of x 2 terms:
ab 15 5
  b
2 16 2

Question 3 [7 Marks]
(a) d x2
e  2 xe  x
2
sgexamguru.com
dx

x e
3 x2
dx
 1 
=    x 2 (2 xe  x )dx
2

 2 
1
=  x 2e  x   xe  x dx
2 2

2
1 1
=  x 2e  x   (2 x)e  x dx
2 2

2 2
1 2  x2 1  x2
=  x e  e c
2 2
(b) π


2
π
4
sinx sin x dx
π
0
=  π sinx (  sin x )dx +  2 sinx(sin x)dx
 0
4
π
0
=  π sin x dx +  2 sin 2 x dx
2
 0
4
π
0 1  cos 2 x 1  cos 2 x
=  π dx +  2 dx

4 2 0 2

Pg 2
RVHS P1 Solutions 488
π
0
1 sin 2 x  1 sin 2 x  2
=  x +  2 x  4 
2 4   π 0
4

  π 1   π  
=  0          0
  8 4   4  
π 1
= 
8 4
π2
=
8

Question 4 [8 Marks]
(i)
Mth Start of Month End of Month
1 600 1.003  600
2 1.003  600  600 1.003  600  600  1.003
 1.0032  600  1.003  600
3 1.0032  600  1.003  600  600 1.0033  600  1.0032  600
1.003  600
n 1.003n  600  1.003n1  600
   1.003  600

At the end of December 2012, there are 24 months.

sgexamguru.com
Amount in the account at the end of December 2012 after interest
paid
 1.00324  600  1.00323  600    1.003  600
 600 1.003  1.0032  1.00324 
1.003 1.00324  1 
 600  
 1.003  1 
 14952.62754
 $14952.63
(ii) Let the monthly contribution be $x.
1.003 1.00324  1 
x   23000
 1.003  1 
x  922.9147161

Since his monthly contributions are in multiple of $10, he should


deposit $930 monthly into his savings account.

Pg 3
RVHS P1 Solutions 489
(iii)
Months Amount contributed ($)
1 190
2 190  0.9
3 190  0.92
n 190  0.9n1

Sum of the savings eventually


 190 1  0.9  0.92  0.93    0.9n1  
 1 
 190  
 1  0.9 
 $1900, which is less than $2000
Hence, John would not be able to save $2000 in the long run.

Question 5 [9 Marks]
(i) b a  a b , where b is a unit vector, which gives the length

of projection of the vector OA onto the line OB (or on

vector OB ).
(ii) 1
S  ab
2
2 S  a b sin 
2 2
4 S 2  a b sin 2 
2
sgexamguru.com
4 S 2  a b 1  cos 2  
2

   a b  2 
 a b 1    
2 2

  a b  
 
 a b  a b
2 2 2

 a  a b
2 2
since b  1
(iii) As C lies on AB, A, C and B are collinear.
 
 AC and BC are parallel
 
  c  a    c  b   AC  BC
 
 AC  BC sin 0 n
 0, since sin 0  0

Pg 4
RVHS P1 Solutions 490
(iv) a is perpendicular to b  a  b  0
a
S 
2
Area of triangle OCB, S1
1
 cb
2
1
 c b sin(30 )
2
1
 c , since b  1
4
a c
Hence, S : S1  :  2 a : c
2 4

Question 6 [9 Marks]
(a) 3a  5i
z
1  2i
3a  5i 1  2i
= 
1  2i 1  2i
3a  10  (6a  5)i
=
5
3a  10  (6a  5)i
Therefore, z * 
5
z w
sgexamguru.com
*

3a  10  (6a  5)i
 1  13bi
5
Comparing real parts:
3a  10
1  a  5
5
Comparing imaginary parts:
6a  5 7
 13b  b 
5 13
(b) 1 i
2

z  4
1  3i
2
2 1
 4 
2 8
 (1  i) 2 
arg( z )  arg  4 
 (1  3i) 
 2 arg(1  i)  4 arg(1  3i)
 π   2π 
 2    4 
 4  3 
π 8π 19π 5π
    (principal range)
2 3 6 6

Pg 5
RVHS P1 Solutions 491
1   5π   5π  
 z  cos    i sin   
8  6   6 
1
 z 6  6  cos  5π   i sin  5π  
8
1
 6  1  0
8
1

262144

Question 7 [10 Marks]


(i) 1
Let Pn be the statement un  for n  0 .
2n  1
When n  0 ,
LHS  u0  1 (Given)
1
RHS   1  LHS
2  0  1
 P0 is true.
1
Assume Pk is true for some k  , k  0 , i.e. uk 
2k  1
1
Show that Pk 1 is also true, i.e. uk 1 
2k  3
Using the recurrence relation,
uk 1  uk  sgexamguru.com
2
 2k  1 2k  3
1 2
 
2k  1  2k  1 2k  3
2k  3  2

 2k  1 2k  3
2k  1 1
 
 2k  1 2k  3 2k  3
Pk 1 is true.

Since P0 is true and Pk is true, implies Pk 1 is also true. By


mathematical induction, Pn is true for n  0 .

Pg 6
RVHS P1 Solutions 492
(ii) N
1
  2n  1 2n  3
n 1

1 N
   un  u n1 
2 n 1
1
 u1  u2
2
u2  u3
u3  u4

 u N 1  u N
 u N  u N 1 
1
  u1  uN 1 
2
11 1 
   
2  3 2N  3 
(iii) Let n  r  1 ,
N
1

n  0  2n  3  2n  5 
r 1 N
1
   2r  2  3 2r  2  5
r 1 0
N 1
1

r 1

11 1 
sgexamguru.com
 2r  1 2r  3
   
2  3 2N  5 
(iv) 1
As N   ,  0,
2N  3

1 1

n 1  2n  1 2n  3 

6

Question 8 [10 marks]


(i) dA dB
Let and be the rate of birth and rate of death
dt dt
respectively.
dA dB dB
 2x ,  x2   kx 2
dt dt dt
dx dA dB
   2 x  kx 2
dt dt dt
dx
When x  10,  0:
dt
1
0  2(10)  k (10) 2  k 
5
2
dx x
  2x 
dt 5

Pg 7
RVHS P1 Solutions 493
(ii) dx x2
  2 x   1 .8
dt 5
1 2
=  ( x  10 x  9)
5
1
=  ( x  5) 2  42  , where a = 4 (shown)
5
dx 1
  ( x  5) 2  42 
dt 5
1 1
 ( x  5)2  42 dx    5 dt
1 ( x  5)  4 1
ln  tc
2(4) ( x  5)  4 5
x9 8
ln   t  8c
x 1 5
8 8
x 9  t  8c  t
e 5  Ae 5
x 1
When t = 0, x = 13:
13  9 1
 A  A
13  1 3
8
x9 1 5  t
 e
x 1 3
8
 t
3( x  9)  ( x  1)e 5

x(3  e
8
 t
5
sgexamguru.com
)  27  e
8
 t
8
 t
5

27  e 5
x 8
 t
3e 5

8
 t
As t   , e 5  0
27
x 9
3
In the long term, the population of garoupa will stabilise at
9000.
x (in thousands)

13

Pg 8
RVHS P1 Solutions 494
Question 9 [12 Marks]
(i)

(-4,0)
(0,0) (4,0)

(ii) x  4cos y  sin 2


dx dy
 4sin   2cos 2
d d

dy 2cos 2 cos 2
 
dx 4sin  2sin 

Equation of tangent at P,
cos 2 p
y  sin 2 p    x  4cos p 
2sin p
 cos 2 p  4cos p cos 2 p
y   x   sin 2 p
 2sin p  2sin p
 cos 2 p 
   x  2cot p cos 2 p  sin 2 p
 2sin p 
(iii) sgexamguru.com
Equation of the tangent at  

2
, y
1
2
x

Region S

y = 0.5x x=1

Area of S
1 1

  y dx
x
 dx 
0 2 0
1 1
cos 1  
 x2 
 sin 2  4sin  d 
4
  
 4 0 
2
1
cos 1  
1

4
 8 sin 2  cos  d (shown)
4 
2

 0.00395 units 2 (to 3 s.f.)

Pg 9
RVHS P1 Solutions 495
(iv)  x  2x
C  C ': 
 y  y 1
Sequence of transformations:
- Scaling by factor 0.5 parallel to x-axis
- Translation of 1 unit in the negative y-direction

Question 10 [12 marks]


(i)
y x2

 4, 2.5 
1,1.5
y  g  x
y 1

0 x
 3, 0 

(ii)
y
x2

1
y

sgexamguru.com
 1
 0, 
 3
2g  x  1
y
1
2
 1
 3, 
0 1, 0   5 x

(iii) y
x 1

y   2g  x  1
0  2, 0 
x

y 2
 0,  3 
3,  5 

Pg 10
RVHS P1 Solutions 496
y

y  h ' x

 3, 0 
0 x

x 1

Question 11 [12 marks]


(i)
x
 3, 1

 0, 19 

(ii)
sgexamguru.com
The horizontal line, y  5 , cuts the graph more than once,
hence f is not one-one  f 1 does not exist.
(iii) Turning point of y  f  x  occurs at  3, 1 .
Hence greatest value of k  3

(iv) y  2 x 2  12 x  19
 2  x  3  1
2

y 1
 x  3
2

2
y 1
x3 
2
y 1
x  3
2
y 1
Since x  3 , x  3 
2
x 1
f 1  x   3 
2

Df 1  Rf   , 1

Pg 11
RVHS P1 Solutions 497
(v)

(–1, –1)

y = f f –1(x)

(vi) Rf 1   ,3 and Dg 


Since Rf -1  Dg , hence gf 1 exists.
(vii) Df 1 Rf 1 Rgf -1
 
 , 1  ,3 e 3 ,  

sgexamguru.com

ALTERNATIVELY:
 x 1 
gf 1 ( x)  g  3  
 2 
2
 x 1 
 3 2   3
e  
, x  1
y
2
1
y = gf ( x)

x
From GC, minimum point is at x = –19  y  e 3
 Rgf -1  e 3 ,  

Pg 12
RVHS P1 Solutions 498
1

River Valley High School 2016 H2 Maths Preliminary Examination - Paper 2

Section A: Pure Mathematics [40 Marks]

1. The diagram below shows the structure of a military building which consists of two
parts. For the first part, the rectangular plane OABC is on the ground level and the
point T is 12 m vertically above O. The second part of the building is an underground
store room in the form of a cuboid with sides OP  AQ  BR  CS  12 m,
OA  CB  PQ  SR  20 m and OC  AB  PS  QR  16 m .

T
U

12 m
C B
k j

O A
i

sgexamguru.com
12 m S
R

16 m
P Q
20 m
To better study the structure using vector method, the point O is taken as the origin and
vectors i, j, k, each of length 1 m, are taken along OA , OC and OT respectively.

(i) The military officials decide to install a surveillance camera U along the edge TB
such that TU : UB  1: 3 for better coverage. Determine the position vector of U
and hence the vector equation of the plane UCB in scalar product form. [3]

(ii) Another camera is to be placed at the point P for effective coverage. Find the
acute angle between the planes UCB and PCB . [3]

(iii) As highly explosive items that need to be kept at low temperature are present in
the underground store room, the military officials also place an infra-red sensor
device at the point T. Determine the shortest distance of T from the plane PCB .
[3]

©RIVER VALLEY HIGH SCHOOL 9740/02/2016

Microsoft Word - RVHS_JC2_H2_Maths_2016_Prelim_P2_Question 499


2

2. (i) Solve the equation z 5  16  16 3i  0 , giving the roots in the form re i , where
r  0 and  π    π . [3]

(ii) The roots of the equation z 5  16  16 3i  0 are represented by


z1 , z2 , z3 , z4 and z5 , where  π  arg( z1 )  arg( z2 )  arg( z3 )  arg( z4 )  arg( z5 )  π .
Show all these roots on an Argand diagram. [2]

(iii) Given that the complex number v satisfies the equation v  z3  v  z4 , sketch

the locus of the points which represent v on the same Argand diagram.
Determine if this locus passes through the point which represent the complex
number z1 . [2]

(iv) Another complex number w is such that w  1 and w  z3  w  z4 . Shade on

the same diagram, the region representing w. Determine the range of value of
arg  w  z2  . [3]

sgexamguru.com π π
3. (i) It is given that y cos x  e x where  x .
2 2
dy
Show that  y (1  tan x) . [2]
dx
(ii) By further differentiation, find the Maclaurin series for y, up to and including the
term in x2 . [5]

(iii) Hence, find the set of values of x for which the value of y in part (i) is within
 0.05 of the value found by its Maclaurin series in part (ii). [3]

©RIVER VALLEY HIGH SCHOOL 9740/02/2016

RVHS P2 Questions 500


3

3x
4. The equation of a curve C is given by y  .
4 x

(i) State the largest range of values of x for C to be defined. [1]

For the rest of this question, define C for the range of values found in part (i).

(ii) Sketch C, stating the equations of any asymptotes and the coordinates of any
points of intersection with the axes. [2]

(iii) The region bounded by the curve, the line y  3 and the y-axis is denoted by R.
Find the exact value of the area of R. [5]

(iv) The region R is rotated 2π radians about the line y  3 to form a solid. Write
down the equation of the curve obtained when C is translated by 3 units in the
negative y-direction. Hence find the volume of the solid formed. [3]

Section B: Statistics [60 Marks]

5. A school has 200 teachers of whom 5% are in the 21 – 30 age group, 60% are in the

sgexamguru.com
31 – 40 age group and the rest are in the age group of 41 and above. During a meeting
held in a Lecture Theatre, the principal intends to obtain a sample of 20 teachers for a
survey. She decides to select 20 teachers from the last occupied row for the survey.

(i) Name the sampling method described. State a reason, in the context of the
question, why this sampling method is not desirable. [2]

(ii) Suggest a method of obtaining a representative sample and describe how it may
be carried out. [3]

4 2 11
6. , P( B ' | A ')  and P( A) 
For events A and B, it is given that P( A | B ')  . Give a
7 3 20
reason why events A and B are not independent. [1]
Find

(i) P( A  B) , [3]

(ii) P( A  B ') . [2]

©RIVER VALLEY HIGH SCHOOL 9740/02/2016

RVHS P2 Questions 501


4

7. Find the number of ways to arrange the nine letters of the word PERMUTATE,

(i) in a circle; [2]

(ii) in a row with exactly one pair of identical letters together; [3]

(iii) in a row with identical letters separated (for example “PERMUATET”,


“PERTUTAEM” etc…). [3]

8. In a college Mathematics examination with a large candidature, the percentage mark X


obtained by each male candidates was found to follow a Normal distribution with mean
 and standard deviation  . It is further found that P( X  45)  0.85 and
P( X  85)  0.15 .

(i) Find the value of  and  . [3]

(ii) Find the least integral value of a such that the probability of a male candidate
scoring less than a is at least 0.75. [2]

In the same examination, the percentage mark obtained by each female candidate was

sgexamguru.com
found to follow a Normal distribution with mean of 67 and standard deviation of 22.

(iii) Find the probability that the total percentage mark of two randomly selected
female candidates is more than three times that of a randomly selected male
candidate. [3]

One Mathematics teacher of the college claimed that the percentage mark of each
candidate in the examination followed a Normal distribution of mean of 66 and
standard deviation of 45. Comment on the validity of this claim. [1]

©RIVER VALLEY HIGH SCHOOL 9740/02/2016

RVHS P2 Questions 502


5

9. Company A packs and supplies salt in small packets. The mass of salt in one packet is
denoted by x grams(g). The company claims that the mean mass of salt per packet is at
least 0 g. A random sample of 12 packets of salt is taken and its mean and standard
deviation are found to be 9.81g and 0.217g respectively.

Find the range of value of 0 for which Company A’s claim will not be rejected at the
5% significance level. State any assumption that you have made. [6]

Another company, Company B, claims that the mean mass of salt supplied by them per
packet is 10 g. In a test against Company B’s claim at the  % significance level, the
hypotheses are as follows:

Null hypothesis:  B  10,


Alternative hypothesis:  B  10,

where  B is the population mean mass of salt in a packet of salt from Company B.

The p-value is found to be 0.0438 (corrected to 3 significant figures). Explain the


meaning of this p-value in the context of the question. [1]

sgexamguru.com
State the range of values of  for which Company B’s claim is not rejected. For the
range of values of  found, explain if Company B’s claim is still valid under a one-
tailed test. [3]

©RIVER VALLEY HIGH SCHOOL 9740/02/2016

RVHS P2 Questions 503


6

10. During the Arts Festival period, JC students in a college are encouraged to view the arts
exhibits put up by the Arts Talent Programme students in the canteen during lunch time
from 12 noon to 2pm daily. Given that the JC students do not influence each other in
their decision to view the exhibits, state a condition for the random variable X, which
denotes the number of students who view the exhibits in independent 15-minute
interval, to be well modelled by a Poisson distribution. [1]

Assuming that X has a Poisson distribution with mean 7.5, find

(i) the probability that less than 12 students view the arts exhibits from 12:30 pm to
1 pm on a particular day during the Arts Festival; [2]

(ii) the probability that there are three 30-minute intervals in which at least 12
students view the arts exhibits on a particular day during the Festival, [2]

(iii) the probability that the mean number of students who view the art exhibits daily
from 12 noon to 2 pm on 65 school days is more than 60.5. [3]

The organising committee decides to study the students’ support for the arts exhibits for
the Arts Festival. By using a suitable approximation, determine the probability that for

sgexamguru.com
the first 50 days, there are more than 25 days whereby there is only one 30-minute
interval among the four possible intervals daily from 12 noon to 2 pm, in which there
are less than 12 students who view the arts exhibits. [3]

©RIVER VALLEY HIGH SCHOOL 9740/02/2016

RVHS P2 Questions 504


7

11. The table below gives the time t, in minutes, taken by Andy to complete a particular
stage of a computer game x weeks after he has started playing the game.

x 1 2 3 4 5 6
t 48.2 32.5 22.7 18.0 16.4 14.6
(i) Draw a scatter diagram for the data, labelling the axes clearly. [2]

(ii) Calculate the product moment correlation coefficient and comment on why its
value does not necessarily mean that the best model for the relationship between x
and t is linear. [2]

(iii) Determine which of the following would be a better model for the set of data,
justifying your choice clearly:

(A) t  a( x  10) 2  b ,

(B) t  ce x  d ,

where a, b, c and d are positive constants. [2]

(iv) For the better model identified in part (iii), calculate the product moment

(v)
sgexamguru.com
correlation coefficient.

Andy would like to estimate the time taken by himself to complete the stage of
[1]

the game 10 weeks after he has started playing the game. Find the equation of a
suitable regression line and use it to obtain the estimate. Give two reasons why
the estimate is not reliable. [4]

END OF PAPER

©RIVER VALLEY HIGH SCHOOL 9740/02/2016

RVHS P2 Questions 505


Solutions to 2016 RVHS Y6 H2 Maths Preliminary Exam II (Paper 2)

Section A: Pure Mathematics [40 marks]

Question 1 [9 Marks]
(i) We first have the following position vectors:
0  20   20  0
           
OT   0  , OA   0  , OB   16  and OC   16 
 12  0 0 0
       
Then by Ratio Theorem:
Since U divides TB in the ratio 1:3,
 1  3 
OU  OB  OT
4 4
 20   0   5
1  3   
  16    0    4 
4  4   
0  12   9 

To find the vector equation of the plane UCB, we first have:


     
UB  OB  OU and CB  OB  OC
 20   5   15   20 
  16    4    12  =  0 
 0   9   9  0
       

sgexamguru.com
We then apply cross product of the vectors to obtain normal
vector to the plane UCB:
 15   20   0  0
      
UB  CB   12    0    180   60  3 

 9   0   240  4
       

Thus, the vector equation of the plan UCB in scalar product


0  0  0
    
form is then r  3   16   3   48
 4  0   4
    
(ii) For the plane PCB, we first need to find the normal vector to
the plane. We have:
 0   0  1
        
PC  CB    16    0     0 
  0   12    0 
     
 0  1  0   0
       
  16    0    12   4  3 
 12   0   16   4 
       
 0
Therefore a vector normal to the plane PCB will be  3  .
 4 
 

Pg 1
Microsoft Word - RVHS_JC2_H2_Maths_2016_Prelim_P2_Solutions 506
Thus, the acute angle  between plane UCB and PCB will be
given by:
0  0 
   
 3  3 
 4   4  9  16 7
cos        
32  42 32   4 
2
32  42 32   4 
2 25

That is,   73.7 (3 s.f.)


(iii) Method 1:

T (0, 0,12)
 0
n n = 3 
 
 4 
 
C (0,16, 0)
Plane PCB

We note that C  (0,16,0) is a point on the plane PCB and



thus we projected the vector CT onto n .
 0 
   
CT  OT  OC   16 
sgexamguru.com
 12 
 

The shortest distance of T to the plane PCB is given by:


 0   0
   
 16    3 
  12   4 
   
d  CT  n 
32   4 
2

48  48 96
  units
5 5
Method 2:

T (0, 0,12)
 0
n = 3 
 
 4 
 
F
Plane PCB

Let F be the foot of the perpendicular from point T to the


plane PCB.

Pg 2
RVHS P2 Solutions 507
 0

Then TF   n    3 
 4 
 
 0  0  0 
   
 OF  OT    3   OF  OT    3    3 
 
 4   4   12  4 
     

Since F is also a point on the plane PCB, OF will also
satisfy the vector equation of plane PCB.

 0   0 0  0
We then have  3    3   16    3  =48
12  4   4   0   4 
       
Thus,
9  4 12  4   48
 25  96
96
 
25
Therefore, the perpendicular distance is
 0
 96   96 2 96
3   4  
2
TF   3 units
25   25 5
 4 

Question 2 [10 Marks]


sgexamguru.com
(i) Given that z 5  16  16 3i  0 ,
 
i 
5
we have z  16  16 3i  32e  3  .

Then by solving, the roots are given by:


1 i 1    2 k 
 
z  32 5 e 5  3 

 1 6 k 
i 
 2e  15  where k  2, 1, 0, 1, 2.

Pg 3
RVHS P2 Solutions 508
(ii) Given that
   arg z1   arg z 2   arg z3   arg z 4   arg z5   
 we have the following roots:
 11   5  1   7 
i   i   i   i 
 15   15   15 
z1  2e , z 2  2e , z3  2e  15 
, z 4  2e
 13 
i 
 15 
and z5  2e .

The roots z1 , z 2 , z3 , z 4 and z 5 are then represented in the


following Argand diagram:
Im(z)
z4
2

z5
2 1
 2 
5
 15
5 z3
2 2
2  0  2 Re(z)
5 5
2

5
z1
z2

sgexamguru.com 2

Note that if the points Z1 , Z 2 , Z 3 , Z 4 and Z 5 represent the


complex number z1 , z 2 , z3 , z 4 and z 5 respectively on the
Argand diagram, then
2
Z1OZ 2  Z 2OZ 3  Z 3OZ 4  Z 4OZ 5  Z 5OZ1 
5

Pg 4
RVHS P2 Solutions 509
(iii)
Locus for point representing
Im(z) z4 complex number v
2

z5 p
2
 4π
5 z3
15
2 2
2  0  2 Re(z)
5 5
2

5
z1
z2
2
Locus for
complex no. v

We note that if p is the complex number represented by the


midpoint of the line joining Z3 and Z4, then
1 7
 
arg  p   15 15  4  .
2 15
4 11
Since arg  p   arg z1        , we can conclude
sgexamguru.com 15 15
that the point representing z1 lies on the locus of v.
(iv)
Im(z) z4
2

z5 1

z3

2 1 0 1 2 Re(z)

1
z1
z2
2

Shaded region represents possible region for complex


number w.

Pg 5
RVHS P2 Solutions 510
Im(z) z4
2

z5 1

z3

2 1 5

sgexamguru.com
0 π 1 2 Re(z)
15

1 
z1 T
 z2
2

To find range of arg  w  z2  , we note that :


5 2
OZ 2T     
15 3
1 
and sin     .
2 6

Thus the range of value of arg  w  z2  is


2 1 2 1
    arg  w  z2     
3 6 3 6
1 5
i.e.   arg  w  z2    or 1.57  arg  w  z2   2.62
2 6

Pg 6
RVHS P2 Solutions 511
Question 3 [10 Marks]
(i) Let y cos x  e x      (1)
 dy 
  cos x  y ( sin x)  e
x
 dx 
 dy 
  cos x  y sin x  y cos x
 dx 
dy cos x  sin x
y
dx cos x
dy
 y (1  tan x)      (2) (shown)
dx
(ii) Differentiate with respect to x:
d 2 y  dy 
2
   (1  tan x)  y (sec2 x)      (3)
dx  dx 

At x = 0,
e0
From (1): y  f (0)  1
cos 0
dy dy
From (2):  (1)(1  tan 0)   f '(0)  1
dx dx
From (3):
d2 y 2 d2 y
 (1)(1  tan 0)  (1) sec 0   f ''(0)  2
dx 2 dx 2

sgexamguru.com
 y  f ( x)  f (0) 
f '(0)
1!
x
f ''(0) 2
2!
x  .........

 1  x  x 2  ............ (up to x 2 term)


(iii) ex π π
Let f ( x)  , where   x  ,
cos x 2 2
2
and g( x)  1  x  x (from (ii)).
We thus solve f ( x )  g( x )  0.05 .

Using GC, we plot y  f ( x)  g( x) and y  0.05 :

x-values of the intersection points are:


–1.425019, –1.410526, –0.4704015 and 0.3795259

 Ans:  x : x  , 1.43  x  1.41 or  0.470  x  0.380

Pg 7
RVHS P2 Solutions 512
Question 4 [11 Marks]
(i) The largest range of values of x for curve C to be defined is
0 x  4.
(ii) y Vertical asymptote:
x4

3x
y
4 x

(0, 0) 4 x
(iii) The area R is as shown in the shaded region below:
y Vertical asymptote:
x4

3x
y
4 x

3 y3

sgexamguru.com
R

0 3 4 x

3x 3x
y then y 2 
4 x 4 x
 4 y 2  xy 2  3 x
4 y2
x
3  y2

Thus, the area of the region R


3

 0
x dy
3 4 y2

 0 3  y2
dy

3 12


0
4
3  y2
dy

3
 12  y 
 4 y  tan 1  
 3  3 0

  43 

12
3
tan 1  3    0
Pg 8
RVHS P2 Solutions 513
12 π
 12  
3 3
4 3
 12  π unit 2
3
(iv) The area R is rotated 2 radian about the line y  3 :
y Vertical asymptote:
x4

3x
y
4 x

3 y3

0 3 4 x

The equation of curve when C is translated by 3 units in the


3x
y-direction is y   3.
4 x
Next, let
3x 3x
y3  3  9  x3

sgexamguru.com
4 x 4 x
The volume V generated when R is rotated completely about
y  3 is the same as the volume generated when the new
curve is rotated about the x-axis from x  0 to x  3 .
Therefore,
3
V
0
πy 2 dx
2
3  3x 
π
 0

 4 x
 3  dx

 28.6 unit 3

Section B: Statistics [60 marks]

Question 5 [5 Marks]
(i) Quota Sampling.
This method is non-random as not every teacher has an equal
chance of being selected. OR
This method does not give a representative sample.
(ii) Stratified Sampling.
Principal can draw random samples from each stratum as
follows:
Age group 21 - 30 31 - 40 41 and above
No. of teachers 0.05  20  1 0.6  20  12 20  1  12  7
selected

Pg 9
RVHS P2 Solutions 514
Question 6 [6 marks]
Since P( A)  P( A | B ') , events A and B’ are not independent.
Thus A and B are not independent.

(i) 2 P( B ' A ') 2


P( B ' | A ')  
3 P( A ') 3
2 9 3
 P( B ' A ')   
3 20 10
7
P( A  B)  1  P( B ' A ') 
10

(ii) 4 P( A  B ') 4
P( A | B ')   
7 P( B ') 7
P( A  B ') 4
 
P( A  B ')  P( A ' B ') 7
P( A  B ') 4
 
3 7
P( A  B ') 
10
2
 P( A  B ') 
5

Question 7 [8 Marks]
(i) (9  1)!
 10080
sgexamguru.com
2!2!
(ii) For the case where EE are together and T, T separated:
Arrange EE, P, R, M, U, A first, then slot in T, T to separate
them.
7
No. of ways = 6!     15120
 2
Hence, required no. of ways = 15120  2  30240

(iii) 9!
No. of ways without restrictions =  90720
2!2!
No. of ways with ‘E’s together & ‘T’s together = 7! = 5040

Hence, required no. of ways


= No. of ways without restrictions
– n(each of the pair of identical letters together)
– n(exactly one of the pairs is together)
= 90720 – 5040 – 30240 = 55440

ALTERNATIVELY
Case 1: E, E, T, T all separated
 6  4!
No. of ways = 5!      10800
 4  2!2!

Pg 10
RVHS P2 Solutions 515
Case 2: with exactly 1 E and 1 T together (i.e. ET, E, T
separated)
6
No. of ways = 5!     3!  2  28800
 3

Case 3: 2 pairs of ET, but the pairs separated


6
No. of ways = 5!     (2!) 2  7200
 2

Case 4: ETE or TET together


 6 
No. of ways = 5!     2!  2  7200
  2 

Case 5: ETET or TETE


No. of ways = 6!  2  1440

Hence, required no. of ways


= 10800 + 28800 + 7200 + 7200 + 1440 = 55440

Question 8 [9 Marks]
(i) P( X  45)  0.85  P( X  45)  0.15
Since P( X  85)  0.15 , by symmetry property of normal
45  85
distribution curve,  
sgexamguru.com
 65
2
Next,
P( X  45)  0.15
 45   
 PZ   0.15
  
 45  65 
 PZ   0.15
  
45  65
  1.03643338

   19.296947  19.3 (3 s.f.)

ALTERNATIVE METHOD

y = P(X < 45)

y = 0.15

  19.296947  19.3 (3 s.f.)

Pg 11
RVHS P2 Solutions 516
(ii) If the probability of a male candidate scoring less than a is at
least 0.75, then we have P( X  a )  0.75 .

X ~ N (65,19.296947 2 )
P( X  a )  0.75
 a  78.01559295
Thus, the least integral value of a is 79.

(iii) Let Y denotes the random variable that represents the


percentage marks obtained by a randomly selected female
candidate in the Maths exam. Then Y ~ N (67, 222 ) .
We first note that
P(Y1  Y2  3 X )  P(Y1  Y2  3 X  0)

Since X ~ N (65,19.296947 2 ) , we have


E(Y1  Y2  3 X )  67  67  3  65  61
Var(Y1  Y2  3 X )  222  222  32  19.296947 2
 4319.349472
Then Y1  Y2  3 X ~ N (61, 4319.349472)
Thus,
P(Y1  Y2  3 X )
 P(Y1  Y2  3 X  0)
 0.177 (3 s.f.)

sgexamguru.com
We note that for a random variable W with normal
distribution of mean 66 and standard deviation 45,
P(W  0)  0.0712334139 which is non negligible.
As W  0 , we conclude that the teacher’s claim is not valid.
OR:
If W ~ N (66, 45) , then 99.7% of the values would lie within
66  345, which contains a significant range of negative
values.
Thus, the teacher’s claim is not valid.

Question 9 [10 Marks]


Let  be the mean mass of salt in a packet of salt from
Company A.

Test H 0 :   0 against H1 :   0
at 5% significance level
X  0
Test statistic: Under H0, T  ~ t (11) ,
s
12
12
where s 2   0.217 2   0.0513698182
11
Critical region:
Reject H0 if tcalc < –1.795884781

Pg 12
RVHS P2 Solutions 517
Since H0 is not rejected,
9.81  0
 1.795884781
0.0513698182
12
0  9.927501081
i.e. 0  9.93 (to 3s.f.) (OR 0  9.92)

We assume that mass of salt follows a normal distribution.

p-value of 0.0438 is the lowest level of significance for


which Company B’s claim that the mean mass of salt per
packet is 10 g is rejected.

To not reject Company B’s claim (i.e. to not reject H0),



p-value 
100
   0.0438  100
i.e.   4.38

Supposed a one-tailed test is conducted instead, i.e.


testing H 0 :  B  10 against H1 :  B  10
0.0438
p-value =  0.0219 .
2

sgexamguru.com
Then we would not be certain if p-value <  since
  4.38 . (no conclusion)

Question 10 [11 marks]


X can be well modelled by a Poisson distribution if
(i) the mean number of JC students who viewed the arts
exhibits in each 15-minute interval remains constant, or
(ii) the mean number of JC students who viewed the arts
exhibits in other time intervals is proportional to the mean
number for each 15-minute interval.

(i) Let X’ denotes the random variable that represents the


number of students who view the arts exhibits in a 30-minute
period. Then X’ Po(15) since X Po(7.5)
Then the required probability
=P(X’ < 12) = P(X’  11)
= 0.184751799
= 0.185 (3 s.f.)

Pg 13
RVHS P2 Solutions 518
(ii) The required probability
4
   (1  0.184751799)3 (0.184751799)1
 3
=  4! (0.815248201)3 (0.184751799)1
3!1!
 0.400422262
 0.400 (3 s.f.)
OR:
Let Y denotes the number of 30-minute intervals among the
4 possible ones from 12 noon to 2 pm in which there are at
least 12 students who view the arts exhibits.
Then Y  B(4,10.184751799) = B(4, 0.815248201)
Then required probability = P(Y = 3) = 0.400422262
= 0.400 (3 s.f.)

(iii) Let W denotes the random variable that represents the


number of students who view the arts exhibits from 12 noon
to 2 pm on a randomly chosen day during the Arts Festival.
Since X Po(7.5), we have W Po(7.58) i.e. Po(60)

For the 65-day period, n = 65 (> 50) is large.


By the Central Limit Theorem (CLT),
 60 
W N  60,  approximately.
 65 

sgexamguru.com
Thus, P(W  60.5) = 0.3013866388
= 0.301 (3 s.f.)

Next, let V be the random variable that denotes the number


of days among 50 whereby there is only one 30 minutes
interval among the 4 possible intervals daily, in which there
are less than 12 students who view the arts exhibits.
Then V B(n, p ) where n = 50, p = 0.400422262 (part ii)

Then upon checking,


n = 50 is large,
np = 50  0.40042262 = 20.0211131 > 5,
nq = 50  (1  0.40042262) = 29.9788869 > 5
We conclude that V N (np, npq) approximately.
That is, V N (20.0211131,12.00421727) approximately.

Then, P(V > 25)


 P(V  26)
 P(V  25.5) (continuity correction)
 0.0569000711
 0.0569 (3 s.f.)

Pg 14
RVHS P2 Solutions 519
Question 11 [11 marks]
(i) t
48.2

14.6

1 6 x
(ii) r  0.9168956355  0.917

Although r is close to 1, suggesting a linear relationship


between x and t (i.e t  mx  k ), it does appear that the
points follow a curvilinear trend from the scatter diagram.
(iii) (B) t  ce  x  d is a better model because as x increases, t
decreases at a decreasing rate.
As for the model t  a( x  10) 2  b , t might decrease at a
decreasing rate initially as x increases (for x < 10) but it will
increase eventually (for x > 10). This does make sense for
the context of the question since more practice on a longer
period ought to improve his proficiency thus taking less time
to complete the stage of the computer game.
(iv) For t  ce x  d : r  0.9852371391  0.985
For t  a( x  10) 2  b : r  0.954421565  0.954
(v)
sgexamguru.com
A suitable regression line:
t  89.60094125 e  x  16.73059924
i.e. t  89.6 e  x  16.7

When x = 10,
t  89.60094125 e 10  16.73059924
 16.73466712  16.7 mins

The estimate is not reliable because:


(1) x = 10 is outside of data range (extrapolation)
(2) Andy has already obtained timings less than 16.7
minutes for both week 5 and week 6. It is more likely
that the timing will be shorter than 16.7 min by week 8.
_______________________________________________
If the wrong line is chosen:
t  0.5024319662( x  10) 2  2.706822862
When x = 10.
t  0.5024319662(2) 2  2.706822862
 4.716550727  4.72 mins

The estimate is not reliable because x = 10 is outside of data


range (extrapolation).

Pg 15
RVHS P2 Solutions 520
SAINT ANDREW’S JUNIOR COLLEGE

Preliminary Examination

MATHEMATICS
Higher 2 9740/01

Paper 1

Monday 29 August 2016 3 hours

Additional materials : Answer paper


List of Formulae (MF15)
Cover Sheet

READ THESE INSTRUCTIONS FIRST

sgexamguru.com
Write your name, civics group and index number on all the work you hand in.
Write in dark blue or black pen on both sides of the paper.
You may use a soft pencil for any diagrams or graphs.

Answer all the questions. Total marks : 100

Give non-exact numerical answers correct to 3 significant figures, or 1 decimal place in the
case of angles in degrees, unless a different level of accuracy is specified in the question.
You are expected to use a graphic calculator.
Unsupported answers from a graphic calculator are allowed unless a question specifically
states otherwise.
Where unsupported answers from a graphic calculator are not allowed in a question, you are
required to present the mathematical steps using mathematical notations and not calculator
commands.
You are reminded of the need for clear presentation in your answers.

The number of marks is given in brackets [ ] at the end of each question or part question.
At the end of the examination, fasten all your work securely together.

This document consists of 7 printed pages including this page.


[Turn over

Microsoft Word - 2016 SAJC Prelim Paper 1(updated) 521


2

n
1
1 The sum  (3r  2)(3r  1)
r 1
is denoted by S n .

(i) By using the method of differences, find an expression for S n in terms of n. [3]

(ii) Hence find the value of S n as n tends to infinity. [1]

(iii) Find the smallest value of n for which S n is within 2  104 of the sum to infinity. [3]

n
1
(iv) Using your answer in part (i), find   3r  1 (3r  4)
r 0
and deduce that

n
1 1
 (3r  4)
r 0
2
 .
3
[4]

2 (a) By completing the square, or otherwise, state precisely a sequence of geometrical


transformations which would transform the graph of y  ln(4 x 2  16 x  15) onto the
1
graph of y  ln(4 x 2  1) . [3]
2
(b) The diagram shows the graph of y  f  x  . The graph passes through the origin and
the point (a  2,0) . It has a minimum point at (a, b) , a  1, b  1 . The graph also

sgexamguru.com
has a vertical asymptote x  a and a horizontal asymptote y  a .

y = f (x) 

y =a 

Sketch, on separate diagrams, the graph of:


1
(i) y ; [2]
f  x
(ii) y  f ' x , [2]
showing clearly all the asymptotes, turning points and axes intercepts.

[Turn Over]
SAJC P1 Questions 522
3

3 (a) John depposits $x intto a bank at the beginniing of each year. The bbank pays in nterest at
a fixed raate of 5% of the amounnt at the end d of each yeear. John thhen withdrawws the
interest as
a soon as itt is added. Find, in termms of x andd N, the totaal amount off interest
he will collect
c at thee beginningg of (N +1)th
h year. [3]

(b) An agriccultural farm


m has 2000kkg of vegetaables. At th he end of eacch week, th he farm
sells 10%
% of the veggetables andd grows ano other 80kg ono the farm..
(i) Finnd the amou he end of ntth week, exp
unt of vegettables the faarm has at th pressing
n the form A( B )  C , where A, B and C are cconstants to
youur answer in n
o be
dettermined. [3]
(ii) At which week will the aamount of veegetables in n the farm bbe first less than
t
835kg? [2]

4 (i) Find  u 2 eu du . [3]

(ii) uation y  lln x  1 as shown belo


The curvve C has equ ow.

y
y = ln x +
+1

sgexamguru.com R
y=1

x
O 1 e

nd the lines x  1 , x = e and y = 1..


The region R is bounded by thee curve C an

uation of thee curve by trranslating C one unit inn the negative y-


Write doown the equ
directionn.

Hence, using
u bstitution u = ln x, evalluate the ex
the sub xact volumee generated when
w R is
rotated completely
c about
a y 2 radians.
the linne y  1 by [4]

[Tu
urn Over]
SAJC P1 Questions 523
4

5 The functions f and g are defined by

f : x  e 2x  2e x + 3, x  

g : x  ln (2 – x), x   , x < 2

(i) By sketching a graph, explain why the inverse function f 1 does not exist.
[2]
(ii) Given that the domain of f is restricted to (  ,a] , state the maximum value of a for
which f 1 exist. [1]
(iii) Using the value of a found in (ii) and by completing the square, find the inverse
function f 1. [3]
(iv) Find the exact range of gf –1. [2]

dy
6 Given that y  4  sin 2 x , show that y  cos 2 x . [1]
dx
(i) By further differentiation of the above result, find the Maclaurin series for y in
ascending powers of x up to and including the term in x2 . [3]
(ii) Verify the correctness of the series found in (i) by using an appropriate standard
series expansion. [2]

sgexamguru.com

(iii) Deduce from part (i) the approximate value of
0
0.1
4  sin 2 x dx , giving your

answer to 5 significant figures. [2]

7 Relative to the origin O, the position vectors of two points A and B are a and b
respectively, where a and b are non-zero and non-parallel vectors. The vector a is a unit
vector which is perpendicular to  a   b , where   1 and  > 1 and the angle between
5
a and b is .
6
2 3  
(i) Show that b   . [3]
3  
(ii) Give the geometrical interpretation of a  b and find its value in terms of  and  .
[3]

(iii) The point M divides AB in the ratio  :1   where 0    1 . The point N is such

that OMNB is a parallelogram. Find ON in terms of a and b and the area of the
triangle OAN in terms of  ,  and  . [5]

[Turn Over]
SAJC P1 Questions 524
5

8 The variables w, x and y are connected by the following differential equations:

dw 3
  w  2 (A)
dx 2

dy
w (B)
dx

(i) Solve equation (A) to find w in terms of x. [3]


(ii) Hence find y in terms of x. [2]
(iii) The result in part (ii) represents a family of curves. Some members of the family are
straight lines. Write down the equation of one of these lines. On a single diagram,
sketch your line together with a non-linear member of the family of curves that has
your line as an asymptote, indicating clearly any axes intercepts. [3]

9 (a) (i) Solve z 3  1  i 3 , giving your answers in the form r eiθ , where r  0 , and
     . [3]

(ii)
sgexamguru.com
Show that

z n
 2ei  z n

 2ei  z 2n  4 z n cos  4 ,

Hence find the roots of the equation

z 6  2 z 3  4  0 in the form of r eiθ , where r  0 , and      . [3]

(b) Given that z  cos   i sin  , show that 1  z 2  (  2i sin  ) z . Given also that
0     , find the modulus and argument of 1  z 2 in terms of  . [5]

[Turn Over]
SAJC P1 Questions 525
6

10

3x 4x 3x
5x
10x
5x 5x
5x
10x
4x
4x Figure 1 Figure 2

A designer decided to build a model as shown in Figure 1 above, consisting of a base and
a top. The base is made up of a prism with a cross-section of a trapezium where the length
of the parallel sides are 4x cm and 10x cm (Figure 2). The top is a right pyramid with a
square base of sides 10x cm, height h cm and a fixed slant height of 5 cm.

sgexamguru.com
(i) Find an expression for the volume of the model, V, in terms of x. Given that x  x1 is
the value of x which gives the maximum value of V, show that x1 satisfies the
equation 13563 x 4  7719 x 2  625  0 . [6]

(ii) Find the two solutions to the equation in part (i) for which x > 0, giving your answers
correct to 5 decimal places. [2]

(iii) Using both the solutions found in part (ii), show that one of the values does not give a
stationary value of V. Hence, write down the value of x1 . [3]

1
[Area of trapezium =  sum of parallel sides   height ;
2

1
Volume of pyramid = base area  height ]
3

[Turn Over]
SAJC P1 Questions 526
7

11 The line l1 and the planes p1 and p 2 have equations as follows:

l1 : x  5   y  1, z  4 ;

p1 : xa  z  5a  4 ,

1 0
   
p2 : r = 1  1
0 1
   

where a is a positive constant and  and  are real numbers.


(i) Given that the acute angle between l1 and p1 is , show that a  1 . [2]
6

(ii) The planes p1 and p 2 meet in the line l2 . Find a vector equation of l2 . [2]

(iii) Hence, find the values of  and  such that the system of equations
x z 9
x z  y
5x  4 y   z  

(a) sgexamguru.com
more than one solution;
has

(b) exactly one solution.


If   5,  =10 , give a geometrical interpretation of the relationship between the 3
equations. Explain your answer. [6]

End of Paper

[Turn Over]
SAJC P1 Questions 527
2016 Prelim Paper 1 Solution
n
1(i) 1
 (3r  2)(3r  1)
r 1

1 n  1 1 
    
3 r 1  3r  2 3r  1 
1 1
 [ 1
3 4
1 1
+ 
4 7
+ 
1 1
+ 
3n  5 3n  2
1 1
+  ]
3n  2 3n  1

1 1 
= 1  
3  3n  1 

1(ii) 1 1
As n  , S n  since 0
3 3n  1

(iii)
1 1  1
sgexamguru.com
S n  S  2 104
4
1     2  10
3  3n  1  3
1 1  4
    2 10
3  3n  1 
1 1  4 
   2 10 since n 
3  3n  1 
1 3

3n  1 5000
5000
3n  1 
3
n  555.2

Hence, smallest n = 556.


n
(iv) 1 1 1 1 1
From (i)      ...  =
r 1 (3r  2)(3r  1) (1)(4) (4)(7) (7)(10) (3n  2)(3n  1)
1 1 
1  
3  3n  1 


 
Microsoft Word - 2016 SAJC Prelim Paper 1 Solutions 528
n
1 1 1 1 1 1
  3r  1 (33r  4)  (1))(4)  (4)(7))  (7)(10)  ...  (3n  2)(3
r 0

2 n  1) (3n  1)(3n  4)

n 1
1 1 1 
=  1  
(3r  1) 3  3n  4 
r 1 (3r  2)(

 3r  4    3r  1 (3r  4) for r  0, r 
2

1 1

 3r  4 
2
 3r  1 (3r  4)
n n
1 1 1
 
r  0  3r  1 (3r  4)

 3r  4 
2
r 0 3

1 
siince 1   1 for n     {0}}
3n  4

2 (a) y  ln(4 x 2  16 x  15)


 ln(4( x 2  4 x)  15)
 ln(4( x  2)) 2  16  15)
 ln(4( x  2)) 2  1)

sgexamguru.com
1 1
Let f(x) = ln(4( x  2) 2  1) and y 
L ln(4 x 2  1) = f(x + 2)
2 2

A translationn to the 2 units


u in the direction off the x– axis followed bby;

A scaling of a scale facctor of ½ parrallel to y– axis .

(b)(i) 1
y
f  x y 


 


 
SAJC P1 Solutions 529
(ii) y  f ' x y 

  x 

3(a) Year Amount withdrawn at the beginning of N years


No

1 0

2 0.05x

3 0.05(2 x )

… sgexamguru.com
N +1 TN  0.05 Nx

S N 1  0.05 x(1  2  3...N )

0.05 xN ( N  1)

2
xN ( N  1)

40

(i) Week Amount of vegetables on the farm


No

1 2000 (0.9) +80

2   (2000  0.9)  80   0.9  80

 (2000  0.92 )  (80  0.9)  80


 
SAJC P1 Solutions 530
3   (2000  0.92 )  (80  0.9)  80   0.9)  80
 (2000  0.93 )  (80  0.92 )   80  0.9   80

….

n  (2000  0.9n )  (80  0.9n1 )...   80  0.9   80

 (2000  0.9 n )  80 0.9 n 1  0.9 n  2  ...1

 (1  0.9n ) 
 (2000  0.9n )  80  
 1  0.9 
 (2000  0.9n )  800(1  0.9n )
 (1200  0.9n )  800

A =1200, B = 0.9 , C = 800

(ii) (1200  0.9n )  800  835


Alternatively use GC
Solving,
When n = 33, (1200  0.9n )  800 =837.08

35
sgexamguru.com
(1200  0.9n )  35
When n = 34, (1200  0.9n )  800 =833.38
0.9n 
1200 When n = 35, (1200  0.9n )  800 = 830.04 
n  33.549

Least number is 34 weeks

4(i) dV
U  u2  eu
du
dU
 2u V  eu
du

u e
2 u
du
 u 2 eu   2  ueu du

dV
U u  eu
du
dU
1 V  eu
du


 
SAJC P1 Solutions 531
u 2 eu  2  ueu du


 u 2 eu  2 ueu    eu du 
 u 2 eu  2ueu  2eu  C , where C w
was an arbitrrary constan
nt

4 At x = 1, y = 1
A

At x = e, y = 2
A

y
x=1 x=e

y = ln
nx +1

B(e,22)
A(1,1) y=1

sgexamguru.com
After translaations, the grraph is:
A

y1  ln x , with
w A’ (1 , 0) and B’ (ee , 1)

u  ln x
x  eu
dx
 eu
ddu
x  1, u  0
x  e, u  1

e
V     lnn x   dx
2
1  
1 dx
   u2 du
0 ddu
1
   u 2 e u du
0
1
  u 2 eu  2ueu  2eu 
0

   e  2 e  2e  2 
  (e  2)) units3


 
SAJC P1 Solutions 532
5 y 
y

(i) y = e 2x  2e


  e x + 3 
 

y = 3 

   2 

As the horizoontal line y = k , k   2 ,3 cuts thee graph y = f(x)


A f more thhan once, f is not
onne - one. Heence f 1 do
oes not existt.

(ii) M
Max a = 0.

(iii) Let y = e 2x  2e x + 3
L
y = (e x)2  2e
2 x+ 3
y = (e x  1)2 + 2

x = ln(1 y  2) ( rejeected, since x ≤ 0 ) .


sgexamguru.com
or ln(1  y  2)

 f1 : x  ln(1  x  2) , x  , 2  x  3.

(iv) f
[22,3) 
1
g
 (  ,0] 
  [ln2,)
R gf 1  [ln2,)

Grapph of g when n Dg is 
y  x = 2  resttricted to  (  ,0]

ln2 

x
   2 

   


 
SAJC P1 Solutions 533
6 y  4  sin 2 x

y 2  4  sin 2 x

Differentiating implicitly with respect to x,

dy
2y  2 cos 2 x
dx

dy
y  cos 2 x …. (1)
dx

(i) Differentiating (1) implicitly with respect to x,


2
d 2 y  dy 
y     2sin 2 x …. (2)
d x 2  dx 

When x = 0, y  4  0  2

dy 1
From (1) 
dx 2
2
d2 y  1  d2 y 1
From (2)
sgexamguru.com
2 2   0 
dx  2  dx 2

8

The Maclaurin’s Series of y is

1 1 x2
y  2 x   
2 8 2!

1 x2
y  2 x  , up to and including the term in x2
2 16

(ii) By using the standard series of sin x,

sin 2 x  2 x

4  sin 2 x  4  2 x
1
 4  2x 2
1
 x 2
 2 1  
 2


 
SAJC P1 Solutions 534
 1  x  1  1  x 2
4  sin 2 x  2 1     2 2    
 
 2 2 2!  2  
 
 x x2 
 2 1    
 4 32 
 
x x2
 2  
2 16

(iii) 0.1 0.1

 0
4  sin 2 x dx 
 0
4  sin(2 x) dx


0.1
x x2 

0 
 2    dx (Replace x with  x )
2 16 
 0.19748 (to 5 s.f.)

7 (i) a   a   b   0

2
 a  a  b  0

  a  b  0

ab  

 sgexamguru.com
5
Since angle between a and b is ,
6

 5  ab
cos  
 6  a b



3 
 
2 b

2 3  
b    (shown)
3  
Or


 
SAJC P1 Solutions 535
O

   a
cos   
6  b
a
b 
 
 cos  
6  

b 
3

2

sgexamguru.com
2
3
2 3  
  
3  

(ii) a  b is the length of projection of b onto a

 5 
a  b  a b cos  
 6 
3
b
2
2 3   3
  
3   2
 
 
 

(iii) By Ratio theorem,



OM  b  1    a


 
SAJC P1 Solutions 536
  
ON  OM  MN

 b  1    a   OB
 b  1    a   b
 (  1)b  1    a

Area of triangle OAN

1  
 OA  ON
2
1
 a     1 b  1    a 
2

1
    1 a  b  1    a  a
2
1
    1 a  b since   1    1 as 0< <1
2
1  5 
    1 a b sin  
2  6 
   1  2 3  
sgexamguru.com
 1 
     
2  3    2 


   1 3  
 
6  

8(i) dw 3
 ( w  2)
dx 2
1
3 dw   1 dx
w2
2
3
2 2
3  3 w 2
dw   1 dx

2
2 3
ln w  2   x  A where A is an arbitrary constant
3 2

3 3 3
ln w2   x A
2 2 2

10 
 
SAJC P1 Solutions 537
3 3 3
ln w2   x A
2 2 2
3 3 3 3
3  x A A  x
w2  e 2 2  e .e 2
2
2
3 3 3 3
3 A  x  x A
w  2   e 2 .e 2  Be 2 where B  e 2
2
3
2  x 4
w Be 2 
3 3
3
 x 4 2
w  Ce 2  where C  B
3 3

8(ii) 3
dy  x 4
 Ce 2 
dx 3
dy   3
x 4
 dx d x    Ce 2   dx
3 

3
2  x 4
y   Ce 2  x  E where E is an arbitrary constant
3 3
3
 x 4 2

sgexamguru.com
y  De 2  x  E where D   C
3 3

8(iii) 4
When D = 0 (or C = 0) and E = 0, y   x
3

3
When D = 1 (or C =  ) [or any value of D or C] and E = 0 [or any value of E
2
corresponding to the choice of E above],
3
 x 4
ye 2
 x
3 y

( – 3, 4)
1
0.407
O x

11 
 
SAJC P1 Solutions 538
9 (a) (i)
z3 = 1 - i 3
æ π ö
i ççç- +2 kπ ÷÷÷
çè 3 ø÷
= 2e ,k Î
æ ö
1 i 1 çç- π +2 kπ ÷÷
ç ÷
3ç 3 ø÷
z= 23 e è , k = 0, 1
æ ö
1 i çç- π + 2 kπ ÷÷
ççè 9 3 ÷÷ø
= 23 e , k = 0, 1

1 5π 1 π 1 7π
i i i
z 23 e 9 , 23 e 9 , 23 e 9

(ii)

(z n
- 2eiθ)( z - 2e )
n -i θ

= z2n - 2 z (e + e ) + 4
n iθ -iθ

= z 2 n - 2 z n (cos θ + i sin θ + cos(-θ ) + i sin(-θ )) + 4


= z 2 n - 2 z n (cos θ + i sin θ + cos θ - i sin θ ) + 4

sgexamguru.com
= z 2 n - 4 z n cos θ + 4 (shown)

Hence,

z6  2z3  4  0

1
z6  4   z3  4  0
2

 π
z 6  4  cos  z 3  4  0
 3

 3 i 
π
3
i 
π
 z  2e 3  z  2e 3   0
  
  
π
-i
3
The roots of are z = 2e 3

1 5π 1 π 1 7π
i i i
z 23 e 9 ,2 e 9
3 ,2 e 9
3 (from (i))

Since the coefficients of the equation are all real, complex roots occur in conjugate
12 
 
SAJC P1 Solutions 539
pairs.

π 1 5π 1 π 1 7π
i i i i
Therefore, for z = 2e 3 3 , the roots are z  23 e 9 , 23 e 9 , 23 e 9

9(b) 1  z 2  1  (cos 2  i sin 2 )

 1  cos 2  i(2sin  cos  )

 2 sin 2   i(2 sin  cos  )

 (2i sin  )(cos   isin  )

 (2isin  )z (shown)

Alternatively :

1  z 2  1  ( e i 2 )
 e i (e  i  e i )
 e i (cos   i sin   cos   i sin  )
 z ( 2i sin  ) (Shown)

1  z 2  2i sin  z  2 sin 


sgexamguru.com
arg(1  z 2 )  arg(2i sin  )  arg( z )
 arg(2sin  )  arg(i)  arg( z )


 
2

10(i) Let V be the total volume of the solid and h be the height of the pyramid.

Height of trapezium, l

 25 x 2  9 x 2
 16 x 2
 4x

1
Vbase   4 x  10 x  4 x 10 x 
2
 280 x 3

13 
 
SAJC P1 Solutions 540
10x

m
10x

m  2 10 x 
2

 10 2 x

m
 5 2x
2

5
h

m/2

sgexamguru.com
 
2
h  25  5 2 x

 25  50 x 2

V pyramid 
1
3

10 x  25  50 x 2
2


100 x 2
3

25  50 x 2 
100 x 2
V  280 x3  25  50 x 2
3

14 
 
SAJC P1 Solutions 541
dV 100  2  1  
 840 x 2  x    100 x   2 x 25  50 x 2 
dx 3   2 25  50 x 2  
100  50 x 3 
 840 x 
2
2 x 25  50 x 
2

3  25  50 x 2 
100  25 x 2 
 840 x 2   
2 x 25  50 x 2
 
3  25  50 x 2 
20   25 x 2  
 x 126 x  10  25  50 x 
2

3   25  50 x 2  

For stationary values of V,

dV
0
dx

Since x > 0,

 25 x 2 
126 x  10  25  50 x 2  0
 25  50 x 2 

 25 x 2 
126 x  10  25  50 x 
sgexamguru.com
2

 25  50 x 
2

126 x 25  50 x 2  10  25  50 x 2   25 x 2 

126 x 25  50 x 2  10  25  75 x 2 
 750 x 2  250

Squaring both sides,

    750 x 2  250 
2 2
126 x 25  50 x 2

15876 x 2  25  50 x 2   562500 x 4  375000 x 2  62500


396900 x 2  793800 x 4  562500 x 4  375000 x 2  62500
3969 x 2  7938 x 4  5625 x 4  3750 x 2  625
13563 x 4  7719 x 2  625  0

(ii) Using G.C., since x > 0, the two values of x are 0.6865562≈0.68656 or
0.3126699≈0.31267 (to 5 dp).

(iii) dV 20   25 x 2  
 x 126 x  10  25  50 x 2  
dx 3   25  50 x 2  

15 
 
SAJC P1 Solutions 542
For x = 0.6865562,

dV 20
  0.6865562  126  0.6865562 
dx 3
 
 
2
25 0.6865562  
10  25  50  0.6865562  
2

 25  50  0.6865562 
2 
  
 0.03789
0

For x = 0.3126699,

dV 20
  0.3126699  126  0.3126699 
dx 3
 
 
2
2 5 0.3126699  
10  25  50  0.3126699  
2

 25  50  0.3126699 
2 
 
 164.24
0

Hence x1  0.68656

11 (i) 5
sgexamguru.com
1
l1 : r   1    1
 
4 0
   

a
p1 : r   0   5a  4
1
 


Acute  between l1 and p1 =
6

1 a 
 Acute  between  1 and  0  =
0 1 3
   

16 
 
SAJC P1 Solutions 543
a  1 
 0    1 
1  0 
     
ccos   
3 a2  1 2
1 a

2 2  a 2  1

2  a 2  1  4a 2
2a 2  2
a  1

Since a  0 , a  1 . (proven)

(ii) 1  0
   
p2 : r =   1     1
 0  1
   

1 0  1 
     
Normal vector of p2 =  1    1    1
N
     
0 1  1 

1
sgexamguru.com
p2 : r   1  0
1
 

Using GC, soolving x  z  9 and x  y  z  0 ,


U

x 9 z
y 9
zz

T
Therefore eqquation of l2 ,

 9   1
   
r  9  s  0 , s 
      
0  1 

(iii) x  z  9 1
x  z  y 2
5 x  4 y   z     3

(1) and (2) are the carttesian equattions of plan


nes p1 and p2 respecttively.

17 
 
SAJC P1 Solutions 544
5
Let p3 : r   4   
L
 
 

Since the sysstem of equations is knnown to hav n one solutioon, p1 , p2 and


ve more than
p3 intersect at
a l2 . Therefore, l2 liess in p3 .

 1  5 
   
 0  4   0
 1   
   
5    0
 5

9 5
9  4  
0 5
   
  81

Since the sysstem of equations is knnown to hav ne solution,, p1 , p2 and


ve exactly on d p3
inntersect at a point. Therrefore, l2 inntersects p3 at a point.

 1  5 
    sgexamguru.com
 0  4   0
 1   
   
5    0
 5



Since   5 and   81
1 , the plane s do not intersect at a common
c poiint.

Since the 3 planes


p are not parallel tto each otheer, the 3 plan
nes form a ttriangular prism.
p

18 
 
SAJC P1 Solutions 545
SAINT ANDREW’S JUNIOR COLLEGE

Preliminary Examination

MATHEMATICS
Higher 2 9740/02

Paper 2

Thursday 15 September 2016 3 hours

Additional materials : Answer paper


List of Formulae (MF15)
Cover Sheet

READ THESE INSTRUCTIONS FIRST

sgexamguru.com
Write your name, civics group and index number on all the work you hand in.
Write in dark blue or black pen on both sides of the paper.
You may use a soft pencil for any diagrams or graphs.

Answer all the questions. Total marks : 100

Give non-exact numerical answers correct to 3 significant figures, or 1 decimal place in the
case of angles in degrees, unless a different level of accuracy is specified in the question.
You are expected to use a graphic calculator.
Unsupported answers from a graphic calculator are allowed unless a question specifically
states otherwise.
Where unsupported answers from a graphic calculator are not allowed in a question, you
are required to present the mathematical steps using mathematical notations and not
calculator commands.
You are reminded of the need for clear presentation in your answers.

The number of marks is given in brackets [ ] at the end of each question or part question.
At the end of the examination, fasten all your work securely together.

This document consists of 7 printed pages including this page.


[Turn over

Microsoft Word - 2016 SAJC Prelim Paper 2 546


2

Section A: Pure Mathematics [40 marks]

n
 r   n n 1 
1 Prove by the method of mathematical induction that  ( r  1) ln  
 r 1 
 ln 

 . [5]
r 2  ( n 1)! 

x2  6 x  8
2 Solve the inequality 0 .
x 1

y 2  2 y  15
Hence, by completing the square, solve the inequality   6. [6]
| y  1| 1

5  ax 2
3 It is given that f ( x)  where a  1, a   .
1 x 2

(i) Sketch y  f ( x ) , showing clearly the coordinates of the turning point, any
intersections with the axes and the equation(s) of any asymptote(s). [3]

(ii) By drawing a sketch of another suitable curve on the same diagram, find the number
of real roots of the equation
x4  (a  1) x2  5  0 . [2]

(iii) Let g( x)  x  (a  1) x  5 . Show that g( x)  g( x ) . What can be said about the
4 2

sgexamguru.com
four roots of the equation g( x )  0 ? [3]
 

4 A curve C has parametric equations

x  a sin 2t , y  a sin 3t


where 0  t  and a is a positive constant.
2


(i) Find the gradient of C at the point  a sin 2 , a sin 3  where 0    . Hence, what
2

can be said about the tangent to C as   ? [3]
4

(ii) Find the equation of the normal, in exact form, at the point where t  . [3]
12

[Turn Over]
SAJC P2 Questions 547
3

(iii) With the aid of a sketch, show that the area bounded by the curve C, the y-axis and the
a2
line y  can be written as
2

  cos 3t sin 2t  dt
2 12
3a .
0
Hence, find the exact area of the region bounded by the curve C, the y-axis and the
   
normal to the curve at t  in the form ka 2 b cos  c sin   da 2 , where k,
12  12 12 
b, c and d are constants to be determined. [7]

1 3
5 The complex number z is given by z  rei , where 1  r  2 and     .
6 4
(i) State z and arg(z) in terms of r and  . Hence, draw an Argand diagram to show the
locus of z as r and  varies. You should identify the modulus and argument of the
end-points of the locus. [3]
(ii) Find the exact minimum value of z  5  6 i and the corresponding complex number
z representing the point at which this minimum value occurs, giving your answer in
the form x + iy, where x and y are real numbers. [3]
5
Another complex number w satisfies the equation arg( w  2  3)  .
6
(iii) On the same diagram as part (i), sketch the locus of w and indicate the set of points

sgexamguru.com
that satisfies both the locus of z and w. [2]

Section B: Statistics [60 marks]

6 A survey is to be carried out to obtain feedback from the members of a new female-only
fitness club regarding its various facilities and fitness classes. The membership of this
fitness club comprises 5000 female members and the number of members belonging to the
various age groups are given in the table below:

Age group 18 - 25 26-30 31 – 40 41 – 50 51 & above

Number of 500 1000 1500 1500 500


members

It is proposed to carry out the survey by interviewing members who visit the club on a
particular weekday in the morning.

(i) Explain why this proposed method is inappropriate. [1]

(ii) Suggest an appropriate method of carrying out the survey and describe how you
intend to implement the sampling method to obtain a representative sample of 200
members. [3]

[Turn Over]
SAJC P2 Questions 548
4

5 7
7 (a) For events X and Y, it is given that P( X  Y )  , P( X  Y ')  and
8 24
9
P( X ' | Y )  .
16

(i) Find P( X ' Y ) , [3]


(ii) Find P( X ) and determine if the events X and Y ' are independent. [3]

(b) The Mathematics department consists of 5 female teachers and 6 male teachers.
After a meeting, the department went to a nearby food court for lunch. Due to the
lunch crowd, they only managed to find a circular table for 6 and a long table with a
row of 5 seats as shown below. The seats at both tables are fixed and cannot be
rearranged.

Ms Koh was among the Mathematics teachers who attended the lunch.
 
             

X  X X X X X X

sgexamguru.com
   
 
X  X

(i) Find the probability that Ms Koh is seated between 2 male teachers. [4]

(ii) Given that Ms Koh is seated between 2 male teachers, find the probability that
the male and female teachers alternate at both tables. [3]

8 (i) Given that X ~ N   ,  2  and P(X < 28.4) = P(X > 77.6) = 0.012, find the value of
 and  . [3]

(ii) The mass, in grams, of a randomly chosen packet of sweets is normally distributed
with mean  and variance  2 obtained from part (i). Every packet of sweets is
priced at $1.20 per 100 grams. Find the probability that the sum of 4 packets of
sweets cost at most $2.60. [3]

[Turn Over]
SAJC P2 Questions 549
5

9 The hens on a farm lay either white or brown eggs. The eggs are randomly put into boxes
of six. The farmer claims that the number of brown eggs in a box can be modelled by a
binomial distribution B  6, p  .

(i) State, in context, two assumptions to support the farmer’s claim. [2]
(ii) Given that the probability a box contains at least 5 brown eggs is 0.04096, find the
value of p. [2]

A supermarket orders 100 boxes of eggs daily.

(iii) By using a suitable approximation, find the probability that there are at least 90 boxes
that contains at most 4 brown eggs in a particular day. [3]
(iv) The supermarket places a daily order of 100 boxes of eggs for 8 weeks. Estimate the
probability that the mean number of boxes that contains at least 5 brown eggs in a day
is between 4 and 7.
[You may assume that there are 7 days in a week.] [3]

10 A researcher wishes to investigate the length of time that patients spend with a doctor at a
particular clinic. The time a patient spends with the doctor is denoted by X minutes. Based

sgexamguru.com
on past records, the clinic claims that the mean length of time for the doctor to see a patient
is at most 10 minutes. To test this claim, the researcher recorded the actual times spent by
the doctor to see a random sample of 12 patients.

 x = 147,  x 2
 1927.91

(i) Stating a necessary assumption, carry out an appropriate test, at the 5% significance
level, to determine whether there is any evidence to doubt the clinic’s claim. [5]

(ii) Suppose now that the population standard deviation of X is 15 and that the
assumption made in part (i) is still valid. A new sample of n patients is obtained and
the sample mean length of time is found to be unchanged. Using this sample, the
researcher conducts another test and found that the null hypothesis is not rejected at
the 5% significance level. Obtain an inequality involving n and find the set of values
that n can take. [3]

[Turn Over]
SAJC P2 Questions 550
6

11 An experiment is conducted to calibrate an anemometer*. In this calibration process, the


wind speed X is fixed precisely and the resulting anemometer speed Y is recorded.  
For a particular anemometer, this process produced the following set of measurements:

Wind speed 1.0 2.0 3.0 4.0 5.0 6.0 7.0 8.0
(m/s), X

Anemometer 24 28 47 92 164 236 312 360


(revs/min), Y

(i) Calculate the product moment correlation coefficient between X and Y. [1]
(ii) Sketch a scatter diagram for the data. [1]
Explain why it is advisable to sketch the scatter diagram in addition to calculating the
product moment correlation coefficient before interpreting this set of bivariate data.
[1]
2
A proposed model for the above data is Y = a + bX .

(iii) Calculate the product moment correlation coefficient and the equation of the least
squares regression line for the proposed model.
Explain whether the model for Y on X 2 or Y on X is a better model for the data set.
[3]

sgexamguru.com
(iv) Use an appropriate regression line to estimate the value of X when the value of Y is
120. Give a reason for the choice of your regression line. [2]
  [* An anemometer is a device commonly used in a weather station for measuring wind speed.]

[Turn Over]
SAJC P2 Questions 551
7

12 The managers of 2 branches of a travel agency were discussing whether the number of
customers who bought the Luxury Cruise Package per week could be modelled by a
Poisson distribution. One of the managers said, “It must be assumed that the number of
customers who bought the package per week is a constant.”

(i) Give a corrected version of the manager’s statement, and explain why the correction
is necessary. [1]

It is given that the number of customers who bought the Luxury Cruise Package per week
can be modelled by a Poisson distribution. The average number of customers who bought
the Luxury Cruise Package per week at Branch A and Branch B is 3.5 and 4.5 respectively.
Assume that the number of customers who bought the package at Branch A and Branch B
are independent.

(ii) Find the probability that, in a randomly chosen week, the total number of customers
who bought the Luxury Package at both branches is between 5 and 10. [2]

(iii) Given that the probability that at most one customer bought the Luxury Cruise
Package at Branch A in n weeks is less than 0.1, find the value of the least n. [3]

(iv) Using a suitable approximation, find the probability that, in one month, the number
of customers who bought the Luxury Cruise Package at Branch B exceeds the
number of customers at Branch A by not more than 5. [4]
[You may assume that there are 4 weeks in 1 month.]

sgexamguru.com
Explain why the Poisson distribution may not be a good model for the number of
customers who bought the Luxury Cruise Package in a year. [1]

End of Paper

[Turn Over]
SAJC P2 Questions 552
2016 Prelim Paper 2 Solution

1 n
 r   n n 1 
Let P(n) be the statement,  (r  1) ln    ln   , , n = 2,3,4,... .
r 2  r 1   (n  1)!
When n = 2,
 2 
LHS = (2  1) ln    ln 2
 2 1 
 2 21 
RHS = ln    ln 2
 (2  1)!
 LHS = RHS
P(2) is true.
Assume that P(k) is true for some positive integer k, , k = 2,3,4,…
k
 r   k k 1 
i.e. Assume  (r  1) ln    ln  .
r 2  r 1  (k  1)!
To prove P(k + 1) is true.
k 1
 r   (k  1)k 
i.e. to prove  (r  1)ln    ln  
r 2  r 1  k! 
k 1
 r 
LHS   (r  1) ln  
r 2  r 1 
k
 r   k 1
  (r  1) ln    k ln 
sgexamguru.com

r 2  r 1   k 
 k k 1   k 1
 ln    k ln  
 (k  1)!  k 
 k k 1   k 1 
k

 ln    ln  
 (k  1)!  k 
 k k 1 ( k  1) k 
 ln   
 (k  1)! kk 
 k 1 (k  1) k 
 ln  
 (k  1)! 
 (k  1) k 
 ln  
 k (k  1)!
 (k  1) k 
 ln  
 k! 
 RHS

Since P(2) is true and if P(k) is true, it implies that P(k+1) is true. By Mathematical
Induction, P(n) is true for all positive integers n  2.

Microsoft Word - 2016 SAJC Prelim Paper 2 Solutions 553


2 x2  6 x  8
 0, x  1
x 1

( x  2)( x  4)
0
( x  1)

Use sign test

4  x  2 or x  1. (Ans)

y 2  2 y  15
 6
| y  1| 1

y 2  2 y  15  6(| y  1| 1)
0
| y  1| 1

sgexamguru.com
( y  1) 2  14  6 | y  1| 6
| y  1| 1
0

( y  1) 2  6 | y  1| 8
0
| y  1| 1

Since (y+1)2 = | y+ 1 |2

Replace x by |y+1| in (i)

4  y  1  2 (no solution since y  1  0 for all real y)

or |y + 1| >1

 y  1 > 1 or y  1  1

(or use of the graphical method)

SAJC P2 Solutions 554


From GC, the intersection points are (2,1) and (0,1)

y  0 or y  2 (Ans)

3(i) 5  ax 2 5  a(1  x 2 )  a 5 a
f ( x)    a  ,a>1
1 x 2
1 x 2
1  x2

sgexamguru.com y
5

SAJC P2 Solutions 555


(ii) x 4  (a  1) x 2  5  0
x 4  x 2  5  ax 2
x 2 (1  x 2 )  5  ax 2
5  ax 2
x2 
1  x2

Hence we should sketch the curve y  x 2 .

y
5

sgexamguru.com
From the graph we can see that there are only two real roots.

LHS = g( x)  ( x)  (a  1)( x)  5  x  (a  1) x  5  g( x) =RHS


4 2 4 2

So g(x) = g(x)

As there are only two real roots, the other two roots should be complex roots.

As the coefficients of equation are all real, the remaining two roots must be a pair of
complex conjugates.

As there is only one pair the complex conjugates, and g( x)  g( x)  0 , then the
complex conjugates must be purely imaginary.

Explanation:

If z  x  iy is a root where x, y are real, so g( x  iy)  0 . Since g( z)  g( z) and


g( z )  0 , then g( z )  0 and hence z  ( x  iy)   x  iy is also a root. As the
complex roots need to be in conjugate pairs, then

SAJC P2 Solutions 556


 x  iy    x  iy
*

x  i y   x  iy
Comparing the Real Part,
x  x
2x  0
x0
[Note that the imaginary part is not necessary, as it yields
 y   y which is trivial.]

Hence the complex roots must be purely imaginary.

Alternative explanation:

x 4  (a  1) x 2  5  0
(a  1)  (a  1) 2  4(1)(5)
x  2

2
(a  1)  (a  1) 2  20
x  2

2
(a  1)  (a  1) 2  20 (a  1)  (a  1) 2  20
x2  or x 2 

sgexamguru.com
2 2
(a  1)  (a  1) 2  20 (a  1)  (a  1) 2  20
x or x  
2 2

(a  1)  (a  1) 2  20 (a  1)  (a  1) 2  20
Since  0 and 0
2 2

then the complex roots must be purely imaginary.

4(i) Given x  a sin 2t , y  a sin 3t ,

dx dy
 2a cos 2t ,  3a cos 3t
dt dt

Hence,

At the point  a sin 2 , a sin 3  ,


dy 3cos 3

dx 2 cos 2

  dy
As   , cos 2  cos 0 ,   .
4 2 dx

SAJC P2 Solutions 557


dy
i.e. will be undefined.
dx


Hence, the tangent to C as   will become a vertical line, i.e. parallel to y-axis
4

(ii) 
At t  ,
12

   
3cos  3   
dy
   12  
dx    
2 cos  2   
  12  
 1 
3 
 
2
 3
2 
 2 
6

2

2
Gradient of normal= 
sgexamguru.com
6


Coordinates of point at t  :
12

   
x  a sin  2   
  12  
a

2
   
y  a sin  3   
  12  
a 2

2

Equation of normal:

a 2 2  a
y  x 
2 6 2
6 a
 x 
3  2

SAJC P2 Solutions 558


6 a 6 a 2
y x 
3 6 2


3
6
x
a
6
 6 3 2 
(iii)
y
D

  
B A

x
O

y  0, t  0 sgexamguru.com
a 2 
y ,t   from (ii) 
2 12

2a
Area bounded by curve, y-axis and the line y  , region OAB
2
2a
 2
x dy
0

 a sin 2t  
dy 
 12
 dt
0
 dt 

 12
0
 a sin 2t   3a cos 3t  dt

 3a 2  12  cos 3t sin 2t  dt (Shown)
0

y-intercept of normal is:

y
a
6
 6 3 2 

SAJC P2 Solutions 559


Hence,

Required area, region OADB


 a 2
  cos 3t sin 2t  dt  2  2   6  
1a a
 3a 2 12
6 3 2  
0
 2 

3a 2  
a2   6 3 2  
2

2   sin 5t  sin t  dt 
12
0 4  6 2 
 

3a 2  1  12 a  6 
2
   5 cos 5t  cos t   4  6 
2 0  
3a 2  1 5  1  a 6
2
   5 cos 12  cos 12  5 cos 0  cos 0   24
2
 1
3a 2 5  4  a2 6
 
 5 cos  cos  
2 12 12 5  24
3a 2   1      6 6
  cos  cos      a 2   
2  12 5  2 12    24 5


3a 2
2

cos
sgexamguru.com
 1
 sin
  2 6 6
 12 5 12   a  24  5  units
 
2

3 1 6 6
where k  , b  1, c   , d  
2 5 24 5

5(i), arg( z )  
(ii) z r

SAJC P2 Solutions 560


(‐5,6)
Im(Z) 

A
Locus of Z

2
1

Re(Z)

(ii) sgexamguru.com
The minimum distance, d = (5) 2  62  2  61  2

Let  be the basic angle of the point (5, 6) .

6 6 5
From the diagram, tan   so sin   and cos   .
5 61 61

Hence complex number that corresponds to point A,

 5   6  1
z  2   cos   i sin    2    2i    10  12i 
 61   61  61
 1.28  1.54i

SAJC P2 Solutions 561


(iii) Im(Z) 

(0,2)

Re(Z)

6(i) sgexamguru.com
The people who visits the fitness centre on a particular weekday may not be
representative of the members of the club as some members may be working or
attending school and only go to club later in the day. Thus the sample collected
would contain mainly women who are not working/do not attend school thereby
under representing women of some age groups.

(ii) Stratified sampling method should be used to obtain a representative sample of 200
members.

1. Draw up a list of the members of the fitness centre and calculate the proportion
for the different corresponding age group.

18 – 25 26-30 31 – 40 41 – 50 51 &
above

500 1000 1500 1500 500

500 1000 1500 1500 500


 200  200  200  200  200
5000 5000 5000 5000 5000
 20  40  60  60  20

SAJC P2 Solutions 562


2. Within each age group, use simple random sampling method to select the
required sample size for survey.

7 P( X  Y )  P(Y )  P( X  Y ')
(a)(i) 5 7
 P(Y ) 
8 24
8 1
P(Y )  
24 3

9
Given P( X ' | Y )  ,
16

P( X ' Y ) 9

P(Y ) 16
9 1 3
P( X ' Y )  . 
16 3 16

(ii) 3 9 7
P( X )  1   P( X  Y ) ' P( X ' Y )   1   
8 48 16

7 2 7
P( X ).P(Y ')  . =  P( X  Y ')

sgexamguru.com
16 3 24

X and Y ' are independent.

7(b) 11
Without restriction    (6  1)!5! = 6652800
(i) 6

Case 1: Ms Koh sits at round table with two male teachers

No. of ways

 8  6 
    2!(4  1)!5! =1209600
 3  2 

Case 2: Ms Koh sits at long table with two male teachers

No. of ways

 8  6 
    2!3!(6  1)! =604800
 2  2 

Required Probability

SAJC P2 Solutions 563


 8  6   8  6 
   2!(4  1)!5!    2!3!(6  1)!
     2  2 
3 2
11
  (6  1)!5!
6
3
 or 0.273 (to 3.s.f)
11

(ii) Required Probability

P(Ms koh sits between 2 male teachers and male and female teachers alternate)

P(Ms koh sits between 2 male teachers)
P(male and female teachers alternate)

P(ms koh sits between 2 male teachers)
 65
   (3  1)!3!3!2!
 3 3 6  5  4  5  4  2  3 2
 6652800 OR 6652800
3 3
11 11
1
 231
3
11
1
 or 0.159 (to 3.s.f)

sgexamguru.com
63

SAJC P2 Solutions 564


Alternative Solution:
Required Probability
P(ms koh sits between 2 male teachers and male and female teachers alternate)

P(ms koh sits between 2 male teachers)

Case 1: Ms Koh sits at the long table and the male and female teachers alternate
 6  4 
no. of ways =    3!2!3!(3  1)!  11520
 3  1 
Case 2: Ms Koh sits at the circular table and the male and female teachers alternate
 6  4 
no. of ways =    (3  1)!3!3!2!  17280
 3  2 
Total no. of ways ms koh sits between 2 male teachers and male and female teachers alternate
 6  4   6  4 
    3!2!3!(3  1)!    (3  1)!3!3!2!
  
3 1  3  2 
 28800

Required Probaility
28800
= 6652800
3
11
1

63

[Note that for the males and females to seat on alternate seats, on the round table there must

sgexamguru.com
be 3 males and 3 females and the long table there must be 3 males and 2 females]

8 (i)

Area =0.012
Area =0.012

28.4 77.6

Method 1 (recognise  is the midpoint)

28.4  77.6
By symmetry,   =53
2

P( X  28.4)  0.012
28.4  53
P( Z  )  0.012

Using GC,

SAJC P2 Solutions 565


24.6
 2.25712924

  10.8988
  10.9 (3 s.f.)

Method 2 ( simultaneous equations- not recommended)

P( X  28.4)  0.012 P( X  77.6)  0.988


28.4   77.6  
P( Z  )  0.012 P( Z  )  0.012
 
28.4   77.6  
 2.25712924  2.25712924
 
28.4    2.25712924 77.6    2.25712924

Solve simultaneously,   53,   10.8988

(ii) Let X be the weight of a packet of sweets in grams.

X ~ N (53,10.89882 )

Method 1 (expression in terms of mass)

X 1  X 2  ... X 4 ~ N (212, 475.13536)

$1.20  100 g
sgexamguru.com
2.6  100
$2.60   216.667 g
1.2
P( X 1  X 2  ... X 4  216.667)  0.58476
 0.585 (3 s.f.)

2.6  100
P( X 1  X 2  ... X 4  )  0.58476
1.2
 0.585 (3 s.f.)

Method 2 (expression in terms of cost)

Let C denote the cost of 4 packets of sweets.

1.20
C ( X 1  X 2  ... X 4 )  0.012( X 1  X 2  ... X 4 )
100

Then

E (C )  0.012(4  53)  2.544


Var (C )  0.012 2 (4  10.89882 )  0.0684195

SAJC P2 Solutions 566


P(C  2.60)  0.5847618  0.585 (3 s.f.)

9 (i) The probability of picking a brown egg from a box is constant.

The colour of an egg is independent of other eggs.

(ii) Let X be the r.v. “number of brown eggs in a box of 6 eggs”

X ~ B(6, p)

P  X  5   0.04096
P  X  5   P  X  6   0.04096
6 5 6 6
  p 1  p     p 1  p   0.04096
0

5 6

sgexamguru.com
Using GC, p  0.4

(iii) Let A be the r.v. “number of boxes that contain at most 4 brown eggs in a box out of
100 boxes”

A ~ B(100, p1 ), where p1  1  0.04096  0.95904 .

Let Y be the r.v. “number of boxes that contain at least 5 brown eggs in a box out of
100 boxes”

Y ~ B(100, 0.04096)

Note that A  Y  100

Since n  100 is large, np  100  0.04096   4.096  5 ,

Y ~ P0 (4.096) approximately

P( A  90)  P(100  Y  90)  P(Y  10 ) = 0.997

(iv) Y ~ B(100, 0.04096)

SAJC P2 Solutions 567


E Y   100  0.04096   4.096
Var Y   100  0.04096  0.95904   3.9282

In 8 weeks, there are 56 days altogether.

Mean number of boxes with at least 5 brown eggs is

Y1  Y2  Y3    Y56
Y=
56

Since sample size = 56 is large, by Central Limit Theorem,

 3.9282 
Y ~ N  4.096,  approximately
 56 

i.e. Y ~ N  4.096, 0.0701469 

 
P 4  Y  7 = 0.641 (correct to 3 sig fig)

10(i) Let X be the random variable ‘ length of time a patient spent with the doctor’

sgexamguru.com
Given

 x = 147,  x 2
 1927.91

The unbiased estimates of population mean  and population variance  2 are

1 ( x) 2
x  12.25, s  [ x 
2 2
]  11.56
n 1 n

Assumption:

The length of time, X, a patient spent with the doctor follows a normal distribution.

Test H0:   10

H1:   10

X 
Under H0, T  ~ t 11
S
n

Use a right tailed t-test at the 5% level of significance.

From GC, p-value = 0.0213

SAJC P2 Solutions 568


Since p-value = 0.0213 < 0.05, we reject H0. There is sufficient evidence to
conclude at the 5% level of significance, the mean time spent with a patient is more
than 10 minutes.

ii Since population variance is given, z-test should be used.

Test H0:   10

H1:   10

X 
Under H0, Z  ~ N (0,1)

n

Since H0 is not rejected,

12.25  10
z  1.64485
15 / n
0.15 n  1.64485
n  120.25

Maximum n = 120.

11(i) r = 0.974
sgexamguru.com
Set of values of n is { n  : 0  n  120 }

(ii)


(8.0, 360)

(1.0, 24)

Although the value of product moment correlation coefficient indicates a strong


positive linear correlation however from the scatter diagram, X and Y follows a non-
linear relationship, hence it is advisable to interpret the data using both the scatter
diagram and the value of product moment correlation coefficient.

SAJC P2 Solutions 569


(iii) Using GC,

Y = 9.67+ 5.81X 2

r = 0.993

Since the value of product moment correlation coefficient for Y on X2 is closer to 1,


compared to the value of product moment correlation coefficient for Y on X, the new
proposed model is a better model for the data set.

(iv) Y = 9.6714285+ 5.81190X 2

When y = 120, x = 4.36 ( 3 s.f.)

The line of Y on X 2 is used because the value of X is fixed precisely and hence X is
the independent variable.

12(i) It must be assumed that the average number of customers who bought the package
per week is a constant. The number of customers who bought the package per week
varies, and cannot be a constant.
(ii) Let X and Y be the number of customers who bought the Luxury Cruise Package in
a week at Branch A and Branch B respectively.

X~Po(3.5)

Y~Po(4.5) sgexamguru.com
X+Y~Po(8)

P(5  X  Y  10) = P( X  Y  9)  P( X  Y  5)  0.525

(iii) Let U be the number of customers who bought the Luxury Cruise Package in n
weeks at Branch A.

U~Po(3.5n)

Given P(U  1)  0.1

From GC, when n = 1, P(U  1) = 0.136 > 0.1

when n = 2, P(U  1) = 0.0073 < 0.1

Least n = 2
(iv) Let S and T be the number of customers who bought the Luxury Cruise Package in
one month at Branch A and Branch B respectively.

S~Po(14)

SAJC P2 Solutions 570


Since   10 , S~N(14, 14) approximately.

T~Po(18)

Since   10 , T~N(18, 18) approximately.

T-S ~ N(4, 32)

P(0  T  S  5) 
C .C .
 P(0.5  T  S  5.5)  0.337

The mean number of customers who bought the Luxury Package might not be a
constant from one week to another because of fluatuations such as sales, holidays,
the economic climate etc.

Hence the Poisson distribution may not be a good model for the number of
customers who bought the package in a year.

sgexamguru.com

SAJC P2 Solutions 571


1

SERANGOON JUNIOR COLLEGE

2016 JC2 PRELIMINARY EXAMINATION

MATHEMATICS
Higher 2 9740/1
15 Sept 2016

3 hours

Additional materials: Writing paper

List of Formulae (MF15)

TIME : 3 hours

READ THESE INSTRUCTIONS FIRST


Write your name and class on the cover page and on all the work you hand in.

sgexamguru.com
Write in blue or black pen on both sides of the paper.
You may use a soft pencil for any diagrams or graphs.
Do not use staples, paper clips, highlighters, glue or correction fluid.

Answer all the questions.


Give non-exact numerical answers correct to 3 significant figures, or 1 decimal place in the
case of angles in degrees, unless a different level of accuracy is specified in the question.
You are expected to use a graphic calculator.
Unsupported answers from a graphic calculator are allowed unless a question specifically states
otherwise.
Where unsupported answers from a graphic calculator are not allowed in a question, you are
required to present the mathematical steps using mathematical notations and not calculator
commands.
You are reminded of the need for clear presentation in your answers.

The number of marks is given in brackets [ ] at the end of each question or part question.
At the end of the examination, fasten all your work securely together.

Total marks for this paper is 100 marks

This question paper consists of 6 printed pages (inclusive of this page) and 2 blank pages.

[TURN OVER]

SRJC P1 Questions 572


2

Answer all questions [100 marks].

1 State a sequence of 3 transformations which transform the graph of


g  x   e 6 x  2  1 to the graph of f  x   e 2 x 2 .
2 2

[3]

2 Using the standard series expansions, obtain the Maclaurin series of


ln (1  x)(1  2 x)3  in ascending powers of x , up to and including the term in x 3 . [2]

(i) Find the set of values of x for which the above expansion is valid. [1]
(ii) Hence, find the range of values of x for which the expansion of
  1 


 
ln ln  
e  1 2 x    ln(1  x)   2  7 x 5 is valid.
3
[2]
 
 
 

3 (a) y

y  e2 x – kx

sgexamguru.com
 
x O

The diagram shows the graph of y  e2 x – kx , where k is a positive real number.

The two roots of the equation e2 x – kx  0 are denoted by  and  , where


 < .
It is given that there is a sequence of real numbers x1 , x2 , x3 ... that satisfies the
1 2 xn
recurrence relation, xn 1  e , for n  1.
k
By considering xn 1 – xn , prove that
xn 1 > xn if xn <  or xn >  . [2]
(b) Prove by the method of mathematical induction that

n
sin(n  12 )  sin 12 
 cos r 
r 1 2sin 12 
, for all positive integers n. [5]

SRJC P1 Questions 573


3

4 Andy and his fiancée signed up for a new 4-room flat in Boon Keng. They take
up a housing loan of $450,000 provided by BEST bank for the purchase. The
couple pay a fixed monthly instalment of $A on the first day of each month. A
fixed interest rate of 1.6% is charged on the last day of each year, based on the
remaining loan amount at the beginning of that year before the 1st monthly
instalment is paid. If the first instalment is paid in January 2016,
(i) Show that the amount the couple owe the bank at the end of 2017 is
$  464515.20  24.192 A . [1]
(ii) Given that A is 1500, find the date and amount of the final repayment to the
nearest cent. [5]

x2  x 1
5 (a) It is given that y  , x  , x  1. Without using a graphic
x 1
calculator, find the set of values that y cannot take. [3]
x2  b
(b) The curve C has equation y  , where a > 0, b > a and x  a .
xa
(i) Draw a sketch of the curve C, label clearly the equation(s) of its
asymptote(s) and the coordinates of any intersection with the axes. [3]
(ii) By drawing an additional graph on the diagram drawn in (i), state the
 
number of real root(s) of the equation x 2  b   x  a  x 2  a . [2]

sgexamguru.com
6 (a) The equations of two planes p1 and p2 are
x  4 y  2z  7 ,
3x   y  4 z   ,
respectively, where  and  are constants.
(i) Given that the two planes intersect in a line l, with a vector equation
given by
1  2 
r = 1  s  1  , s  R,
1  1 
   
show that the value of  is 10 and find the value of  . [3]
(ii) If plane p3 is the reflection of p1 in p2 , find the acute angle between p1
and p3 . [2]
(b) Relative to the origin O, the points A, B, C and D have position vectors a, b,
c and d respectively. It is given that λ and μ are non-zero numbers such that
λa + μb − c = 0 and     1 ,
(i) Show that A, B and C are collinear. [3]
(ii) If O is not on the line AC and c  a  b  a    c d  d , determine the
 
relationship between AC and OD , explaining your answer clearly. [2]

7 A piece of metal with negligible thickness has been cut into a shape that is made

[TURN OVER]

SRJC P1 Questions 574


4

up of four isosceles triangles each with base x cm and fixed sides a cm. Their
bases frame to a form a square with sides of length x cm. A right pyramid is
formed by folding along the dotted lines as shown in the diagram below.

a cm a cm

x cm

a cm a cm x cm

1
[Volume of a pyramid =  base area  height ]
3
x2 x2
(i) Show that the volume of the pyramid is a2  cm3.
3 2 [2]

(iii)
sgexamguru.com
(ii) Find the value of x, in terms of a, that will give maximum volume for the
pyramid. [4]

H cm
cm

a cm
To make the pyramid into a paperweight with negligible thickness, a
viscous fluid is pumped into the interior at a rate of 1 cm3/s. Given that H
cm is the perpendicular distance from the apex of the pyramid to the
2
viscous fluid surface, x = a and the height of the pyramid is a cm, find
2
a
the rate at which H is changing when H  , giving your answer in terms
2 [3]
of a.
[The diagram above shows the cross sectional area of the pyramid.]

SRJC P1 Questions 575


5

 1
8 (i) Show that  0,   lies on the locus z  2  z  1  2i . [1]
 4

(ii) Sketch on a single Argand diagram the loci z  1  2i  5 and

z  2  z  1  2i . [4]
(iii) Hence indicate clearly on the Argand diagram the locus of z that satisfies
the relations z  1  2i  5 and z  2  z  1  2i . [1]
(iv) Find the greatest and least possible values of arg  z  1  2i  , giving your

answers in radians correct to 3 decimal places. [4]

9 The path travelled by an object measured with respect to the origin in the
horizontal and vertical directions, at time t seconds, is denoted by the variables x
and y respectively.
dx
It is given that when t  0, x  1 , y = 0 and  1 . The variables are related by
dt
dy d2 x
the differential equations  y  e  4 y  0 and 2  cos 2 2t .
2t 2

sgexamguru.com
dt dt
dw
(i) Using the substitution y  wet , show that   1  4w2 and hence find y
dt
in terms of t. [6]
(ii) Find x in terms of t. [4]

3r 8r  6  3 r

10 Given that f  r   , show that f  r  2   f  r   . [1]


r 1  r  1 r  3
n
 4r  3 3r
(i) Find 
r 1  r  1 r  3
in terms of n . [2]

n
 4r  11 3r
(ii) Hence find 
r 1  r  3 r  5 
in terms of n . [4]

n  r  3r  3n 1 51
(iii) Using the result in (ii), show that   2
r 0   r  5 

4

160
. [3]
 

[TURN OVER]

SRJC P1 Questions 576


6

11 The functions f and g are defined as follows:


f:x  x2  2 x , a x0
g:x  x 1 , x  1
(i) State the least value of a for the inverse function of f to exist. Hence, find f
−1
in similar form. [4]
For the following parts, use the value of a found in part (i).
(ii) Write down ff 1 in similar form. [1]
(iii) Find the rule for gf in the form bx + c, where b, c  . State its range. [3]
 3  3
(iv) Find the exact range of x for which f  x    g f  x   . [3]
 2  2

d 
 ln x   .
2
12 (a)(i) Find [1]
dx 
(ii) The curve C is defined by the parametric equations
x  ln t  t , y  2t  ln  t 2  where t  0 .

Another curve L is defined by the equation  4  y   3  x . The graphs


2

of C and L intersect at the point A  1, 2  as shown in the diagram below.

sgexamguru.com
C
L

Find the exact area of the shaded region bounded by C, L and the line
A  1, 2  . [6]
(b) The region R is the finite region enclosed by the curve  y  1  1  x and the
2

y-axis. The region S is the region in the 2nd quadrant enclosed by the curve
 
y  2 tan  x   and the axes.
 4
Find the total volume generated when region R and S is rotated through 2
radians about the x-axis, leaving your answers in exact form. [4]

END OF PAPER

SRJC P1 Questions 577


1

SRJC Paper 1 Solutions :

Answer all questions [100 marks].

1 State a sequence of 3 transformations which transform the graph of


2 2
g  x   e
6 x 2 
 1 to the graph of f  x   e
2 x 2 
. [3]

Solution
I) Translation in the negative y-direction by 1 unit
2 2
y  e 6 x  2   1  y  e 6 x  2 
II) Stretch parallel to the x-axis by a scale factor 3.

2
 x 
 6   2 
 6 x  22 2
 e
 3  2 x 2
ye  ye
III) Translation in the positive x-direction by 2 units
2 2 2
y= e 2 x  2  y  e
2 x  2   2 
 e 2 x  2 
Alternatively
I) Translation in the negative y-direction by 1 unit
2 2
y  e 6 x  2   1  y  e 6 x  2 

sgexamguru.com
II) Translation in the positive x-direction by 2/3 units
  2 
 6 x    2 
2

 6 x  22 2
 e
  3  6 x  2
ye  ye
III) Stretch parallel to the x-axis by a scale factor 3.
2
  x 
 6   2 
 6 x  2 2 2
 e
 3  2 x 2 
ye  ye

Note: The translation can be step 1, 2 or 3.

2 Using the standard series expansions, obtain the Maclaurin series of ln (1  x )(1  2 x) 3 
in ascending powers of x , up to and including the term in x 3 . [2]
(i) Find the set of values of x for which the above expansion is valid. [1]
(ii) Hence, find the range of values of x for which the expansion
 l n ln 1  
 e  1 2 x 3   ln(1  x)  2  7 x 5 is valid. [2]
  
 

Solution

[TURN OVER]

SRJC P1 Solutions 578


2

ln[(1  x)(1  2 x )3 ]  ln(1  x )  3ln(1  2 x)


x 2 x3  8 x3 
 x   3  2 x  2 x 2   
2 3  3 
13 x 2 23 x 3
 5 x   
2 3
(i) So 1  2 x  1 and 1  x  1
1 1
 x and 1  x  1
2 2
 1 1
x   :   x  
 2 2
 ln ln 1  
  1 2 x 3 
 ln(1  x )   2  7 x    3ln 1  2 x   ln(1  x)   2  7 x 
3 3
(ii) e
 
 
3
  3ln 1  2 x   ln(1  x )   2  7 x 
From (i), replace x by –x,
1 1
  x  and x  
2 2
1 1
 Range of values of x is  x .
2 2

3 (a)
sgexamguru.com y

y  e 2 x – kx

x
O  

The diagram shows the graph of y  e2 x – kx , where k is a positive real number.


The two roots of the equation e 2 x – kx  0 are denoted by  and  , where
 < .
It is given that there is a sequence of real numbers x1 , x2 , x3 ... that satisfies the
1
recurrence relation, xn 1  e 2 xn , for n  1.
k
By considering xn 1 – xn , prove that
xn 1 > xn if xn <  or xn >  .
[2]

SRJC P1 Solutions 579


3

(b) Prove by the method of mathematical induction that


n
sin(n  12 )  sin 12 
 cos r  2sin 12 
, for all positive integers n.
r 1 [5]

Solution
y
(a) xn 1 – xn
1
= e 2 xn  xn y = e2x– kx
k
e 2 xn  kxn
= x
k O  

From given sketch, if x <  or x >  ,


e2 x  kx  0
So if xn <  or xn >  ,
e 2 xn  kxn
xn 1 – xn = >0
k
Therefore, xn 1 > xn (Proven)
(b) Let Pn be the statement
sin( n  12 )  sin 12 
cos   cos 2  cos 3  ...  cos n  for n   .
2 sin 12 

sgexamguru.com
To show P1 is true,
1
LHS   cos r  cos 
r 1

sin 32   sin 12 
RHS 
2cos 12 
2cos sin( 12  )

2sin 12 
 cos   LHS
 P1 is true.
k
sin(k  12 )  sin 12 
Suppose Pk is true for some k    , i.e.  cos r 
r 1 2sin 12 
k 1
sin(k  32 )  sin 12 
To show Pk 1 is true, i.e.  cos r 
r 1 2sin 12 
k 1
LHS   cos r
r 1
k
  cos r  cos( k  1)
r 1
sin( k  12 )  sin 12 
  cos( k  1)
2 sin 12 

[TURN OVER]

SRJC P1 Solutions 580


4

sin( k  12 )  sin 12   2 cos( k  1) sin 12 



2 sin 12 
sin(k  12 )  sin 12   sin(k  23 )  sin(k  21 )

2sin 12 
sin(k  32 )  sin 12 
  RHS
2sin 12 
Hence, Pk 1 is true if Pk is true. (Missing is “P”)
Since P1 is true and Pk+1 is true when Pk is true, by Mathematical Induction, Pn
is true for all positive integers n.

4 Andy and his fiancée signed up for a new 4-room flat in Boon Keng. They take
up a housing loan of $450,000 provided by BEST bank for the purchase. The
couple pay a fixed monthly instalment of $A on the first day of each month.
Interest is charged on the last day of each year at a fixed rate of 1.6% of the
remaining loan amount at the beginning of that year. If the first instalment is paid
in January 2016,
(i) Show that the amount the couple owe the bank at the end of 2017 is
[1]
$  464515.2  24.192 A .
(ii) Given that A is 1500, find the date and amount of the final repayment to the
nearest cent. [5]

Solution
sgexamguru.com
(i) Amt owe at the end of 2017 = (1.016)(1.016)(450000) – 1.016(12A) – 12A
= $  464515.2  24.192 A
(ii)

year Amt owed at the beginning Amt owed at the end of the year
after paying 18000

1st 450000 1.016(450000) -18000

2nd 1.016(450000) -12A (1.016)(1.016)(450000) –

1.016(18000) –

18000

3rd (1.0162)(450000) – (1.016)(1.0162)(450000) –

1.016(12A) – (1.016)(1.016)(18000) –

12A (1.016)(18000) –

18000

SRJC P1 Solutions 581


5

… …

nth (1.016n)(450000) –

(1.016n-1)(18000) –

(1.016n-2)(18000) –

…... –

18000

Amount of money owe at the end of nth year


 450000(1.016)n  18000(1  1.016  1.0162  ...  1.016n1 )
Consider
 11.016 n  1 
n
450000(1.016)  18000  0
 1.016  1 
 
450000(1.016) n  1125000 1.016 n  1  0
Using G.C, n  32.2
When n = 32,
Amount owe at the end of 32 years

sgexamguru.com
 $450000(1.016)32  $1125000 1.01632  1  $3233.601
Since they will be paying $1500 each month, they will finished the payment on
1st March 2048. The last payment is $233.60.

x2  x 1
5 (a) It is given that y  , x   , x  1. Without using a graphic
x 1
calculator, find the set of values that y cannot take. [3]
2
x b
(b) The curve C has equation y  , where a > 0, b > a and x  a
xa
(i) Draw a sketch of the curve C, label clearly the equation(s) of its
asymptote(s) and the coordinates of any intersection with the axes. [3]
(ii) By drawing an additional graph on the diagram drawn in (i), state the

number of real root(s) of the equation x 2  b   x  a  x 2  a . [2]

Solution
(a) Consider any horizontal line y  k , k   .
x2  x 1
Consider the intersection of the graphs y  and y  k , i.e.
x 1
x2  x  1
k
x 1
 x 2  x  1  k ( x  1)
 x 2  x ( 1  k )  ( 1  k )  0
For the equation to have no real solutions,

[TURN OVER]

SRJC P1 Solutions 582


6

Discriminant  0
 (1  k )2  4(1)(1  k )  0
 k 2  6k  5  0
(k  1)(k  5)  0

+ ‒5 – ‒1 +
The set of values that y cannot take is y   : 5  y  1
b
(bi) When x = 0, y = 
a
2
x b a2  b
y  ( x  a) 
xa xa
x = a and
y = x + a , are equations of the asymptotes
(ii)
y

y = x2 + a y=x+a

sgexamguru.com a
0
x
 b
 0,  
 a

x=a
x 2  b  ( x  a )( x 2  a)
x2  b
  x2  a
xa
By adding an additional graph in (i), i.e. y  x 2  a , no. of real root is 1.

6 (a) The equations of two planes p1 and p2 are


x  4 y  2z  7 ,
3x   y  4 z   ,
respectively, where  and  are constants.
(i) Given that the two planes intersect in a line l, with a vector equation
given by
1  2 
r = 1  s  1  , s  ,
1  1 
   
show that the value of  is 10 and find the value of  . [3]

SRJC P1 Solutions 583


7

(ii) If plane p3 is the reflection of p1 in p2 , find the acute angle between p1


and p3 . [2]
(b) Relative to the origin O, the points A, B, C and D have position vectors a, b,
c and d respectively. It is given that λ and μ are non-zero numbers such that
λa + μb − c = 0 and     1 ,
(i) Show that A, B and C are collinear. [3]
(ii) If O is not on the line AC and c  a  b  a    cd  d , determine the
 
relationship between AC and OD , explaining your answer clearly. [2]

Solution
 2  3
   
(ai)  1  .
   0
 1  4
   
  10
1  3 
   
  1   10   17
1  4 
   
(ii)
Let  be the angle between p1 and p2 .

sgexamguru.com
 1  3 
   
 4    10 
 2  4 
cos      
1  4  22 32  102  4 2
2 2

51
cos 
21 125
  5.4869
Acute angle between p1 and p3 = 2(5.4869)  11.0
(bi)

AB  b - a

AC  c - a
 a   b  a
 (  1)a   b
  a  b
  (b  a)
 
Since AC   AB for some   \{0} , A, B, C are collinear.
 
(ii) c  a  b  a    cd  d  AB  k OD for some k   as c  a  0 since O is
not on AC
 
since AB   AC for some   

[TURN OVER]

SRJC P1 Solutions 584


8

 
so AC is parallel to OD .

7 A piece of metal with negligible thickness has been cut into a shape that is made
up of four isosceles triangles each with base x cm and fixed sides a cm. Their
bases frame to a form a square with sides of length x cm. A right pyramid is
formed by folding along the dotted lines as shown in the diagram below.

a cm a cm

x cm

x cm
a cm a cm

sgexamguru.com
[Volume of a pyramid =
1
3
 base area  height ]

x2 x2
(i) Show that the volume of the pyramid is a2  cm3.
3 2 [2]
(ii) Find the value of x, in terms of a, that will give maximum volume for the
pyramid. [4]
(iii)

H cm
cm

a cm
To make the pyramid into a paperweight with negligible thickness, a viscous
fluid is pumped into the interior at a rate of 1 cm3/s. Given that H cm is the
perpendicular distance from the apex of the pyramid to the viscous fluid surface,

SRJC P1 Solutions 585


9

2
x = a and the height of the pyramid is a cm, find the rate at which H is
2
a
changing when H  , giving your answer in terms of a.
2
[The diagram above shows the cross sectional area of the pyramid.]
[3]

Solution
(i) Let the height of the isosceles triangle be k cm.
x2
k2   a2
4
x2
k 2  a2 
4
x2 x 2 x2
Therefore, height of the pyramid = a2    a2 
4 4 2
x2 x2
Volume of pyramid, V = a2 
3 2
dV 4 x 2 x 2 x 2  1  x

sgexamguru.com
(ii)  a    
dx 3 2 3 2 x2
a2 
2
 2 x2  3
4x  a    x
 2 
 
x2
6 a2 
2
4a 2 x  3 x 2

x2
6 a2 
2
 3  3 
2x  a  x a  x
2 2
   
2
x
3 a2 
2
dV
When  0,
dx
 3  3 
2x  a  x  a  x
 2  2 
0
2
x
3 a2 
2

[TURN OVER]

SRJC P1 Solutions 586


10

2 x  2 x 2  x3
a   
3  2  6
2 3 2 3
x a or  a (rejected  x  0)
3 3
2 3 
When x  a ,
3
1 2 3 3 dV
 0 , x  0, a  x  0 and a  x  0 , thus 0.
x 2 3 2 2 dx
a2 
2
2 3 
When x  a ,
3
1 2 3 3 dV
 0 , x  0, a  x  0 and a  x  0 , thus  0.
x 2 3 2 2 dx
a2 
2
2 3
Therefore x  a gives maximum volume.
3
(iii) Volume of pyramidal empty space, W cm3, in the pyramid as it is being
filled up

sgexamguru.com
1
= b 2 H , where b is the length of the square base
3
b
 2
H
2 dW
Therefore W  H 3   2H 2
3 dH
a
When H 
2
d W dH dW
 
dH dt dt
2
 a  dH
2    1
2 dt
dH 2

dt a2
2
H is decreasing at a rate of 2 cm/s.
a

8  1
(i) Show that  0,   lies on the locus z  2  z  1  2i . [1]
 4

SRJC P1 Solutions 587


11

(ii) Sketch on a single Argand diagram the loci z  1  2i  5 and

z  2  z  1  2i . [4]
(iii) Hence indicate clearly on the Argand diagram the locus of z that satisfies the
relations z  1  2i  5 and z  2  z  1  2i . [1]
(iv) Find the greatest and least possible values of arg  z  1  2i  , giving your

answers in radians correct to 3 decimal places. [4]

Solution
1
(i) When z   i
4
1 1
L.H.S   i+2  4
4 16
1 7 1
R.H.S   i+1+2i  1+ i  4
4 4 16
 1
Hence  0,   lies on |z + 2| = |z + 1 + 2i|.
 4

sgexamguru.com
(ii) & (iii)

Locus of z Im

(– 2, 0) (0, 0)
Re

|z + 1 + 2i| =
(–1, –2)

( –1.5, –1)
(0, – 4)
|z + 2| = |z + 1 + 2i|

(iv)

[TURN OVER]

SRJC P1 Solutions 588


12

Im

(– 2, 0) (0, 0)
Re
( – 1.5, – 1)

Locus of z 1
|z + 1 + 2i| =
( – 1, – 2)
0.5

(0, – 4)
|z + 2| = |z + 1 + 2i|

1
 = tan 1 ( )  tan 1 (2)
0.5

Im
|z + 1 + 2i| =

(– 2, 0) (0, 0)
Re

Locus sgexamguru.com
of z
( – 1, – 2)

(0, – 4)
|z + 2| = |z + 1 + 2i|

 5 
 
1  
2  1 1 
 = cos
 5   cos ( 2 )  3
 
 
Greatest value of arg (z +1+2i) =   (    )
= 3.08164
= 3.082
Least value of arg (z +1+2i ) =     
= 0.987246 = 0.987
(iv)
Alternative Method

SRJC P1 Solutions 589


13

Equation of circle: (x + 1)2 + (y + 2)2 = 5


y = – 2  5  ( x  1)2 ------------------ (1)
0  (2)
Gradient of the line passing through the (– 2, 0) and (– 1, – 2) = =–2
(2)  (1)
1 1
Gradient of the perpendicular bisector =  =
2 2
1
Equation of the perpendicular bisector: y + 1 = (x + 1.5)
2
1 1
y=x – ------------------ (2)
2 4
Using GC, the points of intersection are (– 3.232051, –1.866025) and
(0.23205,– 0.1339746).

Im
|z + 1 + 2i| =

(– 2, 0) (0, 0)
Re
( – 1.5, – 1) (0.23205,– 0.1339746)
Locus 1
of z
sgexamguru.com 0.5
( – 1, – 2)

(–3.232051, –1.866025) –
(0, – 4)
|z + 2| = |z + 1 + 2i|

Least value of arg (z + 1 + 2i) when z = 0.23205 – 0.1339746i is 0.987.


Greatest value of arg (z + 1 + 2i) when z = – 3.232051 – 1.866025i is 3.082

9 The path travelled by an object measured with respect to the origin in the
horizontal and vertical directions, at time t seconds, is denoted by the variables x
and y respectively.

[TURN OVER]

SRJC P1 Solutions 590


14

dx
It is given that when t  0, x  1 , y = 0 and  1 . The variables are related by
dt

dy d2 x
the differential equations  y  e 2t  4 y 2  0 and 2  cos 2 2t .
dt dt
dw
(i) Using the substitution y  wet , show   1  4 w2 and hence find y in
dt
terms of t.
[6]
(ii) Find x in terms of t. [4]

Solution
dy
(i)  y  e 2t  4 y 2  0
dt
y  w et
dy dw t
 e  w et
dt dt
dy  dw 
 et   w
dt  dt 
 dw  2
et   w   wet  e 2t  4  wet   0

sgexamguru.com
 dt 
 dw 
et   w   wet  et 1  4w2  0
 dt 
dw
 w  w  1  4w2  0
dt
dw
  1  4w2  shown 
dt
1 dw
 1
2 dt
1  4w
1 1
sin  2 w   t  C
2
1
w  sin  2t  2C 
2
1
y  et sin  2t  C 
2
When t  0, y  0
1
sin  2C   0
2
C 0
1 1
 y  et sin  2t  or y   et sin  2t 
2 2
d2 x
(ii) 2
 cos 2 2t
dt

SRJC P1 Solutions 591


15

1  cos 4t

2
dx 1  sin 4t 
 t    D1
dt 2  4 
1  t 2 cos 4t 
x     D1t  D2
2 2 16 
dx
When t  0, x  1, 1
dt
33
D1  1, D2 
32
2
t cos 4t 33
x  t 
4 32 32

Given that f  r  
3r
, show that f  r  2   f  r  
 8r  6  3r .
10 [1]
r 1  r  1 r  3
n
 4r  3 3r
(i) Find  in terms of n . [2]
r 1  r  1 r  3 

(ii) Hence find 


n
 4r  11 3r in terms of n .
[4]
r 1  r  3  r  5 

sgexamguru.com
(iii) Using the result in (ii), show that 
n  r  3r  3n 1
 2 
r 0   r  5  
 

4

51
160
. [3]

Solution
3r  2 3r
f  r  2  f  r   
r  3 r 1
r  3r  2  3r  2  3r  r  3r 1
=
 r  1 r  3
8r  3r  3r 1  3  1
=
 r  1 r  3
=
8r  6  3r
 r  1 r  3
n
 4r  3 3r 1 n
(i)    f  r  2   f  r  
r 1  r  1 r  3  2 r 1

[TURN OVER]

SRJC P1 Solutions 592


16

 f  3  f 1 
 
 f  4   f  2  
 +f 5  f 3 
     
1  = 1  f 1  f  2  + f  n  1 + f  n  2  
=  ...........
2  2 
 f  n   f  n  2  
 
 f  n  1  f  n  1 
 f  n  2   f  n  
  ]
n 1 n 1
9 3  1 3  9 3  4n  9 
=     

4 2  n  2 n  3  4 2  (n  2)(n  3)  ]
r 2
n
 4r  11 3r 1 n  4r  11 3
(ii)   
r 1  r  3  r  5  9 r 1  r  3 r  5 
r
1 n  2  4r  3 3
 
9 r 3  r  1 r  3
r
1  n  2  4r  3 3 21 99 
=    
9  r 1  r  1 r  3 8 15 
n 3
1  9 3  4n  17  21 99 
     
9  4 2  n  4  n  5  8 15 

sgexamguru.com n 1
51 3  4n  17 
= 
40 2  n  4  n  5 
n
r  3r 1 n 4 r  3r
(iii)  2
 
4 r 1  r  5 2
r 0  r  5
1 n  4r  11  3
r

< 
4 r 1  r  5 2
r
1 n  4r  11  3
 
4 r 1  r  3 r  5 
n 1
n
r  3r 1  51 3  4n  17  
So  2
    
4  40 2  n  4  n  5  
r 0  r  5
n
r  3r 3n1 51 3n 1  4n  17  3n 1
 2
   
r 0  r  5 4 160 8  n  4  n  5  4

51 3n 1  4n  17 
=  1  
160 4  2  n  4  n  5  
51 4n  17 17
< since  for all n  0
160 2  n  4  n  5 18

SRJC P1 Solutions 593


17

11 The functions f and g are defined as follows:


f : x   x2  2x , a  x  0
g : x   x  1 , x  1
(i) State the least value of a for the inverse function of f to exist. Hence, find
f −1 in similar form. [4]
For the following parts, use the value of a found in part (i).
(ii) Write down ff 1 in similar form. [1]
(iii) Find the rule for gf in the form bx + c, where b, c   . State its range. [3]
 3  3
(iv) Find the exact range of x for which f  x    g f  x   . [3]
 2  2

Solution
(i) Least a  1
 
f  x     x2  2 x   x 2
 2x
Let y  f  x   x 2  2 x
2
y   x  1  1
2
y  1   x  1
x  1  y  1
x  1  y  1  1  x  0 

sgexamguru.com
f 1 : x  1  x  1 , 1  x  0
(ii) ff 1 : x  x, 1  x  0
(iii) gf  x    x 2  2 x  1
2
  x  1
  x 1
 x 1  D gf  (1, 0] 
Rgf   1, 0 
 3  3
(iv) f  x    g f  x  
 2  2
2
 3  3  3
 x    2  x     x   1
 2  2  2
9 3
x 2  3x   3 x  3  1   0
4 2
5
x2   0
4
 5  5
 x   x  0
 2 
 2 

[TURN OVER]

SRJC P1 Solutions 594


18

5 5
x or x
2 2
 1 3
But after translation by 1.5 units in the positive x – direction, Df  Dgf   , 
 2 2
5 3
  x
2 2

d  2
12 (a)(i) Find
 ln x   . [1]
dx
(ii) The curve C is defined by the parametric equations
x  ln t  t , y  2t  ln  t 2  where t  0 .
2
Another curve L is defined by the equation  4  y   3  x . The graphs of
C and L intersect at the point A  1, 2  as shown in the diagram below.

sgexamguru.com C
L

Find the exact area of the shaded region bounded by C, L and the line
y  4  ln 4 . [6]
2
(b) The region R is the finite region enclosed by the curve  y  1  1  x and the
y-axis. The region S is the region in the 2nd quadrant enclosed by the curve
 
y  2 tan  x   and the axes.
 4
Find the total volume generated when region R and S is rotated through 2
radians about the x-axis, leaving your answers in exact form. [4]

Solution
d  1 2 ln x
 ln x     2 ln x    
2
(ai)
dx  x x

SRJC P1 Solutions 595


19

4  ln 4 4  ln 4
3   4  y  2  dy  x dy
(ii) Area = 2   2
3 4 ln 4
  4  y  2  2
 3 y      ln t  t   2   dt
  3  2 1
 t
1 3 2 2 ln t 
 3  2  ln 4     ln 4   8    2 ln t   2t  2  dt
3  1 t 
3
10  ln 4   2 2 ln t dt   ln t 2  2  t 2  2  2t 2
  1  1  1
  6ln 2  1
3 3
3
10  ln 4  2 t ln t 2  2 2 dt  ln 2 2  3  2
  6 ln 2   1 1  
3 3
3
25  ln 4  4 ln 2  2t 2  ln 2 2
  6ln 2   1  
3 3
3
31  ln 4   ln 2 2
  2ln 2   
3 3
0   1 2 2
(b) Vol =    4 tan 2  x   dx     1  1  x    1  1  x  dx


4  4 0  
0     1
 4    sec2  x    1 dx    4 1  x dx

4  4  0

sgexamguru.com
1
0
 3 
     1  x  2 
 4  tan  x    x   4 
  4    3 
4
  
 2 0
  8 20
 4 1         2
 4 3 3

END OF PAPER

[TURN OVER]

SRJC P1 Solutions 596


1

SERANGOON JUNIOR COLLEGE

2016 JC2 PRELIMINARY EXAMINATION

MATHEMATICS
Higher 2 9740/2
21 Sept 2016

3 hours

Additional materials: Writing paper

List of Formulae (MF15)

TIME : 3 hours

READ THESE INSTRUCTIONS FIRST


Write your name and class on the cover page and on all the work you hand in.

sgexamguru.com
Write in blue or black pen on both sides of the paper.
You may use a soft pencil for any diagrams or graphs.
Do not use staples, paper clips, highlighters, glue or correction fluid.

Answer all the questions.


Give non-exact numerical answers correct to 3 significant figures, or 1 decimal place in the
case of angles in degrees, unless a different level of accuracy is specified in the question.
You are expected to use a graphic calculator.
Unsupported answers from a graphic calculator are allowed unless a question specifically states
otherwise.
Where unsupported answers from a graphic calculator are not allowed in a question, you are
required to present the mathematical steps using mathematical notations and not calculator
commands.
You are reminded of the need for clear presentation in your answers.

The number of marks is given in brackets [ ] at the end of each question or part question.
At the end of the examination, fasten all your work securely together.

Total marks for this paper is 100 marks

This question paper consists of 7 printed pages (inclusive of this page) and 1 blank page.

[TURN OVER

SRJC P2 Questions 597


2

Section A: Pure Mathematics [40 marks].

b
1 (a) If 0  a  b , solve  x a  x dx , leaving your answers in terms of a and b. [2]
0

d  3 x 
dx  1  x 
(b)(i) Find . [1]

3 x

[2]
(ii) Find dx .
x 2  3x  2
1 x
 1  x 
[3]
(iii) Hence find 3
tan 1 1  x dx .
2

2 B C

D
A
5 cm

P Q
1 cm
O
sgexamguru.com 12 cm R

The cuboid above is formed by the eight vertices O, A, B, C, D, P, Q and R.


Perpendicular unit vectors i, j, k are parallel to OR, OP and OA respectively.
The length of OR, OP and OA are 12 cm, 1 cm and 5 cm respectively.
(i) Find the Cartesian equation of line AC. [2]
(ii) Find the acute angle between CA and CR. Hence, find the shortest distance
from R to AC.
[4]
(iii) The point T is on AC produced such that AT   AC and M is the midpoint
of OR. The unit vector in the direction of OT is represented by the vector

OV . By considering the cross product of relevant vectors, find the ratio of
the area of triangle OVM to the area of triangle ORT in terms of  . [3]

3 (a) The complex number w is such that w = a + ib, where a and b are non-zero [3]

SRJC P2 Questions 598


3

real numbers. The complex conjugate of w is denoted by w*. Given that


( w* )2
 3  ib , solve for a and b and hence write down the possible values
w
of w.
(b) (i) Without the use of a graphic calculator, find the roots of the equation
z 2  2 z  4  0 , leaving your answers in the form rei , r > 0 and
     . [2]

(ii) Let α and β be the roots found in (b)(i). If arg    arg    , find

 10   10 and arg  10   10  in exact form. [3]


(c) (i) Show that the locus of z where arg( z  2 3  i)   passes through
6
the point ( 3, 2) . [1]
(ii) Find the Cartesian equation of the locus of z in the form y  mx  c ,
stating its domain clearly. Leave your answer in exact form. [2]

sgexamguru.com
4 A curve C has parametric equations
x  sin t , y  cos t .
(i) Find the equations of tangent and normal to C at the point with parameter t. [3]
(ii) Points P and Q on C have parameters p and q respectively, where
 
0 p and 0  q  . The tangent at P meets the normal at Q at the
2 2
sin q
point R. Show that the x–coordinate of R is . Hence, find in
cos  p  q 

similar form the y–coordinate of R in terms of p and q. [3]


The tangent at P meets the y-axis at the point A and the normal at Q meets the

y-axis at the point B. Taking q   p,
2
1
(iii) Show that the area of triangle ARB is cosec  2 p  . [3]
2
(iv) Find the Cartesian equation of the locus of point R. [3]

Section B: Statistics [60 marks]

[TURN OVER

SRJC P2 Questions 599


4

5 Nicole decides to celebrate her birthday with 9 boys and 2 girls whose names
are Vanessa and Sally.
(a) (i) They have a dinner at a restaurant that can only offer them a rectangular
table as shown in the following diagram, with seats labelled A to L as
shown.
A B C D E F

G H I J K L
Find the number of ways in which at least one girl must be seated at the
seats A, F, G and L. [2]
(ii) Find the number of ways in which they can sit if instead, the restaurant
offers them 2 indistinguishable round tables of 6. [2]
(iii) After the dinner, they went for a movie together. They bought tickets for
seats in a row. Find the number of ways where Nicole and Vanessa must
be seated together but not Sally. [2]
(b) After the celebration, Nicole plays a card game with Vanessa. The pack of
20 cards are numbered 1 to 20. The two friends take turns to draw a card
from the pack. If a prime number is drawn, the player wins the game. If a
composite number (4, 6, 8, 9, 10, 12, 14, 15, 16, 18, 20) is drawn, the player

sgexamguru.com
loses the game and the other player wins. If the number ‘1’ is drawn, the
card is returned and the other player draws the next card. Nicole draws the
first card. Find the probability of her winning the game. [3]

6 In a telephone enquiry service, 92% of calls to it are successfully connected.


The probability of any call being successfully connected is constant. A random
sample of 60 calls is taken each day.
(i) State, in context, an assumption needed for it to be well modelled by a
binomial distribution. [1]
(ii) On a given day, it is found that at most 55 calls went through successfully.
Find the probability that there are at least 50 successful calls in the sample
of 60. [2]
(iii) Estimate the probability that the number of successful calls on any day is
less than 55 in a sample of 60. [4]
(iv) The number of successful calls is recorded daily for 70 consecutive days.
Find the approximate probability that the average number of successful
calls in a day is not more than 55. [2]
7 (a) Tickets are sold for the closing ceremony of an international swimming
competition. It is desired to sample 1% of the spectators to find their

SRJC P2 Questions 600


5

opinions of the goodie bags received during the closing ceremony.


(i) Give a reason why it would be difficult to use a stratified sample. [1]
(ii) Explain how a systematic sample could be carried out. [2]
(b) The random variable X has the distribution N(18,32 ) and the random
variable Y has the distribution N(  ,  2 ) . The random variable T is related to
X 1  X 2  3Y
X and Y by the formula T  , where X 1 and X 2 are two
4
independent observations of X. Given that P(T  10)  P(T  30)  0.0668 ,
find the value of  and the exact value of  . [5]
(c) A survey done on students in a particular college found that the amount of
time a student spends on social media in a week is normally distributed with
mean 7 hours and variance 4 hours2.
Five students are randomly chosen. Find the probability that the fifth student
is the second student who spends more than 10 hours a week on social
media. [2]

8 An advertising display contains a large number of light bulbs which are

sgexamguru.com
continually being switched on and off every day in a week. The light bulbs fail
independently at random times. Each day the display is inspected and any failed
light bulbs are replaced. The number of light bulbs that fail in any one-day
period has a Poisson distribution with mean 1.6.
(i) State, in the context of the question, one assumption that needs to be made
for the number of light bulbs that fail per day to be well modelled by a
Poisson distribution. [1]
(ii) Estimate the probability that there are fewer than 17 light bulbs that needs
to be replaced in a period of 20 days. [2]
(iii) Using a suitable approximation, find the probability that there will be not
fewer than 20 days with more than two light bulbs that will need to be
replaced per day in a period of 8 weeks. [4]
(iv) The probability of at least three light bulbs having to be replaced over a
period of n consecutive days exceeds 0.999. Write down an inequality in
terms of n to express this information, and hence find the least value of n. [4]

9 (a) Observations of 10 pairs of values (x,y) are shown in the table below. [2]

[TURN OVER

SRJC P2 Questions 601


6

x 1 2 3 4 5 6 7 8 9 10
y 0.5 0.6 0.8 0.95 a 1.21 1.36 1.55 1.87 2.11

It is known that the equation of the linear regression line of y on x is


y  0.17321x  0.24133 . Find a, correct to 2 decimal places.
(b) A student wanted to study the relationship between the number of
commercial crimes (c) and the mean years of schooling (s) of the offenders.
The following set of data was obtained.
Year 2009 2010 2011 2012 2013 2014 2015
Mean years of
9.7 10.1 10.2 10.3 10.5 10.6 10.7
schooling (s)
No. of commercial
3359 3504 4080 3507 3947 5687 8329
crimes (c)
(i) Draw a scatter diagram for these values. [2]
(ii) One of the values of c appears to be incorrect. Circle this point on your
diagram and label it P. [1]

sgexamguru.com
It is thought that the number of commercial crimes (c) can be modelled by
one of the formulae after removing the point P.
(A) c  a  b 100s 

(B) c  a  bs
(C) c  a  b ln s
where a and b are non-zero constants.
(iii) With reference to the scatter diagram, explain clearly which model is
the best model for this set of data. For the case identified, find the
equation of a suitable regression line. [2]
(iv) Using the regression line found in (iii), estimate the number of
commercial crimes (to the nearest whole number) when the mean
years of schooling reaches 11. [2]
(v) Comment on the reliability of your answer in part (iv). [1]

10 In the latest Pokkinon Roll game, players go to a battle arena to use their
Pokkinon character to battle against each other. Alvin and Billy are interested to

SRJC P2 Questions 602


7

know how long it takes before someone wins a battle. The time taken by a
randomly chosen player to win a game follows a normal distribution.
(a) Alvin claims that on average, it will take at most 190.0 seconds to win a
battle.
To verify his belief, he surveyed a randomly chosen sample of 45 Pokkinon
Roll gamers and found out that the mean is 195.0 seconds with a variance of
206.0 seconds2.
Carry out an appropriate test at 1% level of significance whether there is any
evidence to doubt Alvin’s claim. State an assumption needed for the
calculation. [5]
(b) Billy also obtained his own data by recording the time taken, in seconds, by
5 randomly chosen gamers as shown below.
188.0 190.0 k 186.0 187.0
However, he believes that it will take 190.0 seconds on average to win a
battle. When he conducted the test at 4.742% level of significance, his
conclusion is one where the null hypothesis is not rejected. The sample mean

sgexamguru.com
time taken is denoted by x .
Given that s2 is the unbiased estimate of the population variance and that the
maximum range of values of x is 188  x  a , write down an equation
involving s and a. [1]
Hence or otherwise find the values of a and k, leaving your answers to the
nearest integer. [5]

THE END

[TURN OVER

SRJC P2 Questions 603


1

SRJC Paper 2 Solutions

Section A: Pure Mathematics [40 marks].

b
1 (a) If 0  a  b , solve  x a  x dx , leaving your answers in terms of a and b.
0 [2]
d  3 x 
(b)(i) Find . [1]
dx  1  x 
3 x
(ii) Find
2 
x  3x  2
dx . [2]

1 x
tan 1 1  x dx .
(iii) Hence find
 3
1  x  2
[3]

Solution
b a b
(a) 0 x a  x dx   x  a  x  dx   x  x  a  dx
0 a
a b
 ax 2 x3   x3 ax 2 
     
 2 3  
0 
3 2  a
 a3 a3   b3 ab 2   a3 a 3 
      
 2 3   3 2   3 2 

sgexamguru.com
    
a3 b3 ab 2
  
3 3 2
 1  1 
 1  x  3  x      
d  3 x   2  1 x 
(bi) 
dx  1  x  1 x
2  2 x  3  x x 1
 3
 3
2 1  x  2 2 1  x  2
3 x 3 x
(ii)  x2  3 x  2dx    x  2  x  1dx
 1 2 
  dx
 x  2 x  1 
 ln x  2  2 ln x  1  c
(iii)
1 x
 3
tan 1 1  x dx
1  x  2

 3 x  1  3  x   1  1   1 
 2  tan 1 x   2       dx
 1 x   1  x   1  1  x  2   1  x 
 3  x  1 3 x
 2  tan 1 x   dx
 1 x   2  x 1  x 

[TURN OVER]

SRJC P2 Solutions 604


2

 3  x  1
 2  tan 1  x  ln x  2  2 ln x  1  c
 1 x 

2 B C

D
A
5 cm

P Q
1 cm
O 12 cm R

The cuboid above is formed by the eight vertices O, A, B, C, D, P, Q and R.


Perpendicular unit vectors i, j, k are parallel to OR, OP and OA respectively.
The length of OR, OP and OA are 12 cm, 1 cm and 5 cm respectively.
(i) Find the Cartesian equation of line AC. [2]
(ii) Find the acute angle between CA and CR. Hence, find the shortest distance
from R to AC. [4]
(iii) The point T is on AC produced such that AT   AC and M is the midpoint
of OR. The unit vector in the direction of OT is represented by the vector

sgexamguru.com
OV . By considering the cross product of relevant vectors, find the ratio of
the area of triangle OVM to the area of triangle ORT in terms of  . [3]

Solution

(i) AC  12i j
0  12 
   
Equation of line AC: r   0     1  ,   
5 0 
   
x
Cartesian equation of line is  y, z  5.
12
(ii) Let the acute angle between AC and RC be x.
12   0 
   
1   1 
0  5
   
cos x 
145 26
Therefore, x  89.1
Let the shortest distance required be y.
y
sin 89.07 
26
y  5.10 cm

SRJC P2 Solutions 605


3

(iii)
 
OT  (  1) OA
OC 

  
OT   OC  (  1) OA
 12  0
   1   (  1)  0 
 
5  5
   
 12 
   
5 
 
1 
Area of triangle ORT = OT  OR
2
Area of triangle OVM
 
1 OT OR
  
2 2
OT

1 1  
=  OT  OR

sgexamguru.com
2 25145 2 2
Therefore the ratio of triangle OVM to area of triangle ORT is 1: 2 25  145 2 .

3 (a) The complex number w is such that w = a + ib, where a and b are non-zero
real numbers. The complex conjugate of w is denoted by w*. Given that
(w* )2
 3  ib , solve for a and b and hence write down the possible values
w
of w. [3]
(b) (i) Without the use of a graphic calculator, find the roots of the equation
z 2  2 z  4  0 , leaving your answers in the form rei , r > 0 and
     . [2]
(ii) Let α and β be the roots found in (b)(i). If arg    arg    , find
10   10 and arg  10   10  in exact form. [3]

(c) (i) Show that the locus of z where arg( z  2 3  i)   passes through
6
the point (  3, 2) . [1]
(ii) Find the Cartesian equation of the locus of z in the form y  mx  c ,
stating its domain clearly. Leave your answer in exact form. [2]

Solution
(w* )2 ( a  ib ) 2
(a)  3  ib   3  ib
w (a  ib)

[TURN OVER]

SRJC P2 Solutions 606


4

a2  b2  2iab  (3  ib)(a  ib)  3a  b2  i(ab  3b)


Equating the real and the imaginary parts:
a 2  b2  3a  b2 …..(1) and
2 ab  ab  3b …(2)
From (2) a  3 since b  0
From (1), 9  b 2  9  b 2
b2  9
b  3
Possible values of w are 3  3i
(bi) z 2  2 z  4  0
2  4  16
z  1  3i
2
   
i  i  
3 3
  1  3 i = 2e and   1  3i = 2e
 i  10   i  10  
(ii)  10   10 = 210  e  3   e  3  
 
 
 10 
= 210  2i sin 
 3 
  2  
= 210  2i sin   

sgexamguru.com
  3 
  3 
= 210  2     i

  2 
= 1024 3i
 10   10  1024 3

So arg  10   10   
2
(ci) When z   3  2i ,
LHS = arg(  3  2i  2 3  i) = arg( 3  i)
 1  
  tan 1     6 (Shown)
 3
 1
(ii) Gradient of the half line is tan(  ) = 
6 3
Equation :
1
y2
3
 
x  3 , x  2 3

3
y x3
3

SRJC P2 Solutions 607


5

4 A curve C has parametric equations


x  sin t , y  cos t
(i) Find the equations of tangent and normal to C at the point with parameter t. [3]

(ii) Points P and Q on C have parameters p and q respectively, where


 
0 p and 0  q  . The tangent at P meets the normal at Q at the
2 2
sin q
point R. Show that the x – coordinate of R is . Hence, find in
cos  p  q 
similar form the y – coordinate of R in terms of p and q. [3]
The tangent at P meets the y-axis at the point A and the normal at Q meets the

y-axis at the point B. Taking q   p ,
2
1
(iii) Show that the area of triangle ARB is cosec  2 p  .
2
[3]
(iv) Find the Cartesian equation of the locus of point R.
[3]

Solution
(i)
x  sin t y  cos t

sgexamguru.com
dx dy
 cos t   sin t
dt dt

dy  sin t

dx cos t
  tan t

Equation of tangent:
y  cos t    tan t  x  sin t 
y    tan t  x   tan t  sin t   cos t
sin 2 t cos 2 t
y    tan t  x  
cos t cos t
y    tan t  x  sec t

Equation of normal:
y  cos t   cot t  x  sin t 
y   cot t  x  cos t  cos t
y   cot t  x
(ii)
Equation of tangent at P (with parameter p):
y    tan p  x  sec p
Equation of normal at Q (with parameter q):

[TURN OVER]

SRJC P2 Solutions 608


6

y   cot q  x
Equating the equation of tangent at P and the equation of normal at Q, we have
 cot q  x    tan p  x  sec p
 cot q  tan p  x  sec p
 cos q sin p  1
  x 
 sin q cos p  cos p
 cos p cos q  sin p sin q  1
 x 
 cos p sin q  cos p
 cos  p  q   1
 x 
 cos p sin q  cos p
sin q
x (Shown)
cos  p  q 
sin q
Substitute x  into the equation of normal at Q,
cos  p  q 
 sin q 
y   cot q  
 cos  p  q  
 
cos q
y
cos  p  q 

sgexamguru.com
(iii)
Coordinates of A:
When x  0, y  sec p
A is  0,sec p  or  0,sin p tan p  cos p 

Coordinates of B:
When x  0, y  0
B is  0, 0  .

 sin q cos q 
Since R is  ,  ,
 cos  p  q  cos  p  q  
Area of Triangle ARB
1  sin q 
=  sec p   
2  cos  p  q  
1  1  1 
=   
2  cos p  2sin p 
1 1 
=  
2  2sin p cos p 
1
= cos ec  2 p  (Shown)
2

SRJC P2 Solutions 609


7

(iv)
 Alternatively,
When q   p,
2 
Since p   q ,
  2
sin   p  sin q
x 2  x
   
cos  2 p   cos   2q 
 2 2 
cos p sin q
 
sin 2 p sin 2q
cos p sin q
 
2sin p cos p 2sin q cos q
1 1
 
2sin p 2cos q

  Alternatively,
cos   p  cos q
y 2  y
   
cos  2 p   cos   2q 
 2  2 
sin p cos q
 
sin 2 p sin 2q

sgexamguru.com
sin p cos q
 
2sin p cos p 2sin q cos q
1 1
 
2 cos p 2sin q
2 2
Using sin p  cos p  1, Alternatively,
2 2
2 2 Using sin q  cos q  1,
 1   1 
    1 2
 1   1 
2
 2x   2 y 
    1
1 1  2 y   2x 
2
 2  1, x  0, y  0 1 1
4x 4 y  2  1, x  0, y  0
2
4x 4 y

[TURN OVER]

SRJC P2 Solutions 610


8

Section B: Statistics [60 marks]

5 Nicole decides to celebrate her birthday with 9 boys and 2 girls whose names
are Vanessa and Sally.
(a) (i) They have a dinner at a restaurant that can only offer them a rectangular
table as shown in the following diagram, with seats labelled A to L as
shown.
A B C D E F

G H I J K L
Find the number of ways in which at least one girl must be seated at the
seats A, F, G and L. [2]
(ii) Find the number of ways in which they can sit if instead, the restaurant
offers them 2 indistinguishable round tables of 6. [2]
(iii) After the dinner, they went for a movie together. They bought tickets for
seats in a row. Find the number of ways where Nicole and Vanessa must
be seated together but not Sally. [2]
(b) After the celebration, Nicole plays a card game with Vanessa. The pack of
20 cards are numbered 1 to 20. The two friends take turns to draw a card
from the pack. If a prime number is drawn, the player wins the game. If a
composite number (4, 6, 8, 9, 10, 12, 14, 15, 16, 18, 20) is drawn, the player
loses the game and the other player wins. If the number ‘1’ is drawn, the
card is returned and the other player draws the next card. Nicole draws the
first card. Find the probability of her winning the game. [3]

Solution sgexamguru.com 9


(i) Number of ways  12!   4!8!
4
9
–   4! (for the selection and the arrangement of the 4 guys to be seated at A,
 4
F, G and L.)
 357073920
9
OR Number of ways  12!   8!4!
5
9
–   8! (for the selection of the 5 guys that is to be seated at B, C, D, E, H, I, J,
5
K with the 3 girls)
 357073920
8
OR Number of ways  12!   9!3!
 3
8
–   3! (for the selection of the 3 seat in the slot B, C, D, E, H, I, J, K)
 3
 357073920

SRJC P2 Solutions 611


9

12 
 6  5!5!
(ii) Number of ways   
2!
12 
– For   5!5!
6
= 6652800
10 
(iii) Number of ways  9!  2!2!
2
Alternative solution 10! 9  2  65318400
10 
– For   2! (the selection of the slots to separate Nicole and Vanessa with
2
Sally)
 65318400
(b)

Nicole loses Nicole wins


Nicole wins
Vanessa
Vanessa Nicole
draws
draws draws
Nicole loses
Nicole loses Nicole wins

sgexamguru.com
Probability of Nicole winning
2 4
8   1   1   11  1  1 3  1  5 
 1        ......          ...
20   20   20   20  20  20   20  
st nd
First – 1 infinite series, Second – 2 infinite series
1
8 1 11 20 3
=  2
  2

20  1  20  1  7
1   1  
 20   20 

6 In a telephone enquiry service, 92% of calls to it are successfully connected.


The probability of any call being successfully connected is constant. A random
sample of 60 calls is taken each day.
(i) State, in context, an assumption needed for it to be well modelled by a
binomial distribution. [1]
(ii) On a given day, it is found that at most 55 calls went through successfully.
Find the probability that there are at least 50 successful calls in the sample
of 60. [2]
(iii) Estimate the probability that the number of successful calls on any day is
less than 55 in a sample of 60. [4]
(iv) The number of successful calls is recorded daily for 70 consecutive days.
Find the approximate probability that the average number of successful
calls in a day is not more than 55. [2]

[TURN OVER]

SRJC P2 Solutions 612


10

Solution
(i) The event that a call is successfully connected is independent from the event
of other calls being successfully connected.
(ii) Let X be the random variable denoting the number of successful calls, out of
a sample of 60 calls.
X ~ B(60, 0.92)
P( X  50 | X  55)
P(50  X  55)

P( X  55)
P( X  55)  P( X  49)

P( X  55)
0.52982  0.074926

0.52982
= 0.986 (3s.f.)
(iii) Let Y be the random variable denoting the number of calls that are not
successfully connected, out of a sample of 60 calls.
Y ~ B  60, 0.08 
Since n = 60 is large, np  60(0.08)  4.8( 5)
 Y ~ Po  4.8  approx
Number of successful calls Number of calls that are not

sgexamguru.com
successfully connected
54 6
53 7
52 8
P  less than 55 successful calls 
 P  at least 6 calls that are not successfully connected 
=P  Y  6 
 1  P  Y  5
= 0.349 (3s.f.)
(iv) X ~ B(60, 0.92)
Since n = 70 is large, by Central Limit Theorem,
 4.416 
X ~ N  55.2,  approx
 70 
P  X  55   0.213  to 3 s.f 
Alternative solution:
Let W be the random variable denoting the number of successful calls out of a
sample of 4200.
W ~ B(4200, 0.92)
Since n is large, np = 3864>5, nq = 336>5
W ~N(3864, 309.12)approx
CC
P W  55  70   P W  3850.5   0.221  to 3 s.f 

SRJC P2 Solutions 613


11

7 (a) Tickets are sold for the closing ceremony of an international swimming
competition. It is desired to sample 1% of the spectators to find their
opinions of the goodie bags received during the closing ceremony.
(i) Give a reason why it would be difficult to use a stratified sample. [1]
(ii) Explain how a systematic sample could be carried out. [2]
(b) The random variable X has the distribution N(18,32 ) and the random
variable Y has the distribution N(  ,  2 ) . The random variable T is related to
X  X 2  3Y
X and Y by the formula T  1 , where X1 and X 2 are two
4
independent observations of X. Given that P(T  10)  P(T  30)  0.0668 ,
find the value of  and the exact value of  . [5]
(c) A survey done on students in a particular college found that the amount of
time a student spends on social media in a week is normally distributed with
mean 7 hours and variance 4 hours2.
Five students are randomly chosen. Find the probability that the fifth student
is the second student who spends more than 10 hours a week on social
media. [2]

Solution
(ai) Though the tickets issued might have a serial number indicated, but some
people who have purchased the tickets, may not turn up for the closing

sgexamguru.com
ceremony and so it is difficult to obtain the actual sampling frame.
(ii) To have a sample consisting of 1% of the spectators present, the sampling
interval will be 100. Randomly select a number between 1 to 100 say r. So at
the entrance point, every rth person for each interval of 100 will be selected for
the survey until the sample is obtained.
(b) E(T )  20
1
 2(18)  3   20
4
44

3
 X  X 2  3Y 
Var(T )  Var  1 
 4 
1
 2  2Var( X )  32 Var(Y ) 
4
1 9 9
 
 2 2(32 )  9 2    2
4 8 16
P(T  10)  0.0668
 
 10  20 
P Z    0.0668
 9 9 2
   
 8 16 

[TURN OVER]

SRJC P2 Solutions 614


12

10  20
 1.500
9 9 2
 
8 16
2
 10  20  9 9 2
    
 1.500  8 16
  8.77533  8.78 (3sf)
(c) Let X be the random variable “time taken by a randomly chosen student on
social media”.
X ~ N(7, 22 )
Required probability
2 3
 4  P  X  10    P  X  10  
 0.014508
 0.0145 (3 s.f)

8 An advertising display contains a large number of light bulbs which are


continually being switched on and off every day in a week. The light bulbs fail
independently at random times. Each day the display is inspected and any failed
light bulbs are replaced. The number of light bulbs that fail in any one-day period
has a Poisson distribution with mean 1.6.
(i) State, in the context of the question, one assumption that needs to be made

sgexamguru.com
for the number of light bulbs that fail per day to be well modelled by a
Poisson distribution. [1]
(ii) Estimate the probability that there are fewer than 17 light bulbs that needs
to be replaced in a period of 20 days. [2]
(iii) Using a suitable approximation, find the probability that there will be not
fewer than 20 days with more than two light bulbs that will need to be
replaced per day in a period of 8 weeks. [4]
(iv) The probability of at least three light bulbs having to be replaced over a
period of n consecutive days exceeds 0.999. Write down an inequality in
terms of n to express this information, and hence find the least value of n. [4]

Solution
(i) The average number of light bulbs that fail in a given time interval is
proportional to the length of the time interval.
(ii) Let V be the random variable denoting “the number of light bulbs that needs
to be replaced in 20 days.”
V ~ Po(20  1.6)
V ~ Po(32)
Since  = 32 > 10, V ~ N(32, 32) approximately

SRJC P2 Solutions 615


13

c.c
P(V < 17)   P(V < 16.5)
= 0.003071651
= 0.00307
(iii) P(X > 2) = 1 – P(X  2) = 0.21664
Let Y be the random variable denoting “the number of days in which at least
three light bulbs will need to be replaced out of 56 days”
Y~B(56, 0.21664)
Since n = 56 is large, np = 12.132 (>5), nq = 43.868
Y~N(12.132, 9.5036) approximately
c.c
P(Y  20)   P(Y  19.5)
= 0.0084228
= 0.00842
(iv) Let W be the random variable denoting “the number of light bulbs that need
to be replaced in n consecutive days.”
W ~ Po(1.6n)
P(W  3) > 0.999
1 – P(W  2) > 0.999
1 – 0.999 > P(W  2)

sgexamguru.com
e (1.6n ) (1.6n)0 e (1.6n ) (1.6n)1 e (1.6 n) (1.6n) 2
0.001 >  
0! 1! 2!
(1.6 n ) (1.6n )
e e (1.6n)  e(1.6n) (1.28n2 ) < 0.001
n = 7, P(W  2) = 0.00102
n = 8, P(W  2) = 0.000264
n = 9, P(W  2) = 0.0000664
Least value of n is 8.

9 (a) Observations of 10 pairs of values (x,y) are shown in the table below.
x 1 2 3 4 5 6 7 8 9 10
y 0.5 0.6 0.8 0.95 a 1.21 1.36 1.55 1.87 2.11

It is known that the equation of the linear regression line of y on x is


y  0.17321x  0.24133 . Find a, correct to 2 decimal places. [2]
(b) A student wanted to study the relationship between the number of commercial
crimes (c) and the mean years of schooling (s) of the offenders. The following set
of data was obtained.
Year 2009 2010 2011 2012 2013 2014 2015
Mean years of
9.7 10.1 10.2 10.3 10.5 10.6 10.7
schooling (s)
No. of commercial
3359 3504 4080 3507 3947 5687 8329
crimes (c)

[TURN OVER]

SRJC P2 Solutions 616


14

(i) Draw a scatter diagram for these values. [2]


(ii) One of the values of c appears to be incorrect. Circle this point on your
diagram and label it P. [1]
It is thought that the number of commercial crimes (c) can be modelled by one
of the formulae after removing the point P.
(A) c  a  b 100 s 
(B) c  a  bs
(C) c  a  b ln s
where a and b are non-zero constants.
(iii) With reference to the scatter diagram, explain clearly which model is the
best model for this set of data. For the case identified, find the equation of
a suitable regression line. [2]
(iv) Using the regression line found in (iii), estimate the number of commercial
crimes (to the nearest whole number) when the mean years of schooling
reaches 11. [2]
(v) Comment on the reliability of your answer in part (iv). [1]

Solution
(a) Using GC, x  5.5
10.95  a
y
10
Since  x , y  lies on the regression line,

sgexamguru.com
10.95  a
 0.17321  5.5   0.24133
10
a  0.98985  0.99 (correct to 2 decimal places)
(b) (i) and (ii)

(10.7, 8329)

(9.7, 3359)

0 s

SRJC P2 Solutions 617


15

(iii) From the scatter diagram (after removing the outlier), as s increases, c
increases at an increasing rate. Hence model (A) is the best model.
From GC, c  2862.048513  1.965434 10 18 100 s 
(iv) When s = 11,
c  2862.048513  1.965434  1018 10011 
 22516
(v) The estimate is unreliable because the data substituted is outside the data
range and so the linear relationship between c and 100s may not hold.

10 In the latest Pokkinon Roll game, players go to a battle arena to use their
Pokkinon character to battle against each other. Alvin and Billy are interested to
know how long it takes before someone wins a battle. The time taken by a
randomly chosen player to win a game follows a normal distribution.
(a) Alvin claims that on average, it will take at most 190.0 seconds to win a battle.
To verify his belief, he surveyed a randomly chosen sample of 45 Pokkinon
Roll gamers and found out that the mean is 195.0 seconds with a variance of
206.0 seconds2.
Carry out an appropriate test at 1% level of significance whether there is any
evidence to doubt Alvin’s claim. State an assumption needed for the
[5]
calculation.
(b) Billy also obtained his own data by recording the time taken, in seconds, by
5 randomly chosen gamers as shown below.

sgexamguru.com
188.0 190.0 k 186.0 187.0
However, he believes that it will take 190.0 seconds on average to win a
battle. When he conducted the test at 4.742% level of significance, his
conclusion is one where the null hypothesis is not rejected. The sample mean
time taken is denoted by x .
Given that s2 is the unbiased estimate of the population variance and that the
maximum range of values of x is 188  x  a , write down an equation
involving s and a. [1]
Hence or otherwise find the values of a and k, leaving your answers to the
nearest integer. [5]

Solution
(a) Assume that the time taken to win any battle is independent of other battles.
Let Y denote the time taken to win a randomly chosen battle
45
s 2   206   210.68
44
H0 :   190
H1 :   190
 210.68 
Under H0 Y  N 190, 
 45 
  190 , y  195 , n = 45, s = 210.68
Using G.C, p-value is 0.0104
Since p value > 0.01, we do not reject H0 and conclude that there is insufficient
evidence to doubt Alvin’s claim at 1% level of significance.

[TURN OVER]

SRJC P2 Solutions 618


16

(b) H0 :   190
H1 :   190
2-tailed T-test at 4.742% level of significance
T  t  4
2.82844s
a  190
5
a  1.26s  190
188  x  a
2 5 x  190 5  a  190 
 
s s s
5
2 5
Since  2.82844
s
s  1.5811
s 2  2.5
2.82844 1.5811
So a   190  192
5
OR by symmetry of curve, a = 192
 x  751  k

21
sgexamguru.com
 x 2  141009  k 2
s  141009  k  2  751  k 2 

4 5 

1 2  751  k 2 
So 2.5  141009  k  
4 5 

k =189 or k = 186.5 (rejected since 188  x  a )

THE END

SRJC P2 Solutions 619


TEMASEK JUNIOR COLLEGE, SINGAPORE

Preliminary Examination 2016


Higher 2

MATHEMATICS 9740/01
Paper 1
30 August 2016

Additional Materials: Answer paper 3 hours


List of Formulae (MF15)

READ THESE INSTRUCTIONS FIRST

Write your Civics group and name on all the work that you hand in.
Write in dark blue or black pen on both sides of the paper.
You may use a soft pencil for any diagrams or graphs.

sgexamguru.com
Do not use staples, paper clips, highlighters, glue or correction fluid.

Answer all the questions.


Give non-exact numerical answers correct to 3 significant figures, or 1 decimal place in the case of
angles in degrees, unless a different level of accuracy is specified in the question.
You are expected to use a graphic calculator.
Unsupported answers from a graphic calculator are allowed unless a question specifically states
otherwise.
Where unsupported answers from a graphic calculator are not allowed in a question, you are required
to present the mathematical steps using mathematical notations and not calculator commands.
You are reminded of the need for clear presentation in your answers.

The number of marks is given in brackets [ ] at the end of each question or part question.
At the end of the examination, fasten all your work securely together.

This document consists of 6 printed pages.

© TJC 2016 [Turn over

Microsoft Word - 2016 TJC Prelim H2 Maths Paper 1 (Questions) 620


2

1 A fitness assessment walk is conducted where participants walk briskly around a


running path. The participants’ walking time and heart rate are recorded at the end of
the walk.

The formula for calculating the Fitness Index of a participant is as follows:


420 + (Age  0.2)  (Walking Time  a)  (Body Mass Index  b)  (Heart Rate  c)
where a, b and c are real constants.

Data from 3 participants, Anand, Beng and Charlie are given in the table.

Name Age Walking Time Body Mass Index Heart Rate Fitness Index
Anand 32 17.5 25 100 102.4
Beng 19 18.5 19 120 92.6
Charlie 43 17 23 90 121.2

Find the values of a, b and c. [3]

x 2  2a 2
2 Solve the inequality  a , giving your answer in terms of a, where a is a
x
positive real constant. [3]

Hence solve sgexamguru.com


x 2  2a 2
x
 a. [2]

x
3 (i) Use the substitution u = x2 to find  k 2  x2
dx in terms of x and the constant k.

[3]
2
(ii) Find the exact value of  0
f(x) dx , where

 2
 6  x 2 , 0  x  2,
f(x)  
x
 , 2  x  2. [3]
 6  x
2

4 Relative to the origin O, the points A, B, M and N have position vectors a, b, m and n
respectively, where a and b are non-parallel vectors. It is given that m = a + (1  )b
and n = 2(1  )a  b where  is a real parameter.
Show that m  n = (32  4 + 2) (b  a). [2]
π
It is given that∣a∣= 3,∣b∣= 4 and the angle between a and b is . Find the
6
smallest area of the triangle MON as  varies. [4]

TJC/MA 9740/Preliminary Exam 2016

TJC P1 Questions 621


3

5 B

A  P C

In the diagram, A and C are fixed points 500 m apart on horizontal ground. Initially, a
drone is at point A and an observer is standing at point C. The drone starts to ascend
vertically at a steady rate of 3 m s 1 as the observer starts to walk towards A with a
steady speed of 4 m s 1 . At time t, the drone is at point B and the observer is at point P.
 3t 
Given that the angle APB is  radians, show that   tan 1  . [2]
 500  4t 
d
(i) Find in terms of t. [2]
dt
(ii) Using differentiation, find the time t when the rate of change of  is maximum. [4]

6 The functions f and g are defined by


f : x  ln  x 2  x  1 , x  1,
g : x  e x , for all real x.

sgexamguru.com
Sketch the graph of f and explain why f does not have an inverse.
The function h is defined by
[2]

h : x  f ( x), x  k .
State the maximum value of k such that h 1 exists. [1]
Using this maximum value of k,
(i) show that the composite function gh exists, [1]
find  gh  ( x ) , stating the domain of  gh  .
1 1
(ii) [4]

7 (a) The positive integers are grouped into sets as shown below, so that the number
of integers in each set after the first set is three more than that in the previous
set.
{1}, {2, 3, 4, 5}, {6, 7, 8, 9, 10, 11, 12}, …
Find, in terms of r, the number of integers in the rth set. [1]
r
Show that the last integer in the rth set is (3r  1) . [2]
2
Deduce, in terms of r, the first integer in the rth set. [2]

n  1  1 2 1 
r

(b) Find  1      ...     in terms of n. [4]


 2 2
r 1   2  
[Turn over

TJC/MA 9740/Preliminary Exam 2016

TJC P1 Questions 622


4

8 The graph of y = f(x) intersects the axes at A(0, 2), B(2, 0) and C(6, 0) as shown below.
The lines y = 4 and x = 5 are asymptotes to the graph, and B(2, 0) is a minimum point.
y
y = f(x)

y=4
A(0, 2)
x
B(2, 0) C(6, 0)

x=5
On separate diagrams, sketch the graphs of
(i) y = f(|x|), [2]

(ii) y2 = f(x), [3]


1
(iii) y , [3]
f ( x)
stating the equations of any asymptotes, coordinates of any stationary points and points

sgexamguru.com
of intersection with the axes.

9 (a) Given that x is small such that x3 and higher powers of x can be neglected, show
that
π 
2 sin   x 
4   a  bx  cx 2 ,
2  cos x

for constants a, b and c to be determined. [4]

(b) The curve y  f ( x ) passes through the point (0, 1) and satisfies the differential
equation
dy
(1  x 2 )  e y .
dx
(i) Find the Maclaurin series for y, up to and including the term in x2. [3]
(ii) By using an appropriate expansion from the List of Formulae (MF15),
obtain the Maclaurin series for ln  2  y  , up to and including the term in
x2 . [3]

TJC/MA 9740/Preliminary Exam 2016

TJC P1 Questions 623


5

10
y

x
0 x=3

The diagram shows the curve with parametric equations


1
x  2t  t 2 , y  , for t  1.
(1  t ) 2
The curve has a vertical asymptote x = 3.
(i) Find the coordinates of the points where the curve cuts the y-axis. [2]

(ii) Find the equation of the tangent to the curve that is parallel to the y-axis. [4]

(iii) Express the area of the finite region bounded by the curve and the y-axis in the
b
form a f (t ) dt , where a, b and f are to be determined. Use the substitution

sgexamguru.com
u  1  t to find this area, leaving your answer in exact form. [5]

11 On the remote island of Squirro, ecologists introduced a non-native species of insects


that can feed on weeds that are killing crops. Based on past studies, ecologists have
observed that the birth rate of the insects is proportional to the number of insects, and
the death rate is proportional to the square of the number of insects. Let x be the number
of insects (in hundreds) on the island at time t months after the insects were first
introduced.
Initially, 10 insects were released on the island. When the number of insects is 50, it is
3
changing at a rate that is times of the rate when the number of insects is 100. Show
4
that
dx
  x 2  x
dt
where  is a positive real constant. [3]
p
Solve the differential equation and express x in the form , where p and q are
1  qe 2  t
constants to be determined. [6]

Sketch the solution curve and state the number of insects on the island in the long run.
[3]

[Turn over
 
TJC/MA 9740/Preliminary Exam 2016

TJC P1 Questions 624


6

12 (a) The complex numbers z1 and z2 satisfy the following simultaneous equations

2 z1  i z2*  7  6 i ,

z1  i z2  6  6i .

Find z1 and z2 in the form x  y i , where x and y are real. [4]

1 1
(b) It is given that w   i . Find the modulus and argument of w, leaving your
2 2
answers in exact form. [2]

It is also given that the modulus and argument of another complex number v is 2
π
and respectively.
6
v
(i) Find the exact values of the modulus and argument of . [3]
w*
(ii) By first expressing v in the form c  d i where c and d are integers, find
v
the real and imaginary parts of in surd form. [3]
w*
π
(iii) Deduce that tan    2  3 . [2]
 12 

sgexamguru.com
   End of Paper   

TJC/MA 9740/Preliminary Exam 2016

TJC P1 Questions 625


2016 Preliminar
P y Examinaation H2 Mathematics
M s 9740 Papeer 1 (Solutiions)

1
420  66.4  17.5a  25b  100c  102.4 5b  100c  324      (1)
17.5a  25
420  3.8  18.5a  19b  120c  92.6 9b  120c  331.2     (2)
orr 18.5a  19
420  8.6  17 a  23b  90c  121.2
1 17 a  23b  90c  307
7.4      (3)
58 13 14
GC, a 
Using G , b ,c
5 5 25
2
x 2  2a 2
a , x0
x
x 2  axx  2a 2
0
x
x ( x  a )( x  2a )  0
x   a or 0  x  2a

Replacee x by | x |,
| x |  a or 0 | x | 2a
(no reall solution) 2a  x  2a, x  0

3
(i)
sgexamguru.com
du dx 1
ux 2
  2x or x u  
dx du 2 u

x 1 1
 k 2  x2
dx 
2  k2 u
du

1 k2 u
 1 C
2  2 (1)

 k 2  x2  C

(ii)
2 2 2 2 x

0
f(x) dx  
0 6 x 2
dx  
2
6  x2
dx

2
 2  6  x  2
 ln       6  x 2
  2
 2 6  6  x   0
1  6 2
 ln   2 2
6  6  2 

Microsoft Word - 2016 TJC Prelim H2 Maths Paper 1 (Solutions) 626


4

m  n = (a + (1  )b)  (2(1  )a  b)


= 2(1  ) (a  a)  2 (a  b) +2(1  )2 (b  a)  (1  ) (b  b)
= 2(1  )2 (b  a)  2 (a  b) since a  a = 0 = b  b
~

= (2(1  )2 +2) (b  a) since a  b =  b  a


= (32  4 + 2) (b  a)
1 1
Area of triangle MON  m  n  (3 2  4  2)(b  a)
2 2
1
 3 2  4  2 b  a
2
2
1  2  2 π
 3      b a sin
2  3  3 6
2
 2 2
 3 3    
 3 3
2
 smallest area is 3   2 units2
3

Alternative solution

sgexamguru.com
Using GC, the minimum value of 3 2  4  2 occurs when  
2
3
1  2 2 
2
π 1 2
 smallest area  3    4    2 b a sin     6  2 units 2
2   3   3   6 2 3

5
B
At time t, AB = 3t, AP = 500 – 4t
AB 3t
tan   
AP 500  4t
 3t 
  tan 1  (shown)
 500  4t  A P C

(i)
d

1

 500  4t  3  3t  4 
 500  4t 
2 2
dt  3t 
1  
 500  4t 
 500  4t 
2
1500 1500
  
 500  4t    3t   500  4t  9t 2   500  4t 
2 2 2 2

 1500 60 
  2 
 25t  4000t  250000 t  160t  10000 
2

TJC P1 Solutions 627


d  d  d  1500 18t  2  500  4t  4   1500  50t  4000 
2
(ii)     
 2 2
  
2 2
2
dt  d t  dt 9t   500  4t 
2
9t 2   500  4t 
d2
 0   1500(50t  4000)  0  t  80
dt 2
t 80  80 80 
d 2
+ve 0 ve
dt 2
slope

Using first derivative test, rate of change of  is maximum at t = 80


6

1 x
O
The line y = 0 cuts the graph of f twice, thus f is not one-one and so f does not have an
inverse.
1
Using GC, minimum value of f occurs when x 


2
1 3 sgexamguru.com
OR x 2  x  1   x     minimum point:  , 
2
1 3
 2 4 2 4
1
Hence maximum value of k is
2
 3 
(i) Since Rh  ln ,     ,    Dg , the function gh exists.
 4 

  
(ii) gh  x   g ln x 2  x  1  eln( x  x 1)  x 2  x  1 , x 
1
2

2
Let y  gh  x 
2
 1 3
y  x  x 1   x   
2

 2 4
1 3  1 3 1
 x  y  reject x   y   x  
2 4  2 4 2
1 3
  gh   x 
1
 x
2 4
  ,  
3
D 1  R gh
 gh  4 

TJC P1 Solutions 628


7

(a) {1}, {2, 3, 4, 5}, {6, 7, 8, 9, 10, 11, 12}, …


Set 1st 2nd 3rd ,…
No. of terms 1 4 7 ,…

No. of integers in rth set = 1 + (r – 1)3


= 3r – 2

Last integer in rth set = Sum of no. of terms from 1st to rth set
 1  4  7  ...  (3r  2)
r
 2 1   r  1 3 

2
r
or 1  (3r  2) 
2
r
 (3r  1)
2
r
Hence first integer in rth set  (3r  1)  (3r  2)  1
2
1
or (r  1) 3( r  1)  1  1
2
3r 2  7 r  6

sgexamguru.com
2

(b) n  1  1 2 1 
r


r 1 
1      ...    
2 2 2 

 
n 
 1 1   1  r1 
 2  
r 1  1  2 
1

 

 
n
 2 1  12  12 
r

r 1
n
 2n    12 
r

r 1

1  
n
1 1
 
2 2
 2n 
1  12
n
 2n  1   12 

TJC P1 Solutions 629


8
(i) y = f(|x|)

y=4

A(00, 2)

x
(
(−6, 0) (−2, 0)) 2, 0)
B(2 C(6, 0)

x = −5
− x=5

(ii) y2 = f(x)
y

sgexamguru.com
y=2

(0, )

B(2, 0)
0 C(6, 0) x

(0, − )

y = −2

x=5

TJC P1 Solutions 630


1
(iii) y 
f ( x)
y

(0, )

y=

x
(5, 0)

x=2 x=6

9
     
2 sin   x  2  sin cos x  cos sin x 
(a) 4   4 4 
2  cos x 2  cos x
sgexamguru.com

sin x  cos x
2  cos x
 x2 
x  1  
 2 

 x2 
2  1  
 2 
2
x
1 x 
 2
2
x
1
2
1

 x2   1  2
 1  x   1  x 2 
 2  2 
 x2   1 
 1  x  1  x 2  ... 
 2  4 
3
 1  x  x 2  ...
4

dy
(b)(i) Given (1  x 2 )  e y
dx

TJC P1 Solutions 631


d2 y dy dy
Implicit diffferentiate w.r.t.
w x, (1 x 2 )
2
 2x  e y
dx dx dx
2
dy d y
When x  0 , y  1 ,  e andd 2
  e2
dx dx
e2 2
So y  1  e x  x  ...
2

 e2 
(ii) ln  2  y   ln 1  e x  x 2  ...  use reesult in (i)
 2 
2
 e2  1 e2 
  e x  x2  ...    e x  x 2  ...  usee standard seeries for ln
 2  2 2 
2
e 1
 e x  x2   e x 
2

2 2
 ex  e x2 2

10
y
(i) When x = 0, t  2  t   0  t  0 or t  2
W
C
Coordinates are (0, 1) an
nd (0, 1)
9

dx dy 2
(ii)  2  2t , 
dt dt (1  t )3

sgexamguru.com
dy 1
 
dx 1  t  (1  t )3 0
x
x=3
W
When tangennt is parallell to y-axis,
1  t  (1  t )3  0  t  1 or t  1 (vertical asymptote)
Equation of tangent
t is x  1

1
Area   1
(iii) A x dy
9 Let u = 1  t
0 2 du
   (2t  t 2 )  dt  1
2 1  t )3
(1 dt
1 2 When t = 0, u = 1
  (2(1  u )  (1  u )2 )  du
3 u3 When t = 2, u = 3
3 u 2  4u  3
 2 du
u3
1

3 1 4 3 
 2   2  3  du
1 u u u 
3
 4 3 
= 2 lln u   2 
 u 2u 1
 4 3  3  8
= 2  ln 3      4      2 ln
l 3
 3 18
1   2  3

TJC P1 Solutions 632


11
dx
 birth rate – death rate
dt
  x   x2 where  and  are positive real constants

dx 3 dx
Given  
dt x  1 4 dt x 1
2
2
1 1 3
                2
2 2 4
dx
Hence   x 2  x
dt
1
 2x  x 2
dx    dt

1 1 1 
  
2  x 2 x
 dx    dt
1
ln | x |  ln | 2  x |   t  c
2
1 x 
ln  t  c
2  2  x 

x
 Ae 2  t where A  e 2c
2 x

Subst t  0, x  0.1  sgexamguru.com


0.1
1.9
 A  A
1
19
2 2t 1
x e  x e2  t
19 19
2 2t
e
x  19
1
1  e2 t
19
2e 2  t 2
 2t

19  e 1  19e 2  t

Alternative solution:
1
 2x  x 2
dx    dt

1
 dx    dt
 x  1
2
1
1   x 11  
  ln     t  c
2   x 1 1  
1   x2
  ln     t  c
2  x 

TJC P1 Solutions 633


x2
 Ae 2  t where A   e 2 c
x
1.9
Subst t  0, x  0.1   A  A  19
0.1
x 1  19e 2  t   2
2
x
1  19e 2  t

2 x=2

2
x
  1  19e2  t

  0.1
t
  0

The number of insects will approach 200 in the long run.

12
sgexamguru.com
(a) 2 z1  i z 2*  7  6 i --- (1)

z1  i z 2  6  6 i --- (2)

(1) – (2) x 2: i z2  2i z2  7  6i  2  6  6i   5  6i
*

z 2*  2 z 2  6  5 i

Since z2  2z2  3Re  z2   Im  z2  i  6  5i , z2  2  5i


*

Sub z2  2  5i into (2): z1  6  6i  i  2  5i   1 4i

2 2
1  1 1
(b) w        
2  2 2

π
arg  w   tan 1  1  
4

v v v 2
(i)    2 2
w* w* w  1 
 
 2

TJC P1 Solutions 634


 v  π π π
arg    arg  v   arg  w *  arg  v   arg  w     
 w*  6 4 12

 π π
(ii) v  2  cos  i sin   3  i
 6 6

v
  

3  i 2 3  i 1 i 
w* 1  1 i 1 i 1 i
2 2
   
3 1  1 3 i 
 v   v 
 Re    3  1 and Im    1 3
 w*   w* 

Alternative solution

1   π  π 
 2 cos     i sin      1  i
w*   4  4 

v
w*
  
3  i 1  i   3  3i  i  1    
3 1  1 3 i 

sgexamguru.com
(iii) Using results in (i) and (ii),

Im

O
Re

π 3 1 3 1
From the Argand diagram, tan      2 3
 12  3  1 3 1

TJC P1 Solutions 635


TEMASEK JUNIOR COLLEGE, SINGAPORE

Preliminary Examination 2016


Higher 2

MATHEMATICS 9740/02
Paper 2
14 September 2016

Additional Materials: Answer Paper 3 hours


List of Formulae (MF15)
Graph Paper

READ THESE INSTRUCTIONS FIRST

Write your Civics group and name on all the work that you hand in.
Write in dark blue or black pen on both sides of the paper.
You may use an HB pencil for any diagrams or graphs.

sgexamguru.com
Do not use staples, paper clips, highlighters, glue or correction fluid.

Answer all the questions.


Give non-exact numerical answers correct to 3 significant figures, or 1 decimal place in the case of
angles in degrees, unless a different level of accuracy is specified in the question.
You are expected to use an approved graphing calculator.
Unsupported answers from a graphing calculator are allowed unless a question specifically states
otherwise.
Where unsupported answers from a graphing calculator are not allowed in a question, you are
required to present the mathematical steps using mathematical notations and not calculator
commands.
You are reminded of the need for clear presentation in your answers.

The number of marks is given in brackets [ ] at the end of each question or part question.
At the end of the examination, fasten all your work securely together.

This document consists of 6 printed pages.

© TJC 2016 [Turn over

Microsoft Word - 2016 TJC Prelim H2 Maths Paper 2 (Questions) 636


2

Section A: Pure Mathematics [40 marks]

1 A sequence u1 , u2 , u3 , … is such that u1  2 and


1
un  un 1  , for all integers n  2 .
n  n  1
1 n
1
(i) By expressing in partial fractions or otherwise, find  in terms
r  r  1 r  2 r  r  1

of n. [4]

n
(ii) By considering  u
r 2
r  ur 1  and using the result in part (i), show that for all

b
integers n  2 , un can be expressed in the form a  , where a and b are
n
constants to be determined. [3]

2 Solve the equation w4  2  2 3 i  0 , giving the roots in the form r ei , where
 π    π and r > 0. [4]

π
The roots represented by w1 and w2 are such that   arg  w1   0 and
2

sgexamguru.com
π
 arg  w2   π .
2
The complex number z satisfies the relations z  w1  z  w2 and
π
  arg  z   1  i    0 .
4
On an Argand diagram, sketch the region R in which the points representing z can lie.
[3]
Find the exact area of R. [3]

x 1 z  6
3 The line l has equation  , y  4 and the point A has coordinates (1, 3, 5).
1 2

(i) Find the position vector of the foot of the perpendicular from A to l. [3]

(ii) Plane p1 contains l and A. Show that the equation of p1 is 2x + y + z = 4. [3]

Given that the plane p2 has equation x + 2y + cz = 5 where c is a negative constant,


and that the acute angle between p1 and p2 is 60, find the value of c. [3]

(iii) Find the equation of the line of intersection, m, between p1 and p2. [1]

(iv) The plane p3 has equation 3x + y + 7z =  where  and  are constants. Given
that the planes p1, p2 and p3 have no common point, what can be said about the
values of  and  ? [2]

TJC/MA 9740/Preliminary Exam 2016


TJC P2 Questions 637
3

4 (a) The diagram below shows the graphs of y  x (ln x) 2 and y  x . The two graphs
1 1
intersect at the points  ,  and (e, e).
e e
Find the exact area of the shaded region bounded by the graphs of y  x (ln x) 2
and y  x . [5]
y y  x (ln x) 2
y=x

(e, e)

1 1
 , 
e e

x
O 1

Hence, without integrating, find the exact area of the region bounded by the
1
graphs of y  x (ln x) 2 and the lines y  e and x  . [2]
e

(b)
sgexamguru.com
Find the volume of the solid formed when the shaded region bounded by the lines
x = –4, y = 2 and the ellipse ( x  2) 2  4( y  1) 2  4 is rotated through 2π radians
about the y-axis. Give your answer correct to 1 decimal place. [4]

y
( x  2) 2  4( y  1) 2  4
2

x
–4 –2 O

[Turn over
TJC/MA 9740/Preliminary Exam 2016
TJC P2 Questions 638
4

Section B: Statistics [60 marks]

5 An apartment block has 24 two-bedroom apartments and 88 four-bedroom apartments.


A surveyor wishes to conduct interviews on 35 households in this block to learn about
their household expenditure. She decides to use stratified sampling across apartment
types in the block, assuming that only one household occupies each apartment.

(i) Give an advantage of this sampling method. [1]

(ii) Describe how a stratified sample can be obtained. [3]

6 A mathematician arranges all eight letters in the word PARALLEL to form different
8-letter code words. Find the number of different code words that can be formed if

(i) the code words start with an L and end with an A, [1]

(ii) the 2 A’s are not adjacent to each other, [3]

(iii) there is exactly one letter between the first and second L, and exactly one letter
between the second and third L. [3]

7 Anand, Beng and Charlie are selling cupcakes to raise funds for the charity, Boys And

sgexamguru.com
Girls Understand Singapore (BAGUS). Anand will bake 60% of the cupcakes, Beng
will bake 40% of the cupcakes and Charlie will spread frosting on all the cupcakes.

20% of Anand’s cupcakes will turn out flawed, while 12% of Beng’s cupcakes will turn
out flawed. Charlie has a 6% chance of spreading the frosting badly on any cupcake,
regardless of whether it is flawed or otherwise.

If a cupcake is flawed or has frosting spread badly (or both), it is considered sub-
standard. Otherwise, a cupcake is considered “good”.

Find the probability that

(i) a randomly chosen cupcake is “good”, [2]

(ii) a randomly chosen cupcake is either baked by Anand or is sub-standard but not
both, [3]

(iii) a randomly chosen cupcake is baked by Anand given that it is sub-standard. [3]

TJC/MA 9740/Preliminary Exam 2016


TJC P2 Questions 639
5

8 The rate of growth, x units per hour, of a particular family of bacteria is believed to
depend in some way on the controlled temperature T o C. Experiments were undertaken
in the laboratory to investigate this and the results were tabulated as follows:

T 10 20 30 40 50 60 70 80
x 33 37 41 48 55 65 78 94

Draw a scatter diagram for these values, labelling the axes clearly. [1]

State, with a reason, which of the following model is most appropriate for the given
data,
(A) x  a  bT (B) x  aebT (C) x  a  b ln T
where a and b are constants, and b  0 . [1]

In addition, when T = 90, the rate of growth of bacteria was k units per hour.

Use the most appropriate model identified for the subsequent parts of this
question.
(i) A suitable linear regression line was constructed based on all 9 pairs of
transformed data, including the additional pair of data. If the values of a and b
were determined to be 27.06 and 0.01497 respectively, find the value of k, correct
to the nearest whole number. [4]

sgexamguru.com
(ii) Find the product moment correlation coefficient for all 9 pairs of transformed
data. [1]

(iii) Use the regression line in (i) to predict the growth rate of the bacteria when the
temperature is 105  C , giving your answer to the nearest whole number.
Comment on the reliability of this prediction. [2]

9 Small defects in a twill weave and satin weave occur randomly and independently at a
constant mean rate of 1.2 defects and 0.8 defects per square metre respectively.

(i) Find the probability that there are exactly 9 defects in 7 square metres of twill
weave. [2]

(ii) A box is made from 2 square metres of twill weave and 3 square metres of satin
weave, chosen independently. A box is considered “faulty” if there are more than
10 defects. Show that the probability that a randomly chosen box is “faulty” is
0.0104, correct to 3 significant figures. [2]

(iii) A random batch of 50 boxes is delivered to a customer once every week. The
customer can reject the entire batch if there are at least 2 “faulty” boxes in the
batch. Using a suitable approximation, find the probability that the customer will
reject the entire batch in a randomly selected week. [3]
Hence estimate the probability that the customer will reject 2 batches in a
randomly selected period of 4 weeks. [2]

[Turn over
TJC/MA 9740/Preliminary Exam 2016
TJC P2 Questions 640
6

10 Newmob is a mobile phone service provider which sells several brands of mobile
phones. uPhones and Samseng phones are sold at a subsidy to its subscribers. Each
subscriber can either buy one uPhone or one Samseng phone or both one uPhone and
one Samseng phone. The probability that a randomly chosen subscriber buys a uPhone
is 0.3, and independently, the probability that the subscriber buys a Samseng phone is
p.

(i) In a random sample of 50 subscribers, the probability that at most 20 subscribers


buy a Samseng phone is twice the probability that exactly 15 subscribers buy a
uPhone. Find the value of p. [3]
For the remainder of this question, you may take the value of p to be 0.4.
(ii) In a random sample of 50 subscribers, find the probability that the number of
subscribers who buy a uPhone is greater than the expected number of subscribers
who buy a Samseng phone. [2]

(iii) Each subsidy for the uPhone costs Newmob $280 and each subsidy for a
Samseng phone costs $200. Newmob has 1000 subscribers. Using suitable
approximations, find the probability that the total subsidy given by Newmob for
uPhone purchases exceeds the total subsidy given for Samseng phone purchases.
[5]

sgexamguru.com
11 The volume of a packet of soya bean milk is denoted by V ml and the population mean
of V is denoted by  ml. A random sample of 80 packets of soya bean milk is taken
and the results are summarised by

 (v  250)  217,  (v  250)2  30738.


Test, at the 4% significance level, whether  is less than 250 ml. [5]

Explain, in the context of the question, the meaning of ‘at the 4% significance level’. [1]

In another test, using the same data, and also at the 4% significance level, the
hypotheses are as follows.
Null hypothesis:   0
Alternative hypothesis:   0
Given that the null hypothesis is rejected in favour of the alternative hypothesis, find the
set of possible values of 0 . [4]

It is now given that  = 250 and the population variance is 385. A random sample of
50 packets of soya bean milk is taken and the total volume of the packets is denoted by
T ml. By considering the approximate distribution of T and assuming that the volumes
of all packets of soya bean milk are independent of one another, find P(T > 12600). [3]

End of Paper

TJC/MA 9740/Preliminary Exam 2016


TJC P2 Questions 641
1

1i 1 A B
Let  
r  r  1 r r  1

A(r  1)  Br  1
Sub r  0  A  1
Sub r  1  B  1
1 1 1
So  
r  r  1 r  1 r
n
1 n
 1 1

r  2 r  r  1
  
r 2  r  1
 
r

1 1
 
1 2
1 1
 
2 3


1 1
 
n  2 n 1

sgexamguru.com 
1
n 1

1
n
1
 1
n

1ii n n
1
  ur  ur 1 
r 2
 
r  2 r  r  1

u2  u1
 u3  u2
 1
   1  
 n
 un1  un 2
 un  un1

1 1
un  u1  1   1
n n

Microsoft Word - 2016 TJC Prelim H2 Maths P2 (Solution) 642


2

2 w 4  2  2 3 i  0  w 4  2  2 3 i

i
 4e 3

 2π 
i   2 kπ 
 4e  3 

1
1  i  23π  2 kπ   4  π kπ 
i  
 w  4 4 e  
  2e 
6 2 
, where k  0,  1,  2
 
π 2π  5π   π
i i i   i  
w  2e , 6
2e 3
, 2e  6 
, 2e  3

 π 2π
i   i
 w1  2e  3
, w2  2e 3

Im

w2

A(−1, 1)

sgexamguru.com O
Re

w1
Im

w2
A(−1, 1) B C

Re
O

w1
1
tan 
6 BC
BC  3

TJC P2 Solutions 643


3

Area of shaded region


1
 
B
 1  3 1
2
1 3 1

2

3i x 1 z  6
Let    , y4
1 2
x  1   , z  6  2 , y4

 1   1 
   
l: r =  4     0  ,  
 6  2
   

Let N be the foot of the perpendicular from A to l.


 1   1  1   
      
ON   4     0  =  4  for some  
 6   2   6  2  A
     
  
  
sgexamguru.com
 AN   1 
 1  2 
  B d N
l
    1
    2
AN  l i.e.  1   0   0   =
 1  2   2  5
  
 7 
 1  
Thus ON   20 
5 
 26 
3ii Let B be the point on l with coordinates (1, 4, 6)
 1   1   0  2
        1 
BA   3    4    1 or AN    5 
 5   6   1  5 
      1
 1  0   2   1  2   2
           
A normal to p1 is  0    1   1  or  0    5   5  1 
 2   1  1 2 1 1
           
 2   1   2 
    
Equation of p1 is r  1    3   1   4 i.e 2x + y + z = 4
 1   5   1 
    

TJC P2 Solutions 644


4

1
 
Equation of p2 is r  2   5
a
 
 2 1
  
1  2
1 c 
   
cos 60 
6 5  c2
1 4c

2 6 5  c2

30  6c 2  4 c 2  8c  16 
c 2  16c  17  0
(c  17)(c  1)  0
c  17 (rejected since c < 0) or c  1
3iii p1: 2x + y + z = 4
p2: x + 2y + cz = 5

Using GC, x = 1  t
y = 2 + t
sgexamguru.com
z=t
 1   1
   
Equation of m is r =  2   t  1  , t 
0 1
   
3iv Since the 3 planes are not parallel and they have no common point,
m is parallel to p3 but not contained in p3.
p3

p2

p1
3  1
   
m is perpendicular to n3:     1   0    4
7 1
   

Also (1, 2, 0) on m does not lie in p3: 3x + y + 7z = 


Thus 3(1) + (4)(2) + 7(0)      5

TJC P2 Solutions 645


5

4a Area of shaded region


1  1  1  e
  e   e   
2  e  e  
1
e
x(ln x)2 dx

 e 
1  2 1    x2 2
2
e x 2ln x
  e  2    (ln x)   1 dx 
2 e    2  1 e 2 x 
 e 
1 1   e 2
1  e 
  e2  2      2   1 x ln x dx 
2 e    2 2e  e 
 e 
1  2 1  1  2 1    x2  2
e x 1
  e  2    e  2    ln x   1 dx 
2 e  2 
e   2 1 e 2 x 
 e 
e
1  2 1  1  x2 
 e  2    
2 e  2 2  1
e

1 1  1  e2 1 
  e2  2     2 
2 e  2  2 2e 
1 3
  e2  2 
4 e 

sgexamguru.com
y
y=x

(e, e)

x
O 1

2
1 3  1 1
Area =  e2  2    e  
4 e  2 e

4b
Given ( x  2)2  4( y  1)2  4
( x  2) 2  4[1  ( y  1) 2 ]  x  2  2 1  ( y  1) 2
The shaded region is bounded by the section of the ellipse where
x ≤ −2.
Hence x  2  2 1  ( y  1) 2 .

TJC P2 Solutions 646


6

Volume of solid formed

2
 π42 (2  1)  π  x 2 dy
1

 
2 2
 16π  π  2  2 1  ( y  1) 2 dy
1

 9.6 units3

5i Stratified sampling ensures that households occupying two-bedroom


apartments and households occupying four-bedroom apartments are
proportionately represented in the sample.

5ii Total number of units = 24 + 88 = 112

24
Number of 2-bedroom households needed =  35  7.5
112
88
Number of 4-bedroom households needed =  35  27.5
112

sgexamguru.com
She should choose 8 two-bedroom apartments and 27 four-bedroom
apartments (or 7 two-bedroom apartments and 28 four-bedroom apartments)

Using the list of apartment numbers as sampling frames for the two types of
apartments, she would use simple random sampling (or systematic sampling)
to select the 2-bedroom apartments and 4-bedroom apartments to be
interviewed.

6i 6!
Number of ways   360 P AA R LLL E
2!
6ii (By Slotting)
 7  6!
Number of ways     2520
 2  3!
Method 2 (Complement)
8!
Number of ways to arrange letters without restriction   3360
2!3!
Number of ways to arrange letters with both A’s together
7!
  840
3!
8! 7!
Total number of ways    2520
2!3! 3!

TJC P2 Solutions 647


7

6iii 5!
Number of ways  4   240 L_L_L _ _ _
2!

Method 2
5 4!
Number of ways     2!   240
 2 2!
5
C  C1  4!
4
Number of ways  1  240
2!
Method 3:
Case 1: LALAL_ _ _
Number of ways  4!  24
Case 2: LXLAL_ _ _ or LALXL_ _ _
 3
Number of ways  2     4!  144
1
Case 3: LXLYL_ _ _ or LYLXL_ _ _
 3  4!
Number of ways  2      72
 2  2!
Total = 240

7i
sgexamguru.com
P(a randomly chosen cupcake is “good”)

= 0.6 × 0.8 × 0.94 + 0.4 × 0.88 × 0.94

= 0.78208 = 0.782 (3sf)


7ii P(cupcake is either baked by Anand, is sub-standard but not both)

Anand Sub-standard

= P(cupcake is baked by Anand and is good) +


P(cupcake is baked by Beng and is sub-standard)

= 0.6 × 0.8 × 0.94 + 0.4 × (0.12 + 0.88 × 0.06)


= 0.520 (3sf)

Method 2
= 0.6 × 0.8 × 0.94 + 0.4 × (1 – 0.88 × 0.94)
= 0.520 (3sf)

TJC P2 Solutions 648


8

Method 3 Bad  Substandard 


0.06  spread 
flawed
0.2  Good  Substandard 
0.94 spread   
baked by 
Anand 
Bad  Substandard 
0.06  spread   
0.8 
not 
0.6 
flawed  Good  Good 
0.94 spread   

Bad  Substandard 
0.06  spread   

0.4  flawed 
0.12  Good  Substandard 
0.94 spread   
baked by 
Beng 
0.06 Bad  Substandard 
spread 

sgexamguru.com
 
0.88  not 
flawed  Good  Good 
0.94 spread   

=P(baked by Anand) + P(sub-standard) – 2 P(baked by Anand and sub-


standard)
= 0.6 + (1 – 0.78208) – 2 ( 0.6 × (1 – 0.8 × 0.94) )
= 0.520 (3sf)
7iii P(cupcake baked by Anand | cupcake is sub-standard)

P(cupcake is sub-standard and baked by Anand)


=
P(cupcake is sub-standard)

0.6  (0.2  0.8  0.06) 0.6  (1  0.8  0.94)


= or
1  0.78208 1  0.78208

= 0.683 (3sf)

TJC P2 Solutions 649


9

1
8
x
94

33
T C
10 80
8 Model B is the most appropriate as the curve is concave upwards and x
increases with T , which is the same as the shape shown in the scatter
diagram.
OR
Model B is the most appropriate since x increases at an increasing rate as T
increases, which is the same as the shape shown in the scatter diagram.

8i Regression line for Model B:

sgexamguru.com
ln x  ln a  bT
ln x  ln(27.06)  0.01497T

10  20  30  40  50  60  70  80  90
T  50 and
9

 ln x
9
ln 33  ln 37  ln 41  ln 48  ln 55  ln 65  ln 78  ln 94  ln k

9
31.77392262  ln k

9

Since (
 ln x , T ) lies on the regression line,
9

31.77392262  ln k
 ln(27.06)  0.01497  50 
9

 k  104 (nearest whole number)


8ii From GC, r  0.997
When T = 65,
8iii ln x  ln(27.06)  0.01497 105 

TJC P2 Solutions 650


10

x  130 (nearest whole number)

The estimated growth rate cannot be taken as reliable as the temperature


1050 C , from which the value of x  130 is computed from, lies outside the
data range of T i.e. [10, 90].

9i Let T be the number of defects in 7 m2 of twill weave.

T ~ Po (1.2  7) ,i.e. T ~ Po (8.4)

P (T  9)  0.129025899  0.129
9ii Let W be the number of defects in a box.

W ~ Po (2 1.2  3  0.8) ,i.e. W ~ Po (4.8)


P (box is faulty)  P (W  10)
 1  P (W  10)
 0.0104 (shown)
9iii Let F be the number of “faulty” boxes in a random batch of 50 boxes.
F ~ B(50, 0.0104)

sgexamguru.com
Since n is large and np = 0.52 < 5,
F ~ Po (0.52) approximately.
P(customer rejects the entire batch)
 P( F  2)
 1  P( F  1)
 0.096329
 0.0963 or 0.0966 (3 sig fig)

 4
Required probability    (0.096329)2 (1  0.096329)2
 2
 0.0455 or 0.0457 (3 sig fig)

Alternative Solution:
Let X be the number of batches out of 4 that are rejected.
X ~ B(4, 0.096329)
P  X  2   0.0455 or 0.0457

TJC P2 Solutions 651


11

100i Let U and S be the numberr of subscribers who purchased


p a uPhone and a
Samseng phhone respecctively in a sample of size
s n
U ~ B(n,
B 0.3) S ~ B(n, p)
Whenn n = 50,
P( S  20)  2P(U  155)
P( S  20)  0.24469
From GC,

P(S  20)

or
P(S  20)

sgexamguru.com
p = 0.459 (3 sig fig)
100ii Givenn p = 0.4, n = 50, S ~ B
B(50, 0.4)
E(S) = 50 (0.4) = 20
P(U > 20) = 1 – P(U
P ≤ 20)
= 0.04
478 (3 sig fi
fig)

100iii Whenn n = 1000


U ~ B(1000,
B 0.3))
Since n is large, np
n = 300 > 5, n (1 – p)) = 700 > 5
U ~ N(300,
N 210) approximattely

S ~ B((1000, 0.4)
Since n is large, np
n = 400 > 5, n (1 – p)) = 600 > 5
S ~ N(400, 240) approximat
a tely

TJC P2 Solutions 652


12

280U – 200S ~ N ( 280(300) – 200(400), 2802(210) + 2002(240) )


280U – 200S ~ N ( 4000, 26064000)

P ( 280U > 200S )


= P( 280U – 200S > 0 )
= P( 280U – 200S > 0.5 ) (by continuity correction)
= 0.783 (3 sig fig)

11i H0:   250


H1:   250

Level of significance: 4%

217
v  250  247.2875
80
1  217  
2
2
s   30738 
   381.6378165
79  80 
 

sgexamguru.com
Since n = 80 is large, by Central Limit Theorem,
V follows a normal distribution approximately.
V 
Test statistic: ~ N(0, 1) approximately
 s 
 
 n

If H0 is true, p-value = 0.107 > 0.04, we do not reject H0.


(Or zcalc = −1.24 < −1.751)

There is insufficient evidence at 4% level significance to conclude that


  250 .

‘at the 4% significance level’ means there is a 4% chance that we wrongly


concluded that   250 .
(Or wrongly rejected  = 250 when it is true)

H0:   0
H1:   0
v 0
If H0 is true, zcal =
 s 
 
 n
In order to reject H0, zcal must lie in the critical region.

TJC P2 Solutions 653


13

247.2875   0 247.2875   0
 2.053749 or  2.053749
381.6378165 381.6378165
80 80
0 > 251.77 or 0 < 242.80

Set of possible values of 0 ={ 0 : 0 < 243 or 0 > 252}


(Also accept 242 other than 243)

T  V1  V2  V3  ....  V50
Given E(V) =250 and Var(V) = 385

Since n = 50 is large, by Central Limit Theorem,

T ~ N(50250, 50 385) = N(12500, 19250) approximately.

P(T > 12600) = 0.236 (3 sig fig)

sgexamguru.com

TJC P2 Solutions 654


TAMPINES JUNIOR COLLEGE
JC2 PRELIMINARY EXAMINATION

MATHEMATICS 9740/ 01
Paper 1 Tuesday, 13 Sep 2016
3 hours
Additional Materials: Answer paper
List of Formulae (MF15)

READ THESE INSTRUCTIONS FIRST

Write your name and civics group on all the work you hand in, including the Cover Page.
Write in dark blue or black pen on both sides of the paper.
You may use an HB pencil for any diagrams or graphs.

sgexamguru.com
Do not use staples, paper clips, glue or correction fluid.

Answer all the questions.


Give non-exact numerical answers correct to 3 significant figures, or 1 decimal place in the case of
angles in degrees, unless a different level of accuracy is specified in the question.
You are expected to use an approved graphing calculator.
Unsupported answers from a graphing calculator are allowed unless a question specifically states
otherwise.
Where unsupported answers from a graphing calculator are not allowed in a question, you are
required to present the mathematical steps using mathematical notations and not calculator
commands.
You are reminded of the need for clear presentation in your answers.
At the end of the examination, fasten all your work securely together.
The number of marks is given in brackets [ ] at the end of each question or part question.

This document consists of 6 printed pages.

Microsoft Word - 2016 TPJC H2 JC2 Prelim Paper 1 (Questions) 655


e x  y  e   3 y  1 .
2
1 A curve C has equation
dy
(i) By considering , show that C has no stationary points. [5]
dx
(ii) Write down an equation relating x and y at which the tangent is parallel to the y-axis. [1]

 cos t 
2 Referred to the origin O, the points A and B have position vectors given by a    sin t  and
 
 0.5 
 
 cos 2t 
b   sin 2t  respectively, where t is a real parameter such that 0  t  π .
 1 
 

(i) Show that ab  p  cos( qt ) , where p and q are constants to be determined. [2]
(ii) Hence find the exact value of t for which AOB is a maximum. [3]

3 (i) Describe a sequence of transformations that will transform the curve with equation
1 4

sgexamguru.com
y  2 on to the curve with equation y  . [2]
 x  1
2
x

(ii) It is given that


 x2 for 0  x  2,

f  x   4
for 2  x  3,
  x  1
2

and that f  x   f  x  3 for all real values of x.

Sketch the graph of y  f  x  for 2  x  6 . [3]

4 Do not use a calculator in answering this question.


One root of the equation z 3  az 2  bz  15  0 , where a and b are real, is z  1  2i .
(i) Write down the other complex root. [1]
(ii) Explain why the cubic equation must have one real root. [1]
(iii) Find the value of the real root and the values of a and b. [5]

TPJC P1 Questions 656


5 Functions f and g are defined by

3x  1
f:x , x  , x  1,
3x  3
g : x  x  2, x  , 2  x  3.

(i) Find f 1 ( x) . [2]


(ii) Show that f 2 ( x)  x . Hence find the exact value of f 2017 (0) . [2]
(iii) Show that the composite function fg exists. Find an expression for fg  x  and state the
domain and range of fg. [5]

x3
6 It is given that f  x   , where 1  x  1 and n is a positive integer.
1  x n
(i) Find the binomial expansion of f  x  , in ascending powers of x, up to and including the
term in x 2 . [4]
(ii) Given that the coefficient of x 2 in the above expansion is 21, find the value of n. [3]
(iii) Given now that n  2, by substituting a suitable value of x into the expansion in part

4r  1
(i), find the exact value of 
r 1 4
r 1
. [3]

sgexamguru.com
7
2x y

3 3 3

The figure shows a rectangular sheet of length 2x metres and breadth y metres to be placed in a
horizontal position along a garden walkway bounded by low vertical fence of which a
horizontal cross-section is two concentric semicircles of radii 3 metres and 3 3 metres. One
side of the sheet of length 2x metres must be tangential to the inner fence, and the two ends of
the opposite side must touch the outer fence, as shown in the figure. The rectangular sheet is
assumed to have negligible thickness.

(i) By finding x 2 in terms of y, show that the area A square metres of the sheet, is given by
A  2 18 y 2  6 y 3  y 4 . [3]
(ii) Use differentiation to find, the maximum value of A, proving that it is a maximum. [7]

TPJC P1 Questions 657


8 The plane p1 passes through the points A(4,1,1) and B(2,1,0) and is parallel to the vector
x2
4i  j  2k . A line l has equation  y 1  z  4.
2

(i) Show that a vector perpendicular to the plane p1 is parallel to i  2k . Find the
equation of p1 in scalar product form. [3]
(ii) Find the coordinates of the point C at which l intersects p1 . [3]
(iii) The point D with coordinates (2, −1, −4) lies on l . Find the position vector of the foot
of the perpendicular from D to p1 . Find the coordinates of the point E which is the
mirror image of D in p1 . [4]

The plane p2 contains the line l and the point A.


(iv) The planes p1 and p2 meet in a line m. Find a vector equation for m. [2]

9 A sequence u1 , u2 , u3 , ... is given by


u1  1 and 3un 1  2un  1 for n  1 .

sgexamguru.com
(i) Use the method of mathematical induction to prove that
n
2
un  3    1 . [5]
 3
n
(ii) Find u
r 1
r . [3]
n
(iii) Give a reason why the series  u
r 1
r  1 converges, and write down the value of the

sum to infinity. [2]



(iv) Explain, with the aid of a sketch, whether the value of  u
r 1
r  1 is an overestimation

 x
2
or underestimation of the value of

0
3   dx .
 3
[2]

TPJC P1 Questions 658


10 The mass, x grams, of a certain substance present in a chemical reaction at time t minutes
satisfies the differential equation
dx
 k (4  2 x  x 2 ) ,
dt
dx 1
where 0  x  1 and k is a constant. It is given that x  1 and   when t  0 .
dt 2
1
(i) Show that k   . [1]
10
(ii) By first expressing 4  2x  x 2 in completed square form, find t in terms of x . [5]

(iii) Hence find the time taken for there to be none of the substance present in the chemical
reaction, giving your answer correct to 3 decimal places. [1]

(iv) Express the solution of the differential equation in the form x  f (t ) and sketch the part
of the curve with this equation which is relevant in this context. [5]

sgexamguru.com

TPJC P1 Questions 659


11
y

π
2 y  x sin 1  x 2 

y  x cos 1  x 2 

O 1 x

The diagram shows the curves with equations y  x sin 1  x 2  and y  x cos 1  x 2  , where
 1 π  1 
0  x  1 . The curves meet at the point P with coordinates  4 ,  4   .
 2 4  2 

(i) Find the derivative of 1  x 4 . [1]

(ii)
sgexamguru.com
Find the exact value of the area of the region bounded by the two curves.
π 1 
The region bounded by the curve y  x sin 1  x 2  , the line y 
[5]

and the y-axis


4  4 2 
(iii)

is rotated about the y-axis through 360 . By considering the parametric equations
x  t and y  t sin 1  t 2  ,

show that the volume of the solid formed is given by


1
 2t 4 
  t 2 sin 1  t 2  dt .
4
2
π  [3]
 1 t
4
0 

(iv) Hence find the volume of the solid formed when the region bounded by the curve
π
y  x sin 1  x 2  and the line y  x is rotated through 360 about the y-axis. Give
4
your answer correct to 5 significant figures. [3]

End of Paper

TPJC P1 Questions 660


Tampines Junior College
2016 JC2 Preliminary Examination
H2 Mathematics Paper 1
Solution

e x  y  e   3 y  1
1(i) 2

Differentiate w.r.t x :
 dy   dy 
e x  y 1    2  3 y  1  3 
 dx   dx 
dy dy
ex y  ex y  18 y  6 
dx dx
dy
dx
 
18 y  6  e x  y  e x  y

dy e x y
 
dx 18 y  6  e x  y
Since e x  y  0 x  , y   ,
dy
 0
dx
 Curve C has no stationary points.

(ii) dy
Since tangent is parallel to the y-axis, is undefined

sgexamguru.com
dx
18 y  6  e x  y  0
6(3 y  1)  e x  y

2(i)  cos t   cos 2t 


1
ab    sin t    sin 2t   cos t cos 2t  sin t sin 2t 
 0.5   1  2
   
1 1
 cos(t  2t )   cos3t 
2 2
1
p  ,q=3
2

(ii) 1
a b cos AOB  cos(3t ) 
  2
For maximum AOB , since 0  AOB   and cos  is a decreasing
function over  0, , we aim to minimize cos AOB , ie. To minimize
1
cos 3t  .
2

Thus, cos 3t  1  3t   (since 0  t  π ) ie. t  .
3

Microsoft Word - 2016 TPJC H2 JC2 Prelim Paper 1 (Solutions) 661


3(i) 1 4
f(x) = and 4 f(x – 1) =
x2 ( x  1) 2
1
Translate y  by 1 unit in the direction of x-axis followed by a
x2
scaling of a factor of 4 parallel to y-axis (order can be reversed)
(ii) y Note that the
end-points of
y  f ( x) two functions
4 do not meet.
Remember to
3 draw the given
(2, 3) curve first, then
2 repeat.
(6, 1)
1
x
2 1 0 1 2 3 4 5 6

4(i) The other roots is 1 – 2i

(ii) Since the polynomial has all real coefficients, complex roots must occur
in conjugate pair. The polynomial is of degree 3 so there must be a pair
of complex conjugate roots and one real root.

(iii)
sgexamguru.com
z 3  az 2  bz  15  0
(1  2i)3  a (1  2i) 2  b(1  2i)  15  0
11  2i  a ( 3  4i)  b(1  2i)  15  0
(4  3a  b)  i( 2  4a  2b)  0
Equating real and imaginary parts,
4  3a  b  0 or  2  4a  2b  0
Solving the equations,
a = 1, b = −1
z 3  z 2  z  15  ( z 2  2 z  5)( z  3)  0  
 = −3

Alternative method:
Let the real root be .
The factors are (z 1 – 2i) (z – 1 + 2i)(z  ) = [(z – 1)2+ 4] (z  )
3 2
= (z2 – 2z + 5)(z - ) = z  az  bz  15  0
Compare constant term: 5 = −15,  = −3
Compare coefficient of z2: a = 3 – 2 = 1
Compare coefficient of z: b = 5 – 6 = 1

TPJC P1 Solutions 662


5(i) 3x  1
Let y 
3x  3
y  3 x  3  3 x  1

3 x  y  1  3 y  1

3y 1
x
3y  3
3x  1
f 1 ( x )  , x 1
3x  3
(ii) f 2 ( x)  ff 1 ( x) = x  f 3(x) = f(x) , so odd power gives f(x)

f 2017 (0)  f (0)


3  0  1 1
 
3  0  3 3

(iii) Since R g   0,1   ,1  Df ,

fg exists.

fg  x  
3 
x  2 1
3 x  2  3

Dfg  Dg   2, 3 sgexamguru.com
 1
R fg   , 
 3

6(i) x3
f  x 
1  x 
n

  x  31  x 
n

 (  n )(  n  1) 
  x  3  1  (  n )(  x )  (  x ) 2  ... 
 2! 
  n  n  1  2 
  x  3  1  nx    x  ... 
  2  
 3n  n  1  2
 x  nx 2  3  3nx    x  ...
 2 
 3n  n  1  2
 3   3n  1 x   n   x  ...
 2 

TPJC P1 Solutions 663


(ii) 3n  n  1
n  21
2
3n 2  5n  42  0
14
n
3
 reject as n    or n  3
n  3

(iii) x3
When n = 2,  3  7 x  11x 2  ...
1  x  2

2

4r  1 7 11 1 1

r 1 4
r 1
 3   2  ...  3  7    11   ...
4 4 4 4
1 x3
By substituting x  into in part (i),
4 1  x 2
1

4r  1 3 52

r 1 4
r 1
 4
2

9
 1
1  
 4

 
2
7(i)
3  y 
2
 x2  3 3 y
2x
9  6 y  y  x  27
sgexamguru.com
2 2

x 2  18  6 y  y 2

Area of the sheet, A  2 xy


3 3 3
 2 y 18  6 y  y 2
 2 18 y 2  6 y 3  y 4

dA 1
(ii)
dy
2  36 y  18 y 2  4 y 3 
2 18 y  6 y  y
2 3 4

2 y 18  9 y  2 y 2 

18 y 2  6 y 3  y 4

[OR A2  4 18 y 2  6 y 3  y 4 
dA
2A  4  36 y  18 y 2  4 y 3 
dy
dA
A  4 y 18  9 y  2 y 2  ]
dy

TPJC P1 Solutions 664


dA
For maximum A, 0
dy
 
4 y 18  9 y  2 y 2  0
2 y 2  9 y  18  0 or y  0 (reject as y  0 )
 2 y  3 y  6   0
3
y or y  6 (reject as y  0 )
2

 
3 3 3
y      
2 2 2
dA
dy >0 0 <0

3
A is maximum when y 
2
3
When y  ,
2
2 3 4
3 3 3 9 3
Maximum A  2 18    6       =7.79 m2
2
  2
   2 2

8(i)
sgexamguru.com
 4   2    4   2   4   1 
            
1   1    1   0    1   0 
1   0   2  1   2   2 

Hence a vector perpendicular to the plane p1 is parallel to i  2k .


 1   4  1 
Equation of the plane p1 : r 0    1  0   2
 2   1   2 
    

(ii) 2   2 
Equation of l : r =  1     1  ,   
 
 4  1 
   
 2  2   1 
At C,  1    0   2
 4     2 
  
2  2  8  2  2
2

TPJC P1 Solutions 665


 2 
  
 OC  1 
 2 
 
Coordinates C ( 2,1, 2)
(iii) Let F be the foot of perpendicular from D to the plane p1 .
2  1 
Equation of FD: r =  1     0  ,   
 
 4   2 
   

2    1 
At F ,  1  
 0   2
  4  2    2 
  
2    8  4  2
8
 
5
 8  2 
2  5  5 
    
 OF   1    1 
 8   4
 4  2(  )    
 5   5

sgexamguru.com
Let E be the image of D under a reflection in the plane p1 .
2 
 5  2   5 
 6
    
 OE  2  1    1    1 
 4   4   12 
     
 5  5 
 6 12 
Coordinates E   , 1, 
 5 5
(iv) The line of intersection of the planes p1 and p2 is AC,
  2   4   6   2
        
AC   1    1    0   3  0 
  2   1   3  1 
       
4 2
Equation of line m : r = 1    0  ,       
 
   
1  1 
   

TPJC P1 Solutions 666


9(i) 2
n

Let Pn be the statement un  3    1 for all n    .


 3
When n  1 ,
LHS = u1  1
1
2
RHS = 3    1  1
3
LHS = RHS  P1 is true.
k
 2
Assume Pk is true for some k   . i.e. uk  3    1
 3
k 1
2
Prove that Pk 1 is true. i.e. uk 1  3   1
 3
2u  1
uk 1  k
3
  2 k 
2  3    1  1
 3
  
3
k
2
 2   1
3
k
 2  2 
sgexamguru.com
 3    1
 3  3 
2
k 1

 3  1
3

 Pk is true  Pk 1 is true.
Since P1 is true, Pk is true  Pk 1 is true,
by mathematical induction, Pn is true for all n    .

(ii) n  r

n
2
 u r       1
3
r 1   3
r 1  
r
n
2
 3    n
r 1  3 

 2   2 n  
  1     
3  3 
 3  n
2
 1 
 3 
 
  2 n 
 6 1      n
  3 
 

TPJC P1 Solutions 667


n
((iii) 2 n
As n  ,    0.    ur  1  6.
A
 3 r 1

The sum to iinfinity is 6 OR S  6 .


T

((iv)

1 2 3

sgexamguru.com
  r

 ur  1   3    3    3    3    ...
2 2 2 2

r 1 r 1  3   3  3  3


T
The value off  u
r 1
r  1 can be obtaained by addding up the areas of thee

reectangles off width 1 as seen in the sketch. Sinnce the total area of all tthe
reectangles is lesser than the total areea under thee curve for x  0 ,
 x
 2
  ur  1
r 1
iss an underesstimation off 0
3   ddx .
3

10(i) dx
 k (4  2 x  x 2 )
dt
dx 1
when t  0 , x  1 and
w  ,
dt 2
1
  k (4  2  1)
2
1
k
10
((ii) 4  2x  x 2 = 4  (x 2  2 x )
= 4  [(x  1)2  1]
= 5  ( x  1)2

TPJC P1 Solutions 668


dx
 k (4  2 x  x 2 )
dt
1
 4  2 x  x 2 dx   k dt
1 1
 5  ( x  1)2 dx    10 dt
1 5  ( x  1) 1
ln  tc
2( 5) 5  ( x  1) 10

When t  0 , x  1
c  ln1  0
10 5  ( x  1)
t ln
2( 5) 5  ( x  1)
1
5 5  ( x  1)
t ln
5 5  ( x  1)

5  ( x  1)
t  5 ln
5  ( x  1)

t  5 ln
sgexamguru.com
5  x 1
5  x 1
(iii) When x  0 ,
5 1
t  5 ln
5 1
Time taken is 2.152 minutes

(iv) 5  x 1
t  5 ln
5  x 1
5  x 1 t
ln 
5  x 1 5
t
5  x 1
e 5
5  x 1
t

( 5  x  1)  e 5( 5  x  1)
t t
 
e 5x x e 5( 5  1)  5  1

TPJC P1 Solutions 669


t

e 5( 5  1)  5  1
x t

1+e 5

t
0
2.152

d
 2 x3

11 (i)
1  x4 
dx 1  x4
(ii) Area of region
1


  4 2 x cos 1  x 2   sin 1  x 2  dx
0

1
 x 2 
sgexamguru.com
42  x 2   4 x 
1


   cos 1  x 2   sin 1  x 2     4 2     dx
 2   2   1  x4
0
0 
1  1  1  1  1  
1
2 x 3
 cos  sin   dx
4
   
2
2 2  2  2   0 1  x4
1
1 π π  4 4 2
      1  x 0
2 24 4
2
 1
2

(iii) Volume of solid formed


π 1 
4 
= π 4  2
x 2 dy
0
1
 2t 2 
 π 2 2
 sin 1  t 2   dt
4
t 
 1 t
0 4

1
 2t 4 
 π  t 2 sin 1  t 2   dt
4
2

 1 t
0 4

10

TPJC P1 Solutions 670


(iv) Required volume
2
1
 2t 4  1  1   π  1 
= π 2   t 2 sin 1  t 2   dt  π  4    4  
4

0
 1 t
4
 3  2   4  2 
 0.909285  0.489042
 0.42024 5 sig. fig.

sgexamguru.com

11

TPJC P1 Solutions 671


TAMPINES JUNIOR COLLEGE
JC2 PRELIMINARY EXAMINATION

MATHEMATICS 9740/ 02
Paper 2 Thursday, 15 Sep 2016
3 hours
Additional Materials: Answer paper
List of Formulae (MF15)

READ THESE INSTRUCTIONS FIRST

Write your name and civics group on all the work you hand in, including the Cover Page.
Write in dark blue or black pen on both sides of the paper.

sgexamguru.com
You may use an HB pencil for any diagrams or graphs.
Do not use staples, paper clips, glue or correction fluid.

Answer all the questions.


Give non-exact numerical answers correct to 3 significant figures, or 1 decimal place in the case of
angles in degrees, unless a different level of accuracy is specified in the question.
You are expected to use an approved graphing calculator.
Unsupported answers from a graphing calculator are allowed unless a question specifically states
otherwise.
Where unsupported answers from a graphing calculator are not allowed in a question, you are
required to present the mathematical steps using mathematical notations and not calculator
commands.
You are reminded of the need for clear presentation in your answers.
At the end of the examination, fasten all your work securely together.
The number of marks is given in brackets [ ] at the end of each question or part question.

This document consists of 6 printed pages.

Microsoft Word - 2016 TPJC H2 JC2 Prelim Paper 2 (Questions) 672


Section A: Pure Mathematics [40 marks]

1 The curve C has parametric equations


x  2cos t , y  3sin t .

(i) Find the equation of the normal to C at the point P with parameter  , leaving your
answer in terms of  . [3]
(ii) This normal to C at the point P meets the x- and y-axes at points A and B respectively.
Find the cartesian equation of the locus of the midpoint of AB as  varies. [4]

2 Using partial fractions, find the exact value of


1 15 x 2  x  17
0 (2 x  1)( x 2  4)
dx . [8]

3 Mrs X wants to sew handmade gifts for the guests attending her daughter’s wedding. On the
first day of gift preparation, she spends 270 minutes of her time. Subsequently, she will
decrease the amount of time spent each day on gift preparation by a certain amount. The total
time taken to complete her gift preparation is 9000 minutes.

(i)
sgexamguru.com
Mrs X spends, on each subsequent day, 2.5% less time on gift preparation than on the
previous day. Find, to the nearest minute, the time Mrs X spends on the 10th day, and
find the minimum number of days Mrs X requires to complete her gift preparation. [6]

(ii) It takes 20 minutes for Mrs X to complete one gift. If Mrs X has the opportunity to
make more gifts using this model, in theory, find the maximum number of gifts she can
complete. [2]

(iii) After doing some calculations, Mrs X realises that she has to decrease the number of
days spent on gift preparation. She still spends 270 minutes on the first day, but on each
subsequent day, the amount of time spent is 4 minutes less than on the previous day.
Find the minimum number of days Mrs X requires to complete her gift preparation.
[3]

TPJC P2 Questions 673


4 (a) The complex number w is given by 3  3( 3)i .
(i) Find the modulus and argument of w, giving your answers in exact form. [2]
(ii) Without using a calculator, find the smallest positive integer value of n for which
n
 w3 
  is a real number. [4]
 w* 
 

(b) The complex number z is such that z 5  4 2 .


(i) Find the values of z in the form rei , where r  0 and  π    π . [4]
(ii) Show the roots on an Argand diagram. [2]

(iii) The roots represented by z1 and z2 are such that 0  arg( z1 )  arg( z2 )  π . The locus of
all points z such that | z  z1 |  | z  z2 | intersects the line segment joining points
representing z1 and z2 at the point P. P represents the complex number p. Find, in
exact form, the modulus and argument of p. [2]

Section B: Statistics [60 marks]

sgexamguru.com
5 It is desired to conduct a survey among university students regarding the use of the university’s
facilities. A random sample of 100 students is obtained.
(i) Explain what is meant in this context by the term ‘a random sample’. [2]
(ii) Now it is necessary to obtain a representative range by faculties. Name a more
appropriate sampling method and explain how it can be carried out. [2]

6 75% of the employees in a factory own a cell phone.


(i) A random sample of 8 employees is taken. Find the probability that the number of
employees who own a cell phone is between 4 and 6 inclusive. [2]

In an industrial park, every factory has 160 employees.


(ii) Use a suitable approximation to find the probability that in a randomly selected factory,
at least 115 employees own a cell phone. [3]

(iii) A random sample of 15 factories in the industrial park is taken. Find the probability
that at most 11 of these factories each have at least 115 employees who own a cell
phone. [2]

TPJC P2 Questions 674


7 The average number of parking tickets that a traffic warden issues per day is being
investigated.
(i) State, in context, two assumptions that need to be made for the number of parking
tickets issued per day to be well modelled by a Poisson distribution. [2]

Assume that the number of parking tickets issued per day has the distribution Po (3.6).
(ii) Find the probability that, in seven days, the traffic warden issues more than 22 parking
tickets altogether. [3]

(iii) The probability that the traffic warden issues more than N parking tickets altogether in
10 days is less than 0.05. Using a suitable approximation, find the least possible value
of N. [2]

8 A fruit stall sells papayas. The mass of papayas is denoted by X kg. The stall owner claims that
the mean mass of the papayas is at least 1.2 kg. The masses of a random sample of 8 papayas
are summarised by
 x  8.84 ,  x 2  9.95 .
(i) Find unbiased estimates of the population mean and variance. [2]
(ii) Stating a necessary assumption, test at the 5% level of significance whether there is any
evidence to doubt the stall owner’s claim. [5]

sgexamguru.com
9 Two players A and B decide to play two consecutive card games. A fair coin is tossed to
decide which player has the first move in the first game. The loser of the first game has the
first move in the second game. A player must win both games to be declared the overall
winner. If each player wins a game, the result is a draw.

2
When A has the first move in a game, the probability that A wins that game is . When B has
3
3
the first move in a game, the probability that B wins that game is . Every game ends with
5
either A or B as the winner.
142
(i) Show that the probability that two consecutive card games end in a draw is . [2]
225

(ii) Given that A wins the second game, find the probability that two consecutive card
games end in a draw. [3]

(iii) To make their games more enjoyable, A and B agree to change the procedure for
deciding who has the first move in the first game. As a result of their new procedure,
the probability of A having the first move in the first game is p. Find the exact value of
p which gives A and B equal chances of winning both games. [3]

TPJC P2 Questions 675


10(a) Sketch a scatter diagram that might be expected when x and y are related approximately by
y  px 2  q in each of the cases (i) and (ii) below. In each case your diagram should include 5
points, approximately equally spaced with respect to x, and with all x- and y-values positive.
(i) p is positive and q is positive,
(ii) p is negative and q is positive. [2]

(b) A car website gives the following information on the ages in months (m) and resale price in
dollars (P) of used passenger cars of a particular model.

m 10 20 28 35 40 45 56 62 70 74
P 110600 79900 78700 69200 66100 60200 53800 50600 46700 43800

It is thought that the price after m months can be modelled by one of the formulae
P  am  b , P  c ln m  d ,
where a, b, c and d are constants.

(i) Find, correct to 4 decimal places, the value of the product moment correlation
coefficient between
(A) m and P,
(B) ln m and P. [2]

sgexamguru.com
(ii) Explain which of P  am  b and P  c ln m  d is the better model and find the
equation of a suitable line for this model. [3]

(iii) Use the equation of your regression line to estimate the price of a used passenger car
that is 80 months old. Comment on the reliability of your estimate. [2]

11 A group of 5 girls and 7 boys play ice breaker games to get to know each other.

(i) The group stands in a line.


(a) Find the number of different possible arrangements. [1]
(b) The girls have names that start with different letters. Find the number of different
possible arrangements in which all the girls are separated, with the girls’ names in
alphabetical order. [2]

(ii) The group forms two circles of 6, with one circle inside the other, such that each person
in the inner circle stands facing a person in the outer circle. Find the number of
different possible arrangements. [2]

(iii) The group has to split into a group of 3, a group of 4, and a group of 5. Find the
number of possible ways in which the groups can be chosen if there is no girl in at least
one of the groups. [4]

TPJC P2 Questions 676


12 A supermarket sells two types of guava, A and B. The masses, in grams, of the guava of each
type have independent normal distributions. The means and standard deviations of these
distributions are shown in the following table.

Mean Standard deviation


(g) (g)
Type A 200 12
Type B 175 12

Find the probability that

(i) the total mass of 4 randomly chosen guava of type A is more than 810 g, [2]
(ii) the mean mass of 4 randomly chosen guava of type A differs from the mean mass of 3
randomly chosen guava of type B by at least 30 g. [3]

Mr Tan buys 20 guavas, m of them are guava of type A and the rest are guava of type B.

(iii) Find the least value of m such that the probability that the total mass of these 20 guavas
exceeding 3500 g is more than 0.95. (Answers obtained by trial and improvement from
a calculator will obtain no marks.) [4]

sgexamguru.com End of Paper

TPJC P2 Questions 677


Tampines Junior College
2016 JC2 Preliminary Examination
H2 Mathematics Paper 2
Solution

1(i) x  2 cos t , y  3sin t

dx dy
 2sin t  3cos t
dt dt
dy 3cos t

dx 2sin t
3
  cot t
2
2
Gradient of normal to C at P  2cos  , 3sin   is tan  .
3
Equation of normal to C at P is
2 
y  3sin    tan    x  2cos 
 3 
2 
y   tan    x  2cos    3sin 
3 
2  5
y   tan   x  sin 
3  3

sgexamguru.com
(ii) When y = 0,
 2 sin  
3sin      x  2cos 
 3 cos 
9
x   cos   2cos 
2
5
  cos 
2
 5 
A   cos , 0 
 2 

 2 sin  
When x = 0, y     2cos    3sin 
 3 cos 
5
 sin 
3
 5 
B  0, sin  
 3 

 5 5 
Midpoint of AB =   cos  , sin   .
 4 6 
5 5
x   cos , y  sin 
4 6
sin   cos   1
2 2

Microsoft Word - 2016 TPJC H2 JC2 Prelim Paper 2 (Solutions) 678


6  4 
2 2 Note that no
 y  x 1 restriction of 
5  5  given in question
16 x 2  36 y 2  25

2 15 x 2  x  17 A Bx  C
Let 2
  2
(2 x  1)( x  4) 2 x  1 x  4
15 x 2  x  17  A ( x 2  4)  ( Bx  C )(2 x  1)
1
Sub x   , A  5
2
Sub x  0 , 17  4 A  C
C  3
Sub x  1 , 31  25  3( B  3)
B5

15 x 2  x  17 5 5x  3
2
  2
(2 x  1)( x  4) 2 x  1 x  4

115 x 2  x  17
0 (2 x  1)( x 2  4) dx
1 5 1 x 1 1
=  dx  5 2 dx  3 2 dx
0 2x  1 0 x 4 0 x 4
5 1 2
= 
2 2x  1
0
sgexamguru.com
5 1 2x
dx   2
2 x 4
0
1
dx  3 2
0
1
x  22
dx
1
5 5 3 x 
=  ln 2 x  1  ln x 2  4  tan 1 ( ) 
2 2 2 2 0
5 5 3 1 5 5
= ln 3  ln 5  tan 1    ln1  ln 4
2 2 2 2 2 2
5  15  3 1
= ln    tan 1  
2 4 2 2

3(i) Let r be the common ratio.


Let n be the number of days required.
r  0.975
Time spent on 10th day   270  0.975 
10 1

 214.98
 215 minutes

TPJC P2 Solutions 679


270 1  0.975n 
 9000
1  0.975
 10800 1  0.975n   9000
5
 1  0.975n 
6
1
 0.975n 
6
1
ln
n 6  70.771
ln 0.975
Hence, n  71

(ii) 270
Theoretical maximum total time   10800
1  0.975
10800
Maximum number of gifts   540
20

 2  270    n  1 4    9000


(iii) n
Total number of minutes 
2
 2n 2  272n  9000  0
From GC, 56.864  n  79.136.
Hence, n  57 (time spent each day on task must be positive)

sgexamguru.com
4(a) 32  (3 3) 2  6
|w| =
(i)
3 3 π
arg( w ) = tan 1  1
  tan ( 3) 
 3  3

(ii)   i π 3 
n

n   6e 3    216e iπ 
n
 w3     i
4πn
   π 
  = 36n e 3
 w*  i
π
 i 3 
   6e 3   6e 
 
 
Alternatively,
n
 w3 
n
4πn  w3 
Since   is real, sin ( ) = 0.   is real,
 w*  3  w* 
   
4πn means lying on real
Hence, = 0, , 2, 3, 4, …
3 axis, so
3 3 9 4πn
n = 0, , , , 3, … = 0, , 2, 3,
4 2 4 3
Smallest positive integer value of n is 3. 4, …

TPJC P2 Solutions 680


(b) (i) z 5  4 2
= 4 2e iπ

 
1/5
z  4 2ei ( π  2 kπ ) , k  2, 1,0,1, 2
 π 2 kπ 
i  
 2e  5 5 

π
i 
When k  0, z1  2e  5 
 3π 
i 
When k  1, z2  2e  5 

When k  2, z3  2ei π
 3π 
i 
When k  2, z4  2e  5 

 π
i 
When k  1, z5  2e  5

(ii)
Im

sgexamguru.com
π
Re
5
O

(iii)
Im

z2
Pp

z1

Re

TPJC P2 Solutions 681


5(i) A random sample is obtained by selecting 100 students from the
population of university students such that every student has an equal
chance of being selected and the selection of students is made
independently.

(ii) A more appropriate sampling method is stratified sampling. Divide


the students into mutually exclusive strata (groups) such as the
faculties the students belong to and then randomly select the students
with the sample size proportional to the relative size of each stratum
to form the sample of size 100. The students to be included in the
sample can then be selected from each stratum using random
sampling or systematic sampling.

6(i) Let X be the number of employees who own a cell phone out of 8
employees.
Then X ~ B(8, 0.75)

P(4 ≤ X ≤ 6) = P(X ≤ 6) – P(X ≤ 3)


= 0.60562
≈ 0.606 (3 s.f.)

(ii) Let Y be the number of employees who own a cell phone out of 160
employees
Then Y ~ B(160, 0.75)

sgexamguru.com
Since n =160 > 50 is large, np = 160(0.75) = 120 > 5,
n(1 – p) = 160(0.25) = 40 > 5, np(1 – p) = 30,
Y ~ N(120, 30) approximately
P(Y  115) = P(Y > 114.5) (c.c.)
= 0.84235
≈ 0.842 (3 s.f.)

(iii) Let W be the number of factories with at least 115 employees who
own a cell phone
Then W ~ B(15, 0.84235)
P(W ≤ 11) = 0.202 (3 s.f.)

7(i) The possible assumptions are:


(a) The average / mean number of parking tickets issued per day
is constant.
(b) The parking tickets are issued independently / randomly.

(ii) Let Y be the number of parking tickets issued in seven days.


Then Y ~ Po (3.6 × 7) i.e. Y ~ Po (25.2)
P(Y > 22) = 1 – P(Y ≤ 22)
= 0.69634 ≈ 0.696 (3 s.f.)

TPJC P2 Solutions 682


(iii) Let W be the number of parking tickets issued in 10 days.
Then W ~ Po (36)
Since  = 36 > 10 is large , W ~ N(36, 36) approximately
P(W > N) < 0.05  P(W > N + 0.5) < 0.05 (using c.c)
Using G.C.,
When N = 45, P(W > N + 0.5) = 0.05667 > 0.05
When N = 46, P(W > N + 0.5) = 0.04006 < 0.05
Hence, the least possible value of N is 46

Alternative solution
P(W < N + 0.5) > 0.95
N  0.5  36
 1.64485
6
N  45.369
Hence, the least possible value of N is 46

8(i)
Unbiased estimate of population mean: x   x  8.84  1.105
n 8
Unbiased estimate of population variance:
1 ( x )2  1  8.84 2 
s2    x2     9.95  = 0.02597
7 8  7 8 
(ii) sgexamguru.com
Let X be the mass of papayas

To test H 0 :   1.2
against H1 :   1.2
at 5% level of significance

Since n = 8 is small, population variance is unknown,


Use t-test.
Assume that mass of papayas follows a normal distribution.

X  1.2
Test Statistics: T  ~ t  7  under H 0
s2
8
1.105  1.2
ttest   1.67
0.02597
8
Using t-test, by GC,
p-value = 0.0697
Since p-value = 0.0697 > 0.05, we do not reject H0 and conclude that
there is insufficient evidence, at the 5% significance level, to reject
the stall owner’s claim.

TPJC P2 Solutions 683


9(i)

A wins
A wins
B wins
A
first move
A wins
B wins
B wins

A wins
A wins
B wins
B
first move
A wins
B wins
B wins

12 3 1 2 2 3 3 2
P  games end in a draw           
2 3 5 3 3 5 5 5 3

sgexamguru.com
142

2 25
(ii) P  A wins the second game and games end in a draw 
11 2 3 2
     
23 3 5 3
14

45
12 2 1 2 2 2 3 2
P  A wins the second game           
23 5 3 3 5 5 5 3
118

225
P  games end in a draw | A wins the second game 
P  A wins the second game and games end in a draw 

P  A wins the second game 
14
 45
118
225
35

59

TPJC P2 Solutions 684


(iii)

A wins
A wins
B wins
A
first move
A wins
B wins
B wins

A wins
A wins
B wins
B
first move
A wins
B wins
B wins
2 2 2 2
P  A wins   p    1  p   
3 5 5 5
8 4
 p

sgexamguru.com
75 25
1 1 3 1
P  B wins   p    1  p   
3 3 5 3
4 1
 p
45 5
P  A wins   P  B wins 
8 4 4 1
 p  p
75 25 45 5
9
 p
44

10(a)
(i) y

x
x
x
x
0 x

TPJC P2 Solutions 685


(ii)
y

x
x

x
x
x
0
(b)(i) (A) r =  0.9492
(B) r =  0.9916

(ii) P  c ln m  d is a better model since r is closer to −1. As m


increases, P  c ln m  d decreases but cannot be less than zero. / P
decreases at a decreasing rate.

Equation of regression line is


P  31800 ln m  182000

(iii) When m  80 ,
P  31825.69267 ln 80  181762.4239
P  42300

sgexamguru.com
Estimate is not reliable. m  80 is outside data range.
Extrapolation is not a good practice.

11(i) Number of different possible orders  12!  479001600


(a)
(b) Number of different possible orders  7!  8C5
 282240
(ii) 6!
Number of different possible arrangements  12C6   6!
6
 79833600

(iii) Case 1: No girls in group of 3


Number of ways  7C3  9C4  5C5  4410
Case 2: No girls in group of 4
Number of ways  7C4  8C3  5C5  1960
Case 3: No girls in group of 5
Number of ways  7C5  7C3  4C4  735
Case 4: No girls in group of 3 and group of 4
Number of ways  7C3  4C4  5C5  35

Total number of ways  4410  1960  735  35  7070

TPJC P2 Solutions 686


Alternatively, we can use the complement method (not
recommended)
(There are 6 cases for every group to have at least one girl.)

Number of ways
 12C3  9C4  5C5  5C2  7C1  3C2  6C2  1C1  4C4
 5C2  7C1  3C1  6C3  2C2  3C3  5C1  7C2  4C2  5C2  2C2  4C4
 5C3  2C1  7C3  1C1  4C4  5C1  7C2  4C1  5C3  3C3  2C2
 5C1  7C2  4C3  5C1  1C1  4C4
 7070

12(i) Let X be the mass of guava of type A


Let Y be the mass of guava of type B
X ~ N(200, 122) and Y ~ N(175, 122)

Let T be the total mass of 4 guava of type A.


T ~ N(4  200, 4  12 2 )
T ~ N(800,576)
P(T  810)  0.33846  0.338
(ii)
X ~ N  200,36  , Y ~ N 175, 48 , X  Y ~ N  25,84

   
sgexamguru.com
P X  Y  30  1  P X  Y  30
 0.29269
 0.293
(iii) Let V be the total mass of m guava of type A
Let W be the total mass of (20  m) guava of type B

V ~ N (200m,144m) , W ~ N (175(20  m),144(20  m))


V  W ~ N(25m  3500, 2880)

P V  W  3500   0.95  
 25m 
 PZ    0.95
 2880 
 25m 
 PZ    0.95
 2880 
 25m 
 PZ    P(Z  1.64485)
 2880 
25m
 1.64485
2880
m  3.53
Least value of m = 4

10

TPJC P2 Solutions 687


2016 VJC JC2 Prelim Paper 1 Solutions/Comments

Qn Solution
1 y
y = f ( x + 1)

x
O
1  3k , 0 (3k -1,0)
x  1  6k x = 6k -1

2 z - 6 - 5i 2i - 4 - z ³ z + 4 -10i
£4
2 z + 4 - 2i ³ z + 4 -10i
z - (6 + 5i ) £ 8 z - (-4 + 2i) ³ z - (-4 + 10i)
Im

(-4,10)
-
B (6,6) C

(6,5)

 4, 2  8

sgexamguru.com
Re
O
      
A  6, 4 
By Pythagoras theorem, BC 2 + 12 = 82  BC = 63
63
θ = tan-1
10
π 63
Min arg ( z - 6 + 4i) = - tan-1 = 0.900
2 10
π 63
Max arg ( z - 6 + 4i) = + tan-1 = 2.24
2 10
Alternative : Equation of circle is ( x - 6) + ( y - 5) = 64 --(1)
2 2

Equation of perpendicular bisector is y = 6 ---( 2)


Substituting (2) into (1)
( x - 6) + (6 - 5) = 64  x = 6  63
2 2

(
Min arg ( z - 6 + 4i) = arg 6 + 63 + 6i - 6 + 4i )
                                    = arg ( 63 + 10i) = tan
10 -1
= 0.900  
63
Max arg ( z - 6 + 4i) = arg (6 - 63 + 6i - 6 + 4i)

                                   = arg (- 63 +10i) = π - tan-1


10
= 2.24
63

Microsoft Word - 2016 VJC H2 JC2 Prelim Paper 1 (Solutions) 688


Qn Solution
3a 4 - 7x
³x
x -3
4 - 7 x - x ( x - 3)
³0
x-3
x + 4x - 4
2
£0
x-3
( x + 2) - 8
2

£0
x -3 ‐ + ‐ +
( )(
x + 2+ 2 2 x + 2-2 2 )£0 -2 - 2 2
    - 2 + 2 2     3
x -3
\ x £-2 - 2 2 or - 2 + 2 2 £ x < 3
b Let e, f and g be the ages of Edwin, his father and his grandfather respectively.

e + f + g = 53´ 3 = 159 ---- (1)


1 1 1
e + f + g = 65 ---- (2)
4 3 2
g - 22 = 2( f - 22 + e - 22)
2e + 2 f - g = 66 ---- (3)
From GC, e = 24, f = 51, g = 84.
The ages of Edwin, his father and his grandfather are 24, 51 and 84 respectively.
4ia When k = 1 , f : x  ( x - 2) 2 + 1, x Î  , x £ 2
Let y = ( x - 2) 2 + 1
( x - 2) = y - 1
2
sgexamguru.com
x - 2 =  y -1
x = 2  y -1
Since x £ 2 , x = 2 - y -1 ,
f -1 : x  2 - x -1, x ³ 1
b y
 0,5 y=x
y = f ( x)
y = f f -1 ( x )

1, 2 

1,1 (2,1)
y  f 1  x
x
O (5,0)  
ii {k Î  : k > 2}

VJC P1 Solutions 689


Qn Solution
5i 2 -12 1 1
u2 = u1 + = 0+ =
(1 + 2)! 6 6
2 - 22 1 2 1
u3 = u2 + = - =
(2 + 2)! 6 24 12
2 - 32 1 7 1
u4 = u3 + = - =
(3 + 2)! 12 120 40
ii 1 2 -1 1 2 3 -1 1 3 4 -1
u2 = = , u3 = = = , u4 = = =
6 (2 + 1)! 12 24 (3 + 1)! 40 120 (4 + 1)!
n -1
By observation, a conjecture is that un =
(n + 1)!
iii n -1
Let Pn be the statement un = , for all n Î  + .
(n + 1)!
Check P1:
LHS = u1 = 0
1 -1
RHS = =0
(1 + 1)!
\ P1 is true

Assume that Pk is true for some positive integer k


k -1
i.e. uk =
(k + 1)!
sgexamguru.com
We want to show that Pk +1 is true. i.e. uk +1 =
k
(k + 2)!
LHS = uk +1
2-k2
= uk +
(k + 2)!
k -1 2-k2
= +
(k + 1)! (k + 2)!
(k -1)(k + 2) + 2 - k 2
=
(k + 2)!
k2 + k -2 + 2-k2
=
(k + 2)!
k
= = RHS
(k + 2)!

Since P1 is true, and Pk is true  Pk +1 is true,


by mathematical induction, Pn is true for all n Î  + .
6 If the distance AB is the least, the line segment AB is perpendicular to l.

B(b, 2 b ) and A(a, 2a + 1)


2 b - 2a - 1
Gradient of BA =
b-a

VJC P1 Solutions 690


Qn Solution
2 b - 2a -1 1
Since gradient of l is 2, =-
b-a 2
 4 b - 4a - 2 = a - b
1
 a = b +4 b -2
5
( )
( AB ) = (2 b - 2a -1) + (b - a )
2 2 2

( ) ( )
2 2
= 2 b - 2 a - 1 + 4 b - 4a - 2

= 5(2 b - 2a -1)
2

æ ö2
2
(
= 5çç2 b - b + 4 b - 2 -1÷÷÷
çè 5 ø
)
æ2 2 1ö
2
= 5çç b - b - ÷÷÷
çè 5 5 5ø
1
( )
2
= 2 b - 2b -1
5
1
( )
2
= 2b - 2 b + 1
5

dAB 2 æ 1 ö÷
2 AB (
= 2b - 2 b + 1 çç2 -
çè
÷
b ÷ø
)
sgexamguru.com
db 5
dAB æ 1 ö
When
db
2
5
( )
= 0, 2b - 2 b + 1 çç2 - ÷÷÷ = 0
çè bø

(
Consider 2b - 2 b +1 = 0 )
(
Since (-2) - 4(2)(1) < 0 , 2b - 2 b +1 = 0 has no real solution.
2
)
1 1
2- =0b=
b 4
æ1 ö
(
the point on C nearest to l is b, 2 b = çç ,1÷÷÷ .
çè 4 ø )
Alternative :
1
 
2
( AB) 2b  2 b  1
2
=
5
2
4 1
= b  b  
5 2
2
4  1 1 1
2

=  b     
5   2  4 2 
2
4  1 1
2

=  b    
5   2  4 

VJC P1 Solutions 691


Qn Solution
2
 1 1 1
Since  b    0 for all real b, ( AB) is the least when b  , that is, b  .
2

 2 2 4
1 
Hence the point on C nearest to l is b, 2 b   ,1
4 
 
Alternative :
When ( AB) is the least, tangent to C at B is parallel to l.
2

i.e. gradient of tangent to C = 2


y2 = 4x
dy
2y = 4
dx
dy 2
=
dx y
dy
(
At b,2 b , ) =
dx 2 b
2
=
1
b
=2

1
b=
4
æ1 ö
\ coordinates on C nearest to l is çç ,1÷÷÷ .
çè 4 ø
7a d x3 3 3
xe = e x + x3x 2 e x
dx

sgexamguru.com
3
= e x (1 + 3x 3 )

ò x (1 + 3x )e
x3 3 3
2 3
dx = xe x x 2 - ò xe x 2 xdx
3 2 3
= xe x x 2 -
3 ò 3 x 2 e x dx
3 2 3
= x3e x - e x + C
3
b dy sec x
2 u = tan x
=
dx 2sec x + 4 tan x + 7
2 du
= sec2 x
ó sec 2 x dx
y=ô d x u 2 +1
õ 2sec 2 x + 4 tan x + 7
u
x
ó 1 1
=ô d u
õ 2(u 2 + 1) + 4u + 7
sec x = u 2 + 1
ó
=ô 2
1
du  
õ 2u + 4u + 9 sec 2 x = u 2 + 1

1ó 1 Or
= ôô du
2 õ u 2 + 2u + 9 sec 2 x = tan 2 x + 1
2 = u2 +1
1ó 1
= ôô du
2 õ u +1 2 + 7
( )
2

VJC P1 Solutions 692


Qn Solution
1æ 2ö æ 2 (u + 1)ö÷
ç
= ççç ÷÷÷ tan-1 çç ÷÷÷ + C
÷
2 çè 7 ø çè 7 ÷ø
æ 2 ( tan x + 1)ö÷
1 ç ÷÷ + C
= tan-1 çç
çè ÷÷
14 7 ø
8i n é r ( r + 2) ù
å êêê r + 1 2 úúú
ln
r =2
ë( ) û
n
= å (ln r - 2ln (r + 1) + ln (r + 2))
r =2
= ln 2 - 2ln 3 + ln 4
+ ln 3 - 2ln 4 + ln 5
+ ln 4 - 2ln 5 + ln 6

+ ln (n - 2) - 2ln (n -1) + ln n
+ ln (n -1) - 2ln n + ln (n + 1)
+ ln n - 2ln (n + 1) + ln (n + 2)
= ln 2 - ln 3 - ln (n + 1) + ln (n + 2)
2 n+2
= ln + ln
3 n +1

sgexamguru.com
ii n+2 2 n+2 2
As n  ¥ , ln  ln1 = 0 , ln + ln  ln .
n +1 3 n +1 3
Since the series tends to a constant, it converges.
2
The sum to infinity is ln .
3

iii 13 é (2r )(2r + 4)ù 13 æç é r (r + 2)ù ö÷


å ln ê r + 1 2 ú = å çççln 4 + ln êê r + 1 2 ú÷ú÷÷÷÷
ê ú ç
r =2 êë ( ) úû r =2 è êë ( ) úû ø
2 15
= 12ln 4 + ln + ln
3 14
83886080
= ln
7
9i y
 r , r   0, r  (r , r )

x
O

VJC P1 Solutions 693


Qn Solution
ii h
V = π ò x 2 dy
0 Alternative:
h
(r )

h
= πò - ( y - r ) dy

2 2
0  r 2  ( y 2  2ry  r 2 ) dy
0
é 3 ùh
( y - r) ú h
= π êê r 2 y -
êë 3 ú
ú
û0

0 
 
2ry  y 2 dy 
æ (h - r ) r ÷÷
3 3ö h
ç  1 
= π ççr 2 h - - ÷÷    ry 2  y 3 
çç 3 3 ÷ø  3 0
è
æ 2 ö 
ççr h - h + h 2 r ÷÷
3

ç ÷÷
= π ççç ÷
3

÷
çç r 3
r ÷
çç-hr 2 + - ÷÷÷
è 3 3ø
æ h3 ö
= π çççh 2 r - ÷÷÷
çè 3 ø÷
πh 2
= (3r - h)
3

iii 2 3
πr
dV πr 3
=- 3 =-
dt

sgexamguru.com
24 36
dV
= π (2hr - h 2 )
dh
dh æ πr 3 ö÷
1 çç-
= ÷÷
dt π 2hr - h 2 ççè 36 ÷ø
( )
r3
=-
36(2hr - h 2 )
r3
Rate of decrease is cm3 s-1.
36(2hr - h 2 )
iv The rate of decrease of the depth is the least when the bowl is full, i.e. h = r.
dh r3 r
 
dt 36r  2r  r  36
The slowest rate at which the depth of water is decreasing is
r
cm s- 1.
36
10i Let θ be the angle between p1 and p2.
æ 1 ö÷ æ-2÷ö
çç ÷ çç ÷
çç-5÷÷ ⋅ çç 1 ÷÷
çç ÷÷÷ çç ÷÷÷
çè 2 ÷ø çè 5 ÷ø 3
cos θ = =  θ = 84.3
30 30 30
ii x - 5 y + 2 z = 13,
-2 x + y + 5 z = 1,
From GC, x = -2 + 3λ, y = -3 + λ, z = λ

VJC P1 Solutions 694


on
æ-2ö÷ æ3ö÷
çç ÷ çç ÷
quation of l iis r = ç-3÷÷ + λ çç1÷÷÷ , λ Î 
ç ÷
 çç ÷÷ çç ÷÷
èç 0 ø÷ èç1ø÷
point of inteersection of p1 , p2 and p3 is the pointt of intersectiion of l and p 3 . (a,0, b) iss a point on

æ ö æ ö
çç-2÷÷ çç3÷÷ a = -2 + 3 λ
= çç-3÷÷ + λ çç1÷÷÷  λ = 3
÷
çç ÷÷ çç ÷÷
çè 0 ÷ø çè1ø÷ b = λ
= -2 + 3´ 3 = 7 and b = 3

ernatively, suubst x  a, y  0, z  b intto equation of


o planes
a + 2b = 13 --(1)
-2a + 5b = 1 --- (2)
´ 2 2a + 4b = 26 --(3)
+ (3) 9b = 27  b = 3
a=7
perpendiculaar to p3 and intersect
i p3 at A(7,0,3) . Let P be a point on l suuch that AP = 4 11 ,
n P lies in  .
æ3ö÷ æ3ö÷ l
1 ççç ÷÷ çç ÷
÷
çç1÷÷÷ = 4ççç1÷÷÷
sgexamguru.com
= 4 11 ⋅
11 ç ÷÷ çç ÷÷
çè1ø è1ø  P
 
= OP + AP
æ ö æ ö æ ö æ ö
çç7÷÷ çç3÷÷ çç-5÷÷ çç19÷÷
= çç0÷÷  4çç1÷÷÷ = çç-4÷÷÷ or çç 4 ÷÷÷
÷ p3
A(7,
( 0, 3)
çç ÷÷ çç ÷÷ çç ÷÷ çç ÷÷
çè3÷ø çè1ø÷ çè-1÷ø èç 7 ÷ø

æ3ö÷ æ-5ö÷ æ3ö÷ æ3ö÷ æ19ö÷ æ3ö÷


÷÷ çç ÷÷ çç ÷÷ çç ÷ çç ÷ çç ÷
1÷÷ = çç-4÷÷ ⋅ çç1÷÷ = -20 or r ⋅ çç1÷÷÷ = çç 4 ÷÷÷ ⋅ çç1÷÷÷ = 68
÷÷ çç ÷÷ çç ÷÷  çç ÷÷ çç ÷÷ çç ÷÷
è1÷ø çè-1÷ø çè1÷ø çè1÷ø èç 7 ÷ø çè1÷ø
possible carrtesian equatiions of  aree
+ y + z = -220 and 3 x + y + z = 68.

ernatively,
cartesian eqquation of  is of the forrm 3x + y + z = p .
0, y = 0 and z =p satisfy 3x + y + z = p , B(0,0,p) is
oint in P.

tance betweeen  and p3 is 4 11 .

VJC P1 Solutions 695


Qn Solution

BA ⋅ nˆ = 4 11

æ 7 ö÷ æ3ö
çç ÷÷ 1 çç ÷÷÷
çç 0 ÷ ⋅ çç1÷ = 4 11
çç ÷÷ çç ÷÷÷
çè3 - p÷ø ÷ 11
èç1ø÷
1
24 - p = 4 11
11
p - 24 = 44
p - 24 = -44 or 44
p = -20 or 68
two possible cartesian equations of  are
3 x + y + z = -20 and 3 x + y + z = 68.
11a a + 4d a + 11d
=
a + 2d a + 4d
a 2 + 8ad + 16d 2 = a 2 + 13ad + 22d 2
5ad = -6d 2
5
Since the terms are distinct, d ¹ 0 , d = - a
6
n
Required sum = ((a + d ) + (a + (2n -1) d ))
2
n

bi n Beginning
1 40000
2
sgexamguru.com
= (2a + 2nd ) = n (a + nd )

End
40000(1.005)
2 40000(1.005) 40000(1.005)2
3 40000(1.005)2 – x 40000(1.005)3 – 1.005x
4 40000(1.005) – 1.005x - 40000(1.005)3 – 1.0052x -
3

x 1.005x
4
5 40000(1.005) – 1.005 x - 2

1.005x – x

Amount at the beginning of 5th month


= 40000(1.005)4 – 1.0052x - 1.005x – x
x (1.0053 -1)
= 40000(1.005) -
4

1.005 -1
= 40000(1.005) - 200 x (1.0053 -1)
4

ii He wishes to repay his in 5 years, n = 60


40000(1.005) - 200 x (1.00558 -1) £ 0
59

40000(1.005)
59

x³  
200(1.00558 -1)
x ³ 800.17
His minimum repayment is $800.17
iii Amount interest bank earned = $(800.17(58) – 40000)
= $6410.06 = $6410 (nearest dollar)

VJC P1 Solutions 696


Qn Solutioon
12 Sincce f(x) is a quuadratic exprression and f (3) = f (-3) = 0 , f ( x) = k ( x 2 - 9) .

k ( x 2 - 9)1 b
x +1+
=
x+a 2 x+a
æ1 ö÷
ç x + 1÷( x + a) + b
kx 2 - 9k ççè 2 ÷ø
=
x+a x+a
1 2 æç 1 ö
x + ç1 + a÷÷÷ x + a + b
2 ç
è 2 ø
=
x +1
mparing coeffficients,
Com
k =  f ( x) = ( x 2 - 9) \ 1 + a = 0  a = -2 (shown)
1 1 1
2 2 2
9 5
ab  b 
2 2
y

x2  9
y
2  x  2
(0, 2.25)

(-3,0)

sgexamguru.com
x
O
1  3, 0 
y= x +1
2
x2
x2 - 9 1 5
y= = x + 1-
2 ( x - 2) 2 2( x - 2)
dy 1 5
\ = +
dx 2 2( x - 2) 2
dy 1 5 5 1
Wheen = 1, + =1 =
dx 2 2( x - 2) 2
2(xx - 2) 2
2
( x - 2) 2 = 5
x = 2 5
Wheen x = 2 + 5 ,
1 5
y = (2 + 5) + 1- =2
2 2 5
Wheen x = 2 - 5 ,
1 5
y = (2 - 5) + 1 - =2
2 2(- 5 )
The equations of tangent aree
y-2= x- 2 5 ( )
y = x - 5 or y = x + 5

VJC P1 Solutions 697



 

VICTORIA JUNIOR COLLEGE


Preliminary Examination
Higher 2

MATHEMATICS 9740/01
Paper 1
Friday 8am – 11am

16 September 2016 3 hours

READ THESE INSTRUCTIONS FIRST

Write your name and CT group on all the work you hand in.
Write in dark blue or black pen on both sides of the paper.
You may use a soft pencil for any diagrams or graphs.
Do not use staples, paper clips, highlighters, glue or correction fluid.

Answer all the questions.


Give non-exact numerical answers correct to 3 significant figures, or 1 decimal place in the case of
angles in degrees, unless a different level of accuracy is specified in the question.
You are expected to use a graphic calculator.
Unsupported answers from a graphic calculator are allowed unless a question specifically states
otherwise.

sgexamguru.com
Where unsupported answers from a graphic calculator are not allowed in a question, you are
required to present the mathematical steps using mathematical notations and not calculator
commands.
You are reminded of the need for clear presentation in your answers.

At the end of the examination, fasten all your work securely together.
The number of marks is given in brackets [ ] at the end of each question or part question.

This document consists of 6 printed pages

© VJC 2016 VICTORIA JUNIOR COLLEGE [Turn over

Microsoft Word - 2016 VJC H2 JC2 Prelim Paper 1 (Questions) 698



 

1
y y  f  x

O x
 3k , 0 
 0, k 
x  6k

1
The ddiagram shoows the currve with equuation y  f  x  , x < 6k, k  . Thhe curve crrosses the x-
3
a the points (3k , 0) andd (0, -k ) reespectively. Sketch y  f  x  1 .
axis aand y-axis at [33]

2 Indiccate on a single Argan


nd diagram
m, the set off points wh
hose compleex numberss satisfy thee
folloowing inequualities

z - 6 - 5i
£4 andd 2i - 4 - z ³ z + 4 -10i .
2

Hennce, find thee least valuee and greateest value of arg ( z - 6 + 4i) . [77]

3 (a)
sgexamguru.com
Without ussing a calcu
ulator, solvee the inequallity

4 7x
x. [44]
x3

(b) In 2016, Edwin,


E his father
f and hhis grandfatther have an
a average aage of 53. In the samee
year, the suum of one-half of his ggrandfatherr’s age, one-third of hiis father’s age
a and one-
fourth of Edwin’s
E agee is 65. Tw enty-two yeears ago, hiis grandfathher’s age waas twice thee
sum of hiss father’s ag
ge and his age. What are their reespective agges in 2016 6? [You cann
assume thaat Edwin’s age
a in 20166 is more thaan 22.] [33]

4 y f : x  ( x  2) 2  k , x  2 . It is given
The ffunction f iss defined by g that f 1 exists.

(i) When k  1 ,
(a) definne f 1 in a similar
s form
m, [33]

(b) sketcch, on a sing m, the graphss of y  f  x  , y  f 1  x  and y  ff 1  x  .


gle diagram
[33]

(ii) uation f  x   f 1  x  haas no real so


State the seet of values of k, such tthat the equ olutions. [11]

VJC P1Questions 699



 

5 A seqquence u1 , u2 , u3 ,  is such that u1  0 and

2  n2
un1  un  a n  1.
, for all
 n  2 !
1
(i) Show that u2  nd find the vvalues of u3 and u4 .
, an [22]
6

n 1
(ii) Hence, givve a conjectture for un iin the form
m , where
w f (n) iis a function of n to bee
 f ( n ) !
determinedd. [11]

(iii) Use the method


m of mathematica
m al induction
n to prove your
y conjeccture in partt (ii) for alll
positive inttegers n. [44]

6   y
y2  4x

sgexamguru.com
O

2x  y  1  0

A currve C has equation


e y 2  4x and a line l hass equation 2 x  y  1  0 . The diagram abovee
show
ws the graphss of C and l.

B(b, 2 b ) is a fixed pointt on C andd A is an


n arbitrary point
p on l. State the geometricaal
relatioonship betw
ween the lin
ne segment A
AB and l if the
t distancee from B to A is the least. [11]

Takinng the coorddinates of A as (a, 2a  1) , find an equation reelating a and


nd b for which AB is thee
least. [22]

 
Deduuce that wheen AB is thee least,  AB   m 2b  2 b  1 where m is a constant to be foundd.
2 2

Hence or otherw
wise, find thee coordinatees of the point on C thaat is nearestt to l, as b vaaries. [55]

[Turn Over 
VJC P1Questions 700

 

 x 1  3x  e
3
x3
7 (a) Differentiate xe x with respect to x. Hence, find 2 3
dx . [4]

(b) The variables x and y are related by the differential equation

dy sec 2 x
 . 
dx 2sec2 x  4 tan x  7
 

    Using the substitution u  tan x , find the general solution of the differential equation. [5]

8 (i) Use the method of differences to find, in terms of n,

n  r  r  2 
 ln  2 
. [4]
r 2   r  1 

(ii) Give a reason why the series is convergent and state the sum to infinity. [2]

13   2r  2r  4    p p
(iii) Given  ln    ln   , where p and q are integers and is in the simplest
  r  1
2
r 2  q q
form, find the values of p and q. [3]

9 (i) sgexamguru.com
Sketch the graph with equation x 2   y  r   r 2 , where r > 0 and y  r .
2
[2]

A hemispherical bowl of fixed radius r cm is filled with water. Water drains out from a hole at
the bottom of the bowl at a constant rate.

Use your graph in part (i) to show that when the depth of water is h cm (where h £ r ), the
volume of water in the bowl is given by

 h2
V  3r  h  , [3]
3

(ii) Given that a full bowl of water would become empty in 24 s, find the rate of decrease, in
terms of r and h, of the depth of water in the bowl at the instant when the depth of water
is h cm. [3]

(iii) Without any differentiation, determine, in terms of r, the slowest rate at which the depth
of water is decreasing. [1]

VJC P1Questions 701



 

10 The equations of planes p1 and p2 are

x  5 y  2 z  13,
2 x  y  5 z  1,

respectively.

(i) Find the acute angle between p1 and p2 . [2]

The planes p1 and p2 intersect in a line l.

(ii) Find a vector equation of l. [2]

The plane p3 is perpendicular to both p1 and p2 . The three planes p1 , p2 and p3 intersect at the
point  a, 0, b  , where a and b are constants.

(iii) Show that a  7 and b  3 . [2]

The plane  is parallel to p3 and the distance between  and p3 is 4 11 units.

(iv) Find the two possible cartesian equations of  . [4]

11 (a) sgexamguru.com
An arithmetic progression which consists of 2n terms has first term a and common
difference d. The third, fifth and twelfth terms of the arithmetic progression are also
three distinct consecutive terms of a geometric progression. Find the sum of the even-
numbered terms, i.e. the 2nd, 4th, … , (2n)th terms, of the arithmetic progression in terms
of a and n. [5]

(b) To renovate his new HDB flat, Douglas is considering taking up a bank loan of $40, 000
from Citybank on 1st July 2016. The bank charges a monthly interest of 0.5% on the
outstanding amount owed at the end of each month.

Douglas will pay a fixed amount, $x, to the bank at the beginning of each month, starting
from September 2016.

(i) Taking July 2016 as the 1st month, show that the amount of money owed at the
beginning of the 5th month is
 
1.0054  40000   200 x 1.0053  1 . [3]

(ii) If Douglas wishes to pay up his loan within 5 years, find the minimum amount of
each monthly repayment. [2]

(iii) Using the value found in part (ii), calculate the interest (to the nearest dollar) that
Citybank has earned in total from Douglas’s loan at the end of his last repayment.
  [2]

[Turn Over 
VJC P1Questions 702

 

f  x
12 The curve C has equation y  , where f  x  is a quadratic expression, a is a constant and
xa
a  3 . It is given that the coordinates of the points of intersection of C with the x-axis are
1
 3, 0  and  3, 0  , and the equation of the oblique asymptote is y  x  1 .
2

(i) Find f  x  , and show that a  2 . [5]

(ii) Sketch C, indicating clearly the equations of the asymptotes, and the coordinates of the
points of intersection of C with the x- and y-axes. [2]

  A tangent to C is parallel to the line y  x  2 . Find the possible equations of this tangent,
leaving your answer in an exact form. [5]

[End of Paper]

   

sgexamguru.com

VJC P1Questions 703



 

VICTORIA JUNIOR COLLEGE


Preliminary Examination
Higher 2

MATHEMATICS 9740/02
Paper 1
Wednesday 8am – 11am

21 September 2016 3 hours

READ THESE INSTRUCTIONS FIRST

Write your name and CT group on all the work you hand in.
Write in dark blue or black pen on both sides of the paper.
You may use a soft pencil for any diagrams or graphs.
Do not use staples, paper clips, highlighters, glue or correction fluid.

Answer all the questions.


Give non-exact numerical answers correct to 3 significant figures, or 1 decimal place in the case of
angles in degrees, unless a different level of accuracy is specified in the question.
You are expected to use a graphic calculator.
Unsupported answers from a graphic calculator are allowed unless a question specifically states
otherwise.

sgexamguru.com
Where unsupported answers from a graphic calculator are not allowed in a question, you are
required to present the mathematical steps using mathematical notations and not calculator
commands.
You are reminded of the need for clear presentation in your answers.

At the end of the examination, fasten all your work securely together.
The number of marks is given in brackets [ ] at the end of each question or part question.

This document consists of 5 printed pages

© VJC 2016 VICTORIA JUNIOR COLLEGE [Turn over

[Turn Over 
Microsoft Word - 2016 VJC H2 JC2 Prelim Paper 2 (Questions) 704

 

Section A: Pure Mathematics [40 marks]

1 When an object moves through a fluid, it experiences a force that slows it down. This force is
called the drag force. At low speeds, it is known that the drag force causes the rate of change in
the speed of the object to be proportional to its speed. You may assume that the experiment
described below is carried out at low speeds and the only factor that affects the speed is the
drag force.
An experiment is conducted to find out how the speed of an object changes as it moves through
a certain fluid. When the speed of the object slows down to a speed of D m s-1, a sensor is
triggered and the subsequent speeds of the object are recorded.
(i) Show that the speed of the object, v m s-1, at t s after the sensor is triggered, is given by

v  De pt , where p is a positive constant. [4]

(ii) On a single diagram, sketch the curves, C1 and C2, of v against t corresponding to p = e
1
and p  .
e
State a single transformation the maps C1 onto C2. [3]

dy
 e cos 2 x  , show that 2 y
Given that y 
sgexamguru.com  y 2  e x sin 2 x .
x
2 [2]
dx
(i) Find the series expansion of y in ascending powers of x up to and including the term in
x2. [3]

1
(ii) Hence, or otherwise, find the series expansion of in ascending powers of x
  e cos2 x 
x

2
up to and including the term in x . [3]

3 (a) The points A, B, C and D represent the complex numbers 2  5i , z1 , 4  i and z2


respectively. Given that ABCD is a square, labelled in an anti-clockwise direction, show
that z1  1 . Find z2 . [4]

(b) Show that the equations z 5  z* and z  1 can be reduced to z n  1 , where n is a


positive integer to be determined. Find all possible values of z in the form rei , where
r > 0 and 0    2 . [4]
π (1  i)
Given further that 0 < arg(z) < , find the smallest positive real number k for to
2 zk
be purely imaginary. [4]

VJC P2 Questions 705



 

4 (a) Referred to the origin O, the points A and B have position vectors a and b respectively.
The points C on AB and D on OB are such that 2 AC  CB and 2OD  3DB . Show that a
vector equation of the line m passing through C and D can be written as

3
r  b    5a  2b  ,   . [4]
5

It is given that a  2, b  5 and the angle between a and b is 60 . The point F on m is
such that F is nearest to O. Show that the position vector of F can be written as
k  5a  2b  , where k is a constant to be found. [4]

(b) Plane  has equation 3x  2 y  5z  45.

Obtain a vector equation of  in the form

r  t  u   v,  ,  are real parameters,

given that t and u are of the form pi + pj and 2i + qj respectively, where p and q are
constants to be determined, and u is perpendicular to v . [5]

Section B: Statistics [60 marks]

5
sgexamguru.com
The head of the Physical Education department of a school wants to gather students’ views
about the school’s efforts in promoting student participation in physical activities. On a
particular afternoon, he surveys the first 30 students who turn up at the school gymnasium.

(i) Explain why the above method may not be suitable for the purpose of his survey. [1]
(ii) Describe another sampling method that would yield a sample that is more appropriate in
this context. [2]

6 Numbers in this question are formed using only the digits 1, 2, 6, 7 and 9.
(i) How many 4-digit numbers can be formed if repetition of digits is allowed? [1]
(ii) How many even numbers between 10,000 and 30,000 can be formed, if each digit can
only be used once? [2]

(iii) A “trick” number is a 6-digit number formed using exactly 3 different digits, and that
each digit is smaller than or equal to the following digit. How many “trick” numbers can
be formed? [e.g. 127777 and 667799 are “trick” numbers, 111122 and 192992 are not
“trick” numbers.] [5]

[Turn Over 
VJC P2 Questions 706

 

7 Box A contains 10 red, 8 blue and 7 green balls. Box B contains 2 white and 3 black balls. All
the balls are indistinguishable except for their colours. Three balls are taken from Box A and
two balls are taken from Box B, at random and without replacement.

Mr Wong guesses that there are at least 1 red ball and exactly 2 black balls taken, while Mr
Tan guesses that all the balls taken are of different colours.

(i) Show that the probability that Mr Wong is correct is 0.241, correct to 3 significant
figures. [3]

(ii) Find the probability that Mr Tan is correct. [2]

(iii) Find the probability that Mr Wong is correct, given that Mr Tan is wrong. [3]

8 A shop sells two brands of refrigerators which are in the same price range. The number of
Tahichi refrigerators sold per week is a random variable with the distribution Po(1.3) and the
number of Sungsam refrigerators sold per week is a random variable with the distribution
Po(1.1).
(i) Show that the probability of a total of at least 10 refrigerators being sold in a randomly
chosen 4-week period is 0.491, correct to 3 significant figures. [3]
A 4-week period is called a “good” period if at least 10 refrigerators are sold. Find, using
sgexamguru.com
(ii)
a suitable approximation, the probability that, in 52 randomly chosen 4-week periods,
there are more than 25 but at most 32 “good” periods. [4]
(iii) State, in the context of this question, two assumptions needed for your calculations in
part (i) to be valid. Explain why one of these assumptions may not hold in this context. [3]

9 The masses of grade A durians from a plantation are normally distributed with mean 1.96 kg
and standard deviation 0.24 kg and the masses of grade B durians from the same plantation are
normally distributed with mean 1.00 kg and standard deviation  kg.

The probability that a randomly chosen grade B durian has a mass of more than 0.8 kg is 0.95.
Show that  = 0.122, correct to 3 significant figures. [3]

(i) 50 grade A and 1 grade B durians are randomly picked from this plantation. Find the
probability that the average mass of the 50 grade A durians is more than twice the mass
of the grade B durian. Explain whether there is a need to use Central Limit Theorem in
your working. [4]
(ii) A wholesaler buys 50 grade B durians. Using a suitable approximation, find the
probability that more than 47 of the durians will have a mass of more than 0.8 kg. [4]

VJC P2 Questions 707



 

10 An ice-cream shop owner in Singapore wishes to find out how the daily sales of ice-cream
depend on the daily average temperature. The following data are collected over 10 days.

Day 1 2 3 4 5 6 7 8 9 10
Daily average temperature,
24.0 25.1 26.2 31.0 28.4 34.0 27.2 32.9 33.5 29.5
t oC
No. of cups of ice creams
100 130 140 171 158 179 150 176 178 163
sold in one day, x
(i) Without calculating the equation of the regression line of x on t, find the coordinates of a
point that will lie on this line. [2]
(ii) Draw a scatter diagram to illustrate the data and find the product moment correlation
coefficient between x and t. [2]

(iii) Without any calculations, explain whether a quadratic model is more appropriate than a
linear model to fit the data. [1]

The model x  a  34.2  t   b is used to fit the data. Calculate the least squares
2
(iv)
estimates of a and b. [2]
(v) By using the values found in part (iv), estimate the expected number of cups of ice
creams sold in 1 day if the daily average temperature is 31.0oC. [1]

11
sgexamguru.com
The mass X g, of one loaf of “Gardener” wholemeal bread is a random variable with mean μ g,
which is claimed to be 400g. A random sample of 5 loaves of wholemeal bread has masses in g
as follows,
371.3, 399.4, 402.3, 388.3, 400.4.

Carry out a test at the 5% significance level to determine whether this claim is valid, stating
clearly any assumption made. [4]

Another random sample of 50 loaves of wholemeal bread is taken, with results summarised
below,
  x  400   102.4 ,   x  400
2
 8030.2 .

Using the second sample, another test was carried out at the k% significance level to determine
the validity of the claim. Find the set of possible values of k for which the test concludes that
the claim is incorrect.
Explain, in the context of the question, the meaning of “k% significance level”. [5]
n hypothesis tests are carried out at 4% level of significance to test the validity of the claim.
Given that μ is indeed 400g, find the least value of n such that the probability of at most 1 test
making a wrong conclusion is less than 0.05. [3]

[End of Paper]

[Turn Over 
VJC P2 Questions 708
2016 VJC JC2 Preliim Paper 2 Solutions/Co
S omments

Qn Soluttion
1i Sincee speed is decreasing and
d v is positivee,
dv
= -kv, wwhere k is a positive connstant
dt
1 dv
= -k
v dt
ó 1 dv = -k dt
õv ò
lnn v = -kt + C  v > 0
v = Be-kt

Whenn t = 0s, v = D m s-1


B=D
Let k = p, hence v  De pt , where
w p is a ppositive consstant.
ii v
(0, D )

C2
C1
v=0
O sgexamguru.com t

Stretcch C1 paralleel to the t-axiis, factor e2, v-axis is invariant.


2 y  e x cos 2 x
dy e x cos 2 x  2e x sin x cos
c x

dx x
2 e cos x 2

y 2  e x ssin 2 x

2y
dy
2y  y 2  e x siin 2 x
dx

Alterrnative Solu
ution
y  e x cos 2 x
y 2  e x cos 2 x
dy
2y  e x cos 2 x  2e x sin x co
os x
dx
 y 2  e x siin 2 x
i  dy 
2
d2 y dy
2    2 y 2  2 y  e x sin 2 x  2e x cos 2 x
 dx  dx dx

Whenn x = 0, y  e0 cos 2 0  1

Microsoft Word - 2016 VJC H2 JC2 Prelim Paper 2 (Solutions) 709


Qn Solution
dy dy 1
2 1 0  
dx dx 2
2
1 d2 y 1 d2 y 3
2   2 2  2   0  2  2  
2 dx 2 dx 4
1 3æ x ö2
y = 1 + x - çç ÷÷÷ + 
2 4 çè 2! ø÷
1 3
= 1+ x - x2 + 
2 8
ii 1  1 3 
1

 1  x  x 2   
e x cos 2 x  2 8 
  1 2   1
2
1 3 
 1   1  x  x 2      x    
2 8  2! 2 
1 3 1
 1  x  x2  x2  
2 8 4
1 5
 1  x  x2  
2 8
3a From the diagram, Im D ( z2 )
 A (- 2,5)
arg  4  i  z1    arg  2  5i  z1 
2
i (4 + i - z1 ) = (-2 + 5i - z1 )
C (4,1)
4i -1 - iz1 = -2 + 5i - z1
(1 - i) z1 = -1 + i
z1 = -1
sgexamguru.com B ( z1 )O Re

æ -2 + 4 5 + 1ö÷
Midpoint of AC is çç , ÷ = (1,3)
çè 2 2 ø÷
Let z2 = x + iy
Since the diagonals of a square bisect other,
Midpoint of BD is (1,3)
æ x -1 y + 0 ö÷
ççç , ÷ = (1,3)
è 2 2 ø÷
\ x = 3, y = 6
z2 = 3 + 6i
bi 1 Alternatively
Let z = eiθ  z * = e-iθ =
z z 5 = z*
z 5  z 1 z 6 = zz * = z
2

z 1
6
z6 = 1
z 6  e 2 k i , k 
k i
ze 3
, k  0,1, 2,3, 4,5
 2 4 5
i i i i
 1,e 3 ,e 3 ,  1,e 3 ,e 3

VJC P2 Solutions 710


Qn Solution
ii  k
π i i
Since 0 < arg(z) < , z  e 3  zk  e 3
2
 k  k 
(1  i) i  i   i
k
 2e 4
.e 3
 2e 4 3 
z
(1  i)  k π 3
If k
is purely imaginary,    , ,
z 4 3 2 2
 k π 3
Since k is positive,    ,  ,
4 3 2 2
k 3 7
 , ,  
3 4 4
k 3
Smallest positive k  when    
3 4
9
Smallest positive k =  
4
4a  1  3
OC   2 a  b  , OD  b
3   5
 3 1 2
CD  b   2 a  b     5a  2b 
5 3   15  
Since line m passes through D and is parallel to CD, 
 
r = OD + μCD

3 2
= b + μ (2b - 5a )

sgexamguru.com
5  15  
3 2
= b - μ (5a - 2b)
5  15  
3
r = b + λ (5a - 2b), λ Î
 5  
3
Equation of m is r  b    5a  2b  ,   .
 5  
F is a point on m
 3
 OF  b    5a  2b  for a value of  O
5  
 
OF is perpendicular to l  OF   5a  2b   0
 
3  5a - 2b
  b    5a  2b     5a  2b   0  
5      
F
6
 3  a  b    b  b     25  a  a   20  a  b   4  b  b    0
  5        
6 2
 3  a  b   b    25 a  20  a  b   4 b   0
2 2

  5       

1
Since a  b  a b cos 60  2  5   5
    2
6 2
 3  5    5     25  2   20  5   4  5    0
2 2

5  
3
 
20

VJC P2 Solutions 711


Qn Solution
 3 3 3
 OF  b   5a  2b  =  5a  2b 
5  20   20  
Alternative Method O
 çæ  5a - 2b ÷ö 5a - 2b
DF = çç DO ⋅   ÷÷   5a - 2b
èç 5a - 2b ÷ø 5a - 2b
     
1 æ 3 ö
= ç- b ⋅ (5a - 2b)÷÷(5a - 2b) D F

5a - 2b çè 5    ø÷  
 
6 2
-3a ⋅ b + b
=   5 (5a - 2b)
(5a - 2b)⋅ (5a - 2b)  
   
6 2
-3(5) + (5)
= 5 (5a - 2b)
2 2
25 a - 20(a ⋅ b) + 4 b  
   
15
= 2 2
(5a - 2b)
25(2) - 20(5) + 4(5)  
3
= (5a - 2b)
20  
 3 3 3
 OF  b   5a  2b  =  5a  2b 
5  20   20  
b The equation of the plane  is 3x  2 y  5z  45.

 2
sgexamguru.com
 p, p,0  lies in   3 p  2 p  0  45  p  9

 3  2  3
       
 q  is perpendicular to  2   q   2  0
0 5  0 5
       
6  2q  0  q  3

Since v is perpendicular to both u and n ,


  
 3   2   15 
     
u  n =  2    3    10 
       
 5   0   13 
9 2  15 
     
r   9     3     10  ,  ,  
   0  13 
0    

Alternative method to find v



æ x ö÷
çç ÷
Let v = çç y ÷÷÷
 çç ÷÷
çè z ÷ø
 x   3  x  2 
       
 y    2   0 and  y    3   0
 z  5 z  0 
       

VJC P2 Solutions 712


Qn Solution
3x + 2y + 5z = 0 and 2x – 3y = 0
15 10
x = - z, y = - z, z = z
13 13
Let z = 13 (any non-zero number will work)
æ-15ö÷
çç ÷
v = çç-10÷÷÷  
çç ÷
çè 13 ÷÷ø
5i He will not get to survey the students who do not go to the school gymnasium. Hence, the sample
obtained is biased.

ii He can obtain a numbered list of all the students (labelled 1 to N) in the school. Using a random
number generator, he generates 30 distinct numbers. He will survey the students corresponding the
numbers generated.

Alternatively.
Let the total number of students be N
N
Sampling interval =
30
He can obtain a numbered list of all the students (labelled 1 to N) in the school.
N
Using a random number generator, select a starting number k where 1 £ k £ . He can interview the
30
N N 29 N
students corresponding to the numbers k, k + , k + ,, k + . 
30 15 30

sgexamguru.com
6i Number of 4-digit numbers = 54 = 625
ii Case1: Starts with 1 1 2
No. of ways = 2(3!) = 12 1 6

Case 2: starts with 2


No. of ways = 3! = 6 2 6

Total number of ways = 18

iii Case 1: XXXXYZ


No. of ways = 5C3(3C1) = 30

Case 2: XXXYYZ
No. of ways = 5C3(3C1)(2C1) = 60

Case 3: XXYYZZ
No. of ways = 5C3 = 10

Total number of ways = 100


7i æ 15 C ö 3 C
P(Mr Wong is correct) = ççç1 - 25 3 ÷÷÷´ 5 2 = 0.24065 = 0.241
çè C ÷ø C
3 2
ii 10
C1 ´ C1 ´ C1 3 C1 ´2 C1
8 7
84
P(Mr Tan is correct) = 25
´ 5 = = 0.146
C3 C2 575
Alternative method:
10 ´8´ 7 3´ 2
P(Mr Tan is correct) = ´ 3! ´ ´ 2!
25´ 24 ´ 23 5´ 4

VJC P2 Solutions 713


Qn Solution
84
= = 0.146
575
iii P(Mr Wong’s guess is right, given that Mr Tan’s guess is wrong)
P(Mr Wong is correct and Mr Tan is wrong)

P( Mr Tan is wrong)
0.24065

1  0.14609
 0.282

8i Let T be the total number of refrigerators sold in a 4-week period.


T Po((1.3 + 1.1)  4)
T Po(9.6)
P(T  10) = 1 – P(T  9) = 0.49114 = 0.491 (3sf)
ii Let X be number of good periods out of 52.
X B(52, 0.491) or X B  52, 0.49114 

Since np = 25.532 > 5 and np(1 – p) = 26.468 > 5


X N(25.532, 12.996) approx. or X N  25.539,12.996  approx.

P(25 < X  32) = P(25.5 < X  32.5) = 0.477 (or 0.478)


iii We need to assume that the sales of all the refrigerators are independent of one another.

sgexamguru.com
We also need to assume that the average rate of refrigerators being sold is constant.

The first assumption may not hold as the two brands of refrigerator are in the same price range and
they can be competing in terms of sales. 
OR
The average rate of refrigerators sold is unlikely to be a constant due to sale, festive seasons,
economic conditions etc.
9 Let A kg and B kg be masses of a randomly chosen grade A and grade B durian respectively.
A N(1.96, 0.242) and B N(1.00,  2)
P(B > 0.8) = 0.95
 0.8  1.00 
P Z    0.95
  

 0.8  1.00 
P Z    0.05
  
0.2
 1.64485    0.12159  0.122

i A N(1.96, 0.242) and B N(1.00,  2)
æ 0.242 ö÷÷
ç
A ~ N çç1.96, ÷ and
çè 50 ø÷÷

VJC P2 Solutions 714


Qn Soluttion
2B N(2.00,22 (0.1222) or 2 B N (2.000, 22  0.12159
9
2

A - 2 B ~ N (-0.04,0.060688
8) or A - 2 B ~ N (-0.0
04,0.060290)

P ( A - 2 B > 0) = 0.436 (or 0.435)

Centrral limit theoorem is not needed


n becauuse the massees of grade A durians folllow a normall
distriibution.
ii Let Y be the numbber of grade B durians w
with a mass off more than 0.8
0 kg out off 50 durians.

Y B B(50, 0.95)
np = 50  0.95 = 47.5
4 > 5 and n(1– p) = 500  0.05 = 2.5
5<5

Let Y
Y’ be the num with a mass  0.8 kg outt of 50 durianns.
mber of gradee B durians w
Y ' Po (2.5) appprox.

0  Y ' > 47)


P(Y > 47) = P( 50
= P Y   2 
= 0.5544

10i t andd x t = 29.18, x = 154.5

sgexamguru.com
Hencce, (29.18, 1554.5) lies on the regressioon line x on t.
t

ii  
  x

t /o C

r = 0.934 (3s.f.)

iii From
m the scatter diagram,
d x in
ncreases by ddecreasing am
mounts as t increases. Heence, a quadrratic model
mighht be more apppropriate.
 
iv By G
GC, a = -0.6773 (3sf), b = 179 (3sf)

v Substtituting t = 31.0,
x  0.67342  34.22  31.0   179
2
9.28
 1772.388

VJC P2 Solutions 715


Qn Solution
Expected number of cups of ice cream sold is 172.

11 H 0 :   400
H1 :   400
Level of significance: 5%
X  400
Test Statistic: When H 0 is true, T 
S/ 5
Computation:   5  1  4 .
By GC, x  392.34, s  12.971 , p-value = 0.257 (3sf)

Conclusion: Since p-value = 0.257 > 0.05, H0 is not rejected at 5% level of significance. So there is
insufficient evidence to conclude that the claim is invalid.

It is assumed that the masses of loaves of “Gardener” wholemeal bread follow a normal distribution.
102.4
x = 400 - = 397.952
50
1 ççæ (-102.4) ÷÷ö
2
2
s = ç8030.2 - ÷÷ = 159.60
49 ççè 50 ø÷

H 0 :   400
H1 :   400
Level of significance: k%

sgexamguru.com
Test Statistic: When H 0 is true, Z 
X  400
159.6017306
55
Computation:
By GC, x  397.952, p-value = 0.252 (3sf)

For H0 to be rejected at k% level of significance,


k
p-value   k  25.2
100
Set of values = k  : k  25.2

k
“k% significance level” in this context means there is a probability of(or k%) that the test will
100
conclude that the mean mass of “Gardener” wholemeal bread is not 400g, when it is actually 400g.
Let Y be the number of wrong conclusions out of n hypothesis tests
Y B( n,0.04)
P(Y  1)  0.05

By GC,
n P(Y  1)
116 0.05121
117 0.04952
Least n = 117

Alternatively

VJC P2 Solutions 716


Qn Soluttion
P(Y  1)  0.05
y0.966   n  0.96   0.04   0.05
n n1

n n-1
y = (0.96
6) + 0.04n (0..96)

y = 0.05
0 (116.71, 0.05
0 )
       n
O
     
Leastt n = 117

sgexamguru.com

VJC P2 Solutions 717


YISHUN JUNIOR COLLEGE
2016 JC2 PRELIMINARY EXAMINATION

MATHEMATICS 9740/01
Higher 2

Paper 1 18 AUGUST 2016


THURSDAY 0800h – 1100h
Additional materials :
Answer paper
Graph paper
List of Formulae (MF15)
YISHUNJUNIORCOLLEGEYISHUNJUNIORCOLLEGEYISHUNJUNIORCOLLEGEYISHUNJUNIORCOLLEGEYISHUNJUNIOR COLLEGE
YISHUNJUNIORCOLLEGEYISHUNJUNIORCOLLEGEYISHUNJUNIORCOLLEGEYISHUNJUNIORCOLLEGEYISHUNJUNIOR COLLEGE
YISHUNJUNIORCOLLEGEYISHUNJUNIORCOLLEGEYISHUNJUNIORCOLLEGEYISHUNJUNIORCOLLEGEYISHUNJUNIOR COLLEGE
YISHUNJUNIORCOLLEGEYISHUNJUNIORCOLLEGEYISHUNJUNIORCOLLEGEYISHUNJUNIORCOLLEGEYISHUNJUNIOR COLLEGE
YISHUNJUNIORCOLLEGEYISHUNJUNIORCOLLEGEYISHUNJUNIORCOLLEGEYISHUNJUNIORCOLLEGEYISHUNJUNIOR COLLEGE
YISHUNJUNIORCOLLEGEYISHUNJUNIORCOLLEGEYISHUNJUNIORCOLLEGEYISHUNJUNIORCOLLEGEYISHUNJUNIOR COLLEGE
YISHUNJUNIORCOLLEGEYISHUNJUNIORCOLLEGEYISHUNJUNIORCOLLEGEYISHUNJUNIORCOLLEGEYISHUNJUNIOR COLLEGE
YISHUNJUNIORCOLLEGEYISHUNJUNIORCOLLEGEYISHUNJUNIORCOLLEGEYISHUNJUNIORCOLLEGEYISHUNJUNIOR COLLEGE
YISHUNJUNIORCOLLEGEYISHUNJUNIORCOLLEGEYISHUNJUNIORCOLLEGEYISHUNJUNIORCOLLEGEYISHUNJUNIOR COLLEGE
YISHUNJUNIORCOLLEGEYISHUNJUNIORCOLLEGEYISHUNJUNIORCOLLEGEYISHUNJUNIORCOLLEGEYISHUNJUNIOR COLLEGE
YISHUNJUNIORCOLLEGEYISHUNJUNIORCOLLEGEYISHUNJUNIORCOLLEGEYISHUNJUNIORCOLLEGEYISHUNJUNIOR COLLEGE
YISHUNJUNIORCOLLEGEYISHUNJUNIORCOLLEGEYISHUNJUNIORCOLLEGEYISHUNJUNIORCOLLEGEYISHUNJUNIOR COLLEGE
YISHUNJUNIORCOLLEGEYISHUNJUNIORCOLLEGEYISHUNJUNIORCOLLEGEYISHUNJUNIORCOLLEGEYISHUNJUNIOR COLLEGE
YISHUNJUNIORCOLLEGEYISHUNJUNIORCOLLEGEYISHUNJUNIORCOLLEGEYISHUNJUNIORCOLLEGEYSIHUNJUNIOR COLLEGE

sgexamguru.com
YISHUNJUNIORCOLLEGEYISHUNJUNIORCOLLEGEYISHUNJUNIORCOLLEGEYISHUNJUNIORCOLLEGEYSIHUNJUNIOR COLLEGE
YISHUNJUNIORCOLLEGEYISHUNJUNIORCOLLEGEYISHUNJUNIORCOLLEGEYISHUNJUNIORCOLLEGEYSIHUNJUNIOR COLLEGE
YISHUNJUNIORCOLLEGEYISHUNJUNIORCOLLEGEYISHUNJUNIORCOLLEGEYISHUNJUNIORCOLLEGEYSIHUNJUNIOR COLLEGE
TIME
YISHUNJUNIORCOLLEGEYISHUNJUNIORCOLLEGEYISHUNJUNIORCOLLEGEYISHUNJUNIORCOLLEGEYSIHUNJUNIOR COLLEGE
3 hours
YISHUNJUNIORCOLLEGEYISHUNJUNIORCOLLEGEYISHUNJUNIORCOLLEGEYISHUNJUNIORCOLLEGEYSIHUNJUNIOR COLLEGE

READ THESE INSTRUCTIONS FIRST


Write your name and CTG in the spaces provided on all the work you hand in.
Write in dark blue or black pen on both sides of the paper.
You may use a soft pencil for any diagrams or graphs.
Do not use staples, paper clips, highlighters, glue or correction fluid.

Answer all the questions.


Give non-exact numerical answers correct to 3 significant figures, or 1 decimal place in the case
of angles in degrees, unless a different level of accuracy is specified in the question.
You are expected to use a graphic calculator.

Unsupported answers from a graphic calculator are allowed unless a question specifically states
otherwise.
Where unsupported answers from a graphic calculator are not allowed in a question, you are
required to present the mathematical steps using mathematical notations and not calculator
commands.
You are reminded of the need for clear presentation in your answers.

At the end of the examination, write down the question number of the questions attempted,
model of calculator used on the spaces provided on the cover page. Tie your cover page on top
of the answer scripts before submission.
The number of marks is given in brackets [ ] at the end of each question or part question.

This question paper consists of 5 printed pages.

9740/1 718
Yishun Junior College  2016 JC2 Preliminary Exam  H2 Mathematics 9740/01

1 A bakery sells strawberry, blueberry and walnut muffins. During a promotion, a


customer purchased all 3 types of muffins and twice as many strawberry muffins as
walnut muffins. The promotion price of each strawberry, blueberry and walnut muffin
is $1.60, $1.75 and $2.20 respectively. Given that the customer paid $53.40 for 30
muffins, find the number of each type of muffins purchased. [4]

2 x 1
2 Solve the inequality  . [3]
x2 3
2 x2
Hence, find the range of values of x for which  . [2]
x3 3

3 (i) Find
d
dx
 xe  x  . [1]


x  x2
(ii) Hence, find 
dx . [4]
 ex


4 It is given that h( x)  x cos x for 0  x  . It is also known that h( x)  h( x) and
2
h(  x)  h( x) for all real values of x.
(i) Sketch the graph of y  h(x) for 2 ≤ x ≤ 2.
sgexamguru.com
[3]
(ii) On a separate diagram, sketch the graph of y 2  h(x) for 2 ≤ x ≤ 2. [2]

5 The planes p1 , p2 and p3 have equations 3x  4 y  7 z  2 , x  2 y  4 and


5x  4 y  a z  3 respectively, where a is a constant. The point C has position vector
i  2 j  k .
(i) Given that a  2 , find the coordinates of the point of intersection of p1 , p2 and
p3 . [2]
(ii) Find the coordinates of the foot of perpendicular from C to p2 . [3]
(iii) Find the value of a such that p1 , p2 and p3 have no common points. [3]

6 Do not use a calculator in answering this question.


3i
The complex number z is given by z = .
2i
(i) Find |z| and arg z in exact form. [2]
(ii) Hence, find the exact values of x and y, where  < y ≤ , such that
3i
e x i2 y  . [3]
2i
n
 z2 
(iii) Find the smallest positive integer n such that   is purely imaginary. [3]
 z*

Page 2 of 5
YJC P1 Questions 719
Yishun Junior College  2016 JC2 Preliminary Exam  H2 Mathematics 9740/01

7 A company manufactures a container of length 150 mm. The container has a uniform
cross section made up of a rectangle y mm by x mm and 2 semi-circles of diameter
x mm (see diagram).

150 mm

x mm

y mm

Given that the container has a volume of 7200 mm3, find the exact value of x which
gives a container of minimum external surface area. [8]

8 (a) A bowl of hot soup is placed in a room where the temperature is a constant
20 C. As the soup cools down, the rate of decrease of its temperature  C after
time t minutes is proportional to the difference in temperature between the soup
and its surroundings. Initially, the temperature of the soup is 80 C and the rate of
decrease of the temperature is 4 C per minute. By writing down and solving a
sgexamguru.com
differential equation, show that  = 20 + 60 e 15 .

1
t

Find the time it takes the soup to cool to half of its initial temperature.
[6]
[2]

(b) The gradient of a curve C is given by


dy
 ( x  y)2 .
dx
Use the substitution u  x  y to show that the above equation reduces to
du
 1 u2 . [2]
dx
Hence find y in terms of x given that C passes through the origin. [2]

9 (a) Given that y  ln(cos x) , show that


2
d 2 y  dy 
    1 . [2]
dx 2  dx 
(i) By further differentiation of this result, find the Maclaurin series for y, up to
and including the term in x 4 . [5]
(ii) Hence, find the Maclaurin series for tan x , up to and including the term in
x3 . [2]

(b) Using an appropriate expansion from MF15, find the first three terms of the
Maclaurin series for ln(k  x)n , where n and k are positive constants. [3]

Page 3 of 5
YJC P1 Questions 720
Yishun Junior College  2016 JC2 Preliminary Exam  H2 Mathematics 9740/01

 1
10 (i) Find  dy . [1]
 1  3  y 
2

(ii)
y

O x

x
The diagram shows the curve with equation y  3  . Find the exact
4  x2
volume of revolution when the region bounded by the curve, the line y  1 and
the y-axis is rotated completely about the y-axis. [4]

By using the substitution x  2sin  , find the exact area of the region bounded by
sgexamguru.com
the curve, the line x  1 and the axes. [4]

11 In a training session, athletes run from a starting point S towards their coach in a
straight line. When they reach the coach, they run back to S along the same straight
line. A lap is completed when athletes return to S. At the beginning of the training
session, the coach stands at A1 which is 25 m away from S. After the first lap, the coach
moves from A1 to A2 and after the second lap, he moves from A2 to A3 and so on. The
points A1 , A2 , A3 , …, are increasingly further away from S in a straight line where
Ai Ai 1 = 1 m, i   . The training session will stop only when the athletes have run
more than 1500 m.

An athlete completes his first lap in 20 seconds but the time for each subsequent lap is
15% more than the time for the preceding lap. Given that the athlete must complete
each lap he runs and there is no resting time between laps, find the least amount of time
to complete the training session, giving your answer correct to the nearest minute. [5]

Assuming that the athlete runs at a constant speed for each lap, find the number of
complete laps when he has run for 15 minutes. [2]
Hence, find the distance from S and the direction of travel of the athlete after he has run
for exactly 15 minutes. [3]

Page 4 of 5
YJC P1 Questions 721
Yishun Junior College  2016 JC2 Preliminary Exam  H2 Mathematics 9740/01

 2 
 
12 Planes p and q are perpendicular to each other. Plane p has equation r  3   4 and
1
 
z2
plane q contains the line l with equation x  0, y  1  .
4
(i) Find a cartesian equation of q. [4]
(ii) Find a vector equation of the line m where p and q meet. [2]
(iii) Find the coordinates of the point C at which l intersects m. [3]
(iv) The points A and B have coordinates (0, −1, 2) and (x, 0, 0). If the area of triangle
ABC is increasing at a rate of 17 units 2 per second, find the rate of change of x
when x  5 . [5]

~ End of Paper ~

sgexamguru.com

Page 5 of 5
YJC P1 Questions 722
YISHUN JUNIOR COLLEGE
2016 JC2 PRELIMINARY EXAM PAPER 1
H2 MATHEMATICS
SOLUTIONS

Qn Solution
1 Let S, B and W be the number of strawberry, blueberry and walnut muffins purchased
respectively.

S  B  W  30
1.6S  1.75 B  2.2W  53.40
S  2W  S  2W  0

From GC, S  12, B  12, W  6

2 2 x 1

x2 3
2 x 1
 0
x2 3
6   x  1 x  2 
0
3( x  2)

3( x  2) sgexamguru.com
 x 2  3x  4
0

  x  4  x  1
0
3( x  2)
+  + 
4 2 1
Hence, x  4 or  2  x  1

2 x2
For 
x3 3

From above, x  1  4 or  2  x  1  1

x  5 or  3  x  0

YJC P1 Solutions 723


3(i) d
dx
 xe  x    xe  x  e  x

 e  x 1  x 
(ii) x  x2 1  x 
 e x 
dx  x x dx
e
 x 2e  x  xe x dx

 x 2e  x    xe x  e  x dx 

 

 x 2e  x  xe  x  e  x  C

4(i) y

(5.42, 0.561) (0.860, 0.561) (0.860, 0.561) (5.42, 0.561)

 
2 O 2 x

sgexamguru.com
(4.00, 0.561) (2.28, 0.561) (2.28, 0.561) (4.00, 0.561)

4(ii) y

(0.860, 0.749) (0.860, 0.749) (5.42, 0.749)


(5.42, 0.749)

2 O 2 x

(5.42, 0.749)
(5.42, 0.749) (0.860, 0.749) (0.860, 0.749)

YJC P1 Solutions 724


5(i) 3x  4 y  7 z  2
x  2y  4
5x  4 y  2 z  3
15 139 55
From GC, x   ,y ,z  
31 62 31
15 139 55 
 the point of intersection is   ,  , 
 31 62 31 
(ii) Let F be the foot of perpendicular from C to plane.
Equation of CF:
 1  1
r   2     2  ,   
 
1 0
   
 1 1
    
OF   2     2  , for some   
1 0
   
F also lies on plane p2
1
  
OF  2   4
0
 

  sgexamguru.com
 1     1 
 2  2  2   4
 1  0
  
1    4  4  4
5  9
9

5
 1 1  4 
   9    5 
 OF   2    2     8 5 
 1  5 0   1 
     
4 8 
the coordinates is  ,  ,1
5 5 
(iii) 3x  4 y  7 z  2
x  2y  4
From GC, p1 and p2 intersect at the line
2 14 
   
r   1     7  ,   
0 10 
   
No common points  p3 must be parallel to the line.

YJC P1 Solutions 725


 5  14 
 4  7   0
  
 a  10 
  
42  10a  0
a  4.2

6(i) 3  i  3  i  2  i  1
z    5  5i   1  i
2i 22  1 5
Therefore, | z | = 2
3 i 10
[Or z    2]
2i 5


arg z =
4

(ii) e x  i2y  z

i
e x e i2y  2e 4

  7 
i i  
x i2 y 4  4 
e  2 or e e or e

1
 x  ln 2  ln 2 or
2
sgexamguru.com y

8
or 
7
8
(iii)  z2 
n

For   to be purely imaginary,


 z*
n
 z2  
arg   = (2k  1) , k  
 z* 2

n  2 arg z  arg z *  (2k  1)
2
   
n     (2k  1)
2 4 2
2
n  (2k  1)
3

Hence, the smallest positive integer n = 2

YJC P1 Solutions 726


7 Let A and V be the external surface area and volume of the container respectively.
2
 x
A  300 y  150 x  2 xy  2  
 2
2
 x
V  150 xy  150  
2
75
7200  150 xy   x 2
2
48  x
y 
x 4
48  x
Substitute y   into surface area equation:
x 4
2
 48  x   48  x   x
A  300     150 x  2 x    
 x 4   x 4  2
14400
  96  75 x
x
dA
 14400 x2  75
dx

dA
To find least surface area, 0

sgexamguru.com
dx
14400
  75  0
x2
192 192
x or x   (rejected  x  0 )
 

By 2nd Derivative Test


d 2 A 28800

dx 2 x3
192 d 2 A
When x  ,  0 , A is minimum.
 dx 2

8(a) Note that  ≥ 20 or   20 ≥ 0


d
 k   20  , k > 0
dt
d
Given when t = 0,  = 80,  4
dt
1
 4 =  k (80  20)  k =
15
d 1
    20 
dt 15

YJC P1 Solutions 727


 1 d   1 1 dt
15 

   20 Alternatively:
1 1
ln ( 20) =  t  C ln | 20| =  t C
15 15
(20 > 0) 1
 t C
15
1 | 20| = e
 t C 1
15
 20 = e  t
15
1  20 = Ae ,
 t
 20 = Ae 15
, where A  e c
where A   e c
1 1
 t  t
15
 =20 + Ae 15  =20 + Ae

When t = 0,  = 80,
 80 = 20 + A i.e. A = 60
1
 t
15
 = 20 + 60 e

1
 t
40 = 20+60 e 15
1
 t 1
e 15 =
3
1 1
 t  ln   ln 3
15 3

sgexamguru.com
t = 15ln 3
(b) u=x+y
du dy
  1
dx dx
dy du
  1
dx dx

dy
 ( x  y) 2
dx
du
 1  u2
dx
du
  1  u2
dx

 1 du  1 dx

 1 u2 
tan 1 (u )  x  C
tan 1 ( x  y )  x  C

When x = 0, y = 0, C = 0

 x + y = tan x
Hence, y = tan x  x

YJC P1 Solutions 728


9(a) y  ln(cos x)
dy  sin x
   tan x
dx cos x
d2 y
2
  sec 2 x   1  tan 2 x 
dx
d2 y   dy  2 
  1    
dx2   dx  
2
d 2 y  dy 
    1
dx 2  dx 
(i) d3 y  dy  d y
2
 2   2 0
dx 3  dx  dx
4 3
d y  dy  d y d2 y d2 y
 2   3  2 0
dx 4  dx  dx dx 2 dx 2
dy d2 y d3 y d4 y
When x = 0, y = 0,  0 , 2  1 , 3  0 , 4  2
dx dx dx dx
2 4
x x
y  ln(cos x )   1   2   ...
2! 4!
1 1
=  x 2  x 4  ...

sgexamguru.com
2 12
(ii) dy
tan x  
dx
d  1 1 
    x 2  x 4  ...
dx  2 12 
 1 
    x  x 3  ... 
 3 
1
 x  x 3  ...
3

YJC P1 Solutions 729


(b) f  x   ln  k  x 
n

 n ln  k  x 
  x 
 n ln  k 1   
  k 
  x 
 n ln k  ln 1   
  k 
 x
 n ln k  n ln  1  
 k
 x 1  x 2 
 n ln k  n       
k 2k  
 
2
nx nx
 n ln k   2  
k 2k

10  1  3 y 
(i)  2
dy   1 tan 1  C
 1  (3  y )  1 
  tan 1  3  y   C
(ii) x
y  3
4  x2
x
4  x2
 3 y
sgexamguru.com
x2 2
2
 3  y 
4 x
2
x2  3  y   4  x2 
2 2
x2  4 3  y   x2 3  y 
2 2
x 2  x2  3  y   4  3  y 
2
2 4 3  y 
x  2
1  3  y 
4
 4 2
1  3  y 

YJC P1 Solutions 730


Volume of revolution about the y-axis
3
   x 2 dy
1
3 4
 4 2
dy
1
1  3  y 
3 3 1
   4 dy  4  2
dy
1 1
1 3  y 
3 3
   4 y 1  4   tan 1  3  y  1
 8  4 tan 1 (2)
Using the substitution x  2sin 
dx
 2cos 
d
1 
When x  1, sin     
2 6
When x  0, sin   0    0
Area under the curve
1 x
  3 dx
0
4  x2

 2sin  
 6
3   2cos   d

sgexamguru.com
0 2
 4  4sin  

4sin  cos 
6 6cos   d
0 2cos 

  6sin   2cos  06
 1 3

YJC P1 Solutions 731


11 Distance travelled per lap is in AP:
a1 = 50, d = 2  1 = 2.

Given total distance travelled > 1500


n
[ 2(50) + (n  1)2] > 1500
2
n2 + 49n  1500 > 0
(n + 70.33)(n  21.33) > 0
n < 70.33 or n > 21.33

Since n   , least n = 22

Time taken per lap is in GP:


a1 = 20, r = 1.15

Required least time taken


20  (1.15)22  1
=
1.15  1
 2752.6 s
 46 min

20  (1.15)n  1
1.15  1
(1.15)n  7.75
sgexamguru.com
 900

ln 7.75
n
ln1.15
n  14.65
Number of complete laps = 14

S14 
 14
20 1.15  1 
1.15  1
 810.094 s
He needs to run for another 900  810.094  89.906 s

Distance T15 = 50 + (15 1)2 = 78


89.906
14
 78  49.555
20 1.15 

He is running towards S and at a distance 78  49.555 28.4 m away from S.

10

YJC P1 Solutions 732


12 0  0
(i)    
l : r   1    1  ,   .
2  4
   
 2   0 
n 2   3    1 
 1   4
   
 11 
  8 
 2 
 
 11   0   11 
    
q : r  8    1 8 
 2   2   2 
    
 12
 the Cartesian equation is 11x  8 y  2 z  12
(ii) 2 x  3 y  z  4
11x  8 y  2 z  12
4 2 68 1
From GC, x     , y     , z  
49 7 49 7

sgexamguru.com
 4 
  49 
  2
 68   
r      1 ,   
 49 
  7
0  
 
 
 
(iii)   4  2 
 0   49 
   68 
 1       49   
 2  4   
   7
 
3 2
From GC,  =  ,  
7 49
3
Substitute    into l:
7
 
 0 
0  0  
  3    10 
r   1    1   
 2  7  4   7 
   
 2 
 7 
10 2
 the coordinates of C is  0,  ,  .
 7 7

11

YJC P1 Solutions 733


  
(iv) AB  OB  OA
 x  0 
   
  0    1
 0  2 
   
 x
 
 1 
 2 
 
  
AC  OC  OA
 
 0 
  0
 10   
   1
 7   
 2  2
 
 7 
 
 0 
 
3
  
 7 
 12 
 
 7 
sgexamguru.com
1  
Area of ABC , R  AB  AC
2
 
 x  0 
1
  1     3 
2   7
 2
    12 
 7
  18 
 7  6 
1  12 x  3  
  4x
2 7  14  
 3 x   x 
 7  
 

3 2 2 2
  6    4 x     x 
14
3
 36  16 x 2  x 2
14
3
 36  17 x 2
14

12

YJC P1 Solutions 734


1
dR 3 1 
   36  17 x 2  2  34 x 
dx 14 2
102 x

28 36  17 x 2
51x

14 36  17 x 2
dx dx dR
when x  5,  
dt dR dt
14 11
 17
51 5
154
 units per second
3 5

sgexamguru.com

13

YJC P1 Solutions 735


YISHUN JUNIOR COLLEGE
2016 JC2 PRELIMINARY EXAMINATION

MATHEMATICS 9740/02
Higher 2

Paper 2 24 August 2016


WEDNESDAY 0800h – 1100h
Additional materials :
Answer paper
Graph paper
List of Formulae (MF15)
YISHUNJUNIORCOLLEGEYISHUNJUNIORCOLLEGEYISHUNJUNIORCOLLEGEYISHUNJUNIORCOLLEGEYISHUNJUNIOR COLLEGE
YISHUNJUNIORCOLLEGEYISHUNJUNIORCOLLEGEYISHUNJUNIORCOLLEGEYISHUNJUNIORCOLLEGEYISHUNJUNIOR COLLEGE
YISHUNJUNIORCOLLEGEYISHUNJUNIORCOLLEGEYISHUNJUNIORCOLLEGEYISHUNJUNIORCOLLEGEYISHUNJUNIOR COLLEGE
YISHUNJUNIORCOLLEGEYISHUNJUNIORCOLLEGEYISHUNJUNIORCOLLEGEYISHUNJUNIORCOLLEGEYISHUNJUNIOR COLLEGE
YISHUNJUNIORCOLLEGEYISHUNJUNIORCOLLEGEYISHUNJUNIORCOLLEGEYISHUNJUNIORCOLLEGEYISHUNJUNIOR COLLEGE
YISHUNJUNIORCOLLEGEYISHUNJUNIORCOLLEGEYISHUNJUNIORCOLLEGEYISHUNJUNIORCOLLEGEYISHUNJUNIOR COLLEGE
YISHUNJUNIORCOLLEGEYISHUNJUNIORCOLLEGEYISHUNJUNIORCOLLEGEYISHUNJUNIORCOLLEGEYISHUNJUNIOR COLLEGE
YISHUNJUNIORCOLLEGEYISHUNJUNIORCOLLEGEYISHUNJUNIORCOLLEGEYISHUNJUNIORCOLLEGEYISHUNJUNIOR COLLEGE
YISHUNJUNIORCOLLEGEYISHUNJUNIORCOLLEGEYISHUNJUNIORCOLLEGEYISHUNJUNIORCOLLEGEYISHUNJUNIOR COLLEGE
YISHUNJUNIORCOLLEGEYISHUNJUNIORCOLLEGEYISHUNJUNIORCOLLEGEYISHUNJUNIORCOLLEGEYISHUNJUNIOR COLLEGE
YISHUNJUNIORCOLLEGEYISHUNJUNIORCOLLEGEYISHUNJUNIORCOLLEGEYISHUNJUNIORCOLLEGEYISHUNJUNIOR COLLEGE
YISHUNJUNIORCOLLEGEYISHUNJUNIORCOLLEGEYISHUNJUNIORCOLLEGEYISHUNJUNIORCOLLEGEYISHUNJUNIOR COLLEGE
YISHUNJUNIORCOLLEGEYISHUNJUNIORCOLLEGEYISHUNJUNIORCOLLEGEYISHUNJUNIORCOLLEGEYISHUNJUNIOR COLLEGE
YISHUNJUNIORCOLLEGEYISHUNJUNIORCOLLEGEYISHUNJUNIORCOLLEGEYISHUNJUNIORCOLLEGEYSIHUNJUNIOR COLLEGE

sgexamguru.com
YISHUNJUNIORCOLLEGEYISHUNJUNIORCOLLEGEYISHUNJUNIORCOLLEGEYISHUNJUNIORCOLLEGEYSIHUNJUNIOR COLLEGE
YISHUNJUNIORCOLLEGEYISHUNJUNIORCOLLEGEYISHUNJUNIORCOLLEGEYISHUNJUNIORCOLLEGEYSIHUNJUNIOR COLLEGE
YISHUNJUNIORCOLLEGEYISHUNJUNIORCOLLEGEYISHUNJUNIORCOLLEGEYISHUNJUNIORCOLLEGEYSIHUNJUNIOR COLLEGE
TIME
YISHUNJUNIORCOLLEGEYISHUNJUNIORCOLLEGEYISHUNJUNIORCOLLEGEYISHUNJUNIORCOLLEGEYSIHUNJUNIOR COLLEGE
3 hours
YISHUNJUNIORCOLLEGEYISHUNJUNIORCOLLEGEYISHUNJUNIORCOLLEGEYISHUNJUNIORCOLLEGEYSIHUNJUNIOR COLLEGE

READ THESE INSTRUCTIONS FIRST


Write your name and CTG in the spaces provided on all the work you hand in.
Write in dark blue or black pen on both sides of the paper.
You may use a soft pencil for any diagrams or graphs.
Do not use staples, paper clips, highlighters, glue or correction fluid.

Answer all the questions.


Give non-exact numerical answers correct to 3 significant figures, or 1 decimal place in the case
of angles in degrees, unless a different level of accuracy is specified in the question.
You are expected to use a graphic calculator.

Unsupported answers from a graphic calculator are allowed unless a question specifically states
otherwise.
Where unsupported answers from a graphic calculator are not allowed in a question, you are
required to present the mathematical steps using mathematical notations and not calculator
commands.
You are reminded of the need for clear presentation in your answers.

At the end of the examination, write down the question number of the questions attempted,
model of calculator used on the spaces provided on the cover page. Tie your cover page on top
of the answer scripts before submission.
The number of marks is given in brackets [ ] at the end of each question or part question.

This question paper consists of 5 printed pages.

9740/1 736
Yishun Junior College  2016 JC2 Preliminary Exam  H2 Mathematics 9740/02

Section A: Pure Mathematics [40 marks]

1 A sequence u1 , u2 , u3 , ... is given by


u1  1 and un1  nun  1 for n ≥ 1.
Use the method of induction to prove that
n 1
1
un  (n  1)! . [4]
r 0 r !

un
Hence, find the exact value of lim . [1]
n   n  1!

5x  2 A B C
2 Show that f ( x)  can be expressed as   , where A, B and
x  x  1 x  2  x 1 x x  2
C are constants to be determined. [1]
n
Hence, find  f (r ) . (There is no need to express your answer as a single algebraic
r 2
fraction.) [3]
n n 1
Explain, with the aid of a sketch of y  f ( x) , x > 1, why  f (r ) > 2
r 2
f ( x) dx for

n 2. [2]

sgexamguru.com
3 The parametric equations of a curve C are x  t  a sin t , y  t cos t , where
 
 t  and a is a constant. It is given that the normal to C at x = 0 is parallel to
2 2
the x-axis.
(i) Show that a  1 . [3]
(ii) Sketch C, giving the coordinates of any points of intersection with the axes. [2]
(iii) Find the area of the region enclosed by C and the x-axis. [3]

4 The functions f and g are defined as follows.


x2  1
f:x , x  0,
2x
1
g:x , x  0.
x
(i) Determine whether the composite function gf exists. If it exists, define gf in a
similar form and find the range of gf. [4]
(ii) Give a reason why f does not have an inverse function. [1]
(iii) If the domain of f is further restricted to x ≥ k, state the least value of k for which
the function f 1 exists. Find f 1 ( x) and write down the domain of f 1 . [4]

Page 2 of 5
YJC P2 Questions 737
Yishun Junior College  2016 JC2 Preliminary Exam  H2 Mathematics 9740/02

5 (i) Solve the equation


z6 + 64 = 0,
giving the roots in the form rei, where r > 0 and      . [3]

(ii) The roots in part (i) represented by zn , where 1  n  6 , are such that
  arg( z1 )  arg( z2 )   arg( z6 )   . Show the roots on an Argand diagram
and describe geometrically the relationship between the roots. [4]

The complex number w satisfies the equation iw  4  4 3i  2 .

(iii) On the same Argand diagram, sketch the locus iw  4  4 3i  2 . [3]


(iv) Hence, find the maximum possible value of |w  zn|. [2]

Section B: Statistics [60 marks]

6 The CEO of a company with 40 000 employees wishes to investigate employees’


opinions about the food stalls in the staff canteen. 2% of the employees will be chosen
to take part in the survey. Explain briefly how the CEO could carry out a survey using
(i) random sampling, [2]
(ii) quota sampling. [2]

7
sgexamguru.com
In a certain town, every car license plate number is a 4-digit number where the digits
are chosen from 1 to 9 and cannot be repeated.
Find the number of different car license plate numbers if
(i) there are no restrictions, [1]
(ii) the digits of the car license plate number must not be in ascending order from left
to right, [2]
(iii) exactly one of the digits is an even number. [2]
Due to an increasing population, it is decided that the digits used in the car license plate
number can be repeated.
(iv) Find the number of different car license plate numbers where no digits can be
larger than the third digit. [2]

8 (a) Given that events X and Y are independent, prove that events X and Y ' are
independent. [2]

(b) For events A and B, it is given that P( A) = 0.5, P( B) = 0.6 and P( A ' | B ') = 0.3.
Find
(i) P( A  B) , [3]
(ii) P( B ' | A) . [2]

Stating your reason, determine if events A and B are


(iii) mutually exclusive, [1]
(iv) independent. [1]

Page 3 of 5
YJC P2 Questions 738
Yishun Junior College  2016 JC2 Preliminary Exam  H2 Mathematics 9740/02

9 Rickie takes the train home after work on weekdays.


(i) The number of days in a week where Rickie finds a seat on the train is denoted by
A. State, in context, two assumptions needed for A to be well modelled by a
binomial distribution. [2]
Assume now that A has the distribution B(5, 0.65).
(ii) Rickie is contented if he finds a seat on two or three days in a week. Using a
suitable approximation, find the probability that in a year (52 weeks), Rickie is
contented in no more than 30 weeks. [5]

10 In a certain country, it is to be assumed that the number of drug trafficking cases per
week can be modelled by the distribution Po(0.2) and the number of cigarette
trafficking cases per week can be modelled by the independent distribution Po(0.7).

(i) Find the probability that, in a randomly chosen period of 8 weeks,


(a) the country has more than 6 drug trafficking cases, [2]
(b) the total number of drug and cigarette trafficking cases is fewer than 5. [2]

(ii) The probability that the country sees fewer than 2 drug trafficking cases in a
period of n weeks is less than 0.01. Express this information as an inequality in n,
and hence find the smallest possible integer value of n. [3]
(iii) Give two reasons in context why the assumptions made at the start of the question
may not be valid. [2]

11
sgexamguru.com
In this question, you should state clearly the values of the parameters of any normal
distribution you use.

The masses, in grams, of towels manufactured by companies Alpha and Bravo are
modelled as having independent normal distributions with means and standard
deviations as shown in the table.

Mean Standard Deviation


Alpha  20
Bravo 275 15

(i) Given that 6.68% of the towels from Alpha have mass more than 380 grams,
show that the value of  is 350 grams, correct to 3 significant figures. [2]

Towels from Alpha and Bravo are soaked in water to investigate their absorbency. A
soaked towel from Alpha is 60% heavier than its dry towel, while a soaked towel from
Bravo is 50% heavier than its dry towel.

(ii) Find the probability that the total mass of 4 soaked towels from Alpha and 2
soaked towels from Bravo exceeds 3 kilograms. [4]

Page 4 of 5
YJC P2 Questions 739
Yishun Junior College  2016 JC2 Preliminary Exam  H2 Mathematics 9740/02

12 A new Burger Chain, Burger Queen, claims that the mean waiting time for a burger is
at most 4 minutes. The CEO of Burger Jack decides to record the waiting time, x
minutes, for a burger at Burger Queen at 80 different locations. The results are
summarised by
 ( x  4)  25 ,  ( x  4)2  140 .
(i) Find unbiased estimates of the population mean and variance. [2]
(ii) Test, at the 1% level of significance, whether there is any evidence to doubt
Burger Queen’s claim. [4]
(iii) It is assumed that the standard deviation of the waiting time for a Burger Queen
burger is 1.5 minutes. Given that the mean waiting time at another 80 locations is
x , use an algebraic method to find the set of values of x for which Burger
Queen’s claim would not be rejected at the 10% level of significance. [3]

13 (i) Sketch a scatter diagram that might be expected when x and y are related
approximately as given in each of the cases (A), (B), (C) below. In each case your
diagram should include 6 points, approximately equally spaced with respect to x,
and with all x- and y- values positive. The letters a, b, c, d, e and f represent
constants.
(A) y  a  bx 2 , where a is positive and b is negative,
d
(B) y  c  , where both c and d are positive,
x
(C) y  e  fx , where e is positive and f is negative.
sgexamguru.com
[3]

An archaeologist found an unknown substance on an excavation trip. Research is being


carried out to investigate how the mass of the substance varies with time, measured
from when it is placed in a cooled chamber. Observations at successive times give the
data shown in the following table.

Time (x hours) 100 800 1500 3000 6000 8000


Mass (y grams) 25 8 5 4 3.5 3.3

(ii) Draw the scatter diagram for these values, labelling the axes. [1]
(iii) Explain which of the three cases in part (i) is the most appropriate for modelling
these values, and calculate the product moment correlation coefficient for this
case. [2]
(iv) Use the case that you identified in part (iii) to find the equation of a suitable
regression line and estimate the time when the mass of the substance is 10 grams.
Comment on the reliability of the estimate. [3]

~ End of Paper ~

Page 5 of 5
YJC P2 Questions 740
YISHUN JUNIOR COLLEGE
2016 JC2 PRELIMINARY EXAM PAPER 2
H2 MATHEMATICS
SOLUTIONS

Qn Solution
n 1
1 1
Let Pn be the statement “ un  (n  1)! ”
r0 r !
When n =1, L.H.S. = u1 = 1 (Given)
1
R.H.S = (1  1)!  = 1
 0! 
Hence, P1 is true.

Assume that Pk is true for some k  ,


k 1
1
ie, uk  (k  1)! .
r0 r !
k
1
To show Pk+1 is true, ie, uk 1  (k )! .
r0 r !

uk 1  kuk  1
k 1
1
 k (k  1)! 1
r0 r !
k 1

sgexamguru.com
1 k!
 k ! 
r 0 r ! k!
 k 1 1 1 
 k !   
 r0 r ! k ! 
k
1
 k !
r 0 r !

Therefore, Pk is true  Pk+1 is true

By Mathematical Induction, Pn is true for all n ≥ 1.

n 1
un 1
 =
 n  1! r 0 r !

un 1
lim   = ex|x=1 = e
n   n  1 !
r 0 r !

YJC P2 Solutions 741


2 5x  2 1 1 2
f ( x)  =  
x  x  1 x  2  x  1 x x  2
n n
 1 1 2 
 f (r )    r  1  r  r  2 
r2 r 2

1 1 2
  
1 2 4
1 1 2
  
2 3 5
1 1 2
  
3 4 6
1 1 2
  
4 5 7
1 1 2
  
5 6 8

1 1 2
  
n  4 n  3 n 1
1 1 2
  
n 3 n 2 n
1 1 2
  

sgexamguru.com
n  2 n 1 n 1
1 1 2
  
n 1 n n  2
8 1 2 2 
=    
3  n n 1 n  2 
y y=f(x)

f(2)

… n n+1
x=1 2 3 4 5 x

From the above diagram,


Sum of areas of (n1) rectangles
> Area under graph of y = f(x) from x = 2 to n+1
n 1 n n 1
(1)f (2)  (1)f (3)  (1)f (4)    (1)f (n) >  f ( x) dx   f (r ) >  f ( x) dx
2 2
r 2

YJC P2 Solutions 742


3(i) x  t  a sin t y  t cos t
dx dy
 1  a cos t  t sin t  cos t
dt dt
dy dy dt
 
dx dt dx
t sin t  cos t

1  a cos t

normal parallel to x-axis  1  a cos t  0


1
a --- (1)
cos t
When x = 0, t  a sin t  0 ----- (2)
Sub (1) into (2): t  tan t  0
 
From GC, t = 0 (since  t )
2 2
1
Sub into (1): a   1 (shown)
cos 0

(ii)
y

sgexamguru.com
x
(− 0.571, 0) O
(0.571, 0)

(iii) Area of region



0 1
2
=   y dx   y dx
1 0
2

0
   t cos t 1  cos t  dt   2 t cos t 1  cos t  dt
 0
2

 0.408

YJC P2 Solutions 743


4(i) gf exists when R f  D g .

Since R f  1,     0,    D g , therefore gf exists.

 x2 1 
gf ( x)  g  
 2x 
2x
 2
x 1

2x
gf : x  2
, x0
x 1
f
 0,     1,   
g
  0,1

 R gf   0,1

4(ii)
y
y = f(x)

y=2

(1,1)

sgexamguru.com y = 0.5 x

The line y = 2 cuts the graph of f more than once. Hence, f is not a one–one function.
Therefore, f does not have an inverse.

4(iii) From the graph, least value of k = 1

x2  2yx + 1 = 0

2 y  4 y2  4
x=  y  y2 1
2

Since x ≥ 1, x = y  y 2  1

f 1(x) = x  x 2  1 , x ≥ 1

YJC P2 Solutions 744


5(i) z6 =  64 = 64 ei
= 64 ei(2k +1) , k  
 2 k 1 
i 
 6 
z = 2e , k  0,  1,  2, 3

       5     5 
i  i  i   i  i   i  
z  2e  6  , 2e  2  , 2e  6
, 2e  6 
, 2e  2
, 2e  6 

 5     
i   i   i  
(ii) z1  2e  6 
, z2  2e  2
, z3  2e  6

     5 
i  i  i 
6 2  6 
z4  2e , z5  2e , z6  2e

Im(z)

z5

5
z6 6 2 z4
6

2 2 Re(z)

− 6
5

sgexamguru.com
− 2
z1 6 z3

z2

The 6 points representing the 6 roots are the vertices of a regular hexagon.

(iii) |iw + 4 + 4 3 i| = 2
| i | |w  i(4 + 4 3 i)| = 2
|w  (4 3 + 4i)| = 2

Im(z)

A
2
(4√3, 4)
C
z5
|iw + 4 + 4√3i| = 2 z4
z6

2 O 2 Re(z)
z1 z3
z2 B

YJC P2 Solutions 745


(iv) 5
As arg (4 3 + 4i) = , O, C and B are collinear.
6
Hence, maximum |w  zn| =|w  z3| = AB
= OC + CA + OB
2
= 4 3   42  2  2
= 12

6(i) Number the employees from 1 to 40 000. Randomly select 800 numbers using a random
number generator. The employees corresponding to these 800 numbers are selected for
the survey.

The manager can survey the first 400 male employees and first 400 female employees
(ii) who step into the canteen on a particular day.

7(i) No of ways = 9  8  7  6
= 3024
(ii) No of ways = 3024  C49
= 2898
(iii) sgexamguru.com
No of ways = C14  C35  4!
= 960
9
(iv) 3
No of ways= r
r 1
= 2025

8(a) P( X  Y ')  P( X )  P( X  Y )
 P( X )  P( X )  P(Y ) since X and Y are ind.
 P ( X )[1  P (Y )]
 P( X )  P(Y ')
Since P( X  Y ')  P( X )  P(Y ') , events X and Y ' are independent.

(b) P(A’|B’)=0.3
(i) P( A ' B ')
 0.3
P( B ')
P( A ' B ')  0.3  (1  0.6)
=0.12

P( A  B)  1  0.12  0.88
P( A  B)  P( A)  P( B)  P( A  B)
P( A  B) =0.5+0.6-0.88
=0.22

YJC P2 Solutions 746


(ii) P( B ' A)  P( A)  P( A  B)
 0.5  0.22  0.28
P( B ' A)
P( B ' | A) 
P( A)
0.28
  0.56
0.5

(iii) Since P( A  B)  0.22  0 ,


events A and B are not mutually exclusive

(iv) P( A  B)  0.22
P( A)  P( B)  0.5  0.6  0.3

Since P( A  B)  P( A)  P( B) , events A and B are not independent.

9(i) (1) The probability of Rickie finding a seat on the train is constant every weekday.
(2) The event of Rickie finding a seat on the train on one weekday is independent of the
other weekdays.
A~B(5, 0.65)

sgexamguru.com
(ii)
P( A  2 or 3)  P( A  3)  P( A  1)

= 0.51756
Let X be the number of weeks, out of 52, that Rickie is contented.
Then X ~ B(52, 0.51756)

Since n = 52 is large, np = 52(0.51756)=26.91312 > 5,


nq = 52(1−0.51756)=25.08688 > 5,
X ~ N(26.91312, 52  0.51756  (1  0.51756)) approx.

i.e. X ~ N(26.91312,12.98397) approx.

P( X  30)  P( X  30.5) using continuity correction

 0.840 (3 s.f)

YJC P2 Solutions 747


10(i) Let X and Y be the r.v. “number of drug trafficking and cigarette trafficking cases in 8
(a) weeks respectively”.
Then X ~Po(1.6) , Y ~ Po(5.6)

P( X  6)  1  P( X  6)
 0.00134 (3 s.f)

(i) X + Y ~ Po(7.2)
(b) P( X  Y  5)  P( X  Y  4)
 0.156 (3 s.f)

(ii) Let W be the number of drug trafficking cases in a period of n weeks.


Then W~Po(0.2n)
P(W  2)  0.01
P(W  1)  0.01
e 0.2 n  0.2n(e 0.2 n )  0.01

Using GC,

sgexamguru.com
If n = 33, P(W  1)  0.0103  0.01
If n = 34, P(W  1)  0.00869  0.01
 smallest possible integer value of n is 34.

Alternative method: Plot graph


Using GC, n  33.19176
 smallest possible integer value of n is 34.

The number of drug and cigarette trafficking cases per week may decrease after police’s
(iii) raids so the trafficking cases may not occur with a constant average rate.

A person can be both a drug and cigarette trafficker so the drug trafficking cases and
cigarette trafficking cases may not be independent.

11(i) Let A and B be the mass of a randomly chosen towel from Alpha and Bravo
respectively.
P( A  380)  0.0668
P( A  380)  0.9332
380  
P( Z  )  0.9332
20
380  
Using GC,  1.5000556
20

YJC P2 Solutions 748


380    30.001112
  349.998888
 350 (3 s.f)

(ii) T  1.6( A1  A2  A3  A4 )  1.5( B1  B2 )


~ N(1.6  4  350  1.5  2  275, 1.62  4  202  1.52  2  152 )
i.e. T ~ N(3065, 5108.5)

P(T  3000)  0.81843...


 0.818 (3 s.f)

12(i)  ( x  4)  4
Unbiased estimate of population mean, x 
80
25

 4 = 4.3125
80
1  252 
Unbiased estimate of population variance, s 2   140  
80  1  80 
=1.673259…

sgexamguru.com
 1.67 (3 s.f)

H0 :   4
(ii)
H1 :   4
X 
Under H 0 , the test statistic is Z  ~ N (0,1) approx. (by CLT), where
S/ n
  4, x  4.3125, s 2  1.6733, n  80
Level of significance=1%
Using GC, p-value=0.015357 (5 s.f)
Since p-value=0.015357>0.01, we do not reject H 0 and conclude that at the 1% level,
there is no sufficient evidence to doubt Burger Queen’s claim.

Critical value=1.28155
(iii) In order not to reject H 0 ,
x4
 1.28155
1.5 / 80
x  4.2149...
Required set = {x    : x  4.21}

YJC P2 Solutions 749


13(i) (A) y  a  bx 2 , where a is positive and b is negative

y
x
x
x
x
x
x
x

d
(B) y  c  , where both c and d are positive
x

y
x

x
x x x x

sgexamguru.com x

(C) y  e  fx , where e is positive and f is negative


y
x
x
x
x
x
x

10

YJC P2 Solutions 750


(ii)
y
x (100,25)

x (8000,3.3)
x x x x
x

Case (B) is the most appropriate because as x increases, y decreases at a decreasing rate.
(iii)
From GC, r = 0.994981…
 0.995 (3s.f)

2154.915
From GC, y  3.62896 
(iv) x
2150
y  3.63  (3 sf)
x

sgexamguru.com
When y =10, 10  3.62896 
2154.915
x
x  338 (3s.f)
The estimate is likely to be reliable as y =10 is within the range of given y values and r
value is close to 1.

11

YJC P2 Solutions 751

You might also like